0% found this document useful (0 votes)
8 views

MATHS 1 Compressed Combined

The document presents study support material for class XI mathematics, emphasizing the importance of personalized learning for each student. It includes a detailed curriculum outline and a question paper design for the academic year 2024-25, along with key concepts and properties related to sets. The material aims to equip students with essential skills and competencies in mathematics as per the CBSE curriculum.

Uploaded by

repegom612
Copyright
© © All Rights Reserved
We take content rights seriously. If you suspect this is your content, claim it here.
Available Formats
Download as PDF, TXT or read online on Scribd
0% found this document useful (0 votes)
8 views

MATHS 1 Compressed Combined

The document presents study support material for class XI mathematics, emphasizing the importance of personalized learning for each student. It includes a detailed curriculum outline and a question paper design for the academic year 2024-25, along with key concepts and properties related to sets. The material aims to equip students with essential skills and competencies in mathematics as per the CBSE curriculum.

Uploaded by

repegom612
Copyright
© © All Rights Reserved
We take content rights seriously. If you suspect this is your content, claim it here.
Available Formats
Download as PDF, TXT or read online on Scribd
You are on page 1/ 401

P

Vidarbha Youth Welfare Society’s

MATHS
STANDARD XI
PRINCIPAL MESSAGE

It brings me great pleasure to present the study support material


specifically designed for students of class 11th by our dedicated team of
subject experts. The support material epitomizes commendable
endeavour towards harmonizing contain with the latest CBSE
patterns serving as a facilitative tool for comprehending acquiring
and honing essential skills and competencies stipulated within the
curriculum .
In the insightful words of Rabindranath Tagore, "Don't limit a
child to your own learning for he was born in another time."
Every child is unique, with their own interests, abilities and
potential .By allowing children to learn beyond the scope of our own
experience, we support their individual growth and development
helping them to reach their full potential in their own right.
May every student embrace the joy of learning and be
empowered with the tools and confidence to navigate and shape the
future.

Principal

Mr. Ajay Chowbe


INDEX
S.NO. CONTENT PAGE NO

1. CURRICULUM 05 – 08

2. SETS 09 – 24

3. RELATIONS & FUNCTIONS 25 – 40

4. TRIGONOMETRIC FUNCTIONS 41 – 57

5. COMPLEX NUMBERS & QUADRATIC EQUATIONS 58 – 63

6. LINEAR INEQUALITIES 64 – 78

7. PERMUTATIONS & COMBINATIONS 79 – 101

8. BINOMIAL THEOREMS 102 – 110

9. SEQUENCE & SERIES 111 -140

10. STRAIGHT LINES 141 – 161

11. CONIC SECTIONS 162 – 193

12. 3D 194 – 217

13. LIMITS & DERIVATIVES 218 – 229

14. STATISTICS 230 – 254

15. PROBABILITY 255 – 284

16. SAMPLE QUESTION PAPERS WITH BP & MS 285 - 349

4
5
+

6
7
MATHEMATICS (Code No. - 041)
QUESTION PAPER DESIGN CLASS - XI
(2024-25)
Time: 3 hours Max. Marks: 80

S. %
Typology of Questions Total
No. Weightage
Marks

Remembering: Exhibit memory of previously learned material


by recalling facts, terms, basic concepts, and answers. 55
1 44
Understanding: Demonstrate understanding of facts and
ideas by organizing, comparing, translating, interpreting,
givingdescriptions, and stating main ideas
Applying: Solve problems to new situations by applying
2 20 25
acquired knowledge, facts, techniques and rules in a
differentway.
Analysing :
Examine and break information into parts by identifying
motives or causes. Make inferences and find evidence to
support generalizations
Evaluating:
Present and defend opinions by making judgments about
3 information, validity of ideas, or quality of work based on a set 16 20
of criteria.
Creating:
Compile information together in a different way by
combiningelements in a new pattern or proposing
alternative solutions

Total 80 100

1. No chapter wise weightage. Care to be taken to cover all the chapters


2. Suitable internal variations may be made for generating various templates keeping the overall
weightage to different form of questions and typology of questions same.

Choice(s):
There will be no overall choice in the question paper. However, 33% internal choices will be
given in all the section.
INTERNAL ASSESSMENT 20 MARKS
Periodic Tests ( Best 2 out of 3 tests conducted) 10 Marks

Mathematics Activities 10 Marks

8
SETS
KEY POINTS:
Set : a set is a well-defined collection of objects
If a is an element of a set A, we say that “ a belongs to A” the Greek symbol  (epsilon) is used to denote the
phrase ‘belongs to’. Thus, we write a  A. If ‘b’ is not an element of a set A, we write b  A and read “b does
not belong to A”.
There are two methods of representing a set :
(i) Roster or tabular form (ii) Set-builder form.
In roster form, all the elements of a set are listed, the elements are being separated by commas and
are enclosed within brackets { }. For example, the set of all even positive integers less than 7 is described in
roster form as {2, 4, 6}.
In set-builder form, all the elements of a set possess a single common property which is not
possessed by any element outside the set. For example, in the set {a, e, i, o, u}, all the elements possess a
common property, namely, each of them is a vowel in the English alphabet, and no other letter possess this
property. Denoting this set by V, we write V = {x : x is a vowel in English alphabet}
** Empty Set : A set which does not contain any element is called the empty set or the null set or the void
set. The empty set is denoted by the symbol φ or { }.
** Finite and Infinite Sets : A set which is empty or consists of a definite number of elements is
called finite otherwise, the set is called infinite.
** Equal Sets : Two sets A and B are said to be equal if they have exactly the same elements and we write
A = B. Otherwise, the sets are said to be unequal and we write A ≠ B.
** Subsets : A set A is said to be a subset of a set B if every element of A is also an element of B.
In other words, A  B if whenever a  A, then a  B. Thus A  B if a  A  a  B
If A is not a subset of B, we write A  B.
** Every set A is a subset of itself, i.e., A  A.
** φ is a subset of every set.
** If A  B and A ≠ B , then A is called a proper subset of B and B is called superset of A.
** If a set A has only one element, we call it a singleton set. Thus, { a } is a singleton set.
** Closed Interval : [a , b] = {x : a ≤ x ≤ b} where a,b (Set of Real Numbers)
** Open Interval : (a , b) = { x : a < x < b}
** Semi Closed/Semi open Interval : [a , b) = {x : a ≤ x < b}
** Semi Open/Semi closed Interval : (a , b] = { x : a < x ≤ b }
** Power Set : The collection of all subsets of a set A is called the power set of A. It is denoted by P(A)
If A is a set with n(A) = m, then it can be shown that n [ P(A)] = 2 m.
9
** Universal Set : The largest set under consideration is called Universal set.
** Union of sets : The union of two sets A and B is the set C which
consists of all those elements which are either in A or in
B (including those which are in both). In symbols, we write.
A  B = { x : x  A or x  B }.
X  A  B ⇒ x ∈ A or x ∈ B

x  A ∪ B ⇒ x  A and x  B

** Some Properties of the Operation of Union

(i) A  B = B  A (Commutative law)


(ii) ( A  B )  C = A  ( B  C) (Associative law )
(iii) A  φ = A (Law of identity element, φ is the identity of  )
(iv) A  A = A (Idempotent law)
(v) U  A = U (Law of U)

** Intersection of sets : The intersection of two sets A and B is the


set of all those elements which belong to bothA and B.
Symbolically, we write A ∩ B = {x : x  A and x  B}
x ∈ A ∩ B ⇒ x ∈ A and x ∈ B

x  A ∩ B ⇒ x  A or x  B

** Disjoint sets : If A and B are two sets such that A ∩ B = φ, then


A and B are called disjoint sets.

** Some Properties of Operation of Intersection


(i) A ∩ B = B ∩ A (Commutative law).
(ii) ( A ∩ B ) ∩ C = A ∩ ( B ∩ C ) (Associative law).
(iii) φ ∩ A = φ, U ∩ A = A (Law of φ and U).
(iv) A ∩ A = A (Idempotent law)
(v) A ∩ ( B  C ) = ( A ∩ B )  ( A ∩ C ) (Distributive law ) i. e., ∩ distributes over 
** Difference of sets : The difference of the sets A and B in this order
is the set of elements which belong to A but not to B.
Symbolically, we write A – B and read as “ A minus B”.

10
A – B = { x : x  A and x  B }.

* The sets A – B, A ∩B and B – A are mutually disjoint sets,


i.e., the intersection of any of these two sets is the null set.

** Complement of a Set : Let U be the universal set and A a subset of U.


Then the complement of A is the set of all elements of U which
are not the elements of A. Symbolically, we write A′ to denote
the complement of A with respect to U.
Thus, A′ = {x : x  U and x  A }. Obviously A′ = U – A
** Some Properties of Complement Sets
1. Complement laws: (i) A  A′ = U (ii) A ∩ A′ = φ
2. De Morgan’s law: (i) (A  B)´ = A′ ∩ B′ (ii) (A ∩ B )′ = A′  B′
3. Law of double complementation: (A′ )′ = A
4. Laws of empty set and universal set φ′ = U and U′ = φ.

MCQS-
1.The number of elements in the Power set P(S) of the set S = {1, 2,3} is:

A. 4 B. 8 C. 2 D. None of these

Answer: B. 8

Explanation: Number of elements in the set S = 3

Number of elements in the power set of set S = {1,2,3} = 2³= 8

2. Empty set is a _______.

A. Infinite set B. Finite set C. Unknown set D. Universal set

Answer: B. Finite set

Explanation: The cardinality of the empty set is zero, since it has no elements. Hence, the size of the
empty set is zero.

11
3. Cardinality of the power set P(A) of a set A having ‘n’ number of elements is equal to:

A. n B. 2n C. 2n D. n²

Answer: C.

Explanation: The cardinality of the power set is equal to 2n, where n is the number of elements in a given
set.

4. Write X = {1, 4, 9, 16, 25,…} in set builder form.

A. X = {x: x is a prime number} B. X = {x: x is a whole number}

C. X = {x: x is a natural number} D. X = {x: x is a square number}

Answer: D. X = {x: x is a square number}

Explanation: Given,

X = {1, 4, 9, 16, 25,…}

X = {1², 2², 3², 4², 5², …}

Therefore,

X = {x: x is a set of square numbers}

Q5.If A={1,2,3,4,5} the number of proper subset is-

A.120 B.30 C.31 D.32

Answer- 31

Explanation-As every set is a subset of itself but not proper subset of itself.

Q6. The group of honest people in a city is

(a) Vois Set (b) Finite Set (c) Infinite Set (d) Not a set

Answer -d

Explanation -As honest people in a set is not a well defined collection , hence it is not a set

Q.7.The set of circles passing through Origin.

A. finite set B.infinite set C.Null set D.None of these

Answer-infinite set

Explanation-Through Origin many circles can pass.

12
Q.8 The set AUA’ IS

A. A B.A’ C.Ø D.U

Answer-U

Explanation-

Q.9 Set A and B has 3 and 6 elements. Find the minimum number of elements in AUB-

A.3 B.6 C.9 D.8

Answer-6

Explanation- If A is a subset of B then AUB=B

Q.10.If A and B are two disjoint sets then A ∩ B =

A. A B.A’ C.Ø D.U

Answer=Ø

Explanation-

Assertion Reason Type Questions


DIRECTION: In each of the following questions, a statement of Assertion is given followed by a
corresponding statement of Reason just below it. Of the statements, mark the correct
answer as
(a) Both assertion and reason are true, and reason is the correct explanation of assertion.
(b) Both assertion and reason are true, but reason is not the correct explanation of assertion.
(c) Assertion is true, but reason is false.
(d) Assertion is false, but reason is true

Q.11 Assertion: The union of two finite sets is always finite.

Reason: The union of two finite sets contains all the elements from both sets, which is a finite collection.

Answer: a) Both assertion and reason are true and reason is the correct explanation of assertion

13
Q.12.Assertion: The intersection of two finite sets is always finite.

Reason: The intersection of two finite sets contains only the common elements, which is also a finite
collection.

Answer:a)Both assertion and reason are true and reason is the correct explanation of assertion

Q.13.Assertion: The power set of a set with ‘n’ elements contains (2n) subsets.

Reason: Each element in the power set can either be included or excluded from the original set, resulting
in (2n) possible subsets.

Answer: a) Both assertion and reason are true, and reason is the correct explanation of assertion

Q.14.Assertion: If (A is a subset of B) and (B is a subset of A), then (A = B).

Reason: If every element of set (A) is in set (B) and vice versa, the two sets are equal.

Answer: a) Both assertion and reason are true and reason is the correct explanation of assertion

Q.15.Assertion: The empty set is a subset of every set.

Reason: By definition, the empty set has no elements, so it is a subset of any set.

Answer: a) Both assertion and reason are true and reason is the correct explanation of assertion

Short Answer Type


Q.1 Let X = {1, 2, 3, 4, 5, 6}. If n represent any member of X, express the following as sets:

(i) n + 5 = 8

(ii) n is greater than 4

Solution:

(i) Let B = {x | x ∈ X and x + 5 = 8}

Here, B = {3} as x = 3 ∈ X and 3 + 5 = 8 and there is no other element belonging to X such that x + 5 = 8.

(ii) Let C = {x | x ∈ X, x > 4}

Therefore, C = {5, 6}

2. Write the following sets in the roster form.

(i) A = {x | x is a positive integer less than 10 and 2x – 1 is an odd number}

(ii) C = {x : + 7x – 8 = 0, x ∈ R}

14
Solution:

(i) 2x – 1 is always an odd number for all positive integral values of x since 2x is an even number.

In particular, 2x – 1 is an odd number for x = 1, 2, … , 9.

Therefore, A = {1, 2, 3, 4, 5, 6, 7, 8, 9}

(ii) + 7x – 8 = 0

(x + 8) (x – 1) = 0

x = – 8 or x = 1

Therefore, C = {– 8, 1}

3. Let U = {1, 2, 3, 4, 5, 6}, A = {2, 3} and B = {3, 4, 5}.

Find A′, B′, A′ ∩ B′, A ∪ B and hence show that (A∪B )′ = A′∩ B′.

Solution:

Given,

U = {1, 2, 3, 4, 5, 6}, A = {2, 3} and B = {3, 4, 5}

A′ = {1, 4, 5, 6}

B′ = {1, 2, 6}.

Hence, A′ ∩ B′ = {1, 6}

Also, A ∪ B = {2, 3, 4, 5}

(A ∪ B)′ = {1, 6}

Therefore, (A∪ B)′ = {1, 6} = A′ ∩ B’

Q.4.Write the solution set of the equation + x – 2 = 0 in roster form.

Solution-

The given equation can be written as (x – 1) (x + 2) = 0, i. e., x = 1, – 2 Therefore, the solution set of the
given equation can be written in roster form as {1, – 2}.

Q.5. Write the set {x : x is a positive integer and < 40} in the roster form.

Solution-

The required numbers are 1, 2, 3, 4, 5, 6. So, the given set in the roster form is

15
{1, 2,3, 4, 5, 6}

Q.6.Write the set A = {1, 4, 9, 16, 25, . . . }in set-builder form.

Solution

We may write the set A as A = {x : x is the square of a natural number} Alternatively, we can write A = {x : x
= , where n ∈ N}.

Q.7. Let A = {1, 2, 3, 4, 5, 6, 7, 8, 9, 10} and B = { 2, 3, 5, 7 }. Find A ∩ B and hence show that A ∩ B = B.

Solution

We have A ∩ B = { 2, 3, 5, 7 } = B. We note that B ⊂ A and that A ∩ B = B

Q.8. Let A = { 2, 4, 6, 8} and B = { 6, 8, 10, 12}. Find A ∪ B.

Solution

We have A ∪ B = { 2, 4, 6, 8, 10, 12}.

Q.9.Let A = { a, e, i, o, u } and B = { a, i, u }. Show that A ∪ B = A

Solution

We have, A ∪ B = { a, e, i, o, u } = A.

Q.10. Let U = {1, 2, 3, 4, 5, 6, 7, 8, 9, 10} and A = {1, 3, 5, 7, 9}. Find A′.

Solution

We note that 2, 4, 6, 8, 10 are the only elements of U which do not belong to A. Hence A′ = { 2, 4, 6, 8,10 }.

Case Study Based Questions

Q.1 In a school at Bhubaneswar, students of class XI were forming some sets. Two Students Ankita and
Babita form two sets A = {1, 2, 3, 4, 5} and B = {2, 4, 6}.

Based on the above information answer the following:

● Find AUB

16
● Find A B

● Find A-B AND B-A.Are they equal?

Answer- I){1, 2,3,4,5,6} II){2,4} II)A-B={1,3,5},B-A={6}….NO

Q.2.A class teacher Mamta Sharma of class XI write three sets A, B and C are such that A = {1, 3, 5,7, 9}, B =
{2, 4, 6, 8} and C = {2, 3, 5, 7, 11}.

Answer the following questions which are based on above sets.

(i) Find A ∩ B.

(a) {3, 5, 7} (b) φ

(c) {1, 5, 7} (d) {2, 5, 7}

(ii) Find A ∩ C

(a) {3, 5, 7} (b) φ

(c) {1, 5, 7} (d) {3, 4, 7}

(iii) Which of the following is correct for two sets

A and B to be disjoint?

(a) A ∩ B = φ (b) A ∩ B ≠φ

(c) A∪B = φ (d) A∪B ≠φ

(iv) Which of the following is correct for two sets

A and C to be intersecting?

(a) A ∩ C = φ (b) A ∩ C ≠φ

(c) A∪C = φ (d) A∪C ≠φ

(v) Write the n[P (B)].

(a) 8 (b) 4 (c) 16 (d) 12

Answers (i) b (ii) a

(iii) a (iv) b (v) 16

Long Answer Type Questions

Q.1.Use the properties of sets to prove that for all the sets A and B, A – (A ∩ B) = A–B

17
Solution:

A – (A ∩ B) = A ∩ (A ∩ B)′ (since A – B = A ∩ B′)

= A ∩ (A′ ∪ B′) [by De Morgan’s law)

= (A∩A′) ∪ (A∩ B′) [by distributive law]

= φ ∪ (A ∩ B′)

= A ∩ B′ = A – B

Hence, proved that A – (A ∩ B) = A – B.

Q.2. Two finite sets have m and n elements. The total number of subsets of first set is 56 more than the
total number of subsets of the second set. Find the value of m and n.

Solution:

No. of elements in A=m

No. of elements in B=n

According the question

No. of subsets of set A – No. of subsets off set B = 56

= 56 because m

= 8x7

= x7

By comparing to both sides we get

n = 3, +1

m -n = 3

18
m = n+3

m=6 because n=3

Q.3. Consider the sets φ, A = { 1, 3 }, B = {1, 5, 9}, C = {1, 3, 5, 7, 9}. Insert the symbol ⊂ or ⊄ between each
of the following pair of sets: (i) φ . . . B (ii) A . . . B (iii) A . . . C (iv) B . . . C

Solution

(i) φ ⊂ B as φ is a subset of every set.

(ii) A ⊄ B as 3 ∈ A and 3 ∉ B

(iii) A ⊂ C as 1, 3 ∈ A also belongs to C

(iv) B ⊂ C as each element of B is also an element of C.

PRACTICE QUESTIONS

Q1 The number of subsets of a set containing n-elements is

(a) n (b) n2 (c) 2" (d)2" - 1

Q2 For any two sets A and B, A (AUB) =

(a) A (b) B (c)Ø (d)none of these

Q3 If A= {1,3,5,7} B={2,4},then

(a) 4 A (b) {4} A (c) B A (d) none of these

Q4 Let A= {x: x R, x>4} and B={x: x R, x<5}. Then A B =

(a) (4,5] (b) (4,5) (c) [4,5) (d) [4,5]

Q5 Let A and B be two sets such that n(A)=16,n(B)=14, n(AUB)=25.Then n(A B) =

(a) 30 (b) 50 (c) 5 (d) None of these

Q6 If A={1,2,3,4,5), then the number of proper subsets of A is

(a) 120 (b) 30 (c) 31 (d) 32

Q7 In set builder form empty set is represented by

19
(a) {} (b)Ø (c){x: x≠x} (d) {x:x=x}

Q.8.For two sets A B=A if

(a)B A (b )A B (c)A B (d)

Q9 In a city 20% of population travel by car,50% travel by bus and 10% travels by both Car and bus. Then
percentage of persons travelling neither by car nor bus is:-

(a) 80% (b) 40% (c) 60% (d) 70%

Q.10 Two finite sets have m and n elements. The number of subsets of the first set is 112 more than that of
second. Then the values of m and n are respectively.

(a) 4,7 (b)7,4 (c) 4,4 (d) 7,7

Answers

1.(c) 2. (a) 3. (d) 4. (b) 5. (c) 6. (c) 7. (c) 8. (a) 9. (b) 10. (b)

DIRECTION: In each of the following questions, a statement of Assertion is given followed by a


corresponding statement of Reason just below it. Of the statements, mark the correct
answer as
(a) Both assertion and reason are true, and reason is the correct explanation of assertion.
(b) Both assertion and reason are true, but reason is not the correct explanation of assertion.
(c) Assertion is true, but reason is false.
(d) Assertion is false, but reason is true
1. Assertion (A): Set of English alphabets is the universal set for the set of vowels in English alphabets

Reason (R) : The set of vowels is the subset of consonants in the English alphabets

2. Assertion (A): Let A={1,2,3} and B={1,2,3,4} then

Reason (R): A set A is said to be a subset of a set B if every element of A is also an element of B.
3. Assertion (A) : ‘The collection of all natural numbers less than 100’ is a set.

Reason (R) : A set is a well-defined collection of distinct objects.

4. Assertion (A) : The set D = {x : x is a prime number which is a divisor of 60}

Reason(R ) : in roster form is{1, 2, 3, 4, 5}.


20
5. Assertion (A) : The set E = the set of all letters in the word ‘TRIGONOMETRY’, in the roster form is {T, R,
I, G, O, N, M, E, Y}.

Reason (R ) In roaster form distinct elements is written ,separated by comma

Answers

1. c 2. a 3. a 4. c 5. a

Short Answer Type Questions

1. Write the set A = {x: x∈ x, x2 < 20} in roster form.

2. Which of the following sets are empty sets?

A = {x: x2 − 3 = 0 And x is rational}

B = {x ∈ R: 0 < x< 1}

3. Write down all possible subsets of each of the following sets:

(i) {1, {1}} (ii) {1,2,3}

4. Write the following as intervals:

{x: x ∈ R, −12 < x < −10}

{x: x ∈ R, 3 ≤ x ≤ 4}

5.What Universal Set would you propose for each of the following:

(i)the set of isosceles triangles? (ii) the set of right-angle triangles.

6. Let 𝑈 = {1,2,3,4,5,6,7,8,9}, 𝐴 = {2,4,6,8} 𝑎𝑛𝑑 𝐵 = {2,3,5,7}. 𝑉𝑒𝑟𝑖𝑓𝑦 𝑡ℎ𝑎𝑡,

(𝑖)(𝐴 ∪ 𝐵)′ = 𝐴′ ∩ 𝐵′ (𝑖𝑖)(𝐴 ∩ 𝐵)′ = 𝐴′ ∪ 𝐵′.

7. Which of the following sets are finite and which are infinite:

A = {𝑥: 𝑥𝜖 𝑍 𝑎𝑛𝑑 − 5𝑥 + 6 = 0}

𝐵 = {𝑥: 𝑥 𝜖 𝑍 𝑎𝑛𝑑 𝑖𝑠 𝑒𝑣𝑒𝑛}

C = {𝑥 ∶ 𝑥 𝜖 𝑍 𝑎𝑛𝑑 𝑥 > −10}


21
8. Let A and B be two sets. Prove that (𝐴 − 𝐵) ∪ 𝐵 = 𝐴 𝑖𝑓 𝑎𝑛𝑑 𝑜𝑛𝑙𝑦 𝑖𝑓 𝐵 ⊂ A.

9. Let 𝑈 = {1,2,3,4,5,6,7,8,9}

𝐴 = {1,2,3,4}, B = {2,4,6,8}, C = {3,4,5,6}.

Find (i) (𝐴 ∩ 𝐶)′(𝑖𝑖)(𝐴′)′(𝑖𝑖𝑖)(𝐵 − 𝐶)′

10. Which of the following pairs of sets are equal? Justify your answer

(i)𝐴 = {𝑥: 𝑥 𝑖𝑠 𝑎 𝑙𝑒𝑡𝑡𝑒𝑟 𝑜𝑓 𝑡ℎ𝑒 𝑤𝑜𝑟𝑑 "𝐿𝑂𝑌𝐴𝐿"}

B = {𝑥: 𝑥 𝑖𝑠 𝑎 𝑙𝑒𝑡𝑡𝑒𝑟 𝑜𝑓 𝑡ℎ𝑒 𝑤𝑜𝑟𝑑 "𝐴𝐿𝐿𝑂𝑌"}.

(II) A = {𝑥: 𝑥 𝜖 𝑍 𝑎𝑛𝑑 ≤8}

B = {𝑥: 𝑥 𝜖 𝑅, 𝑎𝑛𝑑 − 4𝑥 + 3 = 0}

Answers

1. A = {−4, −3, −2,0,1,2,3,4}

2. 2. A= Empty Set, B= Non – Empty Set

3. (i) ∅, {1}, {{1}}, {1, {1}}

(ii) ∅, {1}, {2}, {3}, {1,2}, {1,3}, {2,3}, {1,2,3}

4. (−12, −10) [3 , 4]

5. The set of all triangles in plane.

The set of all triangles in plane.

6. verification by given sets

7. 𝐴 = {2, 3}

So, A is finite set.

B = {… , −6, −4, −2,0,2,4,6, … }


22
So, B is infinite set.

C= {-9 , -8 ,-7 ,…..}

So C is infinite Set

8. Verification/ Prove Given Set

9.(i) (𝐴 ∩ 𝐶)′ = {1,2,5,6,7,8,9}

(ii) (𝐴′)′ = {1,2,3,4}

(iii) (𝐵 − 𝐶)′ = {1,3,4,5,6,7,9}

10. (i) A

(II) A

Long Answer Type


1.Three friends were having get together. Suddenly they decided to play with their names using sets. Name
of friends were AARTI, CHARVI and AYSHA. They asked each other the following questions.

(i)How letters used for AARTI are written in roster form as a set?

(a) {A, R, T, I} (b) {x: x is a letter of the word AARTI} (c) {A,T,I} (d) none of these

(ii) What is the difference of set of letters of CHARVI and AYSHA?

(a) {C, R, V, I} (b) {C,S,V,I} (c) {C,T,V,I} (d) {C,V,I}

(III)Form a union of sets taking the letters of names of friends.

(𝑎){A,R,T,I,C,H,Y,V,S} (b) {A,R,T,I,C,H,V,} (c) {A,R,C,H,V,Y,S} (d) none of these

Form a set of intersection of sets taking the letters of names of friends.

(a) {A} (b) {A,R,T,I,C,H,V} (c) {A,R,C,H,V,Y,S} (d) none of these

2.For all sets A, B and C. Is (A- B) Justify your answer.


3. A,B and C are subset of universal set U. If A ={2,4,6,8,12,20} , B={3,6,9,12,15},

23
C={ 5,10,15,20} and U is the set of all whole numbers. Draw a Venn diagram showing the relation of U, A,B
and C.

Answers
1.(i) a (ii) a (iii) a (iv) a

2. By Verification

3.

24
RELATIONS AND FUNCTIONS

CARTESIAN PRODUCT OF SETS


● Given two non-empty sets A and B. The cartesian product A x B is the set of all ordered pairs of elements
from A and B .
A, B and C are three set
● A x B = { (a, b): a ∈ A and b ∈ B}, where A and B are non-empty sets A x (B U C) = (A x B) U (A x C)
A x (B ∩ C) = (A x B)∩ (A x C)
● if A =∅ or B =∅, then cartesian product A x B= ∅ A x (B - C) = (A x B) - (A x C)
AxB=BxA⇒A=B
● example if A = { 2, 5, 7} and B = { 3, 9} then A x B = A x C ⇒ B =C
A x B = { (2,3), (2,9), (5,3), (5,9), (7,3), (7,9) }
B x A = { (3,2), (3,5), (3,7), (9,3), (9,5), (9,7) }
● if A and B are two sets then
no. of elements in cartesian product = n(A) x n(B)
● If either A or B is infinite set, then A x B is an infinite set

ORDERED PAIR
● pair of elements in fixed order
● elements can be same or distinct
● example points (3,4), (9,0) representing distinct points in a 2D plane
● in set, if A and B are two sets and a ∈ A and b ∈ B, ordered pair is (a, b)

● if (a, b) = (c, d) a=c and b=d


● A xA xA ={(a,b,c):a,b,c ∈A }Here (a,b,c) is called an ordered triplet.

RELATION
● Let A and B are two non empty sets. Then relation R ⊆ A x B
The subset is derived by describing a relationship between the first element and the second
element of the ordered pairs in A × B. The second element is called the image of the first element.
Domain (R) = {x : (x,y) ∈ R}, Range (R) = {y : (x,y) ∈ R} and codomain = B

● Let A and B are two non empty finite sets consisting of m and n elements respectively.
No. of relation defined = 2mn

FUNCTION
● Function ‘f’ is subset of relation of two non empty sets A and B such that every element in A has one and
only one image in B.
(a, b) ∈ f ⇒ f(a) =b here b is image of a and a is pre image of b
● Domain of f = set A and co-domain of f = set B
● Range of f = set of images of elements in the domain
25
● A function which has either R or one of its subsets as its range is called a real valued function.
Further, if its domain is also either R or a subset of R, it is called a real function.
● Algebra of function : if f: X→ R and g: X→ R are two real functions and k 𝜖 R, then
o f ± g : X→ R is defined as (f ± g)(x) = f(x) ± g(x)
o fg :X→ R is defined as (fg)(x) = f(x)g(x)
𝑓 𝑓 𝑓(𝑥)
o : X – {x : g(x) = 0} → R is defined as (𝑔) (𝑥) = 𝑔(𝑥)
𝑔

o kf :X→ R is defined as (kf)(x) = kf(x)

ILLUSTRATIVE EXAMPLES
MULTIPLE CHOICE QUESTIONS
Q.1 Let A={1,2,3,4,5} and R be a relation from A to A, R = {(x, y): y = x + 1}. Find the domain.
(a) {1,2,3,4,5} (b) {2,3,4,5}
(c) {1,2,3,4} {d) {1,2,3,4,5,6}
Solution:- (c) Make relation from A to A
Put x=1,2,3,4 we get
y=2,3,4,5
∴R={(1,2),(2,3),(3,4),(4,5)}
Domain {1,2,3,4}, Co-domain {1,2,3,4,5} and Range {2,3,4,5}

Q.2 If set A has 2 elements and set B has 4 elements then how many relations are possible?
(a) 32 (b) 128 (c) 256 (d) 64
Solution:- (c) n(A)=4,n(B)=2
Number of relations from A to B=2n(A)n(B) = 28 = 256

Q.3 In a function from set A to set B, every element of set A has___________ image in set B.
(a) one and only one (b) different
(c) same (d) many
Solution:- (b)

|𝑥|
Q.4 . f(x) = { 𝑥 for x≠0 and 0 for x=0}. Which function is this?
(a) Constant (b) Modulus
(c) Identity (d) Signum function
Solution:- (d)

Q.5 If f(x) = x3 – (1/x3), then f(x) + f(1/x) is equal to

26
(a) 2x3 (b) 2/x3 (c) 0 (d) 1

1 1 1
Solution:- (c) 𝑓 (𝑥) = 𝑥 3 − 1
𝑥3
1 1
𝑓 ( ) = 3 − 𝑥3
𝑥 𝑥
1
𝑓(𝑥) + 𝑓 ( ) = 0
𝑥

Q.6 f(x) =√9 − 𝑥 2 . Find the range of the function.


(a) R (b) R+
(c) [-3,3] (d) [0,3]

Solution:- (d) Given that f(x)= 9 − x


2

The domain of the given function is x∈[−3,3]


⇒−3 ≤ x ≤ 3
⇒0 ≤ x 2≤ 9
⇒−9 ≤ −x2 ≤0
⇒0≤ 9−x2 ≤9
⇒0≤ 9 − x ≤3
2

⇒0≤ f(x) ≤3
The range of the function is [0,3].

Q.7 Let A = {-2, -1, 0} and f(x) = 2x – 3 then the range of f is


(a) {7, -5, -3} (b) {-7, 5, -3} (c) {-7, -5, 3} (d) {-7, -5, -3}
Solution:- (d) f(-2) = 2(-2) -3 = -7
f (-1) = 2(-1) -3 = -5
f( 0) = 2(0) -3 = -3
Range = {-7, -5, -3}

Q.8 A function is defined by f(t) = 2t – 5, then the value of f(-3) is


(a) - 11 (b) 11 (c) 1 (d) -1
(b) Solution:- ( a) f(-3) = 2(-3) -5
= -6 -5 = -11

Q.9 Let f = {(1, 1), (2, 3), (0, -1), (-1, -3)} be a function from Z to Z defined by f(x) = mx + c. Determine c.
(a) 1 (b) 0 (c) - 1 (d) – 3
Solution:- (c ) f(1)=1 gives us m⋅1+c= m + c =1.
f(2)=3 gives us m⋅2+c = 2m + c =3
f(0)= −1 gives us m⋅0+c=−1
⇒c = -1
Q.10 If P X Q is an empty set then which of the following is a null set?
(a) only P (b) only Q
(c) either P or Q (d) both P and Q
Solution :- (c)
27
ASSERTION AND REASONING

In the following questions a statement of assertion (A) is followed by a statement of


Reason (R). Choose the correct answer out of the following choices.

(a) Both A and R are true and R is the correct explanation of A.


(b) Both A and R are true but R is not the correct explanation of A.
(c) A is true but R is false.
(d) A is false but R is true.
1 Assertion (A): If (x+1, y-2) = (3,1), then x = 2 and y =3.
Reason (R): Two ordered pairs are equal, if their corresponding elements are equal.
Solution:- x +1 = 3⇒x=3-1⇒x =2
And y-2 = 1⇒y=1+2⇒y= 3
Reason is the property of ordered pair i.e. (a, b) = (c, d) ⇔ a=c and b=d
Both A and R are true and R is the correct explanation of A.

2 Assertion (A):The cartesian product of two non-empty sets P and Q is denoted as P x Q and P x Q = {
(p, q): p 𝜖 P, q 𝜖 Q}
Reason (R): If A = {red, blue} and B = {b, c, s}, then A x B = {(red, b), (red, c), (red, s), (blue, b), (blue ,c),
(blue, s)}.
Solution:- Assertion (A) states that the cartesian product of two non-empty sets P and Q, denoted P x Q ,
consists of all ordered pairs ( (p, q) where p is an element of P and q is an element of Q . This definition is
accurate and corresponds to the formal definition of cartesian product.
Reason (R) provides a valid example that matches the definition of the cartesian product. Both A and R
are true and R is the correct explanation of A.

3 Assertion (A): Let A = {1, 2} and B= {3, 4}. Then, number of relations from A to B is 16.
Reason (R): If n(A) = p and n(B) = q, then number of relations in 2pq.
Solution:- Assertion (A) is incorrect. To find the number of relations from set A to set B, we consider all
possible subsets of A×B, where A x B denotes the cartesian product of A and B. Given A={1,2} and B={3,4},
the cartesian product A×B consists of 4 ordered pairs: {(1,3),(1,4),(2,3),(2,4)}. Reason (R) provides a general
formula
Therefore, while Assertion (A) itself is incorrect in its specific example, Reason (R) provides the correct

4 Assertion (A): f(x) = x2, f: N to N is a function.


Reason (R): All relations are functions.

Solution:- Assertion(A) is correct as each value of x has unique image in f(x)

Reason (R) is false because not all relations are functions; functions are a specific subset of relations
where each input is associated with exactly one output.

5) Assertion(A):The domain of the relation R = {(x + 2, x + 4) : x ∈ N, x < 8} is {3, 4, 5, 6, 7, 8,9).

28
Reason (R): The range of the relation R = {(x + 2, x + 4) : x ∈ N, x < 8} is {1, 2, 3, 4, 5, 6, 7}.
Solution:- on putting the values of x from 1 to 7 in relation we find the domain {3, 4, 5, 6, 7, 8,9) and
range ={5, 6, 7, 8, 9, 10, 11}.So, A is true but R is false.

SHORT ANSWER TYPE QUESTIONS


Q.1 Find x and y, if (x+3, 5) = (6, x-y).
Solution:- x+3 = 6 ⇒x = 3
And x – y = 5
⇒x – 5 = y
⇒y= 3 – 5 ⇒y= -2
Q.2 If A = {4, 9, 6} and B = {5, 4, 7, 6}, find A x B and B x A.
Solution:- A×B: This is the set of all ordered pairs (a, b) where a ∈ A and b ∈ B
A×B={(4,5),(4,4),(4,7),(4,6),(9,5),(9,4),(9,7),(9,6),(6,5),(6,4),(6,7),(6,6)}
And B x A : This is the set of all ordered pairs (b, a) where a ∈ A and b ∈ B
B×A={(5,4),(5,9),(5,6),(4,4),(4,9),(4,6),(7,4),(7,9),(7,6),(6,4),(6,9),(6,6)}.

Q.3 If P ={ 3,5}, then find P x P x P.


Solution:- {((3,3,3),((3,3,5),((3,5,3),((3,5,5),((5,3,3),((5,3,5),((5,5,3),((5,5,5)}.

Q.4 Given R = {(x, y) : x, y ∈ N, x2 + y2 = 25}. Find the domain and range of R


Solution:- We have 32 + 42 = 25 or 42+ 32 = 25
So the domain of R is {3,4}. [ Values corresponding to x for x being natural number]
And the range is {3,4}. [ Values corresponding to y for y being natural number]

Q.5 Redefine the function: f(x) = |x – 1| – |x + 6|


Solution:- Given function is f(x) = |x – 1| – |x + 6|
Redefine of the function is:

The domain of this function is R.

Q.6 The function ‘t’ which maps temperature in degree Celsius in to temperature in degree
Fahrenheit is defined by t(C) = 9C+ 32. Find t(0), t (- 10) and the value of C, when t (C) = 212.

Solution:- For t(0): Substitute C=0 into the function:


t(0)=9⋅0+32
29
So, t(0)=32
For t(-10) : Substitute C=-10 into the function:
t(-10)=9(-10)+32
So, t(-10)=-58
For t(C)=212
212= 9C +32
9C = 180
C = 20

Q.7 Determine the domain and range of the relation R = { (a, b): a 𝜖 N, a ≤ 5, b=4}
Solution:- We have,
R = {(a, b): a ϵ N, a ≤ 5,b=4}
⇒a=1,2,3,4,5 and b = 4
Thus, R = {(1, 4), (2, 4), (3, 4), (4, 4), (5,4)}
Clearly, Domain (R) = {1, 2, 3, 4, 5} and Range (R) = {4}

Q.8 Let A = {1, 2, 3, 4, 5}; B = {2, 3, 6, 7}. Then what is the number of elements in (A × B) ∩ (B × A)?
Solution:- Here, A and B sets have 2 elements in common, so A × B and B × A have 2 2, i.e., 4 elements in
common. Hence, n [(A × B) ∩ (B × A)] = 4.

Q.9 The Cartesian product A × A has 9 elements among which are found (–1, 0) and (0,1). Find the set A
and the remaining elements of A × A
Solution:- We know that,
If n(A) = p and n(B) = q, then n(A × B) = pq
From the given,
n(A × A) = 9
n(A) × n(A) = 9,
n(A) = 3 ……(i)
The ordered pairs (-1, 0) and (0, 1) are two of the nine elements of A × A.
Therefore, A × A = {(a, a) : a ∈ A}
Hence, -1, 0, 1 are the elements of A. …..(ii)
From (i) and (ii),
A = {-1, 0, 1}
The remaining elements of set A × A are (-1, -1), (-1, 1), (0, -1), (0, 0), (1, -1), (1, 0) and (1, 1).
Q.10 Find the domain of function of f(x) = 16 − x .
2

Solution:- Consider the given function.

f(x)= 16 − x
2

For domain under root should not be negative quantity,


16−x2 ≥0
16 ≥ x2
Therefore, x ≤ 4 or x ≥ −4
Then, the domain[−4,4]
LONG ANSWER TYPE QUESTIONS

30
Q.1 The function f is defined by

Draw the graph of f(x).


Solution:f(x) = 1 – x, x < 0, this gives
f(– 4) = 1 – (– 4)= 5;
f(– 3) =1 – (– 3) = 4,
f(– 2) = 1 – (– 2)= 3
f(–1) = 1 – (–1) = 2; etc,
Also, f(1) = 2, f (2) = 3, f (3) = 4, f(4) = 5 and so on for f(x) = x + 1, x > 0.
Thus, the graph of f is as shown in the below figure.

Q.2 Draw the graph of the function f(x) = |x-2|+|x-3|

Solution:- f(x)=|x−2|+|x−3|

When x<2, f(x)=2−x+3−x =5−2x

When 2≤x<3, f(x)=x−2+3−x = 1

When x≥3, f(x)=x−2+x−3 = 2x−5

31
Q.3 Find the domain and range of the function f(x) = 1/ x −  x 

x −  x
Solution:-We have, f(x)=1/
for domain of f: we know that,
0 ≤ x−[x] ≤ 1,∀ x ϵ R
Also, x−[x]=0 ∀ x ϵ Z
∴0 < x−[x] < 1,∀ x ϵ R−Z
x −  x
⇒f(x)=1/ ∀ x ϵ R−Z
∴Domain of f(x)=R−Z
For range of f:we have 0 < x−[x] < 1,∀ x ϵ R−Z
x −  x
⇒0< <1, ∀ x ϵ R−Z
x −  x
⇒1< 1/ <∞,∀ x ϵ R−Z
⇒1<f(x)<∞, ∀ x ϵ R−Z
∴Range of f(x)=(1,∞)

Q.4 Determine the domain and range of the relation R = {(x, y) : x 𝜖 N, y 𝜖 N and x + y = 10}.
Solution:- Given set N = {1, 2, 3,…} A relation R in N is defined as R = {(x, y) : x, y ∈ N, x + y = 10} ∀ x, y ∈ N
So, xRy ⇔ x + y = 10 ⇔ y = 10 – x when x = 1, then y = 10 – 1 = 9 ∈ N then (1, 9) ∈ R
when x = 2, then y = 10 – 2 = 8 ∈ N then (2, 8) ∈ N
when x = 3, then y = 10 – 3 = 7 ∈ N then (3, 7) ∈ R
when x = 4, then y = 10 – 4 = 6 ∈ N then (4, 6) ∈ R
Similarly, (5, 5) ∈ R, (6, 4) ∈ R, (7, 3) ∈ R, (8, 2) ∈ R, (9, 1) ∈ R
R = {(1, 9), (2, 8), (3, 7), (4, 6), (5, 5), (6, 4), (7, 3), (8, 2), (9, 1)}
Domain of R ={1, 2, 3, 4, 5, 6, 7, 8, 9}
Range of R = {9, 8, 7, 6, 5, 4, 3, 2, 1}

Q.5 Assume that A = {1, 2, 3,…,14}. Define a relation R from A to A by R = {(x, y) : 3x – y = 0, such that x, y ∈
A}. Determine and write down its range, domain, and co domain.
Solution:- It is given that the relation R from A to A is given by R = {(x, y): 3x – y = 0, where x, y ∈ A}.It
means that R = {(x, y) : 3x = y, where x, y ∈ A}
Hence, R = {(1, 3), (2, 6), (3, 9), (4, 12)}
We know that the domain of R is defined as the set of all first elements of the ordered pairs in the given
relation.
Hence, the domain of R = {1, 2, 3, 4}
To determine the co domain, we know that the entire set A is the co domain of the relation R.
Therefore, the co domain of R = A = {1, 2, 3,…,14}
As it is known that, the range of R is defined as the set of all second elements in the relation ordered pair.
Hence, the Range of R is given by = {3, 6, 9, 12}

32
CASE STUDY BASED QUESTIONS

Q.1 Maths teacher started the lesson Relations and Functions in Class XI. He explained the following
topics:
Ordered Pairs: The ordered pair of two elements a and b is denoted by (a, b) : a is first element (or first
component) and b is second element (or second component).
Two ordered pairs are equal if their corresponding elements are equal. i.e., (a, b) = (c, d) ⇒ a = c and b = d
Cartesian Product of Two Sets: For two non-empty sets A and B, the cartesian product A x B is the set of all
ordered pairs of elements from sets A and B.

In symbolic form, it can be written as A x B= {(a, b) : a ∈ A, b ∈ B.


Based on the above topics, answer the following questions.
(i) If (a – 3, b + 7) = (3, 7), then find the value of a and b
(ii) If (x + 6, y – 2) = (0, 6), then find the value of x and y
(iii) If (x + 2, 4) = (5, 2x + y), then find the value of x and y
(iv) Find x and y, if (x + 3, 5) = (6, 2x + y).
Solution:-
(i) We know that, two ordered pairs are equal, if their corresponding elements are equal.
(a – 3, b + 7) = (3, 7)
⇒ a – 3 = 3 and b + 7 = 7 [equating corresponding elements]
⇒ a = 3 + 3 and b = 7 – 7 ⇒ a= 6 and b = 0
(ii) (x + 6, y – 2) = (0, 6)
⇒ x + 6 = 0 ⇒ x = -6 and y – 2 = 6 ⇒ y = 6 + 2 = 8
(iii) (x + 2, 4) = (5, 2x + y)
⇒ x + 2 =5 ⇒ x = 5 – 2 = 3 and 4 = 2x + y ⇒ 4 = 2 x 3 + y ⇒ y = 4 – 6 = -2
(iv) x + 3 = 6, 2x + y = 5 ⇒ x = 3, y = 1
Q.2 Hanuman Pareek and Pawan Saini are two students of class XIth in a school. The ages of students is
x + 4 and g(x) = 16 − x . Then answer the
2

represented by f and g be real functions defined by f(x) =


some question based on their ages.
(i). Find the domain of f(x) and g(x).
(ii) Find the domain of sum of both ages.
𝑓
(iii).Find the domain of (𝑔).
Solution:- (i)for domain of f(x)
x+4 ≥ 0
⇒ X+4 ≥ 0 ⇒ x 𝜖 [-4,ꝏ)
for domain of g(x)
16 − x ≥ 0⇒16 -x2 ≥0⇒x2 ≤16⇒ x 𝜖 [-4,4]
2

(ii) domain of f(x) ꓵ domain of g(x) = [-4,4]


𝑓
(iii) domain of (𝑔) = [domain of f(x) ꓵ domain of g(x)] – points where g(x) is not defined
= (-4,4)

33
Q.3 A general election of Lok Sabha is a gigantic exercise. About 969 million people were eligible to
vote and voter turnout was about 66%, the highest ever Let I be the set of all citizens of India who were
eligible to exercise their voting right in general election held in 2024.
A relation ‘R’ is defined on I as follows: R = {(V1, V2) ∶V1, V2 ∈I and both use their voting right in general
election – 2024}
(i). Two neighbours X and Y∈ I. X exercised his voting right while Y did not cast her vote in general election –
2024. Which of the following is true?
a. (X,Y) ∈R b. (Y,X) ∈R c. (X,X) ∉R d. (X,Y) ∉R
(ii). Mr.X and his wife ‘W’both exercised their voting right in general election -2024, Which of the following
is true?
a. both (X,W) and (W,X) ∈ R b. (X,W) ∈ R but (W,X) ∉ R
c. both (X,W) and (W,X) ∉ R d. (W,X) ∈ R but (X,W) ∉
(iii). Three friends x, y and z exercised their voting right in general election-2024, then which of the
following is true?
a. (x, x ) ∈R, (y, z) ∈ R and (x, z) ∈ R b. (x, y ) ∈ R, (y, z) ∈ R and (x, z) ∉ R
c. (x, y ) ∈ R, (y, y) ∈R but (z, z) ∉ R d. (x, y ) ∉ R, (y, z) ∉ R and (x, z) ∉ R
(iv). Mr. Shyam exercised his voting right in General Election – 2024, then Mr. Shyam is related to which of
the following?

a. All those eligible voters who cast their votes


b. Family members of Mr.Shyam
c. All citizens of India
d. Eligible voters of India
Solution :- (i)(d) (ii) (a) (iii) (a) (iv) (a)

Q.4 To make herself self dependent and to earn her living, a college student decided to setup a small scale
business of manufacturing hand sanitizers.
She estimated a fixed cost of Rs. 15000 per month and a cost of Rs. 30 per unit to manufacture.
Based on the above information, answer the following questions

(i)Let x units of hand sanitizers are manufactured per month. What is the function of cost ?
(ii)If each unit is sold for Rs. 45, then what is the selling (revenue) function?
(iii)How much units should be manufactured and sold, for break-even (that is, no profit, no loss situation) in
a month?
(iv)What is the monthly cost borne by the student, if the student decided to manufacture 1500 units in a
month?

Solution:- (i) C(x) = 15000 + 30x


(ii) R(x) =45x
(iii) For break-even, C(x) = R(x)
15000+30x = 45 ⇒ 15x = 15000
⇒ X= 1000
(iv) C(1500) = 15000+ 30(1500) =60000

Q.5 Let A = {1, 2, 3, 4, 6}. Let R be the relation on A defined by

34
{(a, b): a , b ∈A, b is exactly divisible by a}.
(i) Write R in roster form
(ii) Find the domain of R
(iii) Find the range of R.
Solution:- A = {1, 2, 3, 4, 6} then R= {(1,1), (1, 2), (1,3), (1,4), (1,6), (2, 4), (2, 6), (2, 2), (3, 3), (4, 4), (3,6),
(6,6)}
Domain = {1, 2, 3, 4, 6}
Range = {1, 2, 3, 4, 6}

PRACTICE QUESTIONS
MULTIPLE CHOICE QUESTIONS
Q.1 Express the function f: A→R. f(x) = x2 – 1, where A = { -4, 0, 1, 4} as a set of ordered pairs.
(a) {(-4, 15), (0, -1), (1, 0), (4, 15)}
(b) {(-4, -15), (0, -1), (1, 0), (4, 15)}
(c) {(4, 15), (0, -1), (1, 0), (4, 15)}
(d) {(-4, 15), (0, -1), (1, 0)}

Q.2 The number of relations from A={1,2,3} to B={4,6,8,10} is(a) 43 (b) 27 (c) 212 (d) 34
Q.3 If P X Q has 10 elements then which is not possible?
(a) n(P)=1 and n(Q)=10 (b) n(P)=10 and n(Q)=1 (c) n(P)=2 and n(Q)=5 (d) n(P)=5 and n(Q)=4
Q.4 Let n (A) = m, and n (B) = n. Then the total number of non-empty relations that can be defined from
A to B is
(a) mn (b) nm – 1 (c) mn – 1 (d) 2mn – 1
Q.5 If f(x) = ax + b, where a and b are integers, f(–1) = – 5 and f(3) = 3, then a and b are equal to (a) a = –
3, b = –1 (b) a = 2, b = – 3 (c) a = 0, b = 2 (d) a = 2, b = 3
Q.6 Let R be a relation defined by a relation on N by x + 2y = 8 , the domain of R is
(a){2, 4, 8 } (b) { 2 ,4 ,6 ,8 } (c) { 2, 4 ,6 } (d) { 1 , 2 ,3 ,4 }

Q.7 The domain of the function f(x) = 1/(x² – 3x + 2) is


(a) {1, 2} (b) R (c) R – {1, 2} (d) R – {1, -2}

Q.8 If A = {a, b}, B = {c, d}, C = {d, e} then {(a, c), (a, d), (a, e), (b, c), (b, d), (b, e)} is equal to
(a) A∪(B∩C) (b) A∩(B∪C) (c) A×(B∩C) (d) A×(B∪C)

Q.9 Let R be the relation in the set N given by R = {(a, b): a = b – 2, b > 6}. Choose the correct answer.
(a) (2, 4) ∈ R (b) (3, 8) ∈ R (c) (6, 8) ∈ R (d) (8, 7) ∈ R

Q.10 If f(x) = - |x|. Choose the correct option from the following:
35
(a)Domain is set of negative real numbers (b) Range is set of real numbers
(c)Range is set of all negative integers (d) Range is (-∞, 0]

ANSWER
Q.1 Q.2 Q.3 Q.4 Q.5 Q.6 Q.7 Q.8 Q.9 Q.10
A c d d b c c d c d

ASSERTION AND REASONING

In the following questions a statement of assertion (A) is followed by a statement of


Reason (R). Choose the correct answer out of the following choices.

(a) Both A and R are true and R is the correct explanation of A.


(b) Both A and R are true but R is not the correct explanation of A.
(c) A is true but R is false.
(d) A is false but R is true.
1 Assertion (A):If (x,1), (y,2) and (z, 1) are in A x B and n(A)=3, n(B)=2, then A= {x, y, z} and B = {1, 2}.
Reason (R):If n(A) = 3 and n(B) = 2, then n(A x B) = 6
2 Assertion (A): Let A = {1, 2, 3, 4, 6}. If R is the relation on A defined by {(a, b): a, b 𝜖 A, b is exactly
divisible by a}.The relation R in Roaster form is {(6, 3), (6, 2), (4, 2)}.
Reason (R):The domain and range of R is {1, 2, 3, 4, 6}.
3 Assertion (A): R= {(1, 2), (3, 4), (4, 5), (5, 6), (6, 6)} is a function. Reason (R):
The domain of a real valued function is a subset of real numbers.
4 Assertion (A): Let A={a,b} and B={1,2,3}. The number of relations from A to B is 64.
Reason (R):If n(A)=p and n(B)=q, then the number of relation from A to B is qp.
5 Assertion (A): If P = {1, 2}, then P × P × P = {(1, 1, 1), (2, 2, 2), (1, 2, 2), (2, 1, 1)}
Reason (R): A × A × A = {(a, b, c) : a, b, c ∈ A}. Here (a, b, c) is called an ordered triplet.

ANSWER
Q.1 Q.2 Q.3 Q.4 Q.5
a d b c d

SHORT ANSWER TYPE QUESTIONS


Q.1 If A = {-1, 2, 3 } and B = {1, 3}, then determine
(i) A x B (ii) B x A (c) B x B (iv) A x A

Q.2 If A = {x : 𝑥 2 − 5𝑥 + 6 = 0}; B = {2,4}, and C = {4,5}, find the value of A x (B ꓵ C).

Q.3 If A x B = { (a, x), (a, y), (b, x), (b, y) }, then find A and B.

36
Q.4 Let the function f(x) = x2 for all x ∈ X, where X = {-2, -1, 0, 1, 2, 3} define f: X → Y. Express the relation
f in roaster form. Mention if f is a function.

Q.5 Draw the graph of function f(x) = 4 − x 2 . Also write the domain and range of f(x).

Q.6 If A= { 1, 2, 7}; B= {3, 6, 4} and C= {5, 4}. Prove A x (B ꓵ C) = (A x B) ꓵ (A x C).


4−𝑥
Q.7 Find the domain and range of the real valued function f(x) given by f(x) = 𝑥−4.

Q.8 Determine the domain and range of the relation R, where R = { (x, 𝑥 3 ): x is a prime no. less than 10}.
Q.9 Draw the graph of the function f(x) = 5 - |x-2|
Q.10 Calculate the domain and range of f(x) = |2x-3|-3.

ANSWERS
Q. A × B = {(–1, 1), (–1, 3), (2, 1), (2, 3), (3, 1), (3, 3)}
1 B × A = {(1, –1), (1, 2), (1, 3), (3, –1), (3, 2), (3, 3)}
B × B = {(1, 1), (1, 3), (3, 1), (3, 3)}
A × A = {(–1, –1), (–1, 2), (–1, 3), (2, –1), (2, 2), (2, 3), (3, –1), (3, 2), (3,
3)}
Q. {(2,4),(3,4)}
2
Q. A= {a, b}
3 B= {x, y}
Q. {(-2,4), (2,4), (-2,4), (-2,4), (-2,4), (-2,4), (-2,4), (-2,4),
4
Q. Domain= [-2, 2] and range [0, ꝏ)
5
Q. Domain= R- {4} and range = {-1}
7
Q. Domain = {2, 3, 5, 7} and range = {8, 27, 125, 343}
8
Q. Domain = R and Range = [-3,ꝏ)
10

LONG ANSWER TYPE QUESTION


1
Q.1 Find the domain of the function f given by f(x) = .
√[𝑥]2 −[𝑥]−6

Q.2 If a relation R is defined on the set Z of integers as follows {𝑎, 𝑏} ∈ 𝑅 𝑎2 + 𝑏 2 ≤ 25 find domain
range and co-domain
Q.3 Find the domain and range of the function f(x) = |2x + 1|- |x-3|.
Q.4 Let f: R → R be given by f(x) = x2 +3. Find {x : f(x) =28} and find the pre-images of 39 and 2 under f.
Q.5 Let A = {a, b, c, d}. Examine which of the following relations is a function on A and give reason.
37
(i) f= {(a, a), (b, c), (c, d), (d, c)}
(ii) g = {(a, c), (b, d), (b, c)}
(iii) h = {(b, c), (d, a), (a, a)}

ANSWERS

Q. x 𝜖 (-ꝏ,-2) [4,ꝏ)
1

Q. Domain= {0,±3,±4,±5}
2

Q. Domain: R
3 Range: (−∞,−2]∪[7,∞).

Q. {x : f(x)=28}={5,−5}
4 Pre-images of 39 under f are {6,−6}.
There are no real numbers x such that f(x)=2.

Q. f and g are function


5

CASE STUDY BASED QUESTIONS


Q.1 A is the anthills of an ant, at B some sweets are there and ant wants to reach at B. The path traced by
an ant is shown in the following graph:

On the basis of the above graph find the following:


(i) When ordinate is 6 then find abscissa
(ii) Which axis is line of symmetry for the graph?
(iii) Write the function for the graph along with domain and range.
Q.2 In a school at Chandigarh, students of class XI were discussing about the relations and functions. Two
students Ankita and Babita form two sets A = {1,2, 3, 4, 5} and B = {2, 4, 6}.
Based on the above information answer the following:
(i) Find n(A×B)
(ii) A correspondence of elements from A to B given as {(1, 2), (2, 2), (3, 4), (3, 6), (4, 4), (5,6)}. Is it a
function? Justify your answer.
(iii) If the function f: A⟶B such that (a, b) ∈ f and a < b, defined by f = {(1, 2), (x, 4), (2,4), (4, y), (5, 6)}, then
find x and y.
38
Q.3 Method to find set when cartesian product is given
For finding these two sets, we write first element of each ordered pair in first set say A and corresponding
second element in second set B (say).
Number of elements in cartesian product of two sets
If there are p elements in set A and q elements in set B, then there will be pq elements in A x B i.e., if n(A) =
p and n(B)= q, then n(A x B) = pq
Based on the above two topic, answer the following questions.

(i) If A x B = {(a, 1), (b, 3), (a, 3), (b, 1), (a, 2), (b, 2)}. Then, A and B are
(a) {1, 3, 2}, {a, b} (b) {a, b}, {1, 3, 2} (c) {a, b}, {1, 3} (d) none of these
(ii) If the set A has 3 elements and set B has 4 elements, then the number of elements in A x B is
(a) 3 (b)4 (c) 7 (d) 12
(iii) A and B are two sets given in such a way that A x B contains 6 elements. If three elements of A x B
are (1, 3), (2, 5) and (3, 3), then A, B are
(a) {1, 2, 3}, {3, 5} (b) {3, 5}, {1, 2, 3} (c) {1, 2}, {3, 5} (d) {1, 2, 3}{5}
(iv) The cartesian product P x P has 16 elements among which are found (a, 1) and (b,2). Then, the set P
is
(a) {a, b} (b) {1,2} (c) {a, b, 1, 2} (d) {a, b, 1, 2, 4}

Q.4 Given set A = {x: x < 5, x ∈ N} and B = {x: 0 ≤ x ≤ 2 , x ∈ Z} .


Based on the above information, answer the following questions:
(i)A × B is a set of ordered pair:
(a) {(1, 0), (1, 1), (1, 2), (2, 0), (2, 1), (2, 2), (3, 0), (3, 1), (3, 2), (4, 0), (4, 1),(4, 2)}
(b) {(0, 0), (1, 1), (2, 2)}
(c) {(1, 0), (2, 1), (3, 2), (4, 3)}
(d) {(1, 0), (1, 1), (2, 0), (2, 1), (3, 0), (3, 1), (4, 0), (4, 1)}

(ii) (A ∪ B) × (A ∩ B) is a set of ordered pair:


(a) {(1, 0), (1, 1), (1, 2), (2, 0), (2, 1), (2, 2), (3, 0), (3, 1), (3, 2), (4, 0), (4, 1),(4, 2)}
(b) {(0, 1), (0, 2), (2, 2), (1, 1), (1, 2), (2, 1), (3, 1), (3, 2), (4, 1), (4,2)}
(c) {(1, 0), (2, 1), (3, 2), (4, 3)}
(d) {(1, 0), (1, 1), (2, 0), (2, 1), (3, 0), (3, 1), (4, 0), (4, 1)}

(iii) A relation R from set A to B defined by R = {(x, y): x + y = 4, x ∈ A, y ∈ B} as a set of ordered pair is:
(a) {(1, 3), (4, 0), (2, 2)} (b) {(0, 4), (1, 3), (2, 2)}
(c) {(1, 3), (2, 2), (3, 1), (4, 0)} (d) {(2, 2), (3, 1), (4, 0)}

(iv) The range of relation R is:


(a) {1, 2, 3, 4} (b) {0, 1, 2} (c) {2, 3, 4} (d) {0, 1, 2}

39
Q.5 If A = {x : x ∈ N, x ≤ 2}, B = {x : x ∈ N, 1 < x < 5}, C = {3, 5} find
(i) A x B
(ii) A x C
(iii) B x ( A ꓵ C)

ANSWERS
Q.1 (i) {-6, 6} (ii) Y axis (iii) f(x) = |x| , domain = R and range = [0, ꝏ)
Q.2 (i) 15 (ii) f is not a function (iii)x = 3 and y =6
Q.3 (i) (b) (ii)(d) (iii)(a) (iv)(c)
Q.4 (i)(a) (ii) (b) (iii) (d) (iv) (b)
Q.5 (i) A×B={(1,2),(1,3),(1,4),(2,2),(2,3),(2,4)}.(ii) A×C={(1,3),(1,5),(2,3),(2,5)}.(iii) ∅

40
TRIGONOMETRIC FUNCTIONS

CONCEPTUAL NOTES
Angles: Angle is a measure of rotation of a given ray about its initial point.
Measurement of an angle.

The amount of rotation from the initial side to the terminal side is called the measure of an angle. There are
two systems for measuring an angle, which are given below.

Sexagesimal System (Degree Measure)


th
 1 
If a rotation from the initial side to terminal side is  of a revolution, the angle is said to be have
 360 
measure of one degree written as 1o. A degree is divided into 60 minutes &
a minute is divided into 60 second.
1o = 60 minutes = 60’. (iii) 1’ = 60 second = 60’’.
Some of the angles whose angles are 3600 , 1800 , 2700 are shown below

Circular System (Radian Measure)


Angle subtended at the center by an arc of length one unit in a circle of radius 1 unit is said to have a
measure of 1 radian.
In general a circle of radius r having an arc of length r will subtend an angle of 1 radian at the centre.
1 radian is written as 1c or 1 rad.
Also a circle of radius r having an arc of length l will subtend an angle  radian
At the centre, where

l Lengthof Arc
= = or l = r
r Radius

The figure shows the angle whose measures are 1 radian ( 1c )

41
Remembering Points
In Sexagesimal System we measure angles in degree, minute & seconds.
1o = 60 minutes = 60’. (iii) 1’ = 60 second = 60’’.
In circular system, we measure angles in radian.

Relation between Degree & Radian


A circle subtends at its centre an angle whose radian measure is 2 & its degree measures is
3600 .It follows that

2 radian = 360 0 or  rad =1800

180 0
1 rad = = 57 016 22 (approx..)


10 = rad = 0.01746 rad
180

Radian measure =  Degree Measure
180
180
Degree Measure =  radian Measure

The relation between degree measure & radian measure of some common angles are given
in the following table.

Degree 30 0 45 0 60 0 90 0 180 0 270 0 360 0


Radian π/6 π/4 π/3 π/2 π 3π/2 2π

42
2n−4
(iii) Each interior angle of a regular polygon of n sides is equal to right angles.
n
    2 3 5
T-ratios 0 
6 4 3 2 3 4 6
1 1 3 3 1 1
Sin 0 1 0
2 2 2 2 2 2
3 1 1 1 1 3
Cos 1 0 – – – –1
2 2 2 2 2 2
1 1
Tan 0 1 3 n.d – 3 –1 – 0
3 3

Domain & Range of the Trigonometric Functions.

T Domain Range Graph


Functio
n
sin  R − 1,1

cos  R − 1,1

tan  x : x  R & x  (2n + 1) / 2, n  Z  R

43
cot  x : x  R & x  n , n  Z  R

cosec  x : x  R & x  n , n  Z  R –(-1,1)

Sec  x : x  R & x  (2n + 1) / 2, n  Z  R –(-1,1)

Formulae for T-ratios of Allied Angles:


 3
All T-ratio changes in (   and   while remains unchanged in    and 2    .
2 2
   3  
sin     = cos  sin     = = cos 
2   2  2
   3 
cos    =  sin  cos    =  sin 
2   2 

44
II Quadrant I Quadrant
   3 
tan     =  cot  tan     =  cot  sin  > 0 All > 0
2   2 
sin (  ) =  sin  sin (2  ) =  sin   0
cos (   ) = = cos  cos(2  ) = cos  tan  > 0 cos  > 0
tan (   ) =  tan  tan (2  ) =  tan 
III Quadrant IV Quadrant
3
Sum and Difference formule:
2
Sin (A + B) = sin A cos B + cos A sin B
Sin (A – B) = sin A cos B – cos A sin B
Cos (A + B) = cos A cos B – sin A sin B
Cos (A – B) = cos A cos B + sin A sin B
tanA + tanB tanA − tanB   1 + tan A
tan(A + B) = , tan (A – B) = . tan  + A  = ,
1 - tanA tanB 1 + tanA tanB 4  1 − tan A
  1 − tan A cot A. cot B − 1 cot A. cot B + 1
tan  − A  = cot (A + B) = cot (A – B) =
4  1 + tan A cot B + cot A cot B − cot A
2 2 2
sin(A + B) sin(A – B) = sin A – sin B = cos B – cos A 2

cos(A + B) cos(A – B) = cos2A – sin2B = cos2B – sin2A

**Formulae for the transformation of a product of two circular functions into algebraic
sum of two circular functions and vice-versa.
2 sinA cos B = sin (A + B) + sin(A – B)
2 cosA sin B = sin (A + B) – sin(A – B)
2 cosA cos B = cos (A + B) + cos(A – B)
2 sinA sin B = cos (A - B) – cos(A + B)

C+D C−D
sin C + sin D = 2 sin cos ,
2 2
C+D C−D
sin C – sin D = 2 cos sin .
2 2
C+D C−D
cos C + cos D = 2 cos cos ,
2 2
C+D C−D
cos C – cos D = – 2 sin sin .
2 2

Formulae for T-ratios of multiple and sub-multiple angles :


2 tan A
sin 2A = 2 sin A cos A = .
1 + tan 2 A
1 − tan 2 A
cos 2A = cos2A – sin2A = 1 – 2 sin2A = 2 cos2A – 1 =
1 + tan 2 A
A
1 + cos2A = 2cos2A 1 – cos2A = 2sin2A 1 + cosA = 2 cos 2
2
A
1 – cosA = 2 sin 2
2
45
2 tan A 3 tan A − tan 3 A
tan 2A = , tan 3A = .
1 − tan 2 A 1 − 3 tan 2 A
sin 3A = 3 sin A – 4 sin3A, cos 3 A = 4 cos3A – 3 cos A
3 −1 3 +1
sin15o = cos75o = . & cos15o = sin75o = ,
2 2 2 2
3 −1 3 +1
tan 15o = cot 75o = =2– 3 & tan 75o = cot 15o = =2+ 3
3 +1 3 −1
5 −1 5 +1
sin18o = = cos 72o and cos 36o = = sin 54o.
4 4
10 − 2 5 10 + 2 5
sin36o = = cos 54o and cos 18o = = sin 72o.
4 4
o o o
 1 1o  1  1
tan  22  = 2 – 1 = cot 67 and tan  67  = 2 + 1 = cot  22  .
 2 2  2  2

General solutions:
* Sin = 0   = n , n  Z

* cos  = 0   = (2n + 1) , n  Z
2
* tan  = 0   = n , n  Z
* sin  = sin    = n + (− 1)  , n  Z
n

* cos  = cos    = 2n   , n  Z


* tan  = tan    = n +  , n  Z
COMPETENCY BASED SOLVED EXEMPLAR QUESTIONS
MCQs
Examples:
1. The value of tan 1° tan 2° tan 3° … tan 89° is:
(a) 0 (b) 1 (c) ½ (d) Not defined
Correct option: (b) 1
Solution: tan 1° tan 2° tan 3° … tan 89°
= [tan 1° tan 2° … tan 44°] tan 45°[tan (90° – 44°) tan (90° – 43°)… tan (90° – 1°)]
= [tan 1° tan 2° … tan 44°] [cot 44° cot 43°……. cot 1°] × [tan 45°]
= [(tan 1° × cot 1°) (tan 2° × cot 2°) …..(tan 44° × cot 44°)] × [tan 45°]
We know that, tan A × cot A =1 and tan 45° = 1
Hence, the equation becomes as;
= 1 × 1 × 1 × 1 × …× 1= 1 {As 1ⁿ = 1}
2. If α + β = π/4, then the value of (1 + tan α) (1 + tan β) is:
(a) 1 (b) 2 (c) – 2 (d) Not defined
Correct option: (b) 2

Solution: Given, α + β = π/4


Taking “tan” on both sides,
tan(α + β) = tan π/4

46
We know that, tan(A + B) = (tan A + tan B)/(1 – tan A tan B)
and tan π/4 = 1.
So, (tan α + tan β)/(1 – tan α tan β) = 1
tan α + tan β = 1 – tan α tan β
tan α + tan β + tan α tan β = 1….(i)
(1 + tan α)(1 + tan β) = 1 + tan α + tan β + tan α tan β
= 1 + 1 [From (i)] =2
3. Find the radius of the circle in which a central angle of 60° intercepts an arc of length
37.4 cm (use π = 22/7).
Solution: Given, Length of the arc = l = 37.4 cm
Central angle = θ = 60° = 60π/180 radian = π/3 radians
We know that,r = l/θ
= (37.4) * (π / 3)= (37.4) / [22 / 7 * 3]= 35.7 cm

4. Find the value of √3 cosec 20° – sec 20°.


Solution: √3 cosec 20° – sec 20°
a. 1 b. 2 c. 3 d. 4
 3 1 
 cos 20 0 − sin 20 0 
3 1 3 cos 20 − sin 20 0 0
Solutions: − = = 4 2 2 
 2 sin 20 . cos 20 
0 0
sin 20 cos 20 sin 20 0.. cos 20 0 0. 0

 
 sin 60 0 cos 20 0 − cos 60 0 sin 20 0   sin( 60 0 − 20 0 ) 
= 4  = 4   = 4 1 = 4
 sin 40 0   sin 40 0 

5. Find the value of sin75°.


( 3 + 1) ( 3 − 1) ( 3 + 1) ( 3 − 1)
a. b. c. d.
2 2 2 2 3 2 3 2
Solutions: Sin(45+30)=sin45.cos30+cos45sin30
( 3 + 1)
=
2 2
𝟏𝟏
6. Find degree measure: 𝟏𝟔 radian
a. 40.40 b. 39.60 c.39.10 d. 39.30
0
11  180 
  =39.30
Solutions : 16   
7. A pendulum of length 14cm swings, find the length of arc described by the pendulum if angle made at
the center is 600
a. 14.67cm b. 14.76cm c. 14.57cm d. 14.77cm
Solutions: Angle=arc/radius
Length of arc =14.67cm
𝝅
8. Find the value of 𝐜𝐨𝐬(− 𝟑 )
a. 1/2 b.-1/2 c. 0 d. 1
Ans: a. 1/2
9. The value of 2 sin75° sin15° is
(a) ½ (b) -1/2 (c) 1 (d) -1
47
Ans: (a) 1/2
10. If sin y + cosec y = 2, then sin2y + cosec2y is equal to -
(a) 1
(b) 4
(c) 2
(d) None of these
Ans: c) 2
11. The value of cos1° × cos2° × cos3° ……..… cos179° is
1
(a)
√2
(b) 0
(c) 1
(d) -1
Ans: (b) 0

ASSERATION/REASONING TYPE
For Q1 to Q5, a statement of assertion (A) is followed by a statement of reason (R). Choose thecorrect
answer out of the following choices.
(a) Both A and R are true and R is the correct explanation of A.
(b) Both A and R are true but R is not the correct explanation of A.
(c) A is true but R is false.
(d) A is false but R is true.
1. Assertion (A): The ratio of the radii of two circles at the centers of which two equal arcs subtend
angles of 30° and 70° is 21:10.
Reason (R): Number of radians in an angle subtended at the center of a circle by an arc is equal to the
ratio of the length of the arc to the radius of the circle.
Ans: (d) A is false but R is true.
2. Assertion (A): cosec x is negative in third and fourth quadrants.
Reason (R): cot x decreases from 0 to −  in first quadrant and increases from 0 to  in third quadrant.
Ans: (c) A is true but R is false.
3. Assertion (A): (sin  + cos ) 2 = sin 2
Reason (R): sin 2  + cos 2  = 1
Ans: (a) Both A and R are true and R is the correct explanation of A.

4.Assertion (A): IfA>0,B>0 and A + B =  / 3 then the maximum value of tanA tanB is 1/3
Reason (R): If
a1 + a2 + a3 ..... + an = k (cons tan t ) then the value a1a2 a3 ........an is greatest when a1 = a2 = a3 ...... = an Ans: (a)
Both A and R are true and R is the correct explanation of A.
𝜋
As 𝐴 + 𝐵 = so, tan(𝐴 + 𝐵) = √3
3
tan 𝐴+tan 𝐵
Or, 1−tan 𝐴 tan 𝐵 = √3
1
Or, tan 𝐴 tan 𝐵 = 1 − (tan 𝐴 + tan 𝐵)
√3
tan 𝐴 tan 𝐵 will be maximum if tan 𝐴 + tan 𝐵 is minimum. But the minimum value of tan 𝐴 + tan 𝐵 is
𝜋
obtained when tan 𝐴 = tan 𝐵 or 𝐴 = 𝐵 = 6
𝜋 𝜋 1
Hence the maximum value of tan 𝐴 tan 𝐵 = tan 6 tan 6 = 3
5. Assertion (A): tanx+2 tan 2x+4 tan 4x+8 tan 8x+16 cot 16x = cot x
48
Reason (R): cot x –tan x = 2 cot 2x
Ans- Both A and R are true and R is the correct explanation of A.
LHS= tan 𝑥 + 2 tan 2𝑥 + 4 tan 4𝑥 + 8 tan 8𝑥 + 16 cot 16𝑥
= cot 𝑥 − (cot 𝑥 − tan 𝑥) + 2 tan 2𝑥 + 4 tan 4𝑥 + 8 tan 8𝑥 + 16 cot 16𝑥
= 𝑐𝑜𝑡𝑥 − 2(cot 2𝑥 − tan 2𝑥) + 4 tan 4𝑥 + 8 tan 8𝑥 + 16 cot 16𝑥
= cot 𝑥 − 4(cot 4𝑥 − tan 4𝑥) + 8 tan 8𝑥 + 16 cot 16𝑥
= cot 𝑥 − 8(cot 8𝑥 − tan 8𝑥) + 16 cot 16𝑥)
= cot 𝑥 − 16 cot 16𝑥 + 16 cot 16𝑥
= cot 𝑥 = RHS
SHORT ANSWER TYPE
1. Solve: 𝒕𝒂𝒏𝒙 + 𝒔𝒊𝒏𝒙 = 𝟎
𝑡𝑎𝑛𝑥 + 𝑠𝑖𝑛𝑥 = 0
1
𝑠𝑖𝑛𝑥 (1 + )=0
𝑐𝑜𝑠𝑥
𝑠𝑖𝑛𝑥 = 0 𝑂𝑅 𝑐𝑜𝑠𝑥 = −1
𝑥 = 𝑛𝜋 𝑂𝑅 𝑥 = (2𝑛 + 1)𝜋, 𝑛𝜖𝑍
𝑺𝒊𝒏𝟕𝒙−𝑺𝒊𝒏𝟓𝒙
2. Show that : 𝑪𝒐𝒔𝟕𝒙+𝑪𝒐𝒔𝟓𝒙 = 𝒕𝒂𝒏𝒙
7𝑥 − 5𝑥 7𝑥 + 5𝑥
𝑆𝑖𝑛7𝑥 − 𝑆𝑖𝑛5𝑥 2𝑠𝑖𝑛 ( 2 ) cos( 2 )
𝐿𝐻𝑆 = =
𝐶𝑜𝑠7𝑥 + 𝐶𝑜𝑠5𝑥 2𝑐𝑜𝑠 (7𝑥 + 5𝑥 ) cos(7𝑥 − 5𝑥 )
2 2
2 𝑠𝑖𝑛𝑥 𝑐𝑜𝑠6𝑥
=
2 𝑐𝑜𝑠6𝑥 𝑐𝑜𝑠𝑥
= 𝑡𝑎𝑛𝑥 = 𝑅𝐻𝑆
3. Find the value of cosec (-1410°)
𝑐𝑜𝑠𝑒𝑐(−1410°) = −𝑐𝑜𝑠𝑒𝑐(1410°)
= −𝑐𝑜𝑠𝑒𝑐(4 ∗ 360 − 30)°
= cosec30°=2
4. Find the value of tan15°
1
1−
tan 45 − tan 30 √3 = 2 −
tan 15° = tan(45° − 30° ) = = √3
1 + tan 45 tan 30 1 + 1
√3
𝟏𝟗𝝅
5. Find the values of tan ( 𝟑 )
=√3
𝟑 𝟑𝝅 𝒙 𝒙 𝒙
6. If 𝒕𝒂𝒏𝒙 = 𝟒 , 𝝅 < 𝒙 < , find the values of 𝒄𝒐𝒔 𝟐 , 𝐭𝐚𝐧 𝟐d 𝒔𝒊𝒏 𝟐
𝟐
3 3𝜋
𝑡𝑎𝑛𝑥 = , 𝜋 < 𝑥 <
4 2
𝑥
2𝑡𝑎𝑛 2 3
𝑥 =
1 − 𝑡𝑎𝑛2 2 4
𝑥 𝑥
3𝑡𝑎𝑛2 + 8𝑡𝑎𝑛 − 3 = 0
2 2
𝑥 1
𝑡𝑎𝑛 = −3,
2 3

49
3𝜋
𝜋<𝑥<
2
𝜋 𝑥 3𝜋
< <
2 2 4
𝑥 𝑥 𝑥
𝑡𝑎𝑛 2 = −3 is possible. Find the value of 𝑐𝑜𝑠 2, 𝑠𝑖𝑛 2
7. Prove that cos 4x = 𝟏 − 𝟖𝒔𝒊𝒏𝟐 𝒙𝒄𝒐𝒔𝟐 𝒙
1 - 8 𝑠𝑖𝑛2 𝑥𝑐𝑜𝑠 2 𝑥
= 1 − 2(2 sin 𝑥. cos 𝑥)2
=1 − 2𝑠𝑖𝑛2 2𝑥
= cos4x

8. A horse is tied to a post by a ripe. If the horse moves along a circular path always keeping the rope tight
and describes 88 m when it has traced out 720 at the centre, find the length of the rope.
Ans-Apply the formula l = r & find the value of the arc, Ans =70cm
9. Show that cos 600 + cos 1200 + cos 2400 - sin 3300 = 0

Ans:-Reduce the angles in to acute angles & find the value.

10. Show that: √𝟐 + √𝟐 + 𝟐 𝒄𝒐𝒔 𝟒𝒙 = 𝟐𝐜𝐨 𝐬 𝒙


Ans: Apply the formula & prove.

CASE STUDIED BASED QUESTIONS


1. If the arcs of same length in two circles subtend angles 65 degrees and 110 degrees at the Centers, find
the ratio of their radii.
Ans:- Apply the formula l=r  , r1:r2=22:13
2.
During Durga puja, a mela was organized by the house
owner’s society Vikram Plaza apartment complexes.
Ram Narayan, a 15-year-old boy visited the mela with his
younger sister Radha. After enjoying different rides, they
decided to ride Giant wheel. Both of them got seated
after purchasing tickets from the vendor.
The Giant wheel was moving with a speed of 3km/hr and
radius of the Giant wheel was 15m. After two complete
rounds, his sister Radha felt vomiting sensation. So, she
wanted to get down the wheel immediately. The operator stopped the wheel when she reached the exit
point. Look at the figure and find out, how many seconds it takes for Radha to reach the exit point from her
present position? (π ≈ 3.14)
(a)31.4 sec (b) 37.68 sec (c) 62.8 sec (d) 45 sec
Ans: (b) 37.68 sec
3. Two students Sai Chandan & Sarada standing on either side of a flag post, observes its top which is 30m
at the angles of elevation 𝛼 & 𝛽 respectively as shown in the figure. Both are 40√3 m apart from each
other and the distance between Sai Chandan and the flag post is 30√3 m.

50
Find the measure of angle ARQ.
1
Ans:- 𝑡𝑎𝑛𝛼 =
√3
1 𝜋
 𝛼 = tan−1 =6
√3
𝜋
 𝑡𝑎𝑛𝛽 = √3 = tan−1 √3 = 3
𝜋 𝜋 𝜋
 𝑎𝑛𝑔𝑙𝑒 𝐴𝑅𝑄 = +3=
6 2

4. In a classroom, teacher explains the properties of a particular curve by saying that this particular curve
has beautiful ups and downs. It starts at 1 and heads down till 𝜋 radian and then heads up again. The curve
𝜋
is closely related to sine function and both follow each other exactly 2 radians apart as shown in the figure
below.

(i) Name the curve about which the teacher is explaining in the classroom.

Ans:- Cos x
ii) Find the domain range of cosine function.
Ans:- Domain R, range=[−1,1]

51
5. From the top of a tower of 10 m high building the angle of elevation of top of a tower is 75° and
theangle of depression of foot of the tower is 15°. If the tower and building are on the same
horizontal surfaces.

(i) Find the value of tan 15°.


(ii) Find the value of cos 75°.

Ans: (i) By the trigonometry formula, we know, tan (A – B) = (tan A – tan B) /(1 + tan A tan B)Therefore, we
can write, tan(45 – 30)° = tan 45° – tan 30°/1+tan 45° tan 30°
Now putting the values of tan 45° and tan 30° from the table we get;
tan(45 – 30)° = (1 – 1/√3)/ (1 + 1.1/√3)
tan (15°) = √3 – 1/ √3 + 1
Hence, the value of tan (15°) is √3 – 1/√3 + 1.
(ii) Using the formula for cos(A + B) = cos(A)cos(B)- sin(A)sin(B) we can find
the value of cos 75°.
cos(75°) = cos(30° + 45°)
Now, we know that cos 30° = √3/2, cos 45° = √2/2, sin 30° = 1/2, and sin 45° = √2/2 from the
special values of trigonometric functions.
cos(75°) = cos(30°)cos(45°) sin(30°)sin(45°)Substituting the known values:
cos(75°) = (√3)/2)(√2)/2) – (1/2)(√2)/2)
Simplifying:
cos(75°) = (√6)/4) – (√2)/4)
Combining the terms:
cos(75°) = (√6 – √2))/4 = (√3-1) / 2√2

6. The below figure shows the compass. The East direction is along the positive X-axis (0° angle) and
North direction is along the +ve Y-axis (90° angles). Initially the pointer is pointed towards North-
East direction. Pointer is deflected in a magnetic field by some angle.

52
On the basis of above answer the following.
a. If pointer move in anticlockwise direction by an angle of 90°, then find the value of sine of angle
made by pointer from East direction.
b. If pointer moves an angle of 165° from its initial position in anticlockwise direction, then find
the value of cosine of angle made by pointer from East direction.
−1
c. If the sine and cosine of angle made by pointer with East direction is then find where the
2
pointer pointed?
d. How much angle will pointer move in anticlock wise direction if tangent of angle made by
pointer with x-axis is – 1?

Ans: (i) Angle made by pointer with East direction = 45° + 90° = 135°

∴ sin 135° = sin (180° – 45°) = sin 45° = 1/ 2

(ii) Angle made by pointer with East direction = 45° + 165° = 210°
3
cos 210°= cos (180° + 30°) = – cos 30° = −
2

(iii) sine and cosine both are – ve so quadrant is III and we known that sin 45° =1/ 2 . Given that,
−1
sin  =
2
sin  = sin( 1800 + 450 )
⇒ θ = 225°
⇒ South West direction
(iv) If tan θ = – 1
θ = 135° or 315°
Initially the pointer is at 45°.So angle moved by pointer is
= 135° – 45° = 90°Or 315° – 45° = 270°
⇒ 90° or 270°

LONG ANSWER TYPE


53
𝝅 𝟑𝝅 𝟓𝝅 𝟕𝝅 𝟑
1. Prove that: 𝒔𝒊𝒏𝟒 𝟖 + 𝒔𝒊𝒏𝟒 𝟖
+ 𝒔𝒊𝒏𝟒 𝟖
+ 𝒔𝒊𝒏𝟒 𝟖 =𝟐.

𝜋 2 1 2
𝜋 𝜋 2 1−𝑐𝑜𝑠 1− 3−√2
Ans: 𝑠𝑖𝑛4 8 = (𝑠𝑖𝑛2 8 ) = ( 4
) =( √2
) =
2 2 8

3𝜋 3 + √2
𝑠𝑖𝑛4 =
8 8
5𝜋 3 + √2
𝑠𝑖𝑛4 =
8 8
7𝜋 3 − √2 𝜋 3𝜋 5𝜋 7𝜋 3
𝑠𝑖𝑛4 = ⇒ 𝑠𝑖𝑛4 + 𝑠𝑖𝑛4 + 𝑠𝑖𝑛4 + 𝑠𝑖𝑛4 =
8 8 8 8 8 8 2
sin 8 x cos x − sin 6 x cos 3x
2. Prove that = tan 2 x
cos 2 x cos x − sin 4 x sin 3x
2 sin 8𝑥 cos 𝑥−2 sin 6𝑥 cos 3𝑥 (sin 9𝑥+sin 7𝑥)−(sin 9𝑥+sin 3𝑥)
Ans: LHS= 2 cos 2𝑥 cos 𝑥−2 sin 4𝑥 sin 3𝑥 = (cos 3𝑥+cos 𝑥)−(cos 𝑥−cos 7𝑥)
sin 7𝑥−sin 3𝑥 2 cos 5𝑥 sin 2𝑥
= = = tan 2𝑥 = 𝑅𝐻𝑆
cos 3𝑥+cos 7𝑥 2 cos 5𝑥 cos 2𝑥
𝒔𝒊𝒏 𝟓𝒙− 𝟐 𝒔𝒊𝒏 𝟑𝒙 + 𝒔𝒊𝒏𝒙
3. Prove that : = tan x
𝒄𝒐𝒔 𝟓𝒙− 𝒄𝒐𝒔𝒙

𝑠𝑖𝑛 5𝑥 + 𝑠𝑖𝑛 𝑥−2 𝑠𝑖𝑛 3𝑥


ANS-
𝑐𝑜𝑠 5𝑥− 𝑐𝑜𝑠𝑥
2 𝑠𝑖𝑛 3𝑥 𝑐𝑜𝑠 2𝑥 – 2 𝑠𝑖𝑛 3𝑥 sin 3𝑥(cos 2𝑥−1)
= =tanX=RHS
– 2 𝑠𝑖𝑛 3𝑥 𝑠𝑖𝑛 2𝑥 −𝑠𝑖𝑛 3𝑥 𝑠𝑖𝑛 2𝑥
4. If sin = 3/5 and cosy = -12/13, where x and y lie in second quadrant, then find the value of sin(x+y)
and tan(x-y).

Ans: cosx =-4/5 , siny = 5 /13


tanx = -3/4 , tany = -5/12
sin(x+y) = -56/65, tan(x-y) =-16/33
5. Prove that: 𝐜𝐨𝐬 𝟓 𝑨 = 𝟏𝟔 𝒄𝒐𝒔𝟓 𝑨 − 𝟐𝟎 𝒄𝒐𝒔𝟑 𝑨 + 𝟓 𝐜𝐨𝐬 𝑨
𝐿𝐻𝑆. = 𝑐𝑜𝑠(3𝐴 + 2𝐴) = 𝑐𝑜𝑠 3𝐴 . 𝑐𝑜𝑠 2𝐴 − 𝑠𝑖𝑛 3𝐴 . 𝑠𝑖𝑛 2𝐴 .
= 4 𝑐𝑜𝑠 3 𝐴 − 3 cos 𝐴)(2 𝑐𝑜𝑠 2 𝐴 − 1)−(3 sin 𝐴 − 4 𝑠𝑖𝑛3 𝐴)(2 sin 𝐴 cos 𝐴)
=(4 𝑐𝑜𝑠 3 𝐴 − 3 cos 𝐴)(2 𝑐𝑜𝑠 2 𝐴 − 1)−(3 − 4 𝑠𝑖𝑛2 𝐴)(2 sin2 𝐴 cos 𝐴)
=(4 𝑐𝑜𝑠 3 𝐴 − 3 cos 𝐴)(2 𝑐𝑜𝑠 2 𝐴 − 1)−[3 − 4 (1 − 𝑐𝑜𝑠 2 𝐴)][2 (1 − cos2 𝐴) cos 𝐴]
= 16 𝑐𝑜𝑠 5 𝐴 − 20 𝑐𝑜𝑠 3 𝐴 + 5 cos 𝐴
6.If α + β = π/4, then the value of (1 + tan α) (1 + tan β) is
Ans: Given, α + β = π/4
Taking “tan” on both sides,
tan(α + β) = tan π/4
We know that, tan(A + B) = (tan A + tan B)/(1 – tan A tan B)
and tan π/4 = 1.
So, (tan α + tan β)/(1 – tan α tan β) = 1
tan α + tan β = 1 – tan α tan β
tan α + tan β + tan α tan β = 1….(i)
(1 + tan α)(1 + tan β) = 1 + tan α + tan β + tan α tan β
= 1 + 1 [From (i)] =2

54
SELF PRACTICE QUESTIONS
MCQs
Examples:
1. The conversion of 400 20′ into radians is:
15 121π
a) π Radians b) 9 π radians c) 540 radians d) none of these.
√3 1
2. If sin= 2 and cosx= − 2 then x lies in:
a) 1st quadrant b) 2ndquadrant c) 3rdquadrant d) 4thquadrant
3. Value of sec 2 x + cos2 x is:
a) < 0 𝑏) < 1 𝑐) ≥2 d) None of these.
7
4. If coty = 24 and y lies in the third quadrant then value of cosy-siny is:
17 16 14 13
a. b. c. 25 d. 25
25 25
π 7π π
5. Value of 2sin2 6 + cosec 2 cos2 3 is :
6
1 1 3
a. b. − c. 1 d.
2 2 2
1
6. If sin = 3 then value of sin3x is:
23 −23 23 4√2
a.27 b. c.√27 d.
27 9
0
7. Value of tan 15 is:
1 1
a. 0 b.2 c.3 d. 2−√3
8. The minimum value of cosx + sin is:
a. 0 b. 1 c. -1 d.−√2
9. Value of tan (−15750 ) is:
1
a. 1 b. 2 c.0 d. -1
10. The greatest and least values of sin , cosx are respectively
1 1 1 1
a. 1, -1 b. , − c. , − d.2,-2
2 2 4 4
Answer key of practice questions
1. C 2. b 3. C 4. a 5. d
6. d 7. d 8. D 9. d 10 b
ASSERATION/REASONING TYPE
For Q1 to Q5, a statement of assertion (A) is followed by a statement of reason (R). Choose thecorrect
answer out of the following choices.
a. Both A and R are true and R is the correct explanation of A.
b. Both A and R are true but R is not the correct explanation of A.
c. A is true but R is false.
d. A is false but R is true.

 11 
1. Assertion (A): Value of tan  −  is 1 .
 4 
Reason (R): sin (3 + ) = − sin  .
5
2. Assertion (A): Radian measure for the angle 55 0 is
12
Reason (R): Radian measure = Degree measure ×  /180
3. Assertion (A): The function sin x is negative in third &fourth quadrant.

55
Reason (R): The function sin is decreasing in the interval 0  x   / 2
4. Assertion (A): sin2>sin3
Reason (R): If x, y ( / 2,  ), x  y, then sin x  sin y
5. Assertion (A): The maximum value of sin  + cos is 2.
Reason (R): The maximum value of sin  is 1 & that of cos  is also 1.
Answer Key
1. b
2. d
3. c
4. a
5. d

SHORT ANSWER TYPE


1 3
1. 1 If Cos θ = - ,π<θ< , Evaluate 4 tan 2θ – 3 Cosec 2 θ.
2 2
2. Show that cos 600 + cos 1200 + cos 2400 - sin 3300 = 0
3. Show that 2 + 2 + 2 cos 4 x = 2 Cosx
 x− y
4. Show that ( cos x − cos y ) + ( sin x − sin y ) = 4sin 2 
2 2

 2 
 9 5
5. Show that cos 2 .cos − cos3 .cos = sin 5 .sin
2 2 2
6. Show that 1 + Sin 2 x − Cos 2 x
= tan x
1 + Sin 2 x + Cos 2 x
cos3 A
7. Show that Cos A. Cos (60 – A). Cos (60 + A) =
4
1 1  A+ B  1
8. If Cos A + Cos B = ,Sin A + Sin B = , Show that tan   =
2 4  2  2
2
9. Show that 2 sin β + 4 cos(α + β) sinα sinβ + cos2 (α + β) = cos2α.
10. Prove that sin 2 8x − sin 2 3x = sin11x.sin 5 x
Answer Key to Short Questions

Q1. 11

CASE STUDY BASED QUESTIONS


1. Find in degree the angle through which a pendulum, swings if its length is 50 cm and the tip describes an
arc of length 10 cm.

2. If the arcs of same length in two circles subtend angles 65 degrees and 110 degrees at the
Centres, find the ratio of their radii.

3.Lalu constructs two right triangles in the fourth quadrants in such a way that the measure of triangle
3
gives cos A = 4/5 and cos B=12/13, where  A and B  2
2

56
Based on above information, answer the following questions.
I)find the value of cos(A+B)
ii)Find the value of sin(A-B)
Iii)Find the value of tan(A+B)
4. The base of a pole of height 20m,the angle of elevation of the top of a tower is 60 0 .The pole subtends an
angle 30 0 at the top of the tower. Then find the height of the tower.
5. Two poles standing on a horizontal ground are of heights 5m & 10m respectively. The line joining their
top makes an angle of 15 0 with ground. Then find the distance (in meter) between the poles.
Answer Key
1. 11 5 ,
11
2.22:13
3. 33/65,-16/65,-56/33
4. 30m
5. 5(2 + 3 ) m
LONG ANSWER TYPE
 
1. Prove that: cos2 x + cos2  x +  + cos2  x −  =
3
 3  3 2
5 x x x
2. If tan x = and x lies in 2rd quadrant, find the value of sin , cos , tan
12 2 2 2
x 5 x 1 x
Ans. sin = , cos = , tan = 5
2 26 2 26 2
3. Show that cos 6 x = 32cos x − 48cos x + 18cos x −1
6 4 2

(cos x − cos3x)(sin8x + sin 2x)


4. Show that =1
(sin 5x − sin x)(cos 4 x − cos6x)
1
5. Show that Cos200 Cos 400 Cos 600 Cos 800 =
16 .
Answer Key
x 5 x 1 x
2. sin = , cos = , tan = 5
2 26 2 26 2

57
COMPLEX NUMBERS AND QUADRATIC EQUATION
CONCEPTUAL NOTES:-
A number consisting of real number and imaginary number is called complex number. A complex number
can be defined as a number of the form a+𝑖𝑏,where a and b are real numbers.
The symbol 𝑖 is used to denote √−1 and it is called iota.
The complex number is generally denoted by z i.e z = 𝑎 + 𝑖𝑏.
In complex number z, a is called the real part and b is called the imaginary part of z and they are denoted
as Re(z) and Im(z).
Integral Powers of 𝑖
𝑖1 𝑖2 𝑖3 𝑖4 𝑖5 𝑖6
𝑖 -1 −𝑖 1 𝑖 -1
Equality of Complex Numbers
Two complex numbers z1 = 𝑎 + 𝑖𝑏 ,z2 = 𝑐 + 𝑖𝑑 are said to be equal, if a = c and b = d.
Operations on Complex Numbers:-
(i) Let z1 = 𝑎 + 𝑖𝑏, z2 = 𝑐 + 𝑖𝑑 be two complex numbers, then their addition is defined as
Z = z1+ z2 =( a+c) + 𝑖(b+d)
(ii) Let z1 = 𝑎 + 𝑖𝑏 ,z2 = 𝑐 + 𝑖𝑑 be two complex numbers, then their subtraction is defined as
Z = z1- z2 =( a-c) + 𝑖(b-d)
(iii) Let z1 = 𝑎 + 𝑖𝑏 ,z2 = 𝑐 + 𝑖𝑑 be two complex numbers, then their multiplication is defined as
z1 z2 =( ac-bd) + 𝑖(ad+bc)
(iv) Let z1 = 𝑎 + 𝑖𝑏, z2 = 𝑐 + 𝑖𝑑 be two complex numbers, then their division is defined as
𝑧1 1
= z1. z2-1 = z1. ( )
𝑧2 𝑧2

Conjugate of a Complex Number:-


The conjugate of a complex number z, is the complex number, obtained by changing the sign of imaginary
part of z. It is denoted by 𝑧.
If z = 𝑎 + 𝑖𝑏,then 𝑧 = 𝑎 − 𝑖𝑏.
Modulus of Complex Numbers:-
The modulus or Absolute value of a complex number z = 𝑎 + 𝑖𝑏 is defined as the non-negative real number

√𝑎2 + 𝑏 2 .It is denoted by |𝑧| = √𝑎2 + 𝑏 2 .


MCQ QUESTIONS:-
1.If z = 1+i,then the value of z2 is
(a) 1-i (b) i-1 (c) 2i (d) - 2i
2.If a and b are non-negative real numbers,then the value of√−𝑎√−𝑏 is
58
(a)√𝑎𝑏 (b) −√𝑎𝑏 (c) 𝑖 √𝑎𝑏 (d)−𝑖 √𝑎𝑏
1
3.If z = ,then the value of |𝑧| is
𝑖

(a) 1 (b) -1 (c) 0 (d) 2


4.Which of the following is true ?
(a) 1-I< 1 + 𝑖 (b) 2i+1> −2𝑖 + 1 (c) 2i > 1 (d) None of these
5.If z= 𝑖 9 + 𝑖 19 ,then z is equal to
(a) 0 + 𝑜𝑖 (b) 1 + 𝑜𝑖 (c) 0 + 𝑖 (d) 0 + 2𝑖
6.If x = √−16 ,then
(a) x = 4𝑖 (b) x = 4 (c) x = -4 (d) All of these
3
7.If z = 5𝑖(− 5 𝑖),then z is equal to

(a) x = 0+ 3𝑖 (b) x = 3+ 0𝑖 (c) 0+ 𝑖 (d) 1 + 2𝑖


8.The value of 𝑖 1947 + 𝑖 1948 + 𝑖 1950 is
(a) 1 (b) -1 (c) 0 (d) i
9.The value of 𝑖 −1097 is
(a) 1 (b) -1 (c) 𝑖 (d) −𝑖
10.If z1 = 3 + 2 𝑖 and z2 = 2- 𝑖 ,then 𝑧1 + 𝑧2 is equal to
𝑧1
(a) 𝑧1 𝑧2 (b) 𝑧1 + 𝑧2 (c) 𝑧1 𝑧2 (d) 𝑧2

ASSERTION-REASON BASED QUESTIONS:-


Directions(Q.Nos.11 to 15) In the questions given below are two statements labeled as Assertion(A) and
Reason(R) .In the context of the two statements ,which one of the following is correct?
(a)Both A and R are correct ;R is correct explanation of A.
(b)Both A and B are correct ;R is not the correct explanation of A.
(c)A is correct ;R is incorrect
(d)R is correct ;A is incorrect
(1+𝑖)𝑥−2𝑖 (2−3𝑖)𝑦+𝑖
11 .Assertion(A) If x and y are real numbers and + = 𝑖,then x=3 and y= -1.
3+𝑖 3−𝑖
1
Reason(A) If 2a+3b and 2𝑖 represent the same complex number,then a = 0, b = 2.

12. Assertion(A) If z is a complex number, then (𝑧-1)(𝑧) is equal to 4.


𝑧−𝑎
Reason(A) The region of the complex plane for which |𝑧+𝑎| = 1 [𝑅𝑒(𝑎) ≠ 0] is Y-axis.

13. Assertion(A) If √𝑎 + 𝑖𝑏 = x +𝑖𝑦,then √𝑎 − 𝑖𝑏 = x – 𝑖𝑦.


Reason(A) A complex number z is said to be purely imaginary. if Re(z) = 0.
14. Assertion(A) The modulus of the complex number z = 2 + 3𝑖 is 4.

59
𝜋 𝜋 𝜋
Reason(A) The arg z = 3 ,Hence the polar form of the complex number 2 + 3𝑖 is z = 4( cos3 + 𝑖sin 3 ).

15. Assertion(A) The roots of the quadratic equation x2-x-6 = 0 are 3 and -2.
Reason(A) The roots are real and distinct as b2-4ac > 0.
SHORT ANSWER TYPE QUESTIONS:-
1.Find the real value of a for which 3𝑖 3-2a𝑖 2 +(1-a)𝑖 +5 is real.
2.For what value of a and b are (1-𝑖)𝑎 + (1 + 𝑖)𝑏 𝑎𝑛𝑑 1 − 3𝑖 𝑒𝑞𝑢𝑎𝑙 ?
5+√2𝑖
3.Express in the form of a+𝑖𝑏: .
1−√2𝑖

4.Express in the form of 𝑎 + 𝑖𝑏:


𝑖 75 + 𝑖 80 + 𝑖 85 +𝑖 90
5.Evaluate (1 + 𝑖))6 + (1 − 𝑖))3.

6.Find the multiplicative inverse of √5 + 3𝑖.


1−𝑖
7.If 𝑎 + 𝑖𝑏 = (1+𝑖)100,then find (a,b).
2−𝑖
8.Find the conjugate of (1−2𝑖)2 ?
1+𝑖 1−𝑖
9.Find the modulus of 1−𝑖 - .
1+𝑖
(1+𝑖)(2+𝑖)
10.Find the value of | |.
3+𝑖

CASE STUDY BASED QUESTIONS:-


1.Consider z = 𝑎 + 𝑖𝑏 and 𝑧̅ = 𝑎 − 𝑖𝑏,where a and b are real numbers are conjugate of each other.Based
on the above information,answer the following questions.
(i) z1 = 3 + 2𝑖, z2 = 2 − 𝑖,then ̅̅̅̅̅̅̅̅̅
𝑧1 + 𝑧2 is equal to
(a) 5 − 3𝑖 (b) 5−𝑖 (c) 0 (d) 2
(ii)The conjugate of (6 + 5𝑖)2 is
(a) 11-60𝑖 (b) 11+60𝑖 (c) −11 − 60𝑖 (d) −11 + 60𝑖
(iii)If the complex number -3+ 𝑖(𝑥 2 y) and x2+y+4𝑖,where x and y are real,are conjugate to each other,then
the number of ordered pairs(x,y) is
(a) 1 (b) 2 (c) 3 (d) 4
(iv) If z1 =x2−7𝑥 − 9𝑦𝑖 z2 = y2𝑖 + 20𝑖 − 12,then the number of ordered pairs(x,y) is
(a) 1 (b) 2 (c) 3 (d) 4
2.A complex number z is purely real if and only if 𝑧̅ = 𝑧 and purely imaginary if and only if 𝑧̅ = −𝑧Based
on the above information,answer the following questions.
(2+𝑖)𝑥−𝑖 (1−𝑖)𝑥+2𝑖
(i) If x and y are real numbers and the complex number + is purely real,the relation
4+𝑖 4𝑖

between x and y is
60
(a) 8x +7y = 15 (b) 8x- 17y = 16 (c) 17x – 8y = 15 (d) 17x +8y = 15
(ii)If z be a complex number satisfying |𝑅𝑒(𝑧)|+|𝐼𝑚(𝑧)| = 4,then |𝑧| cannot be
17
(a) √7 (b) √ 2 (c) √10 (d) √8

(iii)The smallest positive integer(n) for which (1 + 𝑖)2𝑛 = (1 − 𝑖)2𝑛 is


(a) 2 (b) 3 (c) 1 (d) 4
3.Two complex numbers z1 = 𝑎 + 𝑖𝑏 z2 = 𝑐 + 𝑖𝑑 are said to be equal,if a = c and b = d. In other words they
have same real parts and imaginary parts. Based on the above information ,answer the following
questions.
(i) The real part of the complex number 2𝑖 + 3𝑖 5 – 6𝑖 2 .
(a) 2 (b) 3 (c) 5 (d) 6
(ii) If 4x +𝑖(3𝑥 − 𝑦) = 3 + 𝑖(−6),where x and y are real numbers ,then the values of x is
3 33 3
(a) (b) (c) 3 (d)
4 4 2

(iii)For what values of a and b are (1 − 𝑖)𝑎 𝑎𝑛𝑑 (1 + 𝑖)𝑏 𝑎𝑛𝑑 1 − 3𝑖 equal?
(a) (2,-1) (b) (-1,-1) (c) (2,2) (d) (3, −1)
4.Given two complex numbers z1 = 1 + 𝑖, z2 = 2 − 3𝑖. Based on the above information ,answer the
following questions.
(i) The conjugate of z1 + z2 is
(a) 2+3𝑖 (b) 2-3𝑖 (c) 3 + 2𝑖 (d) 3 − 2𝑖
(ii)The modulus of z1 - z2 is
(a) 15 (b) √15 (c) 17 (d) √17
(iii)The multiplicative inverse of z1 is
1+𝑖 1−𝑖
(a) (b) (c)1 + 𝑖 (d) 1 − 𝑖
2 2

5.The following complex numbers are given.


𝑧1 = −2𝑖, 𝑧2 = √3 + 𝑖 and 𝑧3 = 𝑎 + 𝑖𝑏, where a, b𝜖𝑅. Based on the above information , answer the
following questions.

(i) If |𝑧1 𝑧3 | = 16,find the modulus of z3

(a) 3√2 (b) 2√2 (c) 4√2 (d) 4√3


𝑧 7𝜋
(ii) Given that arg(𝑧3) = 12 ,determine the argument of z3
2

3𝜋 3𝜋 2𝜋 𝜋
(a) (b) (c) (d)
4 2 3 4

(iii)The values of and b are


(a) (-4,4) (b) (−4,2) (c)(2,4) (d) (−2,2)
61
LONG ANSWER TYPE QUESTIONS
1.What is the smallest positive integer n, for which (1+𝑖)2n = (1-𝑖)2n?
2.Find the value of z3-2z2+3z-4,when z=1+𝑖

3.If z = x+𝑖𝑦,then prove that |𝑥|+|𝑦| ≤ √2|𝑧|.


4.Solve for real numbers x and y (1+𝑖)x2 + (6+𝑖) = (2 + 𝑖)𝑦.
5.Find the value of x2+y2 if x+𝑖𝑦 = (1 + 𝑖)(2 + 𝑖)(3 + 𝑖).
ANSWER KEY
MCQ QUESTIONS:-
1.(c)
2.(b)
3.(a)
4.(d)
5.(a)
6.(a)
7.(b)
8.(c)
9(d)
10.(b)
ASSERTION-REASON BASED QUESTIONS:-
11.(b)
12.(d)
13.(b)
14.(d)
15.(a)
SHORT ANSWER TYPE QUESTIONS:-
1. a= -2
2.a=2,b=-1
3.1+2√2𝑖
4.0
5.-2-10𝑖
√5 3
6.14 -14 𝑖

7.(a,b) = (1,0)

62
1
8.25(-2-11𝑖)

9.2
10.2
CASE STUDY BASED QUESTIONS:-
1.(i) b
(ii) a
(iii) b
(iv) d
2.(i) b
(ii) a
(iii) a
3.(i) d
(ii) a
(iii) a
4.(i) c
(ii) d
(iii) b
5.(i) c
(ii) d
(iii) a
LONG ANSWER TYPE QUESTIONS
1. 2
2. -3 +𝑖
3. To prove
4. x = ±2,y = 5
5.100

63
LINEAR INEQUALITIES
Inequation
A statement involving variables and the sign of inequality viz. >, <, ≥ or ≤ is called an inequation or an
inequality.
Numerical Inequalities
Inequalities which do not contain any variable is called numerical inequalities, e.g. 3 < 7, 2 ≥ -1, etc. Literal
Inequalities Inequalities which contains variables are called literal inequalities e.g. x – y > 0, x > 5, etc.
Linear Inequation of One Variable
Let a be non-zero real number and x be a variable. Then, inequalities of the form ax + b > 0, ax + b < 0, ax +
b ≥ 0 and ax + b ≤ 0 are known as linear inequalities in one variable.
Linear Inequation of Two Variables
Let a, b be non-zero real numbers and x, y be variables. Then, inequation of the form ax + by < c, ax + by >
c, ax + by ≤ c and ax + by ≥ c are known as linear inequalities in two variables x and y.
Solution of an Inequality
The value(s) of the variable(s) which makes the inequality a true statement is called its solutions. The set of
all solutions of an inequality is called the solution set of the inequality.
Solving Linear Inequations in One Variable
Same number may be added (or subtracted) to both sides of an inequation without changing the sign of
inequality.
Both sides of an inequation can be multiplied (or divided) by the same positive real number without
changing the sign of inequality.
However, the sign of inequality is reversed when both sides of an inequation are multiplied or divided by a
negative number.
Representation of Solution of Linear Inequality in One Variable on a Number Line
To represent the solution of a linear inequality in one variable on a number line. We use the following
algorithm.
If the inequality involves „>‟ or „ we draw an open circle (O) on the number line, which indicates that the
number corresponding to the open circle is not included in the solution set
If the inequality involves „≥‟ or „≤‟ we draw a dark circle (•) on the number line, which indicates the
number corresponding to the dark circle is included in the solution set

64
MCQ Questions.
1. Mr X has 6 more marks than Mr Y. If they have more than 100 marks. What is the minimum number of
marks does Mr Y have?
(a) 46 (b) 47 (c) 48 (d) 49
2. If |x-2|x-2 ≥ 0, then
a) x ∈ [2, ∞) (b) x ∈ (2, ∞) (c) x ∈ (– ∞, 2) (d) x ∈ (– ∞, 2]
3. The length of a rectangle is three times the breadth. If the minimum perimeter of the rectangle is 160
cm, then
(a) breadth > 20 cm (b) length < 20 cm (c) breadth x ≥ 20 cm (d) length ≤ 20 cm
4. If I x + 3I ≥ 10, then
(a) x ∈ (– 13, 7] (b) x ∈ (– 13, 7] (c) x ∈ (– ∞, – 13] ∪ [7, ∞) (d) x ∈ (– ∞, – 13] ∪ [7, ∞)
5. In a garden there are two types of plants Rose & Jasmine with maximum 500 plants.
The ratio of Rose and Jasmine plants are in the ratio 2 : 3. What is the maximum number of Rose plants?
(a) 200 (b) 300 (c) 350 (d) 400
6. A furniture dealer deals in only two items–tables and chairs. He has Rs60,000 to invest and has storage
space of at most 100 pieces. A table costs Rs3000 and a chair Rs1000.
Let x be the number of tables and y be the number of chairs that the dealer buys. Which of the following
represents the investment constraint/ inequality.
(a) 3000x + 1000y ≤ 60000(b) 3000x + 1000y < 60000
(c) 3000x + 1000y ≥ 60000(d) 3000x + 1000y > 60000
7. If x < 5, then
(a) – x < – 5 (b) – x ≤ – 5 (c) – x > – 5 (d) – x ≥ – 5
8. Given that x, y and b are real numbers and x < y, b < 0, then
(a) x/ b < y/ b (b) x/ b ≤ y/ b (c) x /b > y/ b (d) x/ b ≥ y /b
9. If – 3x + 17 < – 13, then
(a) x ∈ (10, ∞) (b) x ∈ [10, ∞) (c) x ∈ (– ∞, 10] (d) x ∈ [– 10, 10)
10. If x is a real number and | x | < 3, then
(a) x ≥ 3 (b) – 3 < x < 3 (c) x ≤ – 3 (d) – 3 ≤ x ≤ 3
Assertion Reason Questions
Q1.Assertion (A): The minimum value of 2𝑠𝑖𝑛2𝑥+ 2𝑐𝑜𝑠2 𝑥𝑖𝑠 √2 𝑤ℎ𝑒𝑟𝑒𝑥𝜖𝑅
Reason (R): A.M ≥ 𝐺. 𝑀

1)Both A and R are true and R is the correct explanation of A.


2)Both A and R are true but R is not the correct explanation of A.
65
3)A is true but R is false.
4)A is false but R is true. Which one is correct
a) 1 (b) 2 (c)3 (d) 4
Q2. Assertion (A) : The inequality ax + by < 0 is strict inequality
. Reason(R) : The inequality ax + b ≥ 0 is slack inequality
1)Both A and R are true and R is the correct explanation of A.
2)Both A and R are true but R is not the correct explanation of A.
3)A is true but R is false.
4)A is false but R is true. Which one is correct
a) 1 (b) 2 (c)3 (d) 4
Q3. Assertion(A) : If a < b, c < 0, then a/c <b/c
Reason(R) : If both sides are divided by the same negative quantity, then the inequality is reversed.
1)Both A and R are true and R is the correct explanation of A.
2)Both A and R are true but R is not the correct explanation of A.
3)A is true but R is false.
4)A is false but R is true. Which one is correct
a) 1 (b) 2 (c)3 (d) 4
Q4. Assertion: The inequality 3x + 2y ≥ 5 is the linear inequality.
Reason : The solution of 5x – 3 < 7, when x is a real number, is (–∞, 2)
1)Both A and R are true and R is the correct explanation of A.
2)Both A and R are true but R is not the correct explanation of A.
3)A is true but R is false.
4)A is false but R is true. Which one is correct
a) 1 (b) 2 (c)3 (d) 4
Q5. Assertion : Two real numbers or two algebraic expressions related by the symbol < , >, ≥, ≤ forms an
inequality.
Reason : The inequality ax + by < 0 is strict inequality.
1)Both A and R are true and R is the correct explanation of A.
2)Both A and R are true but R is not the correct explanation of A.
3)A is true but R is false.
4)A is false but R is true. Which one is correct
a) 1 (b) 2 (c)3 (d) 4
Short type questions.

66
Q1. Find all pairs of consecutive even positive integers, both of which are larger than 5 such that their sum
is less than 23.
Q2. Abhaya obtained 70 and 75 marks in first two unit tests. Find the number if minimum marks he should
get in the third unit test to have an average of at least 60 marks.
Q3. The length of a rectangle is three times the breadth. If the minimum perimeter of the rectangle is 320
cm, then what is the minimum breadth?
Q4. If Kavita’s marks in first four examinations are 87, 92, 94 and 95, find the minimum mark in the fifth
examination to get grade ‘A’ in the course.
Q5. State which of the following statements is True or False
(i) If | x| ≤ 4, then x ∈ [– 4, 4] (T/F)
(ii) If | x| > 5, then x ∈ (– ∞, – 5) ∪ [5, ∞) (T/F)
Q6. . Solve 4x+32x-5 ˂ 6 and show the solution on the number line.
Q7. Find the solution set of inequalitiesx2x+1>14.
Q8. . Find all pairs of consecutive odd positive integers, both of which are smaller than 18, such that their
sum is more than 20.
Q9. The water acidity in a pool is considered normal when the average pH reading of three daily
measurements is between 7.2 and 7.8. If the first two pH readings are 7.48 and 7.85, find the range of pH
value for the third reading that will result in the acidity level being normal.
Q10. Jitendra, Mahendra and Dharmendra together invest money into a new business. Mahendra gives
twice of Jitendra and Dharmendra gives 20000 more than Mahendra. Find the minimum amount of money
invested by Mahendra if at least 1 crore needed to start business.
Case based questions.
1. Shweta was teaching “method to solve a linear inequality in one variable” to her daughter.
StepI: Collect all terms involving the variable (x) on one side and constant terms on other side with me
help of above rules and then reduce it in the form ax<b or ax𝑎𝑥 ≤ 𝑏 𝑜𝑟 𝑎𝑥 > 𝑏 𝑜𝑟 𝑎𝑥 ≥ 𝑏.
Step II: Divide this inequality by the coefficient of variable (x).This gives the solution set of given
inequality.
Step III: Write the solution set.
Based on the above information ,answer the following questions.
I) Find the solution set of 24x<100 , when x is a natural number is?

ii) Find the solution set of -5x+25>0 where x ∈ 𝑅

67
Q2. Aayansh works in a chemical factory .He regularly needs to keep some chemical at different
temperatures for storage. Suppose boiling point of one of the chemical is less than 144°F.He knows the
relation between Fahrenheit temperature and Celsius temperature as C5=F-325
a)Form the linear inequality from this situation.
b)He needs to keep a solution between 40°C to50°C What could be the range of the temperature in
Fahrenheit?
c)He wants to dilute a 500 litres acid solution of 30% concentration by adding water to it . What could be
the range of water to be added if he wants acid in solution to keep between 20% and 25% ?
Q3. Mitu is a psychology students and now a days she is learning about intelligence Quotient. She know
the result IQ= Mental age/ chronological age× 100
On the basis of the above case answer the following questions
a) What could be the range of mental age if a group of children with chronological age of 15 years have the
IQ range as 90 ≤ IQ ≤ 150?
b) What could be the range of IQ if a group of children with age of 12 years have the mental age range as 9
≤ MA ≤ 15?
Q4. A company produced cassettes, one cassette Cost Company Rs. 30 and also an additional fixed cost
26000 per week. The company sold each Cassette at Rs. 43. If x is number of cassettes produced and sold
by the company in a week. From the following information find
a. The cost function of the company (a) 26000 + 30x (b) 26000 + 43x (c) 30 + 26000x (d) 43 + 26000x
b. The revenue function of the company is
(a) 30x (b) 26000x (c) 43x (d) 13x
c. How many cassettes must be produced by the company in a week to realize some profit?
(a) more than 2000 (b) less than 2000 (c) more than 5000 (d) less than 5000
d. If company incurred an additional cost of Rs. 3 on each cassette per week. How many cassettes must be
produced by the company in a week so that there is no profit no loss?
(a) 2000 (b) 5000 (c) 2600 (d) 1000
Q5. Sweta was teaching “method to solve linear inequality in one variable” to her daughter.
Step-I:collect al terms involving the variable (x) on one side and constants on another side with the help of
above rules and then reduce it in the form of ax< b or ax≤b or ax > b or ax ≥b.
Step-II: divide this inequality by the coefficient of variable (x). this gives the solution set of given inequality.
Step-III: write the solution set.
Based on the above information, answer the following questions.
i) The solution set of 24x< 100 , when x is a natural number is

68
a) {1,2,3,4} b) (1,4) c) [1,4] d) none of these
ii) The solution set of 24x< 100 , when x is an integer is
a) {…….-2,-1,0,1,2…….} b) (-∞,4] c) [4,∞) d) none of these
5 long answer type questions
Q1. In drilling world’s deepest hole it was found that the temperature T in degree Celsius, x km below the
earth’s surface was given by T = 30 + 25(x-3), 3 ≤ x ≤ 15. At what depth will the temperature be between
155° C and 205°C.
Q2. Suppose we have been given 200 litres of 10% concentrated H2 SO4 acid. How many
litres of 30% conc. H2 SO4 acid should be added so that the concentration of the resulting
acid should be more than 15% but less than 20%?

Q3. In an experiment a thermometer only Fahrenheit measurements are shown . Using it a


solution of hydrochloric acid is measured which is to be kept between 300 and 350 Celsius.
What is the range of temperature in degree Fahrenheit if conversion formula is given by C = 59(F-32)
Where C and F represent and degree Celsius and degree Fahrenheit respectively.

Q4. The longest side of a triangle is 3 times the shortest side, and the third side is 2 cm shorter than the
longest side. If the perimeter of the triangle is at least 61 cm, find the minimum length of the shortest side.

Q5.A manufacturer has 600 litres of a 12% solution of acid. How many litres of a 30% acid
must be added to it so that the acid content in the resulting mixture will be
more than 15% but less than 18%?
SOLUTION
MCQ
Q1.(c) 48
x + x + 6 > 100 ⇒ 2x > 94 ⇒ x > 47
⇒ Minimum value of x is 48.
Q2. (b) x ∈ (2, ∞)
Q3. (c) 2(3x+x)≥160 => x ≥ 20
Q4. Since x +3 ≥ 10, ⇒ x + 3 ≤ – 10 or x + 3 ≥ 10⇒ x ≤ – 13 or x ≥ 7 ⇒ x ∈ (– ∞, – 13] ∪ [7, ∞)
Q5. 2x + 3x ≤ 500 ⇒ x ≤ 100 ⇒ 2x ≤ 200
Q6. 3000x + 1000y ≤ 60000
Q7. – x > – 5

69
Q8. x /b > y/ b
Q9. x ∈ (10, ∞)
Q10. – 3 < x < 3
Assertion Reason Questions
Q1. (d) 4
Q2. (b) 2
Q3. (d) 4
Q4. (b) 2
Q5. (b) 2
Short answer type questions
Q1. Let x be the smaller of the two positive consecutive even integers, then the other number is(x + 2)
Given x > 5 and x+ x + 2 < 23
=>2x +2 < 23 or x < 10.5
Value of x may be 6 , 8 , 10 (even integers)
The pairs may be (6, 8), (8, 10), (10, 12)
Q2. (70 + 75 + x)/3 ≥ 60
or 145 + x ≥ 180
or x ≥ 35
Q3.Let breadth be b, so its length l = 3b
Thus 2 (l + b) ≥ 320
or 2(3b + b) ≥ 320
or 8b ≥ 320
or b ≥ 40
Therefore, minimum breadth is 40 cm.
Q4. (87 + 92 + 94 + 95 + x)/5 >= 90
or 368 + x >= 450
or x >= 82
Kavita should obtain at least 82 marks in the fifth examination.
Q5. (i) T
(ii) F
Q6. (-∞,52) U (338,∞)
Q7. No solution.
Q8. (11,13 , 13,15 ) and (15,17) .

70
Q9. Between 6.27 and 8.07.
Q10. Let investment of Jitendra be x.
Therefore, investment of Mahendra is 2x and investment of Dharmendra is 2x
+20000.
ATQ,
⇒X+2x+2x+20000 ≥ 10000000
⇒5x +20000 ≥ 10000000
⇒5x ≥ 9980000
⇒ x ≥ 9980000/5
⇒ x≥ 1996000
Hence investment of Mahendra is 2×1996000 = 3992000
Case based questions
Q1. a) 9.8km& 13.8km
b) x≤ -1913
c) (-5,5)
d) d
Q2. a) x< 144°F
b) 72< F< 82
c) 100 <x < 250
Q3.a) 90 ≤ IQ ≤ 150
=>90 ≤MA15X100≤150
=>90X15100≤MA≤150X15100
=>272≤MA≤452
b) 9 ≤ MA ≤ 15
=> 75 ≤ IQ ≤ 125
Q4. (a) 26000 + 30x
b) 43x
c) more than 2000
d) 2600
Q5. a) {1,2,3,4}
b) {-3,-2,-1,0,1,2,3}

71
Long answer type
Q1. T = 30 + 25(x-3), 3 ≤ x ≤ 15
=> 155 < T < 205
=>155 <30 + 25(x-3)< 205
=>…………..
=>8 < x < 10
Hence, at the depth 8 to 10 km temperature lies between 155° C and 205°C.
Q2. Let x liters of 30% conc. H2 SO4acid solution is required to be added. Then Total mixture = (x + 200)
litres. Therefore 30% x + 10% of 200 > 15% of (x + 200) and 30% x + 10% of 200 < 20% of (x + 200)
………………………..
or
200/3< x< 200
Q3. 30< 59(F-32)<35
=>54 < F-32 < 63
=> 86 < F< 95
Q4. Let x cm be the length of the shortest side of the triangle.
∴ According to the question, length of the longest side = 3x cm
Length of the third side = (3x – 2) cm
The least perimeter of the triangle = 61 cm (given)
Thus, x + 3x + (3x – 2) cm ≥ 61 cm
= >7x – 2 ≥ 61
=> 7x ≥ 63
Dividing by 7 on both sides, we get;
=> 7x/7 ≥ 63/7
=> x ≥ 9
Hence, the minimum length of the shortest side will be 9 cm.
Q5. Let x litres of 30% acid solution be required to be added.
Total mixture = (x + 600) litres
Thus, 30% x + 12% of 600 > 15% of (x + 600)
and
30% x + 12% of 600 < 18% of (x + 600)
⇒ (30x/100) + (12/100) × (600) > (15/100) (x + 600)
And

72
(30x/100) + (12/100) × (600) < (18/100) (x + 600)
⇒ 30x + 7200 > 15x + 9000 and 30x + 7200 < 18x + 10800
⇒ 15x > 1800 and 12x < 3600
⇒ x > 120 and x < 300,
i.e., 120 < x < 300
Hence, the number of litres of the 30% acid solution will have to be more than 120
litres but less than 300 litres.
SELF PRACTICE QUESTIONS

1. Solve when .

2. Solve when .

3. Solve when .

4. Solve: .when X∈ R

5. Solve: .when X∈ R

Write the solutions in the form of intervals

6 .

7. .

8. Solve: . Show the solution on the number line.

9.Solve : . Show the solution on the number line.


10.If 1/x<0.5 then find solution set of x, where x is a real number.

ANSWER

1. .

2. .

3. .

4. .

5. .

73
6.

7.

8.

9.

10. (−∞,0) U (2,∞)


Q1.In the following questions, a statement of Assertion(A)is followed by a statement of Reason (R). Choose
the correct answer out of the following choices.
(a) Both (A) and (R) are true and (R) is the correct explanation of (A).
(b) Both (A) and (R) are true but (R) is not the correct explanation of (A).
(c) (A) is true but (R) is false.
(d)(A) is false but(R) is true.
. Assertion (A): The solution set of the inequality x-3<2 ,x ∈N is {1,2,3,4,5,6,7,8}.
Reason (R) :Solution set of a inequality in x is set of values of x satisfying the inequality .
Answer :(d)
Q2.In the following questions ,a statement of Assertion(A) is followed by a statement of Reason(R). Choose
the correct answer out of the following choices.
(a) Both (A) and (R) are true and (R) is the correct explanation of (A).
(b) Both (A) and (R) are true but (R) is not the correct explanation of (A).
(c) (A) is true but(R) is false.
(d) (A) is false but(R) is true.
Assertion(A) : For x ∈𝐑, x<-3 then -5x >15
Reason (R): when both sides are multiplied (or divided) by the same negative number then the sign of
inequality reverse.

74
Answer: (a)
Q3. In the following questions ,a statement of Assertion(A) is followed by a statement of
Reason(R). Choose the correct answer out of the following choices.
(a) Both (A) and(R) are true and (R) is the correct explanation of (A).
(b) Both(A)and(R)are true but (R)is not the correct explanation of (A).
(c) (A) is true but(R) is false.
(d) (A) is false but(R) is true.
Assertion(A) : For x ∈ R, and x < 2, then x-2 < 0
Reason (R): A number can be added or subtracted from both side of inequality without
changing the sign of inequality.
Answer: (a)
Q1. Ravi scored 70 and 75 marks in the first two-unit test. Calculate the minimum marks he
should get in the third test to have an average of at least 60 marks.
Ans: Assume that x be the marks obtained by Ravi in the third unit test
It is given that the student should have an average of at least 60 marks.
From the given information, we can write the linear inequality as:
(70+75+x)/3 ≥ 60
Now, simplify the expression:
⇒ (145 +x) ≥ 180
⇒ x ≥ 180 -145
⇒ x ≥ 35
Hence, the student should obtain a minimum of 35 marks to have an average of at least 60 marks.
Q2. 3. The cost and revenue functions of a product are given by C(x) = 20 x + 4000 and R(x) = 60x +2000,
respectively, where x is the number of items produced and sold. How many items must be sold to realise
some profit?
Solution:
Given that,
Cost, C(x) = 20 x + 4000
Revenue, R(x) = 60x + 2000
We know that, profit = Revenue – Cost
Now, substitute the given data in the above formula,
Profit = R(x) – C(x)
Profit = (60x + 2000)-(20 x + 4000)

75
Now, simplify it:
Profit = 60x + 2000 -20x -4000
Profit = 40x – 2000
To earn some profit, 40x – 2000 > 0
⇒40x > 2000
⇒ x>2000/40
⇒ x > 50
Thus, the manufacturer should sell more than 50 items to realise some profit.
Q3. Solve 5x + 6 > 1 when x is real number?
Ans: 5x + 6 > 1
5x > -5
x > -1 ......Dividing by 5
Hence the solution (-1, ∞)
Q4. 50x < 540, where x is the natural number?
Ans: Given: 50x < 540
Dividing both sides by 50
x < 540/50
x < 54/5
Hence the solution set {1,2,3,4,5,6,7,8,9,10}
Q5. Solve the equation, 4x-2 ≤ 6 and 9x+3 ≥ -15.
Ans: 4x-2 ≤ 6 and 9x+3 ≥ -15
First,
4x-2 ≤ 6
2x -1 ≤ 3......divide by 2
2x ≤ 4
x ≤ 2 .........divide 2
Second,
9x+3 ≥ -15
3x+1 ≥ -5 ..........Divide by 3
3x ≥ -6
x ≥ -2 .........divide by 3
From both the solutions, -2 ≤ x ≤ 2
Hence the solution [-2, 2]

76
Q6. Solve the inequalities, 2x−1≤3 and 3x+1≥−5
Ans: 14We are given equations 2x-1 <3 and 3x+1≥-5, so now solving them we have
2x-1< 3
2x≤ 4
x≤ 2
So the solution set for this is (-infinity, 2]
Now,
3x+1≥-5
3x≥ -6
X -≥2
So the solution set for this is [-2, Infinity)
Now the combined solution set for both the equations will be, [-2.2]
Hence, the solution set is [-2,2]..
Q7. Find all pairs of consecutive odd natural numbers both of which are larger than 10 such that
their sum is less than 40.
Ana: Let the two consecutive odd positive integer be xx and x+2x+2.
Both number are smaller than 10 Therefore
x+2<10
Adding −2−2 to both sides,
x<10−2
⇒x<8
Also sum of the two integers is more than 40.
So,x+x+2>40
⇒2x+2>40
adding −2to both sides,
2x>40−2
⇒2x>38
Divided by 2 both sides
x>19
Then the number greater then 10 and less then 19 are the consecutive odd number pair
In (C) (11,13),(13,15) ,(15,17) ,(17,19) is right answer
Q8. solve – 3x + 17 < – 13
Ans: Given,

77
-3x + 17 < -13
Subtracting 17 from both sides,
-3x + 17 – 17 < -13 – 17
⇒ -3x < -30
⇒ x > 10 {since the division by negative number inverts the inequality sign}
⇒ x ∈ (10, ∞)
Q9. Solve: 4x + 3 < 6x +7
Ans: Given,
4x + 3 < 6x + 7
Subtracting 3 from both sides,
4x + 3 – 3 < 6x + 7 – 3
⇒ 4x < 6x + 4
Subtracting 6x from both sides,
4x – 6x < 6x + 4 – 6x
⇒ – 2x < 4 or
⇒ x > – 2 i.e., all the real numbers greater than –2, are the solutions of the given
inequality.
Hence, the solution set is (–2, ∞), i.e. x ∈ (-2, ∞)
Q10. Hari obtained 70 and 75 marks in first two unit tests. Find the number if minimum marks he
should get in the third unit test to have an average of at least 60 marks.
Ans:
(70 + 75 + x ) ≥ 60
or 145 + x ≥ 180
or x ≥ 35

Q1. 1. Solve the inequation 3 x + 17 ≤ 2 1 – x.


Q2. 2.Solve the inequality x+3/ x−2≤ 2
Q3. Find all pair of consecutive odd integers , both are smaller than 18, such that their sum is more than20.
Q4. Solve 3x-5 < x+1.Show the solution on number line.
Q5. Solve the inequation2x+4/x-1≥ 5
Answer:
Q1. x≤ −32. Q2. x ∈ (−∞, 2) ∪ [7, ∞) Q3. 11 and 13, 13 and 15, 15 and 17
Q4. x < 2/3 Q5. 1<x ≤ 3

78
PERMUTATIONS AND COMBINATIONS
CONCEPTUAL NOTES
The study of permutations and combinations revolves around determining the number of different ways of
arranging and selecting objects out of a given number of objects. Before moving to the discussion of
permutations and combinations, we require basic knowledge about some symbols and fundamental principle
of counting which are given below.
Factorial:The continued product of first n natural number is known as ‘n factorial’. It is denoted by n! .
i.e. n! = 1 x 2 x 3 x………………………………..x(n-1) x n
For example,
2! = 2 x 1
3! =3 x 2 x 1
5! = 5 x 4 x 3 x 2 x 1

Fundamental Principle of counting: If an event can occur in m different ways, following which another
event can occur in n different ways, then the total number of occurrence of the events in the order is m x n.

• Permutations: A permutation is an arrangement in a definite order of a number of objects taken


some or all at a time.
The number of permutations of 𝑛 different objects taken 𝑟 at a time, where 0 < 𝑟 ≤ 𝑛 and the
objects do not
repeat is 𝑛(𝑛 − 1)(𝑛 − 2) … (𝑛 − 𝑟 + 1), which is denoted by

𝑛
𝑛!
𝑃(𝑛, 𝑟) 𝑂𝑅 𝑃𝑟 = ,0 ⩽ 𝑟 ⩽ 𝑛
(𝑛 − 𝑟)!

• 𝑛
𝑃0 = 1, 𝑛
𝑃𝑛 =n!
• The number of permutations of n different objects taken r at a time, where repetition is allowed, is
n𝑟 .
• The number of permutations of n objects, where p1 objects are of one kind, p2 are of second kind,
n!
..., pk are of k th kind and the rest, if any, are of different kind is .
p1 !p2 !…p𝑘 !

• The number of permutations of an dissimilar things taken all at a time along a circle is ( n -1)!.
• The number of ways of arranging a distinct objects along a circle when clockwise and anticlockwise
1
arrangements are considered alike is 2 (𝑛 − 1)

79
• The number of ways in which (𝑚 + 𝑛) different things can be divided into two groups containing 𝑚
(𝑚+𝑛)!
and 𝑛 things is .
𝑚!𝑛!

𝑛 𝑛!
Combination of 𝑛 different objects taken 𝑟 at a time, denoted by 𝐶𝑟 = 𝑟!(𝑛−𝑟)!

Important Results:

∎ 𝑛 𝑃𝑟 = 𝑛 𝐶𝑟 𝑟! ,0 ⩽ 𝑟 ⩽ 𝑛
∎ 𝑛 𝐶0 = 1 = 𝑛 𝐶𝑛
∎ 𝑛 𝐶1 = 𝑛 = 𝑛 𝐶𝑛−1
𝑛 𝑛(𝑛−1) 𝑛
∎ 𝐶2 = = 𝐶2
2
𝑛(𝑛−1)(𝑛−3)
∎ 𝑛 𝐶3 = = 𝑛
𝐶𝑛−3
3
𝑛 𝑛
∎ 𝐶𝑟 = 𝐶s ⇒ 𝑟 = 𝑠 or 𝑟 + 𝑠 = 𝑛
𝑛 𝑛
∎ 𝐶𝑟 + 𝐶𝑟−1 = 𝑛+1 𝐶𝑟

MULTIPLE CHOICE QUESTIONS


Q1.In a class there are 8 boys and 10 girls. The teacher wants to select a boy and a girl to represent the class
in a function. The number of ways in which a teacher can make the selection is
(a) 18
(b) 108
(c) 810
(d) 80
Ans (d)
Solution:- The teacher has to select
(i) a boy among 8 boys
(ii) a girl among 10 girls
The first one can be done in 8 ways and second one in 10 ways.
Therefore, by fundamental principle of multiplication, total numbers of ways = 8  10 = 80

Q2.The number of ways in which n distinct objects can be put into two different boxes
(a) n2
(b) 2n
(c) 2n
(d) 2 + n
Ans (b)
80
Solution: Let the two boxes be B1 and B2. We have two choice for each of n object. So totalnumber of ways is
2 × 2 × 2 … … .× 2 = 2𝑛

𝑛times

Q3.The number of ways in which 8 district toys can be distributed among 4 children is
(a) 48
(b) 84
(c) 8 P4
(d) 32
Ans: (a)
Solution: Each toy can be distributed in 4 ways so total number of ways = 4  4 4 4 4 4 4 4 = 48
Q4.How many 5-digits number can be formed using the three digits 0, 1 and Z, is
(a) 34
(b) 2  34
(c) 35
(d) 3  24
Ans: (b)
Solution: As 0 cannot be filled at first place and remaining each place can be filled in 3 ways
2  3 3 3 3

So, required number of numbers = 2  34


Q5.In how many ways a committee consisting of 3 men and 3 women can be selected from 5 men and 7
women?
(a) 45
(b)4200
(c) 210
(d) 350
Ans (d)
Solution: out of 5 men, 3 can be chosen in5C3 ways. Out of 7 women, 3 can be chosen in 7C3 ways
So committee can be chosen 5C3 x 7C3 = 350 ways.
Q6.The number of signals that can be sent by 4 flags of different colours taking one or more at a time is
(a) 64
(b) 24
(c) 16
(d) 74

81
Ans (a)
Number of signals using one flag =4P1 = 4
Number of signals using two flag = 4 P2 = 12

Number of signals using three flag = 4 P3 = 24

Number of signals using four flag = 4 P4 = 24

So, total number of signal = 4 P1 + 4 P2 + 4 P3 + 4 P4


= 4 + 12 + 24+ 24 = 64
Q7.The number of different seven-digit numbers that can be written using only the three digits 1, 2 and 3
with the condition that the digit 2 occurs twice in each number is
(a) 7 P2  25
(b) 7C2 .25
(c) 7C2 . 52
(d) 7 p2  25
Ans (b)
Solution: We can choose any two of the seven digits (in seven digits number). This can be done in 7 c2 ways.
Put 2 in these two digits. The remaining 5 places can be filled using 1 and 3 in 25 ways.
So, required number of numbers = 7C2 . 25
Q8.If 18 cr =18 cr −10 , then 16 cr is equal to
(a) 820
(b) 420
(c) 116
(d) none of these
Ans: (d)
Solution: We have 18 cr =18 cr −10

 r + r – 10 = 18
 2r – 10 = 18
If nCa = nCb either a=b or a+b = n
 r = 14
16  15
 16 cr = 16 c14 = 16
c2 = = 120
2!
Q9.A gentleman has 5 friends to invite. In how many ways can be send invitation cards to them, if he has 2
servants to carry the cards?

82
(a) 25
(b) 52
(c) 10
(d) none of these
Ans (a)
Solution: As a card can be sent by any of the two servants, so the number of ways of sending the invitation
card to the first friend = 2. Similarly, invitation card can be sent to each of the 5 friends in 2 ways
 Required number of ways = 2  2 2  2  2 = 25
Q10.How many different words, each containing 2 vowels and 2 consonant can be formed with 5 vowel
and 17 consonants?
(a) 5 c2  17 c2

(b) 5 c2  4!
(c) 17C2 x 4!
(d) 5 c2  17 c2  4!
Ans (d)
Solution: There are 5 vowels and 17 consonant. Two vowels out of 5 vowels, 3 consonants out of 17
consonants can be chosen in 5 c2  17 c2 ways. Now, we have 4 letters to arrange which can be done in 4! Ways

So total numbers of words = 5 c2  17 c2  4!

ASSERTION REASON BASED QUESTIONS


In the following question ,a statement of assestion (A) is followed by a statement of Reason (R).choose
the correct answer out of the following choices.
a) Both A and R are true and R is the correct explanation of A.
b) Both A and R are true but R is not correct explanation of A.
c) A is true but R is false.
d) A is false but R is true.
Q1.Assestion (A): 12C = 12C
2 10

Reason (R): Selection of the ɤ distinct thing out of n is equal to the rejection of (n-ɤ) thing out of n.
Ans-(a)
Solution:- Attributing to the reason, selection of 2 distinct thing out of 12,is equal to rejection of (12-
2=10) thing out of 12 12C = 12C 2 10

Q2.Assertion (A): 3! +4! = 7!


83
Reason (R): For any positive integer n, n! = 123…..n and 0! =1
Ans-(d)
Solution - (R) is the definition of factorial which is true but assertion is false.
As 3! + 4! = ( 3  2 1) + ( 4  3  2 1)

= 6 + 24 = 30
7! = 7  6  5  4  3  2 1  30
Q3.Assertion (A):-For any natural number n, nC = 1 n

n!
Reason (R): nCr =
(n − r )!

Ans (c)
n! n! n!
Solution: nCr =  nCn = = =1
(n − r)!r! (n − n)!n! n !

Q4.Assertion (A): nPr = nCr  r! 0 < 𝑟 ≤ 𝑛

Reason(R): For each combination of nCr , there are r! permutations.


Ans (a)
Solution: r objects in every combination can be arranged in r! ways. nPr = nCr  r! 0 < 𝑟 ≤ 𝑛

Q5.Assertion (A):-The number of ways of choosing 4 cards from a pack of cards is 52C 4

Reason (R): permutation is an arrangement in a definite order of a number of objects taken some or all at a
time.
Ans (b)
Solution: Selection of 4 card out of 52 can be made in 52C ways. And (R) is the definition for permutation
4

(b) is correct

SHORT ANSWER TYPE QUESTIONS


Q1. How many different words can be formed by using all the letters of the word ALLAHABAD?
Sol :-There are 9 letters in given word ,of which 4 are A’&,2 are L’&. So total number of words is the
number of arrangements of 9 things of which 4 are similar of one kind, two are similar of one kind 
9!
Total number of words = specific.
4!2!

84
Q2. There are 10 points on a circle. How many chords can be drawn by using these points?
Sol:- One chord can be drawn by using two points. So total number of chords = number of ways of
choosing two points out of 10.i.e 10C 2

Q3. Find the total number of five-digit numbers that can be formed by using the digits 0,1,2,3, ….,9
Sol:- Total number of five digit numbers 910101010=90000
As 0 cannot be used as first place so it has 9 choices and rest all 4 places can be filled in 10 ways.
Q4. At an election, a voter may vote for any number of candidates, not greater than the number to be
elected there are 8 candidates and 3are to be elected. If a voter votes for at least one candidate, then
find number of ways in which he can vote?
Sol:-The number of ways in which a voter can vote is
8C1 + 8C2 + 8C3 = 8 + 28 + 56 = 92

Q5. Find the total number of ways of answering 6 objective type question, each question having 4choies:
Sol:-since each question can be answered in 4 ways ,So total number of ways of answering 6 question is
444444 = 46
Q6. There are 3 candidates for a classical ,4 for a social science and 2 for a natural science scholarship In
how many ways can these scholarship be awarded?
Sol:-Since, classical scholarship can be awarded to any one of three candidates. So awarding the classical
scholarship.
Similarly the social science and natural science scholarship can be awarded in 4 and 2 ways respectively.
So number of ways 342=24
Q7. A bag contains 4 black and 3 red balls determine the number of ways in which 2 black and 2 red balls
can be selected
Sol:-Out of 4 black balls ,2 black balls can be selected in 4C ways and out of 3 red balls ,2 red balls can be
2

selected in 3C ways
2

Total number of ways = 4C  3C = 6  3 = 18


2 2

Q8. In how many ways can a student choose programme of 4 courses if 8courses are available and 2
specific courses are compulsory for every student?
Sol:- As 2 courses are compulsory for every student. Therefore, every student has to choose 2 courses out
of remaining 6 courses this can be done in 6C = 15 ways.
2

Q9. There are 8 true –false questions in an exam. Find the number of ways in which these questions can
be answered
Sol:-Each question cab be answered in two ways . So total number of ways of answering 8question is

85
2 × 2 × … … × 2, = 28 = 256

8𝑡𝑖𝑚𝑒𝑠

Q10.If there are 10 persons in a room Everybody shakes hands with everybody else How many
handshakes will be there?
Sol:-Two hands are required for a handshakes Therefore total number of handshakes = 10C = 45
2

CASE STUDY BASED QUESTIONS

Q.1 A state cricket authority has to choose a team of 11 members, to do it so the authority ask 2 coaches
of a government academy to select the team Members that have experience as well as best performin the
last 15 matches. They can make up a team of 11cricketers amongst 15 possible candidates in which 5
players can bowl.

Based on the information answer the following

(i) In how many ways can the final eleven be selected from 15 cricket players?

(ii) In how many ways can the final eleven be selected if exactly 4 bowlers must be included.

(iii)In how many ways can the final eleven be selected if all bowlers must be included.

Solution. (i) 15C11= 1365

(ii) 4 bowlers can be select by = 5C4

Remaining 7 players can be select out of 10 (15-5) is = 10C7

Total no. of ways is = 5C4,10C7 = 600

(iii) all 5 bowlers can be select by = 5C5

Remaining 6 players can be select out of 10 (15-5) is = 10C6


86
Total no. of ways is = 5C5 ,10C6 = 210

Q 2:In an examination, a question paper consists of 12 questions divided into two parts i.e.,Part land Part
II, containing 5 and 7 questions, respectively. A student is required to attempt 8 questions in all, selecting
at least 3 from each part

. Based on the information answer the following

In how many ways can a student select the questions in such a way that

(a) 3 questions from part I and 5 questions from part II

(b) 4 questions from part I and 4 questions from part II

(c) 5 questions from part I and 3 questions from part II

d) In how many ways can a student select the questions

Answer:It is given that the question paper consists of 12 questions divided into two parts - Part I and Part
II, containing 5 and 7 questions, respectively.

A student has to attempt 8 questions, selecting at least 3 from each part.

This can be done as follows.

a)3 questions from part I and 5 questions from part II can be selected in 5C37C5ways.

b) 4 questions from part I and 4 questions from part II can be selected in 5C47C4ways.
87
c) 5 questions from part I and 3 questions from part II can be selected in 5C57C3 ways.

d) Thus, required number of ways of selecting questions

= 5C3  7C5 + 5C4  7C4 + 5C5  7C3

5 7 5 7 5 7
=  +  + 
23 25 41
  43 50 34

= 210 + 175 + 35 = 420

Q3. The number of different words that can be formed from the letters of the wordINTERMEDIATE such
that two vowels never come together is_________.

6
[Hint: Number of ways of arranging 6 consonants of which two arc alike is and number of ways of
2
1 1
arranging vowels = 7 P6   ]
3 2

Solution. Letters of the word INTERMEDIATE are: Vowels (I, E, E, I, A, E) and consonants (N, T, R, M, D, T)

Now we have to arrange these letters if no two vowels come together.

6
So, first arrange six consonants in ways.
2

Arrangement of six consonants creates seven gaps.

6
Six vowels can be arranged in these gaps in 7C6  ways.
23

6 7 6
So, total number of words  C6  = 360  7  60 = 151200
2 23

Que4. Four persons entered the lift cabin on the ground floor of 7 floor house suppose each of them can
leave the cabin independently at any floor beginning with the first. What is. The total number of ways in
which each of the four persons can leave the cabin at any of the six floor?

88
Solution: suppose that A, B, C, D are 4 persons a can leave the cabin at any of the 6 floors so A can leave
the cabin in 6 ways, similarly each of B,C and D can leave the cabin in 6 ways therefore the total number of
ways in which each of the 4 persons can leave the cabin at any of the six floor is 6  6  6  6= 1296.

Que5.Three married couples are to be seated in a row having six seats in a cinema hall. If spouses are to
be seated next to each other, in how many ways can they be seated? Find also the number of ways of their
seating if all the ladies sit together.

Solution: Let us denote married couples by S1, S2, S3 where each couple is considered to be a single unit
as shown in the following figure:

Then the number of ways in which spouces can be seated next to each other is 3! = 6 ways.

Again each couple can be seated in 2! ways. Thus the total number of seating arrangement so that spouces
sit next to each other = 3!  2!  2!  2! = 48.

Again, if three ladies sit together, then necessarily three men must sit together.

Thus, ladies and men can be arranged altogether among themselves in 2! ways.

Therefore, the total number of ways where ladies sit together is 3! 3!  2! = 144.

Long Answer Type questions

Q1. In how many ways can final eleven be selected from 15 cricket players’ if

89
(i) there is no restriction

(ii) one of then must be included

(iii) one of them, who is in bad form, must always be excluded

(iv) Two of then being leg spinners, one and only one leg spinner must be included?

Ans. (i) 11 players can be selected out of 15 in 15C11 ways

15  14  13  12
= 15C11 , ways = ways=1365 ways
1 2  3  4

(ii) Since a particular player must be included, we have to select 10 more out of remaining 14 players.

This can be done in 14C10 ways 14C4 ways

14  13  12  11
= ways = 1001 ways
1 2  3  4

(iii) Since a particular player must be always excluded, we have to choose 11 ways out of remaining 14

This can be done in 14C11 ways = 14C3 ways

14  13  12  11
= ways = 364 ways.
1 2  3

(iv) One leg spinner can be chosen out of 2 in 2C1 ways. Then we have to select 10 more players get of 13

(because second leg spinner can’t be included). This can be done in 13C10 ways of choosing 10 players. But

these are 2C1 ways of choosing a leg spinner, therefore, by multiplication principle of counting the required
number of ways

= 2C1  13C10

2 13  12  11
= 2C1  13C3 =  = 572
1 1 2  3

90
Q2. A bag contains six white marbles and five red marbles. Find the number of ways in which four marbles
can be drawn from the bag, if (i) they can be of any colour (ii) two must be white and two red and (iii) they
must all be of the same colour.

Ans: Total number of marbles = 6 white +- 5 red = 11 marbles

(a) If they can be of any colour means we have to select 4 marbles out of 11

Required number of ways = 11C4

(b) Two white marbles can be selected in 6C2

Two red marbles can be selected in 5C2 ways.

Total number of ways = 6C25C2 = 15  10 = 150

(c) If they all must be of same colour,

Four white marbles out of 6 can be selected in 6C4 ways.

And 4 red marbles out of 5 can be selected in 5C4 ways.

Required number of ways = 6C4 + 5C4 = 15 + 5 = 20

Q3. A group consists of 4 girls and 7 boys. In how many ways can a team of 5 members be selected, if the
team has

(i) no girls

(ii) at least one boy and one girl

(iii) at least three girls

Ans: Number of girls = 4;

Number of boys = 7

We have to select a team of 5 members provided that

91
(i). Team having no girls

7×6
Required number of ways = 7C5 = = 21
2!

(ii) Team having at least one boy and one girl

A Required number of ways

= 7C1  4C4 + 7C2  4C3 + 7C3  4C2 + 7C4  4C1

= 7 1 + 21 4 + 35  6 + 35  4 = 7 + 84 + 210 + 140 = 441

(iii) Team having at least three girls

Required number of ways = 4C3  7C2 + 4C4  7C1

= 4  21 + 7 = 84 + 7 = 91

Q4.The number of 6-digit numbers that can be made with the digits 0,1,2,3,4 and 5 so that even digits

occupy odd places, is

SOLUTION: There are 3 odd and 3 even places. Three odd places viz. first, third and fifth can be occupied

by even digits 0,2 and 4 in 3! ways. But, 0 cannot occupy the first place from the left. Therefore, the

number of ways in which 3 odd places can be filled is (3! - 2!). Three even places can be filled in 3! ways.

Hence, total number of six digit numbers = (3! - 2!)  3! = 24.

Q5.The number of words that can be made by down the letters of the word CALCULATE such that each

word starts and ends with a consonant, is

SOLUTION: Required number of words

= Number of words which begin and end with C

+ Number of words which begin and end with L

92
+ Number of words which begin with C and end with L

+ Number of words which begin with L and end with C

+ Number of words which begin with C and end with T

+ Number of words which begin with T and end with C

+ Number of words which begin with L and end with T

+ Number of words which begin with T and end with L

7 7 7 7 7 7 7 7
= + + + + + + +
22 22 2 2 22 22 22 22

7 7 5  7
= 6 + 2 =
22 2 2

SELF-PRACTICE QUESTIONS

MULTIPLE CHOICE QUESTIONS

Q1. The number of 5-digit number that can be made using the digit 1 and 2 and in which at least one digit
is different is

(a) 30 (b) 31

(c) 32 (d) 33

Q2. Total number of 6-digit number in which all the odd digits appear ,is

5
(a)  6 (b) 6
2

6
(c) (d) none of these
2

Q3. The number of all permutations of n different objects taken ɤ at a time when a particular object is
never taken in each arrangement, is

93
(a) n −1
Cr  ( r − 1) (b) n −1
Cr −1  r

n −1
(c) Cr  r (d) nCr  r

Q4. How many diagonals are there in a polygon with n sides?

n ( n − 1) n ( n − 3)
(a) (b)
2 2

n ( n + 1) n ( n + 2)
(c) (d)
2 2

Q5. If n Pr = 720 nCr then the value of ɤ is

(a) 5 (b) 4

(c) 6 (d) 7

Q6. If nC12 = nC8 then n is equal to

(a) 12 (b) 6

(c) 30 (d) 20

Q7. The number of triangles that are formed by choosing the vertices from a set of 12 point ,7 of which lie
on same line, is

(a) 15 (b) 175

(c) 185 (d) 105

Q8. The number of ways in which a team of 11 players can be selected from 22 players always including 2
of them and excluding 4 of them is

(a) 16C11 (b) 16C5

(c) 16C9 (d) 20C9

Q9. The total number of five digit numbers with atleast one repeating digit is
94
(a) 27218 (b) 15120

(c) 90000 (d) 62784

Q10. The number of ways in which n distinct objects can be put into two different boxes is

(a) n 2 (b) 2 n

(c) 2n (d) none of these

ASSERTION REASON BASED QUESTIONS

In the following questions, a statement of Assertion (A) is followed by a statement of Reason (R). Choose
the correct answer out of the following Choices.

(a) Both (A) and (R) are true and (R) is the correct explanation of (A).

(b) Both (A) and (R) are true but (R) is not the correct explanation of (A).

(c) (A) is true but (R) is false.

(d) (A) id false but (R) is true.

Q1.Assertion: The number of rectangles on a chess board is 8C2  8C2

Reason: To from a rectangle, we have to select any two of horizontal line and any two of the vertical line

Q2.Assertion: The number of ways of ways of distributing 10 identical balls in 4 distinct boxes such that no
box is empty is 9C3

Reason: The number of ways of choosing any 3 places from 9 different places is 9C3

Q3. Assertion: The number of ways in which n different prizes can be distributed among m(< 𝑛) person if
each is entitled to receive at most (n-1) prizes is mn − m

Reason: Required number of ways=

Total number of ways ( mn ) –number of ways in which one gets all prizes (m)

95
Q4.Assertion; If 15𝐶3𝑟 = 15
𝐶𝑟+3,then r = 4

Reason: nCa = nC6  either a = b or a + b = n

Q5.Assertion; Number of lines formed by joining n points on a circle is n(n − 1) / 2

Reason: C = ( n,3) = n(n − 1) / 2

SHORT ANSWER TYPE QUESTIONS

Q1. In a city, all telephone numbers have six digit, the first two digit always being 41 or 42 or 46 or 62 or
64. How many telephone numbers have all six digit distinct?

Q2. Out of 18 points in a plane, no three are in the same line except five points which are collinear find the
number of lines that can be formed joining these points

Q3. There are 5 lamps in a hall, each of them can be switched on independtly. Find the number of ways in
which hall can be illuminated.

Q4. How many committee of 5 persons (including a chairperson) can be selected form 12 persons?

Q5. The number of ways in which n distinct balls can be put into 3 boxes so that no two boxes remain
empty, is __

Q6. A box contains two white, three black and four red balls. Find the number of ways in which three balls
can be drawn from the box if atleast one black ball is to be included in the draw.

Q7. Out of all possible permutations of the letters of the word ENDEANOEL, find the number of
permutations in which letters A,E,O occur only in odd positions.

Q8. Given 1 2 flags of different colours, how many different signals can be generated if each signalrequires
the use of 2 flags, one below the other?

Q9. There are four bus routes between A and B; and three bus routes between B and C. A man cantravel
round-trip in number of ways by bus from A to C via B. If he does not want to use a bus route more than once,
in how many ways can he make round trip?

96
Q10. In an examination there are three multiple choice questions and each question has 4 choices.

Find the number of ways in which a student can fail to get all answer correct.

CASE STUDY BASED QUESTIONS

Q1. In a company, CEO wants to establish a new branch. New branch required a committee of 5 members
is to be formed out of 6 gents and 4 ladies.

In how many ways this can be done, when

(i) At least two ladies are included?

(ii) II. At most two ladies are included?

Q2.Read the following passage and answer the questions given below.

Every person has Independence thought.

Find the number of arrangements of the letters of the word INDEPENDENCE. In how many of these
arrangements,

(i) Do the words start with P


(ii) Do all the vowels always occur together
(iii) Do the vowels never occur together
OR
Do the words begin with I and end in P?

97
Q3. Read the following passage and answer the question given below.

The longest river of North America is Mississippi river

In how many ways can the letters of the word MISSISSIPPI be arranged such that

(i) AII letters are used

(ii) All I’s are together

(iii) All I's are not together

OR

All S 's are not together

Q4. A bag contains six white marbles and five red marbles. four marbles can be drawn from the bag .

Based on the information answer the following .

Find the number of ways in which

(a) They can of any color

98
(b) Two must be white and two red

(c) They must be all of same color

Q5.Read the following passage and answer the question given below.

In a village, there are 87 families of which 52 families have at most 2 children.

In a rural development program, 20 families are to be helped chosen for assistance. In how many ways can
the choice be made in which

a) At least 18 families must have at most 2 children


b) All the families must have at most 2 children

LONG ANSWER TYPE QUESTIONS

Q1. If nCr −1 = 36; nCr = 84 and nCr +1 = 126 then Find the value of r C2

Q2. Find the number of ways in which a miXed double game can be arranged from amongst nine married
couples if no husband and wife play in the same game.

Q3. What is the number of ways of choosing four cards from a pack of 52 playing cards? In how many of there
(a) Four cards one of the same suit
(b) Four cards belong to four different suits
(c) Are face cards.
(d) Two are red cards & two are black cards.
(e) Cards are of the same colour?
Q4. If n Pr = n Pr +1 and n cr = n cr −1 find the value of n and r

99
Q5. How many four letter words can be formed using the letters of the letters of the word ‘FAILURE’ so that

(i) F is included in each word

(ii) F is excluded in each word.

Answer key for self-practice questions

Multiple choice questions

Q1. (a)

Q2. (a)

Q3. (c)

Q4. (b)

Q5. (c)

Q6. (d)

Q7. (c)

Q8. (c)

Q9. (d)

Q10. (b)

Short-answer questions

Q1. 8400 Q2. 144 Q3. 31 Q4. 3960 Q5. 3 n-3

Q6. 64 Q7. 2 x 5! Q8. 132 Q9. 72 Q4. 63

Assertion-Reason based questions

Q1. (d) Q2. (b) Q3. (a) Q4. (d) Q5. (c)
100
Case -Study based questions

Q1. (i) 186 (ii) 186

Q2. (i) 138600 (ii) 16800 (iii) 1646400

Q3. (i) 34650 (ii) 840 (iii) 33810

Q4. (i) 330 (ii) 150 (iii) 20

Q5. 52C x 35C2 + 52C19 x 35C1 + 52C20


18

Long -answer type questions

Q1. 3

Q2. 3024

Q3. (i) 2860 (ii) (13)4 (iii) 495 (iv) 105625

Q4. n = 3, r = 2

Q5. (i) 480 (ii) 360

101
BINOMIAL THEOREM
MAIN CONCEPTS AND RESULTS
An algebraic expression containing two terms is called binomial expression
1
For example, (2𝑥 − 𝑦), (𝑥 + 𝑦), (𝑥 + 𝑥) etc.
The general form of binomial expression is (𝑥 + 𝑎) and the expansion of (𝑥 + 𝑎)𝑛 , 𝑛𝜖𝑁 is called Binomial
theorem.
We have learnt that:
(𝑥 + 𝑎)0 = 1
(𝑥 + 𝑎)1 = 𝑥 + 𝑎
(𝑥 + 𝑎)2 = 𝑥 2 + 2𝑎𝑥 + 𝑎2
(𝑥 + 𝑎)3 = 𝑥 3 + 3𝑥 2 𝑎 + 3𝑥𝑎2 + 𝑎3
(𝑥 + 𝑎)4 = 𝑥 4 + 4𝑥 3 𝑎 + 6𝑥 2 𝑎2 + 4𝑥𝑎3 + 𝑎4
We observe that the coefficients in the above expansion follow a particular pattern as
Index of the binomial Coefficient of various terms
0 1
1 1 1
2 1 2 1
3 1 3 3 1
4 1 4 6 4 1
In the above pattern it can be seen that the addition of 1’s in the row for the index 1 gives rise to 2 in the
row for index 2. The addition of 1,2 and 2,1 in the row for index 2, gives rise to 3 and so on. This can be
continued to any index.
Pascal’s Triangle
Index Coefficient
0 1
1 1 𝛁 1
2 1 𝛁 2 𝛁 1
3 1 𝛁 3 𝛁 3 𝛁 1
4 1 𝛁 4 𝛁 6 𝛁 4 𝛁 1
The above structure looks like triangle with 1 at the top vertex and running down the two slanting sides.
This array of numbers is known as Pascal’s triangle.
Expansion for higher powers of binomial are also possible by using Pascal’s triangle but for that we have to
find the coefficient for the higher index which is time consuming
Here use of combination to rewrite the numbers of Pascal’s triangle.
𝑛!
We know that 𝐶(𝑛, 𝑟) = 𝑟!(𝑛−𝑟)! ; 0 ≤ 𝑟 ≤ 𝑛, where 𝑛 is non negative integer.
𝐶(𝑛, 0) = 𝐶(𝑛, 𝑛) = 1
Index Coefficient
0 𝑪(𝟎. 𝟎)
1 𝑪(𝟏, 𝟎) 𝑪(𝟏, 𝟏)
2 𝑪(𝟐, 𝟎) 𝑪(𝟐, 𝟐) 𝑪(𝟐, 𝟐)
3 𝑪(𝟑, 𝟎) 𝑪(𝟑, 𝟏) 𝑪(𝟑, 𝟐) 𝑪(𝟑, 𝟑)

102
4 𝑪(𝟒, 𝟎) 𝑪(𝟒, 𝟏) 𝑪(𝟒, 𝟐) 𝑪(𝟒, 𝟑) 𝑪(𝟒, 𝟒)
Observing this pattern, we can now write the rows of Pascal’s triangle for any index
Binomial theorem for any positive integer n:
(𝑎 + 𝑏)𝑛 = 𝐶(𝑛. 0)𝑎𝑛 + 𝐶(𝑛, 1)𝑎𝑛−1 𝑏 + 𝐶(𝑛, 2)𝑎𝑛−2 𝑏 2 + 𝐶(𝑛, 3)𝑎𝑛−3 𝑏 3 + ⋯ + 𝐶(𝑛, 𝑛)𝑏 𝑛
Some important conclusions from the Binomial Theorem:
i. (𝑥 + 𝑎)𝑛 = ∑𝑛𝑟=0 𝐶(𝑛, 𝑟)𝑥 𝑛−𝑟 𝑎𝑟
Or (𝑥 + 𝑎)𝑛 = 𝐶(𝑛. 0)𝑥 𝑛 + 𝐶(𝑛, 1)𝑥 𝑛−1 𝑎 + 𝐶(𝑛, 2)𝑥 𝑛−2 𝑎2 + 𝐶(𝑛, 3)𝑥 𝑛−3 𝑎3 + ⋯ + 𝐶(𝑛, 𝑛)𝑎𝑛
ii. The sum of indices of 𝑥 and 𝑎 in each term is n
iii. Since 𝐶(𝑛, 𝑟) = 𝐶(𝑛, 𝑛 − 𝑟), 𝑓𝑜𝑟 𝑟 = 0,1,2,3, … , 𝑛
𝐶(𝑛, 0) = 𝐶(𝑛, 𝑛); 𝐶(𝑛, 1) = 𝐶(𝑛, 𝑛 − 1); 𝐶(𝑛, 2) = 𝐶(𝑛, 𝑛 − 2); ..
iv. Replacing 𝑎 𝑏𝑦 − 𝑎
(𝑥 − 𝑎)𝑛 = 𝐶(𝑛. 0)𝑥 𝑛 − 𝐶(𝑛, 1)𝑥 𝑛−1 𝑎 + 𝐶(𝑛, 2)𝑥 𝑛−2 𝑎2 − 𝐶(𝑛, 3)𝑥 𝑛−3 𝑎3 + ⋯ + (−1)𝑛 𝐶(𝑛, 𝑛)𝑎𝑛
i.e (𝑥 − 𝑎)𝑛 = ∑𝑛𝑟=0(−1)𝑛 𝐶(𝑛, 𝑟)𝑥 𝑛−𝑟 𝑎𝑟
v. putting 𝑥 = 1 𝑎𝑛𝑑 𝑎 = 𝑥 in the expansion, we get
(1 + 𝑥)𝑛 = 𝐶(𝑛. 0) + 𝐶(𝑛, 1)𝑥 + 𝐶(𝑛, 2)𝑥 2 + 𝐶(𝑛, 3)𝑥 3 + ⋯ + 𝐶(𝑛, 𝑛)𝑥 𝑛
(1 + 𝑥)𝑛 = ∑𝑛𝑟=0 𝐶(𝑛, 𝑟)𝑥 𝑟
vi. putting 𝑥 = 1 𝑎𝑛𝑑 𝑎 = −𝑥 in the expansion, we get
(1 − 𝑥)𝑛 = 𝐶(𝑛. 0) − 𝐶(𝑛, 1)𝑥 + 𝐶(𝑛, 2)𝑥 2 − 𝐶(𝑛, 3)𝑥 3 + ⋯ + (−1)𝑛 𝐶(𝑛, 𝑛)𝑥 𝑛
(1 − 𝑥)𝑛 = ∑𝑛𝑟=0(−1)𝑛 𝐶(𝑛, 𝑟)𝑥 𝑟
vii. the coefficient of (𝑟 + 1)𝑡ℎ term in the expansion of (1 + 𝑥)𝑛 is 𝐶(𝑛, 𝑟)
viii. the coefficient of 𝑥 𝑟 in the expansion of (1 + 𝑥)𝑛 is 𝐶(𝑛, 𝑟)
ix. if n is odd than ((𝑥 + 𝑎)𝑛 + (𝑥 − 𝑎)𝑛 ) and ((𝑥 + 𝑎)𝑛 − (𝑥 − 𝑎)𝑛 ) both have same number of
𝑛+1
terms equal to ( )
2
𝑛 𝑛
if n is even , than ((𝑥 + 𝑎)𝑛 + (𝑥 − 𝑎)𝑛 ) has ( 2 + 1) terms and ((𝑥 + 𝑎)𝑛 − (𝑥 − 𝑎)𝑛 ) has ( 2)
terms
Example 1:Which of the following represents the binomial theorem for positive integral indices?
A. (𝑎 + 𝑏)𝑛 = 𝑎𝑛 + 𝑏 𝑛 B. (𝑎 + 𝑏)𝑛 = ∑𝑛𝑟=0 𝐶(𝑛, 𝑟)𝑎𝑛−𝑟 𝑏 𝑟

C. (𝑎 + 𝑏)𝑛 = 𝑎𝑛+1 + 𝑏 𝑛+1 D. none of these

Answer: B

Example2: Which of the following is the correct binomial coefficient?


𝑛! (𝑛−𝑟)‼ 𝑟!(𝑛−𝑟)! 𝑛!
A. 𝐶(𝑛, 𝑟) = 𝑟!(𝑛−𝑟)! B. (𝑛, 𝑟) = C. (𝑛, 𝑟) = D. (𝑛, 𝑟) =
𝑛!𝑟! 𝑛! 𝑟!

Answer: A

Example3: What is the value of 𝐶(6,2) ?

A. 10 B. 15 C. 20 D. 30

103
Answer: B

Example4: Which of the following corresponds to Pascal’s triangles coefficients of the expansion (𝑎 + 𝑏)4

A. 1,3,3,1 B. 1,4,6,4,1 C. 1,5,10,10,5,1 D. 1,2,1

Answer: B

Example5: The expansion ((𝑥 + 𝑎)51 + (𝑥 − 𝑎)51 ) has__________ terms after simplification.

A. 50 B. 52 C. 26 D. none of these

Answer : C

Example6: The total number of terms in the expansion ((𝑥 + 𝑎)100 + (𝑥 − 𝑎)100 ) after simplification
is____________

A. 202 B. 51 C. 50 D. none of these

Answer: B

Example7: Number of terms in the expansion (1 − 2𝑥 + 𝑥 2 )7 are _______

A. 14 B. 15 C. 16 D. 17
Answer : B

Example 8. The coefficient of 𝑥 5 in (𝑥 + 3)9 is ___________

A. 𝐶(9,4)32 B. 𝐶(9,4)33 C. 𝐶(9,4)34 D. none of these

Answer: C

Assertion –Reason based questions:


In the following questions a statement of assertion (A) is followed by a statement of Reason (R). Choose the
correct answer out of the following choices
a). Both A and R are true and R is correct explanation of A.
b). Both A and R are true but R is not the correct explanation of A.
c). A is true but R is false.
d). A is false but R is true
Example1: Assertion(A) : The sum of the coefficients in the expansion (𝑥 + 𝑦)𝑛 is 2𝑛
Reason (R ): The binomial coefficients in the expansion of (𝑥 + 𝑦)𝑛 represent the elements in the nth row of
Pascal’s triangle.
Answer: A
Example 2: Assertion (A): in the expansion (𝑥 + 𝑦)𝑛 , the coefficient of 𝑥 𝑛−𝑘 𝑦 𝑘 is given by 𝐶(𝑛, 𝑘)
𝑛!
Reason( R ): The binomial coefficient 𝐶(𝑛, 𝑘) is equal to 𝑘!(𝑛−𝑘)!
Answer: A
Example 3: Assertion (A) : Pascal’s triangle can be used to find the coefficients in the expansion (𝑎 + 𝑏)5
Reason ( R ): The nth
104
Example 4: Assertion( A) : the number of terms I the expansion of {(3𝑥 + 𝑦)8 − (3𝑥 − 𝑦)8 } is 4
𝑛
Reason ( R ) : if n is even, than ((𝑥 + 𝑎)𝑛 − (𝑥 − 𝑎)𝑛 ) has ( ) terms
2

Answer: A
Example 5: Assertion ( A) : the coefficient of the expansions are arranged in an array. This array is called
Pascal’s triangle.
Reason ( R ): There are 11 terms in the expansion {(3𝑥 + 4𝑦)10 + (3𝑥 − 4𝑦)10 }
Answer: B
Short Answer Type Questions:
Example 1: Expand (2𝑥 − 3𝑦)4 by Binomial theorem
Answer: using
(𝑥 − 𝑎)𝑛 = 𝐶(𝑛. 0)𝑥 𝑛 − 𝐶(𝑛, 1)𝑥 𝑛−1 𝑎 + 𝐶(𝑛, 2)𝑥 𝑛−2 𝑎2 − 𝐶(𝑛, 3)𝑥 𝑛−3 𝑎3 + ⋯ + (−1)𝑛 𝐶(𝑛, 𝑛)𝑎𝑛
=16𝑥 4 − 96𝑥 3 𝑦 + 216𝑥 2 𝑦 2 − 216𝑥𝑦 3 + 81𝑦 4
Example 2: Expand (𝑥 2 + 2𝑎 )5 by Binomial theorem.
Answer: using
(𝑥 + 𝑎)𝑛 = 𝐶(𝑛. 0)𝑥 𝑛 + 𝐶(𝑛, 1)𝑥 𝑛−1 𝑎 + 𝐶(𝑛, 2)𝑥 𝑛−2 𝑎2 + 𝐶(𝑛, 3)𝑥 𝑛−3 𝑎3 + ⋯ + 𝐶(𝑛, 𝑛)𝑎𝑛
=𝑥10 + 10𝑥 8 𝑎 + 40𝑥 6 𝑎2 + 80𝑥 4 𝑎3 + 80𝑥 2 𝑎4 + 32𝑎5
Example 3: which is larger (1.01)100000 or 10000
Answer : (1.01)100000 = (1 + 01)100000
= 𝐶(100000,0) + 𝐶(100000.1)(0.01) + 𝑜𝑡ℎ𝑒𝑟 𝑝𝑜𝑠𝑖𝑡𝑖𝑣𝑒 𝑡𝑒𝑟𝑚𝑠
=1 + 100000(0.01) + 𝑜𝑡ℎ𝑒𝑟 𝑝𝑜𝑠𝑖𝑡𝑖𝑣𝑒 𝑡𝑒𝑟𝑚𝑠 = 1 + 10000 + 𝑜𝑡ℎ𝑒𝑟 𝑝𝑜𝑠𝑖𝑡𝑖𝑣𝑒 𝑡𝑒𝑟𝑚𝑠
> 10000
So, (1.01)100000 > 10000
Example 4: using binomial theorem prove that 6𝑛 − 5𝑛 always leaves reminder 1 when divided by 25
Answer : 6𝑛 = (1 + 5)𝑛 = 1 + 𝐶(𝑛, 1)5 + 𝐶(𝑛, 2)52 + 𝐶(𝑛, 3)53 + ⋯ + 5𝑛
= 1 + 5𝑛 + 52 (𝐶(𝑛, 2) + 𝐶(𝑛, 3)5 + ⋯ + 5𝑛−2)
6𝑛 − 5𝑛 − 1 = 25𝑘( 𝑤ℎ𝑒𝑟𝑒 𝑘 = 𝐶(𝑛, 2) + 𝐶(𝑛, 3)5 + ⋯ + 5𝑛−2 )
Hence proved.
Long questions
Example-1: find the coefficient of 𝑥 5 in the expansion of the product (1 + 2𝑥)6 (1 − 𝑥)7
Answer (1 + 2𝑥)6 (1 − 𝑥)7 = (1 + 𝐶(6,1)(2𝑥) + 𝐶(6,2)4𝑥 2 + 𝐶(6,3)8𝑥 3 + 𝐶(6,4)16𝑥 4 + 𝐶(6,5)32𝑥 5 +
𝐶(6,6)64𝑥 6 )(1 − 𝐶(7,1)𝑥 + 𝐶(7,2)𝑥 2 − 𝐶(7,3)𝑥 3 + 𝐶(7,4)𝑥 4 − 𝐶(7,5)𝑥 5 + 𝐶(7,6)𝑥 6 − 𝐶(7,7)𝑥 7 )
= (1 + 12𝑥 + 60𝑥 2 + 160𝑥 3 + 240𝑥 4 + 192𝑥 5 + 64𝑥 6 )(1 − 7𝑥 + 21𝑥 2 − 35𝑥 3 + 35𝑥 4 − 21𝑥 5 + 7𝑥 6
− 𝑥7)
Coefficient of 𝑥 5 is
= 1(−21) + 12(35) + 60(−35) + 160(21) + 240(−7) + 192(1)
= 171
4 4
Example 2: find (𝑎 + 𝑏)4 − (𝑎 − 𝑏)4. Hence evaluate (√3 + √2) − (√3 − √2)
Answer: (𝑎 + 𝑏)4 − (𝑎 − 𝑏)4 = (𝐶(4,0)𝑎4 + 𝐶(4,1)𝑎3 𝑏 + 𝐶(4,2)𝑎2 𝑏 2 + 𝐶(4,3)𝑎𝑏 3 + 𝐶(4,4)𝑏 4 ) −
(𝐶(4,0)𝑎4 − 𝐶(4,1)𝑎3 𝑏 + 𝐶(4,2)𝑎2 𝑏 2 − 𝐶(4,3)𝑎𝑏 3 + 𝐶(4,4)𝑏 4 )
= 2(𝐶(4,1)𝑎3 𝑏 + 𝐶(4,3)𝑎𝑏 3 ) = 2(4𝑎3 𝑏 + 4𝑎𝑏 3 ) = 8𝑎𝑏(𝑎2 𝑏 2 )
4 4
(√3 + √2) − (√3 − √2) , put 𝑎 = √3 and𝑏 = √2
4 4
(√3 + √2) − (√3 − √2) = 8√3√2(3 + 2) = 40√3√2 = 40√6

105
3 𝑚
Example 3: The sum of coefficients of the 1st three terms of the expansion of (𝑥 − 𝑥 2 ) is 559, where m is a
natural number. Find the number of terms in the expansion.
Answer: Here, coefficient of 1st three terms is 𝐶(𝑚, 0), − 3𝐶(𝑚, 1), 9𝐶(𝑚, 2)+…
According to the question, 𝐶(𝑚, 0) − 3𝐶(𝑚, 1) + 9𝐶(𝑚, 2) = 559
9𝑚(𝑚 − 1)
1 − 3𝑚 + = 559
2
On solving we get m=12
So, number of terms in the expansion are 13
Example 4: If P be the sum of odd terms and Q that of even terms in the expansion (𝑎 + 𝑏)𝑛
Prove that
i. 𝑃2 − 𝑄 2 = (𝑥 2 − 𝑎2 )𝑛
ii. 4𝑃𝑄 = ( 𝑥 + 𝑎)2𝑛 − (𝑥 − 𝑎 )2𝑛
Answer: (𝑥 + 𝑎)𝑛 = 𝐶(𝑛. 0)𝑥 𝑛 + 𝐶(𝑛, 1)𝑥 𝑛−1 𝑎 + 𝐶(𝑛, 2)𝑥 𝑛−2 𝑎2 + 𝐶(𝑛, 3)𝑥 𝑛−3 𝑎3 + ⋯ + 𝐶(𝑛, 𝑛)𝑎𝑛 = 𝑡1 +
𝑡2 + 𝑡3 + ⋯ + 𝑡𝑛+1
= ( 𝒕𝟏 + 𝒕𝟑 + 𝒕𝟓 + ⋯ ) + (𝒕𝟐 + 𝒕𝟒 + 𝒕𝟔 + ⋯ )
(𝑥 + 𝑎)𝑛 = 𝑃 + 𝑄 − − − − − (𝑖)
(𝑥 − 𝑎)𝑛 = 𝐶(𝑛. 0)𝑥 𝑛 − 𝐶(𝑛, 1)𝑥 𝑛−1 𝑎 + 𝐶(𝑛, 2)𝑥 𝑛−2 𝑎2 − 𝐶(𝑛, 3)𝑥 𝑛−3 𝑎3 + ⋯ + (−1)𝑛 𝐶(𝑛, 𝑛)𝑎𝑛
= 𝑡1 − 𝑡2 + 𝑡3 − 𝑡4 + ⋯
= ( 𝒕𝟏 + 𝒕𝟑 + 𝒕𝟓 + ⋯ ) − (𝒕𝟐 + 𝒕𝟒 + 𝒕𝟔 + ⋯ )
(𝑥 − 𝑎)𝑛 = 𝑃 − 𝑄 − − − − − (𝑖𝑖)
Multiply ( i ) with (ii), we get
𝑃2 − 𝑄 2 = (𝑥 2 − 𝑎2 )𝑛

by squaring ( i) and (ii) and subtract, we get


4𝑃𝑄 = ( 𝑥 + 𝑎)2𝑛 − (𝑥 − 𝑎 )2𝑛

Case Study based question:


1. A bakery sells cookies in packs. Each pack contains either chocolates chips or oatmeal cookies. The
owner wants to find out the different combinations of chocolate chip and oatmeal cookies in a pack of
5 cookies
Question 1: How many different combinations of chocolate chip and oatmeal cookies can be made in a pack
of 5 cookies?
Answer: The number of combinations corresponds to the coefficients in the expansion (𝑥 + 𝑦)5,
where x represents chocolate chips and y represents oatmeal cookies. The number of different
combinations is given by the 5th row of Pascal’s triangle 1,5,10,10,5,1. Therefore there are 6 different
combinations.
Question 2: What is the probability of having exactly 3 chocolate chip cookies in a pack of 5 cookies?
Answer: The coefficient of 𝑥 3 𝑦 2 in the expansion (𝑥 + 𝑦)5 gives the number of ways to have 3
chocolate chip and 2 oatmeal cookies. The coefficient 𝐶(5,3) = 10. The total number of combinations
10
is 25 = 32. Therefore the probability is 32

106
2.

3.

107
Practice Questions:
1. If P be the sum of odd terms and Q that of even terms in the expansion (𝑎 + 𝑏)𝑛
Prove that: 2(𝑃2 + 𝑄 2 ) = {(𝑥 + 𝑎)2𝑛 + (𝑥 − 𝑎)2𝑛 }
2. If three successive coefficients in the expansion of (1 + 𝑥)𝑛 are 220, 495 and 792, then find n
3. The coefficients of three consecutive terms in the expansion (1 + 𝑎)𝑛 are in the ratio 1: 7: 42. Find n
4. Expand: (1 − 𝑥 + 𝑥 2 )4
1 5
5. Expand (𝑥 + 𝑥 )
6. Compute: 985
7. Prove that ∑𝑛𝑟=0 3𝑟 𝐶(𝑛, 𝑟) = 4𝑛
8. Find the number of terms in the expansion of ( 1 − 2𝑥 + 𝑥 2 )7
9. Evaluate: 1014

ASSERTION AND REASONING TYPE QUESTIONS

108
CASE BASED QUESTIONS:

109
Answers: (PRACTICE QUESTIONS)
Q2 Q3 Q6 Q8 Q9
12 55 9039207968 15 104060401
Answers (Assertion and reason)
Q1 Q2 Q3 Q4
A C A D

Answers (case-based questions)


Q1 Q2 Q3 Q4 Q5
64 32 256 128 12

110
SEQUENCE AND SERIES
CONCEPTS:
Sequence: A sequence is an arrangement of numbers in a definite order according to some rule. A sequence
can also be defined as a function whose domain is the set of natural numbers or some subsets of the set {1,
2, 3....}.
A sequence containing a finite number of terms is called a finite sequence. A sequence is called infinite if it is
not a finite sequence.
Series: If 𝑎1 , 𝑎2 , 𝑎3 ,….𝑎𝑛 … be a given sequence. Then, the expression
𝑎1 + 𝑎2 + 𝑎3 +…+ 𝑎𝑛 + …
is called a series.
Arithmetic Progression (A.P.) : A sequence in which terms increase or decrease regularly by the same
constant.
A sequence a1, a2, a3,…, an,… is called arithmetic sequence or arithmetic progression if 𝑎𝑛+1 = an + d,
n ∈ N, where 𝑎𝑛 is called the nth term and the constant term d is called the common difference of the A.P.
The nth term (general term) of the A.P. a, a + d, a + 2d, ... is an = a + (n – 1) d.
If a, b, c are in A.P. and k(  0) is any constant, then
(i) a + k, b + k, c + k are also in A.P.
(ii) a – k , b – k , c – k are also in A.P.
(iii) ak, bk, ck are also in A.P
a b c
, ,
(iv) k k k are also in A.P.
If a, a + d, a + 2d, …, a + (n – 1) d be an A.P. Then l = a + (n – 1) d.

Sum to n terms Sn = [2a + (n − 1)d


n
2
n
= [a + l]
2
a+b
Arithmetic mean (A.M.) between two numbers a and b is 2 .

a+
(b − a ) , a + 2(b − a ) , a + 3(b − a ) , ... , a + n(b − a )
n arithmetic means between two numbers a and b are n +1 n +1 n +1 n +1 .

a+b
Sum of n A.M’s = n( 2 )
111
❖ Three consecutive terms in A.P. are a – d , a, a + d.
❖ Four consecutive terms in A.P. are a – 3d , a – d , a + d, a + 3d.
❖ Five consecutive terms in A.P. are a – 2d , a – d , a, a + d, a + 2d.
These results can be used if the sum of the terms is given.
In an A.P. the sum of terms equidistant from the beginning and end is constant and equal to the sum of first
and last terms.
❖ mth term from end of an A.P. = (n – m + 1)th term from the beginning.

Geometric Progression (G . P.) :A sequence is said to be a geometric progression or G.P., if the ratio of any
term to its preceding term is same throughout.
A sequence a1, a2, a3,…, an,… is called geometric progression, if each
term is non-zero and
a k +1
= r (constant )
ak , for k ≥ 1.
By taking a1 = a, we obtain a geometric progression, a, ar, ar2, ar3,….,
where a is called the first term and r is called the common ratio of the G.P.

n −1
General term of a G .P. = a n = ar .

a (r n − 1) a (1 − r n )
if r  1 if r  1
Sum to n terms of a G .P. = r − 1 and 1 − r .
a
Sum of terms of an infinite G.P. = 1 − r.

Geometric Mean (G .M.) of two positive numbers a and b is the number is ab .


1 2 3 n
 b  n +1  b  n +1  b  n +1  b  n +1
a  , a  , a  , ... a 
n geometric means between two numbers a and b are  a  a a a .
𝑛
Product of n G.M’s between a and b is (√𝑎𝑏)
a
, a, ar
❖ Three consecutive terms in G.P. are r .
a a
3
, , ar, ar3.
❖ Four consecutive terms in G.P. are r r

112
a a
2
, , a , ar, ar 2
❖ Five consecutive terms in G.P. are r r .
These results can be used if the product of the terms is given.
Harmonic Progression: A series of quantities is said to be in harmonic progression if their reciprocals are in
arithmetic progression.

1 1 1 1
is a + (n − 1)d .
, , , ...
nth term of the H.P. a a + d a + 2d
2ab
Harmonic Mean between two quantities a & b is a + b
Relations b/w A(A.M.), G(G.M.) & H(H.M.)
(i) A, G, H are in G.P.
(ii) A, G, H are in descending order of magnitude i.e. A > G > H.
Arithmetico-geometric series : A type of series in which each term is the product of the corresponding terms
of an A.P. and a G.P.

a + (a + d )r + (a + 2d )r 2 + (a + 3d )r 3 + ... is an arithmetico-geometric series.

Sum to n terms of the arithmetico-geometric series : a + (a + d )r + (a + 2d )r + (a + 3d )r + ...


2 3

S=
a
+
( )
dr 1 − r n −1 [a + (n − 1)d]r n

is 1− r (1 − r )2 1− r

a dr
+
❖ Sum of an Infinite arithmetico-geometric series = 1 − r (1 − r ) .
2

Some useful results :


n (n + 1)
 n = 1 + 2 + 3 + ... + n = 2
n (n + 1)(2n + 1)
 n 2 = 12 + 22 + 32 + ... + n 2 = 6
 n (n + 1)
2

 n = 1 + 2 + 3 + ... + n =  2 
3 3 3 3 3

MCQ’s
1) If a, 4, b are in Arithmetic Progression; a, 2, b are in Geometric Progression; then a, 1, b are in

113
a. A.P
b. G.P
c. H.P
d. None of these
Answer: (c) H.P
Explanation:
Given that a, 4, b are in A.P
Hence, 4-a = b-4
a+b = 8 …(1)
Also, given that a, 2, b are in G.P.
Hence, 2/a = b/2
So, ab = 4…(2)
If a, 1, b are in H.P, then 1 = 2(ab)/(a+b) ..(3)
Now substitute (1) and (2) in (3)
1 = 2(4) /(8)
1 = 8/8
1=1.
Therefore, a, 1, b are in H.P.
2) If “a” is the first term and “r” is the common ratio, then the nth term of a G.P is:
a. arn
b. arn-1
c. (ar)n-1
d. None of these
Answer: (b) arn-1
Explanation:
If “a” is the first term and “r” is the common ratio, the terms of infinite G.P are written as a, ar, ar2, ar3, ar4,
…arn-1.
Hence, the nth term of a G.P is arn-1.
Therefore, option (b) is the correct answer.

3) If a, b, c are in arithmetic progression, then


114
a. b = a+c
b. 2b = a+c
c. b2 = a+c
d. 2b2 = a+c
Answer: (b) 2b = a+c
Explanation:
Given that a, b, c are in arithmetic progression.
So, the common difference is b-a = c-b
Rearranging the same terms, we get
b+b = c+a
2b = a+c.
Hence, if a, b, c are in A.P, then 2b = a+c.
4) The sum of arithmetic progression 2, 5, 8, …, up to 50 terms is
a. 3775
b. 3557
c. 3757
d. 3575
Answer: (a) 3775
Explanation:
Given A.P. = 2, 5, 8, …
We know that the sum of n terms of an A.P is Sn = (n/2)[2a+(n-1)d]
Here, a = 2, d = 3 and n=50.
Now, substitute the values in the formula, we get
S50 = (50/2)[2(2)+(50-1)(3)]
S50 = 25[4+(49)(3)]
S50 = 25[4+147]
S50 = 25(151)
S50 = 3775.
Hence, the sum of A.P 2, 5, 8, …up to 50 terms is 3775.
5) The 3rd term of G.P is 4. Then the product of the first 5 terms is:
a. 43
115
b. 44
c. 45
d. None of these
Answer: (c) 45
Explanation:
We know that the terms of infinite G.P are written as a, ar, ar2, ar3, ar4, …arn-1.
Hence, the 3rd term, (i.e) ar2= 4
Thus, the product of the first 5 terms = (a)(ar)(ar2)(ar3)(ar4)
= a5r10
= (ar2)5
Now, substitute ar2 = 4 in the above form, we get
Product of the first 5 terms = (4)5 = 45.
Hence, option (c) 45 is the correct answer.
6) Which of the following is an example of a geometric sequence?
a. 1, 2, 3, 4
b. 1, 2, 4, 8
c. 3, 5, 7, 9
d. 9, 20, 21, 28
Answer: (b) 1, 2, 4, 8
Explanation:
Among the options given, option (b) 1, 2, 4, 8 is an example of a geometric sequence.
We know that in a geometric sequence each term is found by multiplying the previous term by a constant.
In option (b) 1, 2, 4, 8, each term is found by multiplying 2 to the previous term. Here, the common ratio is 2.
7) The next term of the given sequence 1, 5, 14, 30, 55, … is
a. 80
b. 90
c. 91
d. 96
Answer: (c) 91
Explanation: The next term in the sequence 1, 5, 14, 30, 55, … is 91.
Ist term = 12 = 1
116
2nd term = 12+22 = 1+4 = 5
3rd term = 12+22+32 = 1+4+9 = 14
4th term = 12+22+32+42 = 1+4+9+16 = 30
5th term = 12+22+32+42+52 = 1+4+9+16+25 = 55
6th term = 12+22+32+42+52+62 = 1+4+9+16+25+36 = 91.
Thus, option (c) is the correct answer.
8) If the nth term of an arithmetic progression is 3n-4, then the 10th term of an A.P is
a. 10
b. 12
c. 22
d. 26
Answer: (d) 26
Explanation:
Given that the nth term of A.P = 3n-4.
To find the 10th term of A.P, substitute n = 10
Therefore, 10th term of A.P = 3(10) -4 = 30-4 = 26.
9) 3, 5, 7, 9 is an example of
a. Arithmetic sequence
b. Geometric sequence
c. Harmonic sequence
d. Fibonacci sequence
Answer: (a) Arithmetic sequence
Explanation: 3, 5, 7, 9 is an example of an arithmetic sequence. In this sequence 3, 5, 7, 9, the difference
between each term is 2.
(i.e) 5-3 = 2, 7-5 = 2, 9-7 = 2.
Hence 3, 5, 7, 9 is an arithmetic sequence.
10) The first term of a G.P is 1. The sum of the 3rd and 5th terms is 90. Then the common ratio is:
a. 1
b. 2
c. 3
d. 4
117
Answer: (c) 3
Explanation:
Given that first term of G.P, a = 1.
The sum of the 3rd and 5th term = 90
(i.e) ar2+ar4 = 90
Substitute a = 1,
⇒ r2+r4 = 90
⇒ r4 + r2 – 90 = 0
⇒ r4 + 10r2 – 9r2 – 90 = 0
Now, factorize the above equation,
⇒ r2 (r2+10) – 9 (r2+10) = 0
⇒ (r2-9)(r2+10) = 0
⇒ r2 = 9 or r2 = -10
Here, r2= -10 is not possible, as the square of a number cannot be negative.
So, r2 = 9
r = 3 or r= -3
Therefore, option (c) 3 is the correct answer.

Case Study Based Questions


1. Question No. 1 to 5 are based on the given text. Read the text carefully and answer the questions:
Ratan wants to open an RD for the marriage of his daughter, He visited the branch of SBI at sector 3,
Gurgaon. There he made an agreement with the bank.

According to this agreement, he would deposit₹ 100 × n 3 every month ( here n =1 to 15). Other terms and
conditions areas follows:
1. He has to pay a minimum of six instalments.
2. If he continues the deposit up to 15 months then the bank will pay 20% extra as a bonus.
3. If he breaks the deposit after 6 months then the bank will pay 10% extra as a bonus
4. If he breaks the deposit after 10 months then the bank will pay 15% extra as a bonus.
118
No other interest will be paid by the bank.
Ques.1 How much amount would be accumulated after 15 months?
a) ₹ 10,00,000
b) ₹ 11,02,500
c) ₹ 15,00,000
d) ₹ 14,40,000
Answer. (d) ₹ 14,40,000
Ques.2 How much total amount would Ratan get after 15 months?
a) ₹ 14,40,000
b) ₹ 13,23,000
c) ₹ 17,28,000
d) ₹ 15,00,000
Answer. (c) ₹ 17,28,000
Ques.3 How much total amount would Ratan get if he breaks the deposit after 10 months?
a) ₹ 3,50,000
b) ₹ 3,23,000
c) ₹ 3,47,875
d) ₹ 3,45,875
Answer. (c) ₹ 3,47,875
Ques.4 How much total amount would Ratan get if he breaks the deposit after 6 months?
a) ₹ 60,000
b) ₹ 50,715
c) ₹ 50,000
d) ₹ 65,875
Answer. (b) ₹ 50,715
Ques.5 How much amount did Ratan pay in the 10th month?
a) ₹ 729,000
b) ₹ 50,715
c) ₹ 1,00,000
d) ₹ 60,000
Answer. (c) ₹ 1,00,000
119
2. Question No. 6 to 10 are based on the given text. Read the text carefully and answer the questions:
Shamshad Ali buys a scooter for₹ 22,000. He pays ₹ 4,000 cash and agree to pay the balance in annual
instalments of ₹ 1000 plus 10% interest on the unpaid amount.

Ques.6 Interest paid by Ali on 3rd instalment?


a) ₹ 1500
b) ₹ 1700
c) ₹ 1800
d) ₹ 1600
Answer. (d) ₹ 1600
16000×10×1
Explanation: Unpaid amount = ₹ 17000 - ₹ 1000 = 16000 Interest on 2nd instalment = = 1600
100

Ques.7 Interest paid by Ali on 2nd instalment?


a) ₹ 1500
b) ₹ 1800
c) ₹ 1600
d) ₹ 1700
Answer. Explanation: Unpaid amount = 18000 - 1000 = 17000.
17000×10×1
Interest on 2nd instalment= = 1700
100

Ques.8 Interest paid by Ali on 1st instalment?


a) ₹ 1700
b) ₹ 1900
c) ₹ 1800
d) ₹ 1600
Answer. (c) ₹ 1800
Explanation: Scooter cost = ₹ 22,000 Down payment = ₹ 4,000.
Balance payment = ₹ 18,000
18000×10×1
Now, interest on 1st instalment= = 1800
100
120
Ques.9 How much will scooter cost him?
a) ₹ 17100
b) ₹ 39100
c) ₹ 22000
d) ₹ 29100
Answer. (b) ₹ 39100
Explanation: Total amount or actual cost = 22000 + 17100 = ₹ 39100.
Ques.10 Total interest paid by Ali is:
a) ₹ 22000
b) ₹ 17100
c) ₹ 25000
d) ₹ 39100
Answer. (b) ₹ 17100
Explanation: Total interest paid by him
= 1800 + 1700 + 1600 + ... + 18 terms
which is an A.P. with a = 1800, d = 1700 - 1800 = -100 Therefore, total interest

3. Question No. 11 to 15 are based on the given text. Read the text carefully and answer the questions:
A student of class XI draw a square of side 10 cm. Another student join the mid - point of this square to form
new square. Again, the mid - points of the sides of this new square are joined to form another square by
another student. This process is continued indefinitely.

Ques.11 The sum of the perimeter of all the square formed is (in cm)
a) 40
b) 40 + 40√2
c) 80 + 40√2
d) 40√2
Answer. (c) 80 + 40√2
121
Ques.12 The sum of areas of all the square formed is (in sq cm)
a) 250
b) 300
c) 200
d) 150
Answer. (c) 200
Ques.13 The perimeter of the 7th square is (in cm)
5
a) 2

b) 10
c) 5
d) 20
Answer. (c) 5
Ques.14 The area of the fifth square is (in sq cm)
25
a) 4

b) 25
c) 50
25
d) 2
25
Answer. (a) 4

Ques.15 The side of fourth square is (in cm)


√5
a)
2

b) √5
c) None of these
d) 5
Answer. (c) None of these
4. Question No. 16 to 20 are based on the given text. Read the text carefully and answer the questions:
A farmer ,Ramgarh, took a bank loan from SBI for repairing his house. But he could not pay the amount on
time.
This resulted in the accumulation of interest and the amount to pay reached ₹ 1,00,000.

122
After a few months, the farmer opened a shop that resulted in enough income and the income increased on
a regular basis. So he decided to pay the bank loan in a different manner.
The farmer visited the bank. He made an agreement with the bank that he will start paying the amount of ₹
1,00,000 without interest from Jan 2020. In January he will pay ₹ 5000 and will increase the payment by ₹
100 in each month, as shown in the figure.

Ques.16 In how many months will the farmer clear the loan amount?
a) 18
b) 15
c) 16
d) 20
Answer. (a) 18
Ques.17 How much amount he has to pay in last month in rupees?
a) 1500
b) 2000
c) 1400
d) 1800
Answer. (c) 1400
Ques.18 In which month he will pay₹ 6000?
a) 14th
b) 10th
c) 11th
d) 12th
Answer. (c) 11th
Ques.19 How much amount he will pay in 10thmonth in₹ ?
a) 6000
b) 7000
c) 7500
d) 6400

123
Answer. (d) 6400
Ques.20 How much amount in rupees till 10th month he will have paid?
a) 60000
b) 55000
c) 54500
d) 50000
Answer. (c) 54500
5. Question No. 21 to 25 are based on the given text. Read the text carefully and answer the questions:
Father of Ashok is a builder, He planneda 12 story building in Gurgaon sector 5. For this, he bought a plot of
500 square yards at the rate of₹ 1000 /yard².The builder planned ground floor of 5 m height, first floor of
4.75 m and so on each floor is 0.25 m less than its previous floor.

Ques.21 What is the height of the last floor?


a) 2.5 m
b) 3 m
c) 2.75 m
d) 2.25 m
Answer. (d) 2.25 m
Ques.22 Which floor no is of 3 m height?
a) 10
b) 7
c) 5
d) 9
Answer. (d) 9
Ques.23 What is the total height of the building?
a) 40.5 m

124
b) 44 m
c) 40 m
d) 43.5
Answer. (d) 43.5
Ques.24 Up to which floor the height is 33 m?
a) 8
b) 7
c) 9
d) 10
Answer. (a) 8
Ques.25 Which floor no. is half in height of ground floor?
a) 10
b) 12
c) 9
d) 11
Answer. (d) 11
Assertion Reasoning Type Questions
Ques.1 Assertion (A): A sequence is said to finite if it has finite no of terms.
𝑡ℎ 8 2𝑛
Reason (R): The n term of the sequence: 2, 2, 3 , 4..is 𝑛

a) Both A and R are true and R is the correct explanation of A.


b) Both A and R are true but R is not the correct explanation of A.
c) A is true but R is false.
d) A is false but R is true.
Answer (b) Both A and R are true but R is not the correct explanation of A
𝑎
Ques.2 Assertion (A): The sum of infinite terms of a geometric progression is given by 𝑆∞ = , provided
1−𝑟

|r| < 1.
𝑎(𝑟 𝑛 −1)
Reason (R): The sum of n terms of Geometric progression is S 𝑛 = .
𝑟−1

a) Both A and R are true and R is the correct explanation of A.


b) Both A and R are true but R is not the correct explanation of A.
c) A is true but R is false.
125
d) A is false but R is true.
Answer (b) Both A and R are true but R is not the correct explanation of A.
−2 −7
Ques.3 Assertion (A): If the numbers , K, are in GP, then k = ±1 .
7 2
𝑎 𝑎
Reason (R): If a 1 , a 2 , a 3 are in GP, then 𝑎2 = 𝑎3 .
1 2

a) Both A and R are true and R is the correct explanation of A.


b) Both A and R are true but R is not the correct explanation of A.
c) A is true but R is false.
d) A is false but R is true.
Answer (a) Both A and R are true and R is the correct explanation of A
Ques.4 Assertion (A): If the sum of first two terms of an infinite GP is 5 and each term is three times the sum
1
of the succeeding terms, then the common ratio is 4 .

Reason (R): In an AP 3, 6, 9, 12 ........ the 10th term is equal to 33.


a) Both A and R are true and R is the correct explanation of A.
b) Both A and R are true but R is not the correct explanation of A.
c) A is true but R is false.
d) A is false but R is true.
Answer (c) A is true but R is false
Ques.5 Assertion (A): The sum of first 6 terms of the GP 4, 16, 64, ... is equal to 5460.
𝑎(𝑟 𝑛 −1)
Reason (R): Sum of first n terms of the G.P is given by S 𝑛 = , where a = first term r = common ratio
𝑟−1

and |r| > 1.


a) Both A and R are true and R is the correct explanation of A.
b) Both A and R are true but R is not the correct explanation of A.
c) A is true but R is false.
d) A is false but R is true.
Answer (a) Both A and R are true and R is the correct explanation of A.
Short Answer Type Questions
5
Ques.1 Find the sum of first 8 terms of the G.P. 10, 5,2 ,...

Answer. We have a=10 and r=1/2 and n=8


Now we have to find 𝑆8

126
𝑎(1−𝑟 𝑛 )
so, 𝑆𝑛 = 1−𝑟
1
10(1−( )8 )
2
𝑆8 = 1
1−
2

2550
After calculating we have 𝑆8 = .
128

Ques.2 Find the sum of the geometric series 3 + 6 + 12 + ... + 1536.


Answer. Given series 3 + 6 + 12 + + 1536.
Here we have
r = 6÷3= 2 𝑎𝑛 = 𝑎𝑟 𝑛−1
⇒ 1536 = 3 × 2n-1
⇒ 1536 ÷ 3 = 2n ÷ 2
⇒ 1536 ÷ 3 × 2 = 2n
⇒ 1024 = 2n
⇒ 210 = 2n (on comparing)
∴ n = 10
Now, a = 3 and r = 2 and n = 10 terms
𝑎(𝑟 𝑛 −1)
𝑠𝑛 =
𝑟−1

Sn = 3 × (1024 - 1)
Sn = 3069
Therefore, the sum of n term of G.P. is 3069
Ques.3 Find three numbers in GP whose sum is 65 and whose product is 3375.
Answer. Let the terms of the given G.P. be
𝑎
, ar and ar.
𝑟

then, their product of the = 3375


⇒ a3 = 3375
⇒ a = 15
Similarly, sum of the G.P. = 65
𝑎
⇒ 𝑟 + ar + ar = 65 Substituting the value of a
15
+ r15 + 15r = 65
𝑟

127
⇒ 15r2 - 50r + 15 = 0
⇒ 5(3r2 - 10r + 3) = 0
⇒ 3r2 - 10r + 3 = 0
1
(3𝑟 − 1)(𝑟 − 3) = 0 ⇒ 𝑟 = ,𝑟 = 3
3
for a = 15 and r = 3
Therefore the required three numbers are 5, 15, 45

Ques:4 A GP’s 5th, 8th and 11th terms are 𝑝, 𝑞 and 𝑠 respectively. Prove that 𝑞 2 = 𝑝𝑠
Ans. As a result from the given question, 𝑝 is the 5th term, 𝑞is the 8th term and 𝑠 is the 11th term
We have to prove that 𝑞 2 = 𝑝𝑠. we may express the equation as 𝑎5 = 𝑎𝑟 4 = 𝑝 using the above
information_______(1)
𝑎8 = 𝑎𝑟 7 = 𝑞 … … . (2)
𝑎11 = 𝑎𝑟 10 = 𝑠 … … (3)
𝑞
when we divide (2) by (1), we get 𝑟 3 = 𝑝 … … . (4)
𝑠
when we divide (3) by (2), we get 𝑟 3 = 𝑞 … … . (5)

so from equations (4) and (5) we get q/p=s/q⇒ 𝑞 2 = 𝑝𝑠.


Ques.5 A man saved Rs. 66000 in 20 years. In each succeeding year after the first year, he saved Rs. 200
more than what he saved in the previous year. How much did he save in the first year?
Sol: Let us assume that the man saved Rs. 𝑎 in the first year.
In each succeeding year, an increment of Rs. 200 is made. So, it forms an A.P. whose
First term = a, Common difference, d = 200 and n=20

=> 6600 = 2a + 19 x 200 => 2a = 2800


∴a = 1400
Ques.6 The third term of a geometric progression is 4. When the first five terms are multiplied together,
what is the result?
Ans: 𝑇3 = 4 is deduced from the given question.
⇒𝑎𝑟 2= 4
The product of the first five terms can now be defined as = a.ar.ar2.ar3.ar4.
128
= 𝑎5 𝑟 10
=(𝑎𝑟 2 )5
= 45
As a result, the product of the first five terms is 45 .
Ques.7 13/6 is the sum of two numbers. An even number of mathematical means are placed between them,
and their sum exceeds their number by one. What is the number of inserted means multiplied by two?
Assume a and b are two numbers such that
a + b = 13/6
Take𝐴1 , 𝐴2 ,, 𝐴3 ,,………..𝐴2𝑛 , is the arithmetic mean of a and b.
Then, 𝐴1 + 𝐴2 + 𝐴3 +………..+ 𝐴2𝑛 = 2n(n + 1)/2
⇒ n(a + b) = 13n/6
Given this, the series 𝐴1 + 𝐴2 + 𝐴3 +………..+ 𝐴2𝑛 = 2n + 1
13n/6 = 2n + 1, so
⇒n=6
Ques.8 Show that the sum of (m + n)th and (m – n)th terms of an A.P. is equal to twice the mth term.
Ans: Let a and d represent the A.P.'s initial term and common difference, respectively. It is well known that
an A.P.'s nth term is given by 𝑎𝑛 = 𝑎 + (𝑛 − 1)𝑑
∴ 𝑎𝑚+𝑛 = 𝑎 + (𝑚 + 𝑛 − 1)𝑑 and 𝑎𝑚−𝑛 = 𝑎 + (𝑚 − 𝑛 − 1)𝑑
Adding these two ,we have
𝑎𝑚+𝑛 + 𝑎𝑚−𝑛 = 2𝑎 + (2𝑚 − 2)𝑑
= 2{𝑎 + (𝑚 − 1)𝑑}
= 2𝑎𝑚
Hence proved.
Ques.9 Find the sum of integers from 1 to 100 that are divisible by 2 or 5.
Ans: The divisible by 2 integers from 1 to 100 are 2, 4, 6,.... 100.
This results in an A.P. in which the first term and the common difference are both equal to 2.
⇒ 100=2+(n-1)2
⇒ n= 50
So, the sum of integers from 1 to 100 which are divisible by 2 is given as:
2+4+6+…+100 = (50/2)[2(2)+(50-1)(2)]
= (50/2)(4+98)
129
= 25(102)
= 2550
Divisible by 5, 10,.... 100 integers from 1 to 100.
This generates an A.P. with a common difference of 5 and a first term of 5.
Then, 100= 5+(n-1)5
⇒5n = 100
⇒ n= 100/5
⇒ n= 20
As a result, the total of all divisible by 2 integers from 1 to 100 is:
5+10+15+…+100= (20/2)[2(5)+(20-1)(5)]
= (20/2)(10+95)
= 10(105)
= 1050
As a result, the divisible by both 2 and 5 integers from 1 to 100 are 10, 20,.... 100.
This also creates an Arithmetic Progression. because the first term and the common difference are both
equal to ten.
Then, 100= 10+(n-1)10
⇒10n = 100
⇒ n= 100/10
⇒ n= 10
10+20+…+100= (10/2)[2(10)+(10-1)(10)]
= (10/2)(20+90)
= 5(110)
= 550
So, the required sum is:
= 2550+ 1050 – 550
= 3050
As a result, the sum of the divisible by 2 or 5 integers from 1 to 100 is 3050.
Ques.10 Between 1 and 31, m numbers have been introduced in such a way that the resulting sequence is an
A.P., with a 7:(m – 1) ratio of 5:9. Calculate the value of m.
Ans: Let assume 1, A1, A2, …., Am, 31 are in A.P.
130
a = 1, an = 31
𝑎𝑚+2 = 31
𝑎𝑛 = a + (n - 1)d
31 = a + (m + 2 - 1)d
d = 30/(m + 1)
Now,𝐴7 =7 inserted number=a+7d
Similarly,𝐴𝑚−1 =a+(m-1)d
𝐴7 : 𝐴𝑚−1 = 5:9(given)
𝑎+7𝑑 5
=
𝑎+(𝑚−1)𝑑 9
30
1+7 5
𝑚+1
30 =9
1+(𝑚−1)
𝑚+1

On simplification
⇒m = 14
Long Answer Type Questions
Ques.1 :If a, b, and c are in G.P., ax² + 2bx + c = 0 and dx² + 2ex + f = 0 share a common root then prove that
d/a, e/b, and f/c are in A.P.
Ans. (a) A.P
Solution:
Given that a, b, c are in GP (series)
⇒ b2 = ac
⇒ b2 – ac = 0
Hence, ax2 + 2bx + c = 0 have equal roots.
Now the D = 4b2 – 4ac
and the root of the above equation is -2b/2a = -b/a
So -b/a is the common root of the above quadratic equation.
Now,
dx2 + 2ex + f = 0
⇒ d(-b/a)2 + 2e×(-b/a) + f = 0
⇒ db2 /a2 – 2be/a + f = 0
⇒ d×ac /a2 – 2be/a + f = 0
⇒ dc/a – 2be/a + f = 0
131
⇒ d/a – 2be/ac + f/c = 0
⇒ d/a + f/c = 2be/ac
⇒ d/a + f/c = 2be/b2
⇒ d/a + f/c = 2e/b
⇒ d/a, e/b, f/c are in arithmetic progression (AP).
Ques.2 :Let the 𝑇𝑟 be the 𝑟 𝑡ℎ term of an A.P Series., for r = 1, 2, 3, … If for some positive integers m, n, we
have 𝑇𝑚 = 1/n and 𝑇𝑛 = 1/m, then find the value of 𝑇𝑚𝑛 .
Ans: Let’s assume the first term is a has the common difference of d in the AP
Then, 𝑇𝑚 = 1/n
⇒ a + (m-1)d = 1/n ………… (1)
and 𝑇𝑛 = 1/m
⇒ a + (n-1)d = 1/m ………. (2)
From equation (1) – (2), we get
(m-1)d – (n-1)d = 1/n – 1/m
⇒ (m-n)d = (m-n)/mn
⇒ d = 1/mn
From equation (1), we get
a + (m-1)/mn = 1/n
⇒ a = 1/n – (m-1)/mn
⇒ a = {m – (m-1)}/mn
⇒ a = {m – m + 1)}/mn
⇒ a = 1/mn
Now, then 𝑇𝑚𝑛 = 1/mn + (mn-1)/mn
⇒𝑇𝑚𝑛 = 1/mn + 1 – 1/mn
⇒𝑇𝑚𝑛 = 1
Ques.3: If the sum of the roots of the quadratic equation ax2 + bx + c = 0 is equal to the sum of the squares of
their reciprocals, then a/c, b/a, c/b are in which progression.
Ans: From the given question, the equation is
ax2 + bx + c = 0
Let p and q are the roots of the above equation.
Now p+q = -b/a
132
and pq = c/a
Given that, the p + q = 1/p2 + 1/q2, then
⇒ p + q = (p2 + q2)/(p2 ×q2)
⇒ p + q = {(p + q)2 – 2pq}/(pq)2
⇒ -b/a = {(-b/a)2 – 2c/a}/(c/a)2
⇒ (-b/a)×(c/a)2 = {b2/a2 – 2c/a}
⇒ -bc2/a3 = {b2 – 2ca}/a2
⇒ -bc2/a = b2 – 2ca
Divide both sides by bc, we get the following
⇒ -c /a = b/c – 2a/b
⇒ 2a/b = b/c + c/a
⇒ b/c, a/b, c/a are in AP
⇒ c/a, a/b, b/c are in AP
1 1 1
⇒ 𝑐 , 𝑎 , 𝑎𝑟𝑒 𝑖𝑛 𝐻𝑃
𝑏
𝑎 𝑏 𝑐
⇒ a/c, b/a, c/b are in Harmonic Progression
Ques 4.Three numbers are in A.P.If the sum of these numbers is 27 and the product is 648,find the numbers.
Ans:Let the three numbers are 𝑎 − 𝑑, 𝑎 & 𝑎 + 𝑑
∴ 𝑎 − 𝑑 + 𝑎 + 𝑎 + 𝑑 = 27 and(𝑎 − 𝑑)𝑎(𝑎 + 𝑑) = 648
∴ 3𝑎 = 27 and 𝑎(𝑎2 − 𝑑 2 ) = 648
∴ 𝑎 = 9 and 9(81 − 𝑑 2 ) = 648
𝑜𝑟, (81 − 𝑑 2 ) = 72
𝑜𝑟, 𝑑 2 = 9
∴ 𝑑 = ±3
Hence,the numbers are 6,9 and 12.
Ques.5 Find the sum of integers from 1 to 100 that are divisible by 2 or 5.
Ans: The divisible by 2 integers from 1 to 100 are 2, 4, 6,.... 100.
This results in an A.P. in which the first term and the common difference are both equal to 2.
⇒ 100=2+(n-1)2
⇒ n= 50
So, the sum of integers from 1 to 100 which are divisible by 2 is given as:
133
2+4+6+…+100 = (50/2)[2(2)+(50-1)(2)]
= (50/2)(4+98)
= 25(102)
= 2550
Divisible by 5, 10,.... 100 integers from 1 to 100.
This generates an A.P. with a common difference of 5 and a first term of 5.
Then, 100= 5+(n-1)5
⇒5n = 100
⇒ n= 100/5
⇒ n= 20
As a result, the total of all divisible by 2 integers from 1 to 100 is:
5+10+15+…+100= (20/2)[2(5)+(20-1)(5)]
= (20/2)(10+95)
= 10(105)
= 1050
As a result, the divisible by both 2 and 5 integers from 1 to 100 are 10, 20,.... 100.
This also creates an Arithmetic Progression. because the first term and the common difference are both
equal to ten.
Then, 100= 10+(n-1)10
⇒10n = 100
⇒ n= 100/10
⇒ n= 10
10+20+…+100= (10/2)[2(10)+(10-1)(10)]
= (10/2)(20+90)
= 5(110)
= 550
So, the required sum is:
= 2550+ 1050 – 550
= 3050
As a result, the sum of the divisible by 2 or 5 integers from 1 to 100 is 3050.

134
Practice Questions
Multiple choice questions
1 1 1
1.The 𝑛𝑡ℎ term of the sequence {1, 8 , 27 , 64} will be
1 1 1 1
(a)𝑛 (b)𝑛2 (c) )𝑛3 (d)𝑛3 −1

2.The sum of five numbers which are in A.P is 50,the third number is
(a)2 (b)5 (c)10 (d)15
3.Let 𝑎1 ,𝑎2 , 𝑎3 , … , 𝑎40 are in A.P and 𝑎1 + 𝑎5 + 𝑎15 + 𝑎26 + 𝑎36 + 𝑎40 = 105 then sum of that series is
(a)650 (b)700 (c)1400 (d)None of these
4.In an A.P 𝑝𝑡ℎ term is q and (𝑝 + 𝑞)𝑡ℎ term is 0.Then the 𝑞 𝑡ℎ term will be
(a)−𝑝 (b) )𝑝 (c) )𝑝 + 𝑞 (d) )𝑝 − 𝑞
5.A G.P of positive numbers is such that each of its term is equal to the sum of the next two terms. Then the
common difference is
√5−1 √5 √5+1
(a)√5 (b) ) (c) 2 (d)
2 2

6.The third term of a G.P is 9,then the product of the first five term is
(a)39 (b)310 (c)311 (d)312
7.If 1,𝑎and p are in A.P and 1,𝑔and p are in G.P then
(a)1 + 2𝑎 + 𝑔2 = 0 (b)1 + 2𝑎 − 𝑔2 = 0(c)1 − 2𝑎 − 𝑔2 = 0 (d)1 − 2𝑎 + 𝑔2 = 0
8.The first term of a G.P is 1.The sum of the 3rd and 5th term is 90,then the common ratio
(a)1 (b)2 (c)3 (d)4
2𝜃 2𝜃
9.The minimum value of 2𝑠𝑖𝑛 + 2𝑐𝑜𝑠 is
(a)√2 (b) )2√2 (c)4 (d)2
10.If 𝑎 𝑥 = 𝑏 𝑦 = 𝑐 𝑧 , where 𝑎, 𝑏, 𝑐 are in G.P and 𝑎, 𝑏, 𝑐, 𝑥, 𝑦, 𝑧 ≠ 0 then 𝑥, 𝑦, 𝑧 are in
(a)A.P (b)G.P (c)H.P (d) none of these.
Assertion -Reason based questions:
In the following questions, a statement of Assertion(A) is followed by a statement of Reason (R).Choose the
correct answer out of the following choices:
(a)Both (A) and (R) are true and (R) is the correct explanation of (A)
(b)Both (A) and (R) are true but (R) is not the correct explanation of (A)
(c)(A) is true and (R) is false
(d)(A) is false and (R) is true
135
1. Assertion: If x, 2y and 3z are in A.P. where the distinct numbers x,y and z are in G.P., then the common
1
ratio of the G.P. is3.

Reason: The G.P relation between x, y and z is 𝑦 2 = 𝑥𝑧


2.Assertion :the sum of odd integers from 1 to 2001 is 1002001
Reason :Then the 𝑛𝑡ℎ term (general term) of the A.P. is 𝑎𝑛 = 𝑎 + (𝑛 − 1)𝑑.
3. Assertion :The minimum value of 4𝑥 + 41−𝑥 , 𝑥 ∈ 𝑅 is 4
Reason: A.M is greater than or equal to G.M
4. Assertion : the products of the corresponding terms of the sequences 𝑎, 𝑎𝑟 2 , … , 𝑎𝑟 𝑛−1 and
𝐴, 𝐴𝑅 2 , … , 𝐴𝑅 𝑛−1 form a G.P
Reason:Products of the corresponding terms of two sequences form a G.P
5.Assertion: Six numbers between 3 and 24 such that the resulting sequence is an A.P are 6,9,12,15,18 and
21
Reason : Let 𝐴1 , 𝐴2 , … , 𝐴𝑛 be n numbers between a and b such that a,𝐴1 , 𝐴2 , … , 𝐴𝑛 ,
b is an A.P then 𝐴𝑛 = 𝑎 + 𝑛𝑑, 𝑑 =common difference.

Short answer type questions:


𝑥 𝑛+1 +𝑦 𝑛+1
1.If is the A.M of 𝑥 𝑎𝑛𝑑 𝑦 ,then find the value of n.
𝑥 𝑛 +𝑦 𝑛

2.If 51+53+55+…+𝑡𝑛 =5151 then fint the value of 𝑡𝑛


3.The first and 𝑛𝑡ℎ term of a G.P are a and b respectively. If p is the product of first n terms then show that
𝑝2 = (𝑎𝑏)𝑛 .
2 2
4.If 29 and 41 are the 10th and 14th term of a H.P then find 𝑡𝑛 .

5.If the 𝑝𝑡ℎ , 𝑞 𝑡ℎ , 𝑎𝑛𝑑 𝑟 𝑡ℎ term of a G.P are a ,b,c respectively then show that 𝑎𝑞−𝑟 . 𝑏 𝑟−𝑝 . 𝑐 𝑝−𝑞 = 1.
6.Find three in G.P whose sum is 19 and product is 216.
7.In a G.P the first term is 3 and the last term is 48.If each term is twice the previous term then find the total
term and summation of that G.P.
8.In an A.P sum of the n terms is 3𝑛2 + 5𝑛;then which term will be 152?
9.There are n arithmetic mean between 4 and 31.If 2nd A.M: last A.M=5:14 ,then find the value of n.
7
10.Insert four geometric mean between 4 and 56.

Long answer type questions

136
1.Find the sum of the following series upto n terms:
(i).8+.88+.888+…
(ii)1+11+111+…
(iii)1+4+13++40+121+…
1 1 1
(iv)1.3 + 3.5 + 5.7 + ⋯

2.𝑝𝑡ℎ , 𝑞 𝑡ℎ 𝑎𝑛𝑑 𝑟 𝑡ℎ terms of an A.P are P, Q and R respectively. Then show that 𝑝(Q-R)+𝑞(R-P)+𝑟(P-Q)=0.
3.If the A.M of two numbers is twice their G.M ,then show that the numbers are in the ratio
(2 + √3): (2 − √3).
4.The sum of n,2n and 3n terms of an A.P are 𝑆1 , 𝑆2 𝑎𝑛𝑑 𝑆3 respectively. Then show that 𝑆3 = 3(𝑆2 − 𝑆1 ).
5.If 𝑎1 = 2 and 𝑎𝑛 − 𝑎𝑛−1 = 2𝑛, 𝑛 ≥ 2 then find the value of 𝑎1 + 𝑎2 + 𝑎3 + ⋯ + 𝑎20 .
Case Based Questions
1.India is competitive manufacturing location due to the low cost of manpower and strong technical and
engineering capabilities contributing to higher quality production runs.The production of tv sets in a factory
increases uniformly by a fixed number every year. It produced 16000 sets in 6th year and 22600in 9th year.

Based on the above information , answer the following questions:


(i)Find the production during the first year.
(ii)Find the production during the 8th year.
(iii)Find the production during first 3 years.
(iv)In which year, the production is 29200.
2.Your elder brother wants to buy a car and plans to take loan from a bank for his car. He repays his total
loan of Rs 1,18,000 by paying every month starting with the first instalment of Rs 1000. If he increases the
instalment by Rs 100 every month

137
Based on the above story , answer the following:
(i)Find the amount paid by him in 30th instalment .
(ii) What amount paid by him in the 30 instalment?
(iii)What amount does he still have to pay after 30th instalment?
3. In Rangoli competition in school, Preeti made Rangoli in the equilateral shape. Each side of an equilateral
triangle is 24 cm. The mid-point of its sides are joined to form another triangle. This process is going
continuously infinite.

Based on above information, answer the following questions.


(i) Find the side of the 5th triangle is (in cm)
(ii) Find the sum of perimeter of first 6 triangle is (in cm)
4.Rahul being a plant lover decides to open a nursery and he bought few plants with pots. He wants to place
pots in such a way that number of pots in first row is 2,in second row is 4 and in third row is 8 and so on…

138
Based on the above information answer the following questions:
(i)The constant multiple by which the number of pots is increasing in every row is
(a) 2 (b) 4 (c)8 (d) 1
(ii)The number of pots in 8th row is
(a)156 (b)256 (c)300 (d)456
(iii)The difference in number of pots placed in 7th row and 5th row is
(a)86 (b)50 (c)90 (d)96
(iv) Total number of pots upto 10th row is
(a)1046 (b)2046 (c) 1023 (d)1024
5.A student of class xi draw a square of 10 cm. Another student join the mid points of this square to form
new square. Again, the mid points of the sides of this new square are joined to form another square by
another student. This process is continued indefinitely

Based on the above answer the following questions


(i)what is the measurement of the side of the 4th square?
(ii)what is the area of the 5th square ?
(iii)what is the perimeter of the 7th square?
(iv)Find the sum of the all squared formed.

139
Answer key:
MCQ:
1.(c) 2(c) 3(b) 4(b) 5(b) 6(b) 7(d) 8(c) 9(b) 10(c)
ASSERTION-REASON TYPE
1(a)2(a)3(a)4c()5(c)
SHORT ANSWER TYPE
1.0
2.151
2
4.3𝑛−1

6.4,6,9
7.5&93
8.25th
9.8
7
10.2,7,14,28

LONG ANSWER TYPE:


8𝑛 8
1.(i) 9 − 81 (1 − 10−𝑛 )
10 𝑛
(ii)81 (10𝑛 − 1) − 9
3 𝑛
(iii)4 (3𝑛 − 1) − 2
𝑛
(iv)2𝑛+1

5.3080
CASED BASE QUESTIONS
1.(i)5000 (ii)20400 (iii)21600 (iv)12
2.(i)3900 (ii)73500 (iii)44500
567
3.(i)1.5 cm (ii) 𝑐𝑚
4

4.(i)(a) (ii)(b) (iii)(d) (iv)(b)


25
5.(ii) 4 (iii)5 (iv)200s

140
STRAIGHT LINES
CONCEPTS:
1. Distance formula
Distance between the points P (x1, y1 ) and Q (x2 , y2 )

PQ = √(𝑥2 − 𝑥1 )2 + (𝑦2 − 𝑦1 )2
2. The coordinates of a point dividing the line segment joining the points (x1, y1) and (x2-y2) in the ratio m:n .
𝑚𝑥2 +𝑛𝑥1 𝑚𝑦2 +𝑛𝑦1
i) Internal division : ( , )
𝑚+𝑛 𝑚+𝑛
𝑚𝑥2 −𝑛𝑥1 𝑚𝑦2 −𝑛𝑦1
ii) External division : ( , )
𝑚−𝑛 𝑚−𝑛

3 Mid point formula :


𝑥2 +𝑥1 𝑦2 +𝑦1
( , )
2 2

4 Area of a triangle , whose vertices are (x1,y1) (x2,y2) and (x3 , y3) is
1
= 2 |𝑥1 (𝑦2 − 𝑦3 ) + 𝑥2 (𝑦3 − 𝑦1 ) + 𝑥3 (𝑦1 − 𝑦2 )|

5 Slope of a line: A line in a coordinate plane forms two angles with the x-axis,
which are supplementary. The angle θ made by the line l with positive direction of
x-axis and measured anticlockwise is called the inclination of the line. Obviously 0°
≤ θ ≤ 180°
Definition 1 : If θ is the inclination of a line l, then tan θ is called the slope or
gradient of the line l. The slope of a line whose inclination is 90° is not defined.
The slope of a line is denoted by m. Thus, m = tan θ, θ ≠ 90° It may be observed that the slope of the x-axis is
zero and slope of the y-axis is not defined.
6. Slope of a line when coordinates of any two points on the line are given.
Let P(x1 ,y1) and Q(x2 ,y2) be two points on non-vertical line l whose inclination
is θ. Obviously x 1 ≠ x 2 , otherwise the line will become perpendicular to x-axis
and its slope will not be defined. The inclination of the line l may be acute or
obtuse.
𝑦 −𝑦
Therefore slope of line is m = 𝑥2−𝑥1
2 1

7. Conditions for parallelism and perpendicularity of lines in terms of their slopes in a coordinate plane.
suppose that non-vertical lines l1 and l2 have slopes m1 and m2 , respectively. Let their inclinations be α and β,
respectively. If the line l1 is parallel to l2 then their inclinations are equal, i.e., α = β, and hence,
141
tan α = tan β
Therefore m1 = m2 , i.e. their slopes are equal.
Conversely, if the slope of two lines l 1 and l 2 is same, i.e., m1 = m2 . Then tan α = tan β.
By the property of tangent function (between 0° and 180°), α = β.
Therefore, the lines are parallel.
If the lines l1 and l2 are perpendicular . Then β = α + 90°.
1 −1
Therefore, tanβ = tan (α + 90°) = – cot α = 𝑡𝑎𝑛𝛼 i.e. m2 = 𝑚 or m1m2 = – 1
1

Conversely, if m1m2 = – 1 i.e. tanαtanβ = – 1.


Then tan α = – cot β = tan (β + 90°) or tan (β – 90°)
Therefore, α and β differ by 90°. Thus, lines l1 and l2 are perpendicular to each
other. Hence, two non-vertical lines are perpendicular to each other if and only if
−1
their slopes are negative reciprocals of each other. i.e. m2 =𝑚 or, m1 m2 = – 1.
1

8. Angle between two lines.


𝑚 −𝑚1
2
Case I if1+𝑚 is positive, then tan θ will be positive and tan φ will be negative,
1 𝑚2

which means θ will be acute and φ will be obtuse.


𝑚 −𝑚1
2
Case II if 1+𝑚 is negative, then tan θ will be negative and tan φ will be positive, which means that θ will be
1 𝑚2

obtuse and φ will be acute.


Thus, the acute angle (say θ) between lines L1 and L2 with slopes m1 and m2 , respectively, is given by tan
𝑚 −𝑚1
2
θ=|1+𝑚 | as 𝑚1 𝑚2 + 1 ≠ 0
1 𝑚2

9. Various Forms of the Equation of a Line


i) horizontal and vertical line:
a) x=±a which is a equation of b) y=±a which is a equation of line parallel to
line parallel to y-axis x-axis

142
ii. point slope form: Suppose that P(x0,y0)
is a fixed point on a non-vertical line L. Whose
slope is m. Let P (x, y) be an arbitrary point on L. Then by the
definition,
𝑦−𝑦
the slope of L is given by m=𝑥−𝑥1 and equation of line is y-
1

y1=m(x-x1)

iii. Two-point form: Let the line L passes through two given
points P1 (x1 ,y1 ) and P2 (x2 , y2 ).
Let P (x, y) be a general point on L .The three
points P1 , P2 and P are collinear. Therefore we
have slope of P1 P = slope of P1 P2 i.e. Thus equation of the line passing through the points (x1 , y1 ) and (x2 , y2
) is given by
𝑦 −𝑦
y-y1 = 𝑥2−𝑥1 (𝑥 − 𝑥1 )
2 1

iv. Slope-intercept form: Sometimes a line is known to us with its slope and an intercept on one of the axes.
Case I: Suppose a line L with slope m cuts the y-axis at a distance c from the origin. The distance c is called the
y-intercept of the line L. Obviously, coordinates of the point where the line meet the y-axis are (0, c). Thus, L
has slope m and passes through a fixed point (0, c). Therefore, by point-slope
form, the equation of L is
y-c = m(x-0) or y = mx + c
Thus, the point (x, y) on the line with slope m and y-intercept c lies on the line
if and only if
143
y = mx +c
Note that the value of c will be positive or negative according as the intercept is made on the positive or
negative side of the y-axis, respectively.
Case II: Suppose line L with slope m makes x-intercept d. Then equation of L is
y = m(x – d)
v) Intercept – form: Suppose a line L makes
x-intercept a and y-intercept b on the axes.
Obviously L meets x-axis at the point (a,0)
and y-axis at the point (0, b). By two-point
form of the equation of the line, we have
(𝑏−𝑜)
y-0 = (0−𝑎) (𝑥 − 𝑎).

ay=-bx+ab
ax+by=ab
Thus, equation of the line making intercepts a and b on x- and y-
axis, respectively, is
𝑥 𝑦
+ =1
𝑎 𝑏

10) Distance of a Point from a Line.


The distance of a point from a line is the
length of the perpendicular drawn from
the point to the line. Let L :Ax + By + C = 0
be a line, whose distance from the point
P(x1,y1) is d.
𝐴𝑥1 +𝐵𝑦1 +𝐶
d=| |
𝐴2 +𝐵2

11) Distance between two parallel lines: We


know that slopes of two parallel lines are equal.
Therefore, two parallel lines can be taken in the
form y = mx + c1 ... (1)
and y = mx + c2 ... (2)
𝑐 −𝑐
d=|𝑚12 +12 |
144
MCQ
1. If the slope of the line joining the points A( x, 2) and B (6, -8) is − 5/4 , find the value of x.
A) 2 B) 3 C) -2 D)-3

Answer :(C)
If a line passing through (x1 ,y1) and (x2, y2) then slope of the line is given by :
𝑦 −𝑦
Slope = (𝑥2 −𝑥1 )
2 1

Given points are A ( x, 2 ) and B (6,-8) and the slope is -5/4 and
−8−2 5
=-
6−𝑥 4

x = -2
2. Find the value of x so that the line through (3, x) and (2, 7) is parallel to the line through (-1, 4) and (0, 6).
A) 3 B) 6 C)9 D)-9

Ans: (C)
We know that for two lines to be parallel, their slope must be the same. The given points are A(3,x),B(2,7) and
C(-1,4),D(0,6).
6−4 7−𝑥
Slope=(0+1) = (2−3)
2 7−𝑥
therefore = ( −1 )
1

≫-2=7-x
therefore x=9
3. The slope of a line is not defined when the line is
A) parallel to x – axis B) parallel to y – axis
C) parallel to the line x + y = 0 D) parallel to the line x - y = 0 (B)
4. The slope of any line parallel to X – axis is
A) 1 B) - 1 C) 0 D) not defined (C)
5. The equation of the line through ( 0, 0 ) and parallel to the line ax + by + c =0 is (B)
A) ax – by –c = 0 B) ax + by = 0
C) bx + ay = 0 D) bx – ay = 0
6. Slope of a line which cuts off intercepts of equal lengths on the axes is (A)
a) -1 b) 0 c)2 d) 1

145
𝑥 𝑦
Answer Given that equation of a line in intercept form is𝑎 +𝑏 =1

Since a=b therefore x+y=a

Now slope of the line is -1


7. For specifying a straight line ,how many geometrical parameters should be know
a) 2 b) 3 c) 1 d)4 (A)

8. The equation of a line passing through (1,2) and and perpendicular to the line x+y+1=0 is
a) y-x+1=0 b) y-x-1=0 c) y-x+2=0 d)x+y=3 (D)
given that equation of line is x+y+1=0
now equation of line perpendicular to this line y-x+k=0
since given line passing throughthe point (1,2)
therefore 1-2+ k=0
therefore k=1 now equation of line is y-x+1=0 (A)
9. Slope of a line which makes 450 with x-axis is
a) 1 b) 2 c) -2 d) -1

given that 𝜃 = 450


Then slope of line is m=tan 𝜃=tan450 =1 (D)
10. find the angle between a line , whose slope is √3 and x-axis is
a) 150 b) 300 C) 450 D) 600

Given that slope of line is m= √3


Now m= tan 𝜃=√3=tan600
𝜃 =600
` (D)
ASSERTION AND REASON
Read assertion and reason carefully and write correct option for each question
(a) Both A and R are correct, R is the correct explanation of A
(b) 𝐵𝑜𝑡ℎ 𝐴 𝑎𝑛𝑑 𝑅 𝑎𝑟𝑒 𝑐𝑜𝑟𝑟𝑒𝑐𝑡, 𝑅 𝑖𝑠 𝑛𝑜𝑡 𝑡ℎ𝑒 𝑐𝑜𝑟𝑟𝑒𝑐𝑡 𝑒𝑥𝑝𝑙𝑎𝑛𝑎𝑡𝑖𝑜𝑛 𝑜𝑓 𝐴
(c) 𝐴 𝑖𝑠 𝑐𝑜𝑟𝑟𝑒𝑐𝑡, 𝑅 𝑖𝑠 𝑖𝑛𝑐𝑜𝑟𝑟𝑒𝑐𝑡
(d) 𝑅 𝑖𝑠 𝑐𝑜𝑟𝑟𝑒𝑐𝑡, 𝐴 𝑖𝑠 𝑖𝑛𝑐𝑜𝑟𝑟𝑒𝑐𝑡

146
1) ASSERTION slope of x-axis is 0
REASON x-axis make zero angle with the positive direction of x-axis
Answer since 𝜃 = 0 therefore m=0 and x-axis are parallel to positive direction of x-axis (A)
2) ASSERSION two parallel lines have same slope
REASON any line parallel to x axis have slope 0 (A)
Answer since slope of x-axis is zero and any line parallel to x-axis have same slope
Therefore any line parallel to x-axis have 0
3) ASSERSION product of slope of two perpendicular line is -1
REASON product of slope of x-axis and y-axis is -1
Answer since we know that slope of produt of two perpendicular line is -1 and also we know that x-axis
and y-axis is perpendicular therefore product of slope of x-axis and y-axis are -1 . (A)
4) ASSERSION parallel lines have same slope
REASON line P: 2x+ 3y=5 and Q: 6x+ 9y=20 have same slope
Slope of line P= -2/3 and slope of line Q=-2/3 (A)
5) ASSERTION eqn of line Passing through origin is y=mx
REASON line y=mx have intercept on the x-axis is 0 and y-axis is 0
Since assertion and reason are true but R is not correct explanation of A (B)

SHORT ANSWER TYPE QUESTION


1. Show that the line through the points (5, 6) and (2, 3) is parallel to the line through the points (9, -2) and (6,
-5)
Answer: We know that for two lines to be parallel, their slope must be the same. Given points are A(5,6),B(2,3)
and C(9,-2),D(6,-5)
𝑦 −𝑦
Slope = (𝑥2 −𝑥1)
2 1

3−6 −5+2
( )=( )
2−5 6−9

1=1
Hence proved.
2. If a point P(x, y) is equidistant from the points A(6, -1) and B(2, 3), find the relation between x and y.
Answer: Given: Point P(x, y) is equidistant from points A(6, -1) and B(2, 3) i.e., distance of P from A = distance of
P from B Squaring both sides,
147
√(𝑥 − 6)2 + (𝑦 + 1)2 =√(𝑥 − 2)2 + (𝑦 − 3)2

(x-6)2 + (y-1)2 = (x-2)2 + (y-3)2


x2-12x+36+y2-2y-1 = x2-4x+4+y2-6y+9
-12x +36+2y +1 = -4x +4 -6y +9
x-y = 3
Therefore, x – y = 3 is the required relation.
3. Find a point on the x-axis which is equidistant from the points A(7, 6) and B(- 3, 4).
Answer: Let the point on x-axis be P(x, 0). Given: Point P(x, 0) is equidistant from points A(7, 6) and B(-3, 4) i.e.,
distance of P from A = distance of P from B

√(𝑥 − 7)2 + 36 = √(𝑥 + 3)2 + 16


Squaring both sides,

(x-7)2+36 = (x+3)2 + 16
x2 – 14x +49+36 = x2 + 6x+ 9 +16
-20x = -60
x = 3 Therefore, the point on the x-axis is (3, 0).
4. Show that the points A(-5, 1), B(5, 5) and C(10, 7) are collinear.
Answer: Given: The points are A(-5, 1), B(5, 5) and C(10, 7). Note: Three points are collinear if the sum of lengths
of any sides is equal to the length of the third side.

AB = √(5 + 5)2 + (5 − 1)2 = √100 + 16


= 2√29𝑢𝑛𝑖𝑡𝑠

BC = √(10 − 5)2 + (7 − 5)2 = √25 + 4

= √29𝑢𝑛𝑖𝑡𝑠

AC = √(10 + 5)2 + (7 − 1)2 = √225 + 4


= 3√29𝑢𝑛𝑖𝑡𝑠
From equations 1,2,3 we have AB+BC=AC
Therefore, the three points are collinear.
5. If the three points A (h, k), B(x1, y1) and C(x2, y2) lie on a line then show that (h – x1)(y2 – y1) = (k – y1)(x2 – x).
Answer: For the lines to be in a line, the slope of the adjacent lines should be the same. Given points are
A(h,k),B(x1,y1) and C(x2,y2) So slope of AB = BC = CA
148
𝑦 −𝑦
Slope = (𝑥2 −𝑥1)
2 1

𝑦 −𝑘
Slope of AB = 𝑥1−ℎ
1

𝑦 −𝑘
Slope of BC = 𝑥1−ℎ
1

𝑦 −𝑘
Slope of AC = 𝑥2−ℎ =
2

𝑦1 −𝑘 𝑦 −𝑘 𝑦 −𝑘
= 𝑥1−ℎ = 𝑥2−ℎ
𝑥1 −ℎ 1 2

Now Cross multiplying the first two equality,

(y1-k)(x2-x1) = (x1-h)(y2-y1)
(h-x1)(y2-y1) = (k-y1)(x2-x1)
Hence proved.
6. Find the slope of the line which makes an angle of 300 with the positive direction of the y-axis, measured
anticlockwise.
Answer: According to the given figure, the angle made by the line from X-axis is 90+30 = 120°
Slope =(y2-y1/x2-x1)
We also know that slope of a line is equal to tanθ, Where θ = 120° tan (120°) = tan (90°+30°) = -cot (30°) = -√3
Therefor the slope of the given line is -√3.
1
7. Find the angle between the lines whose slopes are, √3𝑎𝑛𝑑
√3

Answer: To find out the angle between two lines, the angle is equal to the difference in θ.
𝑦 −𝑦
The slope of a line = tan𝜃 =( 𝑥2−𝑥1)
2 1

So slope of the first line = √3 = tan𝜃


𝜃1 = 600
The slope of the second line is = 1/√3
𝜃1 = 300
Now the difference between the two lines is θ1-θ2
= 60°-30°
= 30°
8. Find the equation of a line which is equidistant from the lines x = - 2 and x = 6.
Answer: For the equation of line equidistant from both lines, we will find point through which line passes and
is equidistant from both line. As any point lying on x = - 2 line is ( - 2, 0) and on x = 6 is (6, 0), so mid - point is

149
−2+6 0+0
(x,y) =( , )
2 2

(x, y) = (2,0)
So, equation of line is x = 2.
9. Find the equation of a line passing through the origin and making an angle of 1200 with the positive direction
of the x - axis.
Answer: As angle is given so we have to find slope first give by m = tanθ m = tan120°
m = tan(1800-600) = - tan600 = -√3
(tan (180° - θ) is in II quadrant, tanx is negative) Now equation of line passing through origin is given as y = mx
y=-√3𝑥
y+√3𝑥 = 0
so the required equation of line is y+√3𝑥 = 0
10. Find the slope and the equation of the line passing through the points: (i) (3, - 2) and ( - 5, - 7).
Answer: Slope of equation can be calculated using
𝑦 −𝑦
m = (𝑥2 −𝑥1)
2 1

m = 5/8
now using two point formula of the equation of a line
𝑦 −𝑦
y- y1 = 𝑥2−𝑥1( x-x1)
2 1

y – (-2) = 5/8 (x-3)


8 (y+ 2) = 5 ( x-3)
8y + 16 = 5x -15
5x-8y-16-15 = 0
5x-8y -31 = 0
So the required equation of the line is 5x-8y -31 = 0
LONG ANSWER TYPE QUESTION
1. Find the area of the quadrilateral whose vertices are A(-4, 5), B(0, 7), C(5, -5) and D(-4, -2).
Answer: Given: The vertices of the quadrilateral are A(-4, 5), B(0, 7), C(5, -5) and D(-4, -2).
Area of quadrilateral ABCD = Area of Δ ABC + Area of Δ ADC
area of ABC = 1/2[x1(y2-y3) + x2(y3-y1) + x3( y1-y2)]
area of quadrilateral ABCD = Area of ABC +Area of ACD
= 1/2 [-4(7+5) + 0 + 5(5-7)]
150
= 1/2 [-48-10]
= -29
Taking modulus (∵ area is always positive),
Area of Δ ABC = 29 sq. units …..(1)
Area of ACD = 1/2 [ -4 ( -2 + 5) -4 ( -5-5 ) + 5 ( 5+2)]
= 31.5 sq. units …..(2)
From 1 and 2,
Area of quadrilateral ABCD = 29 + 31.5
= 60.5 square units.
Therefore, the area of quadrilateral ABCD is 60.5 square units.
2. Find the area of ΔABC, the midpoints of whose sides AB, BC and CA are
D(3, -1), E(5, 3) and F(1, -3) respectively.
Answer:
The figure is as shown.
x1 + x2 = 2 × 3 = 6 …..(1)
x1 + x3 = 2 × 1 = 2 …..(2)
x2 + x3 = 2 × 5 = 10 …..(3)
Equation 1 – Equation 2 gives us
x2 – x3 = 4 …..(4)
Equation 3 + Equation 4,
2x2 = 14 ⇒ x2 = 7
∴ x1 = -1 and x3 = 3
Similarly,
y1 + y2 = 2 × -1 = -2 …..(5)
y1 + y3 = 2 × -3 = -6 …..(6)
y2 + y3 = 2 × 3 = 6 …..(7)
Equation 5 – Equation 6 gives us
y2 – y3 = 4 …..(8)
Equation 7 + Equation 8,
2y2 = 10 ⇒ y2 = 5
∴y1=-7 and
151
y3=1
area of ABC = 1/2[x1(y2-y3) + x2(y3-y1) + x3( y1-y2)]
= ½ [-1(5-1) + 7(1+7) +3 (-7-5)]
=1/2 (-4+56-36)
=8
3. Find the coordinates of the point which divides the join of A(-5, 11) and B(4, -7) in the ratio 2 : 7.
Answer: Let P(x, y) be the point that divides the join of A(-5, 11) and B(4, -7) in the ratio 2 : 7
Formula: If m1 : m2 is the ratio in which the join of two points is divided by another point (x, y), then
𝑚1 𝑥2 + 𝑚2 𝑥1
x= 𝑚1 +𝑚2
𝑚1 𝑦2 + 𝑚2 𝑦1
y= 𝑚1 +𝑚2

Here, x1 = -5, x2 = 4, y1 = 11 and y2 = -7 Substituting,


2.4+7.−5
x = 2+7

x = -3
2.−7+7.11
y= 2+7

y=8
Therefore, the coordinates of the point which divided the join of A(-5, 11) and B(4, -7) in the ratio 2 : 7 is
(-3, 8).
4. Find the ratio in which the x-axis cuts the join of the points A(4, 5) and B(-10, -2). Also, find the point of
intersection.
Answer: Let the point which cuts the join of A(4, 5), and B(-10, -2) in the ratio k : 1 be
P(x, 0).
Formula: If k : 1 is the ratio in which the join of two
points is divided by another point (x,
y), then
𝑘𝑥2 +𝑥1
x= 𝑘+1
𝑘𝑦2 +𝑦1
y== 𝑘+1

Taking for the y co-ordinate


𝑘.(−2)+5
0= 𝑘+1

2k = 5
152
k=5/2
5
(−10)+4
therefore x = 2 5
+1
2

x = -6
Therefore, the ratio in which x-axis cuts the join of the points A(4, 5) and B(-10, -2) is 5 : 2and the point of
intersection is (-6, 0).
5. In what ratio is the line segment joining the points A(-4, 2) and B(8, 3) divided by the y-axis? Also, find the
point of intersection.
Answer: Let the point which cuts the join of A(-4, 2) and B(8, 3) in the ratio k : 1 be P(0, y)
Formula: If k : 1 is the ratio in which the join of two points are divided by another point
(x, y), then
𝑘𝑥2 +𝑥1
x= 𝑘+1
𝑘𝑦2 +𝑦1
y== 𝑘+1

Taking for the x co-ordinate,


𝑘.8+(−4)
0 = 𝑘+1

8k = 4
K = 1/2
Therefore,
3
+2
Y = 21
+1
2

7
Y=3

Therefore, the ratio in which the line segment joining the points A(-4, 2) and B(8, 3) divided by the y-axis is 1 : 2
7
and the point of intersection is (0, 3).

CASE BASED QUESTIONS


1. Draw a quadrilateral in the Cartesian plane, whose vertices are (–4, 5), (0, 7), (5, –5) and (– 4, –2). Also,
find its area.

153
Answer: Let ABCD be the given quadrilateral with vertices A (–4, 5), B (0, 7), C (5, –5), and D (–4, – 2). Then,
by plotting A, B, C, and D on the Cartesian plane and joining AB, BC, CD, and DA, the given quadrilateral can
be drawn as
To find the area of quadrilateral ABCD, we draw one diagonal, say AC.
Accordingly, area (ABCD) = area (ΔABC) + area (ΔACD).
We know that the area of a triangle whose vertices are (x1, y1),
(x2, y2), and (x3, y3) is
1
|𝑥1 (𝑦2-𝑦3 )+𝑥2 (𝑦3 -𝑦1 )+𝑥3 (𝑦1 -𝑦2 )|
2

Therefore, area of ΔABC

1
| − 4(7+5)+ 0(−5-5)+ 5(5-7)| unit2
2
1
= 2 | − 4(12)+ 5(-2)| unit2
1
= 2 | − 48 - 10| unit2
1
= 2 | − 58 | unit2
1
= 2 × 58 unit2

= 29 unit2
Area of ΔACD
1
= 2 | − 4(5+2)+ 5(−2-5)+ (−4)(5+5)| unit2
1
=2 | − 4(−3)+ 5(−7)+ (−4)(10)| unit2
1
=2 |12 − 35 − 40| unit2
1
=2 | − 63| unit2
−63
= unit2
2
63 58+63 121
Thus, area (ABCD) =(29 + ) unit2 = unit2 = unit2
2 2 2

2. Assuming that straight lines work as the plane mirror for a point, find the image of the point (1, 2) in the

line x – 3y + 4 = 0.

Let Q (h, k) is the image of the point P (1, 2) in the line


154
x – 3y + 4 = 0 ... (1)
Therefore, the line (1) is the perpendicular bisector of line segment PQ.
−1
Hence slope of line PQ = 𝑆𝑙𝑜𝑝𝑒𝑜𝑓𝑙𝑖𝑛𝑒𝑥−3𝑦+4=0
𝑘−2 −1
So that ℎ−1 = 1 𝑜𝑟 3ℎ + 𝑘 = 5 … (2)
3

ℎ+1 𝑘+2
And the mid-point of PQ, i.e., point( , ) will satisfy the equation (1) so that
2 2
ℎ+1 𝑘+2
−3( ) + 4 = 0 or h-3k = -3 … (3)
2 2
6 7
Solving (2) and (3) we get, ℎ = 𝑎𝑛𝑑𝑘 =
5 5
6 7
Hence, the image of point (1, 2) in the line (1) is, (5 , 5)

3. Show that the path of a moving point such that its distances from two lines 3x – 2y = 5 and 3x + 2y = 5 are
equal is a straight line.

Given lines are 3x – 2y = 5 … (1) And 3x + 2y = 5 … (2) Let (h, k) is any point, whose distances from the lines (1)
and (2) are equal. Therefore

which gives 3h – 2k – 5 = 3h + 2k – 5 or – (3h – 2k – 5) = 3h + 2k –


5.
Solving these two relations we get k = 0 or h =3/5. Thus, the
point (h, k) satisfies the equations y = 0 or x =3/5 which
represent straight lines. Hence, path of the point equidistant
from the lines (1) and (2) is a straight line.

4. Find the distance of the line 4x – y = 0 from the point P (4, 1)


measured along the line making an angle of 135° with the
positive x-axis.

Given line is 4x – y = 0 ... (1).

155
In order to find the distance of the line (1) from the point P (4, 1) along another line, we have to find the point
of intersection of both the lines.
For this purpose, we will first find the equation of the second line Slope of second line is tan 135° = –1.
Equation of the line with slope – 1 through the point P (4, 1) is y – 1 = – 1 (x – 4) or x + y – 5 = 0 ... (2)

Solving (1) and (2), we get x = 1 and y = 4


so that point of intersection of the two lines is Q (1, 4).
Now, distance of line (1) from the point P (4, 1) along the line (2) the distance between the points P (4, 1) and
Q (1, 4).

PQ = √(1 − 4)2 + (4 − 1)2

= √(−3)2 + (3)2
= √9 + 9
= 3√2

5. The slope of a line is double of the slope of another line. If tangent of the angle between them is 1/3.find
the slopes of the lines
Let m1 and m be the slopes of the two given lines such that m1 = 2m
We know that if 𝜃 is the angle between the lines l1 and l2 with slopes m1 and m2 m2
𝑚 −𝑚1
2
Then tan𝜃 = |1+𝑚 |
1 𝑚2

It is given that the tangent of the angle between two lines is 1/3
1 𝑚 − 2𝑚
= | |
3 1 + 2𝑚2
1/3 = |-m/1+2m2|
1/3=-m/1+2m2 or 1/3=m/1+2m2

Case 1

156
Case 2

MCQ
1. The equation of the line joining the points ( a , 0 ) and ( 0 , b ) is
A) ax + by = 0 B) bx + ay = 0
C) x/b + y/a = 1 D) x/a + y/b = 1 (D)
2. The equation of a line through ( 1 , -2) and which makes equal intercepts on the axes is
A) x + y =1 B) x - y =1 C) x + y +1 =0 D) x - y - 2 =0 (C)
3. A line passes through the point ( 2 , 2 ) and is at right angles to the line 3x + y =3 . Its y-intercept is (C)
A) - 4 B) 4/3 C) - 4/3 D) None of these
4. The inclination of the line x – y + 3 = 0 with the positive direction of x-axis is
(A) 45° (B) 135° (C) – 45° (D) –135° (A)
5. The coordinates of the foot of the perpendicular from the point (2, 3) on the line x + y – 11 = 0 are
(A) (–6, 5) (B) (5, 6) (C) (–5, 6) (D) (6, 5) (B)

157
6. A line passes through P (1, 2) such that its intercept between the axes is bisected at P. The equation of the
line is
(A) x + 2y = 5 (B) x – y + 1 = 0
(C) x + y – 3 = 0 (D) 2x + y – 4 = 0 (A)
7. The reflection of the point (4, – 13) about the line 5x + y + 6 = 0 is
(A) (– 1, – 14) (B) (3, 4) (C) (0, 0) (D) (1, 2) (A)
8. Slope of a line which cuts off intercepts of equal lengths on the axes is
(A) – 1 (B) – 0 (C) 2 (D) 3 (A)
9. The equation of the line passing through the point (1, 2) and perpendicular to
the line x + y + 1 = 0 is
(A) y – x + 1 = 0 (B) y – x – 1 = 0 (A)
(C) y – x + 2 = 0 (D) y – x – 2 = 0
10. If the coordinates of the middle point of the portion of a line intercepted
between the coordinate axes is (3, 2), then the equation of the line will be
(A) 2x + 3y = 12 (B) 3x + 2y = 12
(C) 4x – 3y = 6 (D) 5x – 2y = 10 (A)

ASSERTION AND REASON


Read assertion and reason carefully and write correct option for each question
(a) Both A and R are correct, R is the correct explanation of A
(b) Both A and R are correct, R is not the correct explanation of A
(c) A is correct, R is incorrect
(d) R is correct, A is incorrect
1. ASSERTION: Slope of a line passing throught the two point (2,2) and (4,4) is 1
REASON: A line whose equation is y=x have slope 1 (A)
2. Assertion: Slope of x axis is 0 and slope of y axis is not defined.
Reason: Slope of y axis is 0 and slope of x axis is not defined. (C)
3. Assertion: Line 3x-4y = 6 have slope 3/4.
Reason: Line 3x-4y = 6 have x intercept is 2 and y intercept is – 3/2. (A)
4. Assertion: The distance between line 4x+3y=15 and 8x+6y=7 is 23/10.

158
𝑐−𝑑
Reason: the distance between line ax+by = c and ax+by = d is |√𝑎2 | (A)
+𝑏2

5. Assertion: Three points are co- linear if they lies in a line. (A)
Reason: Three points are co- linear if the area of triangle is 0.

SHORT ANSWER TYPE QUESTIONS


1. Find the equation of line intersecting the y-axis at a distance of 2 units above the origin and making an angle
of 300 with positive direction of the x-axis.
Answer : x-√3y+2√3 = 0
2. Find the equation of line when a point R (h, k) divides a line segment between the axes in the ratio 1: 2.
Answer : 2kx+hy =3hk
3. Find angles between the lines √3𝑥 + 𝑦 = 1 𝑎𝑛𝑑𝑥 + √3𝑦 = 1
Answer : 300 or 1500
4. The line through the points (h, 3) and (4, 1) intersects the line 7x-9y-19=0 at right angle. Find the value of
h.
Answer : 22/9
5. Find the distance between the lines 3x + 4y = 9 and 6x + 8y = 15.
Answer : 3/10
6. A line which made intercept on the axis are a and b, and p be the perpendicular distance from origin. Then
find the value of p-2 .
Answer : p-2=a-2+b-2
7. Find the centroid of the triangle whose vertex are A ( 2, 3) , B ( 4 - 2 ) and C( -5 , 1 ).
Answer : G(1/3 , 2/3)
8. If the angle between two lines is π /4 and the slope of one of the lines 1⁄2 . Find the slope of the other line.
Answer : 3 and -1/3
9. Lines through the points ( -2 , 6 ) and ( 4 , 8 ) is perpendicular to the line through the points ( 8 , 12 ) and ( x
, 24 ) find x .
Answer : x=4
10. Write the equation of the lines for which tan θ = 1⁄2 where θ is the inclination of the line an
i. y- intercept is – 3/2
ii. x- intercept is 4

159
Answer: i ) 2y = x-3
Answer : ii ) 2y= x-4
LONG ANSWER TYPE QUESTIONS
1. Find the equation of the line passing through the point ( 2 , 2 ) and cutting off intercepts on the axes
whose sum is 9 .
Answer : 2x+y-6=0 and x+2y-6=0
2. In a triangle ABC with vertices A( 2,3 ) B( 4 , -1 ) and C ( 1 , 2 ) , find the equation and length of altitude
from the vertex A .
Answer : y-x=1
3. Find the equation to the straight line passing through the point of intersection of the lines 5x – 6y – 1 = 0 and
3x + 2y + 5 = 0 and perpendicular to the line 3x – 5y + 11 = 0.
Answer : 5x+3y+8=0
4. The slope of a line is double the slope of another line . If the tangent of the angle between them is 1/3 . Find
the slope of the lines .
Answer : -1 and -2 or -1/2 and -1 or 1 and 2 or 1/2 and 1
5. The area of a triangle is 5. Two of its vertices are (2, 1) & (3, – 2). The third vertex lies on y = x + 3. Find the
third vertex.
Answer : (7/2 , 13/2 ) and (-3/2, 3/2)
CASE BASED QUESTIONS
1. The base of an equilateral triangle with side 2a lies along Y-axis such that the midpoint of the base is at
the origin. Find vertices of the triangle.
Answer : (0, a ) , (0,-a) (±√3 , 0 )
2. The owner of a milk store finds that, he can sell 980 litres of milk each week at Rs 14/litre and 1220 litres
of milk each week at Rs 16/litre. Assuming a linear relationship between selling price and demand, how
many litres could he sell weekly at Rs 17/litre?
Answer: 1340
3. A ray of light passing through the point (1, 2) reflects on the x-axis at point A and the reflected ray passes
through the point (5, 3). Find the coordinates of A.
Answer : (13/5,0)

160
4. A person standing at the junction (crossing) of two straight paths represented by the equations 2x – 3y
+ 4 = 0 and 3x + 4y – 5 = 0 wants to reach the path whose equation is 6x – 7y + 8 = 0 in the least time.
Find equation of the path that he should follow.
Answer : 119x+102y =125

5. Find the product of the lengths of the perpendiculars drawn from the points (√𝑎2 − 𝑏 2 , 0)and

(−√𝑎2 − 𝑏 2 , 0)to the line (x/a )cosθ+(y/b)sinθ = 1.


Answer : b2

161
CONIC SECTIONS
CONCEPTS:
CIRCLES
A circle is a locus of a point which moves in a plane such that its distance from a fixed point in that plane is
always constant. The fixed point is said to be the centre of the circle and the constant distance is said to be
the radius of the circle.
** The equation of a circle with centre (h, k) and the radius r is given by (x – h)2 + (y – k)2 = r2.
** The equation of a circle with centre (0, 0) and the radius r is given by x 2 + y2 = r2.

** General equation of a circle is x2 + y2 + 2gx + 2fy + c = 0 whose centre is (–g, –f) and radius is g2 + f 2 − c

** Equation of a circle when end points of diameter as A(x1, y1), B(x2, y2) is given by
(x – x1) (x – x2) + (y – y1) (y – y2) = 0.

** Length of intercepts made by the circle x2 + y2 + 2gx + 2fy + c = 0 on the X and Y-axes are 2 g − c and
2

2 f2 −c .

CONICS
Conic Section or a conic is the locus of a point which moves in such a way that its distance from a fixed point
bears a constant ratio to its distance from a fixed line.

The fixed point is called the focus, the straight line is called the directrix and the constant ratio
denoted by e is called the eccentricity of the conics.
distance between P(x, y) & Focus
Eccentricity (e) e = distance between P(x, y) & Directrix .
If e = 1, then conic is a parabola.
If e < 1, then conic is an ellipse.
If e > 1, then conic is a hyperbola.
If e = 0, then conic is a circle.
PARABOLA
A parabola is the set of all points in a plane that are equidistant from a fixed line and a fixed point in the plane

162
S.No y2 = 4ax y2 = -4ax x2 = 4ay x2 = -4ay

1. Vertex (0,0) (0,0) (0,0) (0,0)


2. Focus (a,0) (-a,0) (0,a) (0,-a)
3 Equation of x+a=0 x–a=0 y+a=0 y–a=0
directrix
4. Equation of (x-axis), y = 0 (x-axis), y = 0 (y-axis), x = 0 (y-axis), x = 0
Axis
5. Length of 4a 4a 4a 4a
Latus Rectum
6. Focal x+a x–a y+a y–a
distance of a
p(x, y)

** Latus rectum of a parabola is a line segment perpendicular to the axis of the parabola, through the focus
and whose end points lie on the parabola.

** Position of a point with respect to a parabola: The point (x1, y1) with respect to the parabola y2 = 4ax lies

(i) outside , if y1 − 4ax1  0 (ii) on it if y1 − 4ax1 = 0 (iii)inside if y1 − 4ax1  0 .


2 2 2

ELLIPSE:

An ellipse is the set of all points in a plane, the sum of whose distances from two fixed points in the plane is a
constant.

163
S.
No.

1. Equation x 2 y2 x 2 y2
+ =1 + =1
a 2 b2 b2 a 2
2. Centre (0 , 0) (0 , 0)
3. Vertices (± a , 0) (0 , ± a)
4. Foci (± ae, 0) or (± c , 0)where c2 = a2 – b2 (0 ± ae) or (0 ± c)where c2 = a2 – b2
5. Eccentricity e = c/a e = c/a
6. Length of major axis 2a 2a
7. Length of minor axis 2b 2b
8. Length of latus 2b2 2b2
rectum a a

HYPERBOLA
A hyperbola is the set of all points in a plane, the difference of whose distances from two fixed points in the
plane is a constant.

S.
N

1 Equation x 2 y2 y2 x 2
− =1 − =1
a 2 b2 a 2 b2
2. Vertices (± a, 0) (0, ± a)
3. Foci (± ae, 0) or (± c , 0) (0 ± ae) or (0 ± ae)
where c2 = a2 + b2 where c2 = a2 + b2
4. Eccentricity e = c/a e = c/a

164
5. Centre (0, 0) (0, 0)
6. Length of Transverse axis 2a 2a
7. Length of conjugate axis 2b 2b
8. Length of latus rectum 2b 2 2b 2
a a
9. Equation of Transverse axis y=0 x=0
10. Equation of conjugate axis x=0 y=0
Competency Based Exemplar Questions
MCQs Marks

1 The equation y2 - x 2 + 2x - 1 = 0 represents [1]


a) a pair of straight lines
b) a circle
c) a parabola
d) an ellipse
2 The length of the latus rectum of the ellipse 3x2 + y 2 = 12 is [1]
a) 3
4
b)
√3

c) 4
d) 8
3 The line 2x - y + 4 = 0 cuts the parabola y2 = 8x in P and Q. The mid - point of [1]
PQ is
a) (1, - 2)
b) (1, 2)
c) ( - 1, - 2)
d) ( - 1, 2)
4 If the parabola y2 = 4ax passes through the point (3, 2), then the length of its [1]
latus rectum is
a) 4
2
b) 3

165
4
c) 3
1
d) 3

5 At what point on the parabola y2 = 4x the normal makes equal angles with the [1]
axes?
a) (4, 4)
b) (9, 6)
c) (1, - 2)
d) (4, - 4)
6 The length of latus rectum of an ellipse is one - third of its major axis. Its [1]
eccentricity would be
2
a) 3

2
b) √3
1
c)
√3
1
d)
√2
3
7 Equation of the hyperbola with eccentricity2 and foci at (± 2, 0) is [1]
𝑥2 𝑦2 4
a) − =9
9 9
𝑥2 𝑦2
b) 16 − 27 = 1
𝑥2 𝑦2
c) − =1
4 9
𝑥2 𝑦2 4
d) 4
− 5
=9

8 If (2, 4) and (10, 10) are the ends of a latus rectum of an ellipse with eccentricity [1]
1
, then the length of semi - major axis is
2
40
a) 3
20
b) 7
15
c) 3
20
d) 3

166
9 The slope of the tangent at the point (a, a) of the circle 𝑥 2 + 𝑦 2 = 𝑎2 is [1]
a) - 2
b) - 1
c) 0
d) depends upon h
10 The equation x2 + y 2 + 2x - 4y + 5 = 0 represents. [1]
a) A circle of non - zero radius
b) A circle of zero radius
c) A pair of straight lines
d) A point
ASSERTION AND REASONING TYPE QUESTIONS

11 If the distances of foci and vertex of hyperbola from the centre are c and a [1]
respectively, then
Assertion (A): Eccentricity is always less than 1.
Reason (R): Foci are at a distance of ae from the centre.
a) Both A and R are true and R is the correct explanation of A.
b) Both A and R are true but R is not the correct explanation of A.
c) A is true but R is false.
d) A is false but R is true.
12 Assertion (A): The length of major and minor axes of the ellipse 5x 2 + 9y 2 - [1]
54y + 36 = 0 are 6 and 10, respectively.
Reason (R): The equation 5x 2 + 9y 2 - 54y + 36 = 0 can be expressed as 5x 2 +
9(y - 3) 2 = 45.
a) Both A and R are true and R is the correct explanation of A.
b) Both A and R are true but R is not the correct explanation of A.
c) A is true but R is false.
d) A is false but R is true.
13 Assertion (A): The sum of focal distances of a point on the ellipse 9x 2 + 4y 2 - [1]
18x - 24y + 9 = 0 is 4.

167
Reason (R): The equation 9x 2 + 4y 2 - 18x - 24y + 9 = 0 can be expressed as
9(x - 1) 2 + 4(y - 3) 2 = 36.
a) Both A and R are true and R is the correct explanation of A.
b) Both A and R are true but R is not the correct explanation of A.
c) A is true but R is false.
d) A is false but R is true.
14 Parabola is symmetric with respect to the axis of the parabola. [1]
Assertion (A): If the equation of standard parabola has a term y 2 , then the axis
of symmetry is along the x - axis.
Reason (R): If the equation of standard parabola has a term x 2 , then the axis of
symmetry is along the x - axis.
a) Both A and R are true and R is the correct explanation of A.
b) Both A and R are true but R is not the correct explanation of A.
c) A is true but R is false.
d) A is false but R is true.
15 Assertion (A): The circle x 2 + y 2 + 2ax + c = 0, x 2 + y 2 + 2by + c = 0 touch if [1]
1 1 1
+ 𝑏2 = 𝑐
𝑎2

Reason (R): The circles with centre C 1 , C 2 and radii r 1 , r 2 touch each other if
r1 ±r2 =C1 C2
a) Both A and R are true and R is the correct explanation of A.
b) Both A and R are true but R is not the correct explanation of A.
c) A is true but R is false.
d) A is false but R is true.
SHORT ANSWER TYPE QUESTIONS

16 If the distance between the foci of an ellipse is equal to the length of the latus [2]
rectum, write the eccentricity of the ellipse.

17 The circle x2 + y 2 - 2x - 2y + 1 = 0 is rolled along the positive direction of x - [2]


axis and makes one complete roll. Find its equation in new - position.

168
18 Find the lengths of major and minor axes, coordinates of foci, vertices and the [2]
eccentricity: 3x2 + 2y 2 = 6.

19 Find the equation of the hyperbola whose focus is at (5, 2), vertex at (4, [2]
2)andcentre at (3, 2).

20 Find the equation of the ellipse whose vertices are (± 13, 0) and foci are ( ± 5, [2]
0).

21 Find the vertex, focus, axis, directrix and latus - rectum of the following [2]
parabolas y2 - 4y - 3x + 1 = 0

22 Write the equation of the parabola with focus (0, 0) and directrix x + y - 4 = 0. [2]

23 Find the distance between the directrices of the hyperbola x2 - y 2 = 8. [2]

24 Find the equation of the ellipse the ends of whose major and minor axes [2]
are(±4,0) and (0, ±3) respectively.

25 ABCD is a square whose side is a; taking AB and AD as axes, prove that the [2]
equation of the circle circumscribing the square is x2 + y 2 - a (x + y) = 0.
CASE STUDY BASED QUESTIONS

26 Read the text carefully and answer the questions: Arun is running in a [4]
racecourse note that the sum of the distances from the two flag posts from him
is always 10 m and the distance between the flag posts is 8 m.

1. Path traced by Arun represents which type of curve. Find the length of
major axis?

2. Find the equation of the curve traced by Arun?

3. Find the eccentricity of path traced by Arun?

4. Find the length of latus rectum for the path traced by Arun.

169
27 Read the text carefully and answer the questions: Indian track and field athlete [4]
Neeraj Chopra, who competes in the Javelin throw, won a gold medal at Tokyo
Olympics. He is the first track and field athlete to win a gold medal for India at
the Olympics.

1. Name the shape of path followed by a javelin. If equation of such a


curve is given by x2 = - 16y, then find the coordinates of foci.

2. Find the equation of directrix and length of latus rectum of parabola x2 =


- 16y.

3. Find the equation of parabola with Vertex (0,0), passing through (5,2)
and symmetric with respect to y - axis and also find equation of
directrix.

4. Find the equation of the parabola with focus (2, 0) and directrix x = - 2
and also length of latus rectum.

28 Read the text carefully and answer the questions: The girder of a railway [4]
bridge is a parabola with its vertex at the highest point, 10 m above the ends.
Its span is 100 m.

1. Find the coordinates of the focus of the parabola.

170
2. Find the equation of girder of bridge and find the length of latus rectum
of girder of bridge.

3. Find the height of the bridge at 20m from the mid - point.

4. Find the radius of circle with centre at focus of the parabola and passes
through the vertex of parabola.

29 Read the text carefully and answer the questions: A satellite dish has a shape [4]
called a paraboloid, where each cross section is parabola. Since radio signals
(parallel to axis) will bounce off the surface of the dish to the focus, the receiver
should be placed at the focus. The dish is 12 ft across, and 4.5 ft deep at the
vertex.

1. Name the type of curve given in the above paragraph and find the
equation of curve?

2. Find the equation of parabola whose vertex is (3, 4) and focus is (5, 4).

3. Find the equation of parabola Vertex (0, 0) passing through (2, 3) and
axis is along x - axis. and also find the length of latus rectum.

4. Find focus, length of latus rectum and equation of directrix of the


parabola x2 = 8y.

30 Read the text carefully and answer the questions: A farmer wishes to install 2 [4]
hand pumps in his field for watering.

171
The farmer moves in the field while watering in such a way that the sum of
distances between the farmer and each handpump is always 26m. Also, the
distance between the hand pumps is 10 m.

1. Name the curve traced by farmer and hence find the foci of curve.

2. Find the equation of curve traced by farmer.

3. Find the length of major axis, minor axis and eccentricity of curve along
which farmer moves.

4. Find the length of latus rectum.

LONG ANSWER TYPE QUESTIONS

31 Find the equation of the ellipse whose foci are (4, 0) and ( - 4, 0), eccentricity = [5]
1/3.

32 Find the equation of a circle passing through the points (2, - 6), (6, 4) and ( - 3, [5]
1).

33 Find the equation of the hyperbola, the length of whose latus rectum is 4 and [5]
the eccentricity is 3.

34 Find the equation of a circle concentric with the circle x2 + y $^{$ + 4x + 6y + 11 [5]
= 0 and passing through the point (5, 4).

35 Find the (i) lengths of major and minor axes, (ii) coordinates of the vertices, (iii) [5]
coordinates of the foci, (iv) eccentricity, and (v) length of the latus rectum of
𝑥2 𝑦2
ellipse: 25 + 9
= 1.

172
SOLUTIONS:
MCQs Marks

1 The equation y2 - x 2 + 2x - 1 = 0 represents [1]


a) a pair of straight lines
2 The length of the latus rectum of the ellipse 3x2 + y 2 = 12 is [1]
4
b)
√3

3 The line 2x - y + 4 = 0 cuts the parabola y2 = 8x in P and Q. The mid - point of PQ is [1]
d) ( - 1, 2)
4 If the parabola y2 = 4ax passes through the point (3, 2), then the length of its latus [1]
rectum is
4
c) 3

5 At what point on the parabola y2 = 4x the normal makes equal angles with the [1]
axes?
c) (1, - 2)
6 The length of latus rectum of an ellipse is one - third of its major axis. Its [1]
eccentricity would be
2
b) √3
3
7 Equation of the hyperbola with eccentricity2 and foci at (± 2, 0) is [1]
𝑥2 𝑦2 4
d) − =9
4 5
1
8 If (2, 4) and (10, 10) are the ends of a latus rectum of an ellipse with eccentricity 2 , [1]

then the length of semi - major axis is


20
d) 3

9 The slope of the tangent at the point (a, a) of the circle 𝑥 2 + 𝑦 2 = 𝑎2 is [1]
b) - 1
10 The equation x2 + y 2 + 2x - 4y + 5 = 0 represents. [1]
d) A point

173
ASSERTION AND REASONING TYPE QUESTIONS
11 If the distances of foci and vertex of hyperbola from the centre are c and a [1]
respectively, then
Assertion (A): Eccentricity is always less than 1.
Reason (R): Foci are at a distance of ae from the centre.
d) A is false but R is true.
12 Assertion (A): The length of major and minor axes of the ellipse 5x 2 + 9y 2 - 54y + [1]
36 = 0 are 6 and 10, respectively.
Reason (R): The equation 5x 2 + 9y 2 - 54y + 36 = 0 can be expressed as 5x 2 + 9(y -
3) 2 = 45.
d) A is false but R is true.
13 Assertion (A): The sum of focal distances of a point on the ellipse 9x 2 + 4y 2 - 18x [1]
- 24y + 9 = 0 is 4.
Reason (R): The equation 9x 2 + 4y 2 - 18x - 24y + 9 = 0 can be expressed as 9(x -
1) 2 + 4(y - 3) 2 = 36.
d) A is false but R is true.
14 Parabola is symmetric with respect to the axis of the parabola. [1]
Assertion (A): If the equation of standard parabola has a term y 2 , then the axis of
symmetry is along the x - axis.
Reason (R): If the equation of standard parabola has a term x 2 , then the axis of
symmetry is along the x - axis.
c) A is true but R is false.
1
15 Assertion (A): The circle x 2 + y 2 + 2ax + c = 0, x 2 + y 2 + 2by + c = 0 touch if 𝑎2 + [1]
1 1
=𝑐
𝑏2

Reason (R): The circles with centre C 1 , C 2 and radii r 1 , r 2 touch each other if r

1 ±r2 =C1 C2
a) Both A and R are true and R is the correct explanation of A.

174
SHORT ANSWER TYPE QUESTIONS
16 If the distance between the foci of an ellipse is equal to the length of the latus [2]
rectum, write the eccentricity of the ellipse.
Ans:
Given that, the distance between the foci of an ellipse is equal to the length of the
latus rectum.

2𝑏 2
i.e. = 2𝑎𝑒
𝑎

𝑏2
⇒𝑒=
𝑎2
𝑏2
But, 𝑒 = √1 − 𝑎2

Hence, 𝑒 = √1 − 𝑒

Squaring both sides, we get:

𝑒2 + 𝑒 − 1 = 0
−1±√1+4
⇒𝑒= (∵ Eccentricity cannot be negative )
2

√5 − 1
⇒𝑒=
2

17 The circle x2 + y 2 - 2x - 2y + 1 = 0 is rolled along the positive direction of x - axis [2]


and makes one complete roll. Find its equation in new - position.
Ans:
It is told that the circle is rolled along the positive direction of the 𝑥-axis and makes
one complete roll.

We know that the complete roll of a circle covers the distance 2𝜋𝑟, where 𝑟 is the
radius of the circle.

The centre of the circle moves 2𝜋𝑟𝑛 the positive direction of the 𝑥-axis.

175
Let the 𝑑 be the distance moved by the centre on completion of one roll.

𝑑 = 2𝜋𝑟(1)
⇒ 𝑑 = 2𝜋
The new position of the centre is (1 + 𝑑, 1) = Centre = (1 + 2𝜋, 1)

We know that the equation of the circle with centre(𝑝, 𝑞) and having radius ' 𝑟 ' is
given by:

(𝑥 − 𝑝)2 + (𝑦 − 𝑞)2 = 𝑟 2
Now, Equation of circle with centre(1 + 2𝜋, 1) and having radius 1 units.

⇒ (𝑥 − (1 + 2𝜋)2 + (𝑦 − 1)2 = 12
⇒ (𝑥 − 1 − 2𝜋)2 + (𝑦 − 1)2 = 1
⇒ 𝑥 2 − (2 + 4𝜋)𝑥 + (1 + 2𝜋)2 + 𝑦 2 − 2𝑦 + 1 = 1
⇒ 𝑥 2 + 𝑦 2 − (2 + 4𝜋)𝑥 − 2𝑦 − (1 + 2𝜋)2 = 0
The equations of the circles is𝑥 2 + 𝑦 2 − (2 + 4𝜋)𝑥 − 2𝑦 − (1 + 2𝜋)2 = 0

18 Find the lengths of major and minor axes, coordinates of foci, vertices and the [2]
eccentricity: 3x2 + 2y 2 = 6.
Ans:
We have,

3𝑥 2 + 2𝑦 2 = 6
𝑥2 𝑦2
⇒ + =1
2 3
𝑥2 𝑦2
This equation is of the form + 𝑏2 = 1, where 𝑎2 = 2 and 𝑏 2 = 3 i.e. 𝑎 = √2
𝑎2

and 𝑏 = √3.

Clearly, 𝑎 < 𝑏, so the major and minor axes of the given ellipse are along 𝑦 and 𝑥-
axes respectively.
∴ Length of the major axis = 2𝑏 = 2√3

and Length of the minor axis = 2𝑏 = 2√2

176
The coordinates of the vertices = (0, 𝑏) and (0, −𝑏) = (0, √3) and (0, −√3)

𝑎2 2 1
The eccentricity e of the ellipse is 𝑒 = √1 − 𝑏2 = √1 − 3 =
√3

The coordinates of the foci = (0, be ) and (0, −𝑏𝑒) = (0,1) and (0, −1).

19 Find the equation of the hyperbola whose focus is at (5, 2), vertex at (4, [2]
2)andcentre at (3, 2).
Ans:
Equation of hyperbola with centre ( 𝑥0 , 𝑦0 ) is given by;

(𝑥 − 𝑥0 )2 (𝑦 − 𝑦0 )2
− =1
𝑎2 𝑏2
Where, focus = (𝑎𝑒 + 𝑥0 , 𝑦0 )

and vertex = (𝑎 + 𝑥0 , 𝑦0 )

∴ ae = 2

and 𝑎 = 1

𝑏 2 = (2)2 − 𝑎2
⇒ 𝑏 2 = (2)2 − (1)2
⇒ 𝑏2 = 3
(𝑥 − 3)2 (𝑦 − 2)2
⇒ − =1
1 3
⇒ 3(𝑥 − 3)2 − (𝑦 − 2)2 = 3

20 Find the equation of the ellipse whose vertices are (± 13, 0) and foci are ( ± 5, 0). [2]
Ans:
𝑥2 𝑦2
Since the vertices are on 𝑥-axis, the equation will be of the form 𝑎2 + 𝑏2 = 1,

where a is the semi-major axis.

177
Given that vertices are (±13,0) and foci are (±5,0)➔ > 𝑎 = 13, 𝑐 = 𝑎𝑒 = ±5

Therefore, from the relation 𝑐 2 = 𝑎2 − 𝑏 2 , we get

25 = 169 − 𝑏 2 , i.e., 𝑏 = 12

𝑥2 𝑦2
Hence the equation of the ellipse is 169 + 144 = 1

21 Find the vertex, focus, axis, directrix and latus - rectum of the following parabolas [2]
y2 - 4y - 3x + 1 = 0
Ans:
21. We are given that:

𝑦 2 − 4𝑦 − 3𝑥 + 1 = 0
⇒ (𝑦 − 2)2 − 4 − 3𝑥 + 1 = 0
⇒ (𝑦 − 2)2 = 3(𝑥 + 1)
⇒ (𝑦 − 2)2 = 3(𝑥 − (−1))
Let 𝑌 = 𝑦 − 2

𝑋 =𝑥+1
Then, we have:

𝑌 2 = 3𝑋
Comparing the given equation with 𝑌 2 = 4𝑎𝑋

3
4𝑎 = 3 ⇒ 𝑎 =
4
∴ Vertex = (𝑋 = 0, 𝑌 = 0) = (𝑥 = −1, 𝑦 = 2)

3 −1
Focus = (𝑋 = 𝑎, 𝑌 = 0) = (𝑥 + 1 = 4 , 𝑦 − 2 = 0) = (𝑥 = , 𝑦 = 2)
4

Equation of the directrix:

𝑥 = −𝑎

178
−3 −7
i.e. 𝑥 + 1 = ⇒𝑥=
4 4

Axis = 𝑌 = 0

i.e. 𝑦 − 2 = 0 ⇒ 𝑦 = 2

Therefore, Length of the latus rectum = 4𝑎 = 3 units

22 Write the equation of the parabola with focus (0, 0) and directrix x + y - 4 = 0. [2]
Ans:
Let 𝑃(𝑥, 𝑦) be any point on the parabola whose focus is 𝑆(0,0) and the directrix is
𝑥+𝑦 =4

Now draw PM perpendicular to 𝑥 + 𝑦 = 4

Therefore, we have: SP = PM

⇒ 𝑆𝑃2 = 𝑃𝑀2

2 2
𝑥+𝑦−4 2
⇒ (𝑥 − 0) + (𝑦 − 0) = ( )
√1 + 1
2
2 2
𝑥+𝑦−4
⇒𝑥 +𝑦 =( )
√2
⇒ 2𝑥 2 + 2𝑦 2 = 𝑥 2 + 𝑦 2 + 16 + 2𝑥𝑦 − 8𝑦 − 8𝑥
⇒ 𝑥 2 + 𝑦 2 − 2𝑥𝑦 + 8𝑥 + 8𝑦 − 16 = 0
This is the required equation of parabola.

179
23 Find the distance between the directrices of the hyperbola x2 - y 2 = 8. [2]
Ans:
Given equation of the hyperbola is 𝑥 2 − 𝑦 2 = 8.

𝑥2 𝑦2
⇒ − = 1, 𝑎 = 2√2, 𝑏 = 2√2
(2√2)2 (2√2)2
√𝑎2 +𝑏 2 √16 2
eccentricity, 𝑒 = = 2√2 = = √2
𝑎 √2

2𝑎
∴ Distance between the directrices = 𝑒

2.2√2
=
√2
= 4 unis

24 Find the equation of the ellipse the ends of whose major and minor axes [2]
are(±4,0) and (0, ±3) respectively.
Ans:
Given that:

End Points of Major Axis = (±4,0) and End Points of Minor Axis = (0, ±3)

∴ The Equation of Ellipse is of the form,

𝑥2 𝑦2
+ =1
𝑎2 𝑏 2
where, 𝑎 is the semi-major axis and 𝑏 is the semi-minor axis.

So, 𝑎 = 4 and 𝑏 = 3

Putting the value of 𝑎 and 𝑏 in Eq. (i), we get

𝑥2 𝑦2 𝑥2 𝑦2
+ = 1 ⇒ + =1
42 32 16 9

180
25 ABCD is a square whose side is a; taking AB and AD as axes, prove that the [2]
equation of the circle circumscribing the square is x2 + y 2 - a (x + y) = 0.
Ans:
We are given that 𝐴𝐵𝐶𝐷 is a square with side 'a' units.

Let 𝐴𝐵 and 𝐴𝐷 represent the 𝑥-axis and the 𝑦-axis, respectively.

Thus, the coordinates of 𝐵 and 𝐷 are ( 𝑎, 0 ) and (0, a), respectively.

The endpoints of the diameter of the circle circumscribing the square are B and D.

Therefore, equation of the circle circumscribing the square is (𝑥 − 𝑎)(𝑥 − 0) +


(𝑦 − 0)(𝑦 − 𝑎) = 0

or, 𝑥 2 + 𝑦 2 − 𝑎(𝑥 + 𝑦) = 0
CASE STUDY BASED QUESTIONS

26 Arun is running in a racecourse such that the sum of the distances from the two [4]
flag posts from him is always 10𝑚 and the distance between the flag posts is 8𝑚.
(i) An ellipse is the set of all points in a plane, the sum of whose distances from two
fixed points in the plane is a constant. Hence path traced by Arun is ellipse.

Sum of the distances of the point moving point to the foci is equal to length of
major axis = 10𝑚

(ii) Given 2𝑎 = 10&2𝑐 = 8

⇒ 𝑎 = 5&𝑐 = 4
𝑐 2 = 𝑎2 + 𝑏 2
⇒ 16 = 25 + 𝑏 2
⇒ 𝑏 2 = 25 − 16 = 9
𝑥2 𝑦2
Equation of ellipse 𝑎2 + 𝑏2 = 1

𝑥2 𝑦2
Required equation is 25 + 9
=1

181
𝑥2 𝑦2
(iii) equation is of given curve is 25 + =1
9

𝑎 = 5, 𝑏 = 3 and given 2𝑐 = 8 hence 𝑐 = 4

𝑐 4
Eccentricity = 𝑎 = 5

𝑥2 𝑦2
(iv) 25 + =1
9

Hence 𝑎 = 5 and 𝑏 = 3

2𝑏 2 2×9 18
Length of latus rectum of ellipse is given by = =
𝑎 5 5

27 Indian track and field athlete Neeraj Chopra, who competes in the Javelin throw, [4]
won a gold medal at Tokyo Olympics. He is the first track and field athlete to win a
gold medal for India at the Olympics.

(i) The path traced by Javelin is parabola. A parabola is the set of all points in a
plane that are equidistant from a fixed line and a fixed point (not on the line) in the
plane.

compare 𝑥 2 = −16𝑦 with 𝑥 2 = −4𝑎𝑦

⇒ −4𝑎 = −16
⇒𝑎=4
coordinates of focus for parabola 𝑥 2 = −4𝑎𝑦 is (0, −𝑎)

⇒ coordinates of focus for given parabola is (0, −4)

(ii) compare 𝑥 2 = −16𝑦 with 𝑥 2 = −4𝑎𝑦

182
⇒ −4𝑎 = −16 ⇒ 𝑎 = 4
Equation of directrix for parabola 𝑥 2 = −4𝑎𝑦 is 𝑦 = 𝑎

⇒ Equation of directrix for parabola𝑥 2 = −16𝑦 is 𝑦 = 4

Length of latus rectum is 4𝑎 = 4 × 4 = 16

(iii)Equation of parabola with axis along 𝑦 - axis

𝑥 2 = 4𝑎𝑦
which passes through (5,2)

⇒ 25 = 4𝑎 × 2
25
⇒ 4𝑎 =
2
hence required equation of parabola is

25
𝑥2 = 𝑦
2
⇒ 2𝑥 2 = 25𝑦
Equation of directrix is 𝑦 = −𝑎

Hence required equation of directrix is 8𝑦 + 25 = 0.

(iv)Since the focus (2,0) lies on the 𝑥-axis, the 𝑥-axis itself is the axis of the
parabola.

Hence the equation of the parabola is of the form either 𝑦 2 = 4𝑎𝑥 or 𝑦 2 = −4𝑎𝑥.

Since the directrix is 𝑥 = −2 and the focus is (2,0), the parabola is to be of the
form 𝑦 2 = 4𝑎𝑥 with 𝑎 = 2.

Hence the required equation is 𝑦 2 = 4(2)𝑥 = 8𝑥

length of latus rectum = 4𝑎 = 8

183
28 (i) From the diagram equation of parabola is 𝑥 2 = −4𝑎𝑦 [4]

Vertex is 10𝑚 hight and spam is 100𝑚

parabola passes through ( 50, −10 )

Hence, 502 = −4𝑎(−10)

⇒ 2500 = 40𝑎
2500
⇒𝑎= = 62.5
40
Hence coordinates of focus = (−𝑎, 0) = (−62.5,0)

2500
(ii) Equation of parabola is 𝑥 2 = −4 ay and 𝑎 = = 62.5
40

2500
Equation is 𝑥 2 = −4 ( )𝑦
40

⇒ 𝑥 2 = −250𝑦
Length of latus rectum is 4𝑎 = 4 × 62.5 = 250𝑚

(iii)Equation parabola 𝑥 2 = −250𝑦

Coordinates of the point at 20𝑚 from mid point = (20, 𝑦)

Substituting in the equation of parabola

⇒ 400 = −250𝑦
−400
⇒𝑦= = −1.6
250
height of the bridge = 10 − 1.6 = 8.4𝑚

(iv)vertex of parabola is (0,0) and focus is (0, −62.5)

⇒ (0, −62.5) is center and (0,0) is on the circle

⇒ 𝑟 = 0 - (-62.5) = 62.5𝑚

184
29 (i) Given curve is a parabola [4]

Equation of parabola is 𝑥 2 = 4𝑎𝑦

It passes through the point (6,4.5)

⇒ 36 = 4 × 𝑎 × 4.5
⇒ 36 = 18𝑎
⇒𝑎=2
Equation of parabola is 𝑥 2 = 8𝑦

(ii) Distance between focus and vertex is = 𝑎 = √(4 − 4)2 + (5 − 3)2 = 2

Equation of parabola is (𝑦 − 𝑘)2 = 4𝑎(𝑥 − ℎ)

where (ℎ, 𝑘) is vertex

⇒ Equation of parabola with vertex (3,4)&𝑎 = 2

⇒ (𝑦 − 4)2 = 8(𝑥 − 3)
(iii)Equation of parabola with axis along 𝑥 - axis

𝑦 2 = 4𝑎𝑥
which passes through (2,3)

⇒ 9 = 4𝑎 × 2
9
⇒ 4𝑎 =
2
hence required equation of parabola is

9
𝑦2 = 𝑥
2
⇒ 2𝑦 2 = 9𝑥
Hence length of latus rectum = 4𝑎 = 4.5

(𝑖𝑣)𝑥 2 = 8𝑦𝑎 = 2

185
Focus of parabola is (0,2),length of latus rectum is 4𝑎 = 4 × 2 = 8

Equation of directrix y +2 = 0

30 (i) The curve traced by farmer is ellipse. Because An ellipse is the set of all points in [4]
a plane, the sum of whose distances from two fixed points in the plane is a
constant.

Two positions of hand pumps are foci Distance between two foci = 2𝑐 = 10 Hence
𝑐 = 5 Here foci lie on 𝑥 axis & coordinates of foci = (±𝑐, 0)

Hence coordinates of foci = (±5,0)

𝑥2 𝑦2
(ii) 169 + 144 = 1

Sum of distances from the foci = 2𝑎

Sum of distances between the farmer and each hand pump is = 26 = 2𝑎

⇒ 2𝑎 = 26 ⇒ 𝑎 = 13𝑚
Distance between the hand pump = 10𝑚 = 2𝑐

⇒ 𝑐 = 5𝑚
𝑐 2 = 𝑎2 − 𝑏 2
⇒ 25 = 169 − 𝑏 2
⇒ 𝑏 2 = 144
𝑥2 𝑦2
Equation is 169 + 144 = 1

𝑥2 𝑦2
(iii)Equation of ellipse is 169 + 144 = 1 comparing with standard equation of ellipse

𝑎 = 13, 𝑏 = 12 and 𝑐 = 5 (given)

Length of major axis = 2𝑎 = 2 × 13 = 26

Length of minor axis = 2𝑏 = 2 × 12 = 24

186
𝑐 5
eccentricity 𝑒 = 𝑎 = 13

𝑥2 𝑦2
(iv) Equation of the ellipse is 169 + 144 = 1 hence 𝑎 = 13 and 𝑏 = 12

2𝑏 2 2×144
length of latus rectum of ellipse is given by =
𝑎 13

LONG ANSWER TYPE QUESTIONS

31 Find the equation of the ellipse whose foci are (4, 0) and (-4, 0), eccentricity = 1/3. [5]
Ans:
Let the equation of the required ellipse be

𝑥2 𝑦2
+ =1
𝑎2 𝑏 2
The coordinate of foci are (+ ae, 0) and (-ae, 0).

∴ ae = 4[∵foci ☹ ± 4,0)]

1 1
⇒𝑎× = 4 [∵ 𝑒 = ]
3 3
⇒ 𝑎 = 12
⇒ 𝑎2 = 144
Now, 𝑏 2 = 𝑎2 (1 − 𝑒 2 )

2
1 2
⇒ 𝑏 = 144 [1 − ( ) ]
3
1
⇒ 𝑏 2 = 144 [1 − ]
9
8
⇒ 𝑏 2 = 144 ×
9
⇒ 𝑏 2 = 16 × 8 = 128
Substituting, 𝑎2 = 144 and 𝑏 2 = 128 in the above equation of ellipse , we get

𝑥2 𝑦2
= + =1
144 128

187
1 𝑥2 𝑦2
⇒ [ + ]=1
16 9 8
𝑥2 𝑦2
⇒ + = 16
9 8
This is the required equation of the ellipse.

32 Find the equation of a circle passing through the points (2, -6), (6, 4) and (-3, 1). [5]
Ans:
Let the equation of the circle passing through the given points be

𝑥 2 + 𝑦 2 + 2𝑔𝑥 + 2𝑓𝑦 + 𝑐 = 0 ……………. (i)

Since, circle passes through the point(2, −6).

So, put 𝑥 = 2, 𝑦 = −6 in Eq. (i), we get

4 + 36 + 4𝑔 − 12𝑓 + 𝑐 = 0
⇒ 4𝑔 − 12𝑓 + 𝑐 = −40 …..(ii)

Also, circle passes through the point(6,4).

So, put 𝑥 = 6, 𝑦 = 4 in Eq. (i), we get

36 + 16 + 12𝑔 + 8𝑓 + 𝑐 = 0
⇒ 12𝑔 + 8𝑓 + 𝑐 = −52...(iii)

Also, circle passes through the point (−3,1).

So, put 𝑥 = −3 and 𝑦 = 1 in Eq. (i), we get

9 + 1 − 6𝑔 + 2𝑓 + 𝑐 = 0
⇒ −6𝑔 + 2𝑓 + 𝑐 = −10...(iv)

On subtracting Eq. (iii) from Eq. (ii), we get

188
−8𝑔 − 20𝑓 = 12 … (v)

On subtracting Eq. (iv) from Eq. (iii), we get

18𝑔 + 6𝑓 = −42 … (vi)

On solving Eqs. (v) and (vi) for 𝑔 and f, we get

32 5
𝑔=− ,𝑓 =
13 13
On putting the values of 𝑔 and 𝑓 in Eq. (ii), we get

332
𝑐=−
13
32 5 332
Now, on putting 𝑔 = − 13 , 𝑓 = 13 and 𝑐 = − in Eq. (i), we get
13

64 10 332
𝑥2 + 𝑦2 − 𝑥+ 𝑦−− =0
13 13 13
⇒ 13𝑥 2 + 13𝑦 2 − 64𝑥 + 10𝑦 − 332 = 0
which is the required equation of circle.

33 Find the equation of the hyperbola, the length of whose latus rectum is 4 and the [5]
eccentricity is 3.
Ans:
Given: The length of latus rectum is 4 , and the eccentricity is 3

𝑥2 𝑦2
Let, the equation of the hyperbola be: 𝑎2 − 𝑏2 = 1

The length of the latus rectum is 4 units.

2𝑏 2
⇒ length of the latus rectum = =4
𝑎

2𝑏 2
⇒ = 4 ⇒ 𝑏 2 = 2𝑎 … (i)
𝑎

And also given, the eccentricity, 𝑒 = 3

189
𝑏2
We know that, 𝑒 = √1 + 𝑎2

𝑏2
⇒ √1 + =3
𝑎2
𝑏2
⇒ 1 + 𝑎2 = 9 [Squaring both sides]

𝑏2
⇒ =8
𝑎2
⇒ 𝑏 2 = 8𝑎2
⇒ 2𝑎 = 8𝑎2 [From (i)]

1 1
⇒𝑎= ⇒ 𝑎2 =
4 16
1 1 1
From (i) ⇒ 𝑏 2 = 2𝑎 = 2 × 4 = 2 ⇒ 𝑏 2 = 2

So, the equation of the hyperbola is,

𝑥2 𝑦2 𝑥2 𝑦2
− =1⇒ − =1
𝑎2 𝑏 2 1 1
16 2
⇒ 16𝑥 2 − 2𝑦 2 = 1

34 Find the equation of a circle concentric with the circle x2 + 𝑦 2 + 4x + 6y + 11 = 0 and [5]
passing through the point (5, 4).
Ans:
34. Here, the equation of circle is 𝑥 2 + 𝑦 2 + 4𝑥 + 6𝑦 + 11 = 0

⇒ (𝑥 2 + 4𝑥) + (𝑦 2 + 6𝑦) = −11


On adding 4 and 9 both sides to make perfect squares, we get

(𝑥 2 + 4𝑥 + 4) + (𝑦 2 + 6𝑦 + 9) = −11 + 4 + 9
⇒ (𝑥 + 2)2 + (𝑦 + 3)2 = (√2)2 … (i)

Its centre is (−2, −3)

190
The required circle is concentric with circle 1, therefore its centre is (−2, −3).
Since, it passes through (5,4), therefore radius is 𝑟 = 𝐶𝑃 =

√(5 + 2)2 + (4 + 3)2 [∵ distance = √(𝑥2 − 𝑥1 )2 + (𝑦2 − 𝑦1 )2 ]

= √49 + 49 = 7√2
Hence, the equation of required circle having centre(−2, −3) and radius 7√2 is,

(𝑥 + 2)2 + (𝑦 + 3)2 = (7√2)2


⇒ 𝑥 2 + 4𝑥 + 4 + 𝑦 2 + 6𝑦 + 9 = 98
⇒ 𝑥 2 + 4𝑥 + 𝑦 2 + 6𝑦 − 85 = 0
35 Find the (i) lengths of major and minor axes, (ii) coordinates of the vertices, (iii) [5]
coordinates of the foci, (iv) eccentricity, and (v) length of the latus rectum of
𝑥2 𝑦2
ellipse:25 + = 1.
9

Ans:
𝑥2 𝑦2
35. Given: 25 + = 1...(i)
9

Equation Major Axis

𝑥2 𝑦2 𝑐2
+ =1 𝑥-axis (±𝑐, 0)
𝑎2 𝑏 2 = 𝑎2 − 𝑏 2

𝑥2 𝑦2 𝑐2
+ =1 𝑦-axis (0, ±𝑐) (±𝑎, 0) 2𝑎 2𝑏
𝑏 2 𝑎2 = 𝑎2 − 𝑏 2

Since, 25 > 9

So, above equation is of the form.

𝑥2 𝑦2
+ =1
𝑎2 𝑏 2
Comparing eq. (i) and (ii), we get

𝑎2 = 25 𝑎𝑛𝑑 𝑏 2 = 9 ⇒ 𝑎 = 5 𝑎𝑛𝑑 𝑏 = 3

191
i. To find: Length of major axes

Clearly, 𝑎 > 𝑏, therefore the major axes of the ellipse are along 𝑥-axes.

∴ Length of major axes = 2𝑎

Length of major axes = 2 × 5

ii. To find: Coordinates of the Vertices

Clearly, 𝑎 > 𝑏

∴ Coordinate of vertices = (𝑎, 0) and (−𝑎, 0)

Coordinate of vertices = (5,0) and (−5,0)

iii. To find: Coordinates of the foci

We know that,

Coordinates of foci = (±𝑐, 0) where 𝑐 2 = 𝑎2 − 𝑏 2

So, firstly we find the value of 𝑐

𝑐 2 = 𝑎2 − 𝑏 2 = 25 − 9
𝑐 2 = 16
𝑐 = √16
c = 4...(ii)

∴ Coordinates of foci = (±𝑎, 0) = (±4,0)

iv. To find: Eccentricity

We know that,

𝑐 4
Eccentricity = 𝑎 ⇒ 𝑒 = 5 [from (ii)]

192
v. To find: Length of the Latus Rectum We know that,

2𝑏 2 2×(3)2
Length of Latus Rectum = =
𝑎 5

18
Length of Latus Rectum = 5

193
INTRODUCTION TO THREE-DIMENSIONAL GEOMETRY
COORDINATE AXES AND COORDINATE PLANE
Let us take three axes in such a way that they form a right-handed system.

(a)RECTANGULAR AXES

Let X’OX, Y’OY and Z’OZ be the three mutually perpendicular straight lines,
(i)The common point O is called Origin.
(ii)X’OX is called the X-axis.
(iii)Y’OY is called the Y-axis.
(iv)Z’OZ is called the Z-axis.
These three, taken together, are called Co-ordinate-axes or simply axes.

(b) CO-ORDINATE PLANES


(i) XOY, the plane containing X and Y axes, is called XY-plane.
(ii) YOZ, the plane containing Y and Z axes, is called YZ-plane.
(iii) ZOX, the plane containing Z and X axes, is called ZX-plane.
These three, taken together, are called co-ordinate planes.

CORDINATE OF A POINT IN SPACE

An arbitrary point P in three-dimensional space is assigned coordinates (x0,y0,z0) provided that


(1)The plane through P parallel to the yz-plane intersects the x-axis at(x0,0, 0);
(2)The plane through P parallel to the xz-plane intersects the y-axis at (0,y0,0);
(3)The plane through P parallel to the xy-plane intersects the z-axis at (0,0,z0).
The space coordinates (x0,y0,z0) are called the Cartesian coordinates of P or simply the rectangular coordinates
of P.

194
SIGN OF CORDINATE OF A POINT
(i)Distances measured upwards XY-plane are taken as +ve and downwards as –ve.
(ii)Distances measured in front of YZ-plane are taken as +ve and back of it as –ve.
(iii)Distances measured to the right of ZX-plane are taken as +ve and left of it as –ve.
The three co-ordinate planes divide the whole space into eight compartments, known as
octants.
Octant→ I II III IV V VI VII VIII
CO-
ORDINATES ↓ XOYZ X’OYZ X’OY’Z XOY’Z XOYZ’ X’OYZ’ X’OY’Z’ XOY’Z’

x + − − + + − − +

y + + − − + + − −

z + + + + − − − −

DISTANCE FORMULA
The distance between the points (x1, y1, z1) and (x2, y2, z2) and is given by:

The distance of the point (x, y, z) from the origin is given by .

SECTION FORMULA
(i)The co-ordinates of the point, which divides the line joining the points (x 1, y1, z1) and (x2, y2,
z2) in the ratio m1 : m2 are:

(ii) The co-ordinates of the mid-point are:

CENTROID OF TRIANGLE
The co-ordinates of the centroid of the triangle having vertices; (x1,y1,z1) and (x2,y2,z2) and

(x3,y3,z3) are: .

195
SUMMERY

PROPERTIES 2 DIMESIONAL GEOMETRY 3-DIMENSIONAL GEOMETRY


1. 1.COORDINATE Two axes namely lines X′OX, Y′OY Three axes namely lines X′OX, Y′OY and Z′OZ
AXIS AND PLANE are called x-axis,y-axis respectively. are called x-axis, y-axis and z-axis.
The three axes taken together in pairs
determine xy, yz, zx plane, i.e., three
coordinate planes.
2.COORDINATE An arbitrary point P in 2D is assigned An arbitrary point P in 3D is assigned co-
OF A POINT IN co-ordinates as (x0,y0) ordinates as (x0,y0,z0)
SPACE
3.DISTANCE If P(x1,y1) and Q(x2,y2) are two points The distance between the points (x1, y1, z1) and
FORMULA then distance between them are (x2, y2, z2) and is given by:

√(𝑥2 – 𝑥1)2 + (𝑦2 – 𝑦1)2 .

SECTION The co-ordinates of the point, which The co-ordinates of the point, which divides the
FORMULA divides the line joining the points (x, line joining the points (x1, y1, z1) and (x2, y2, z2)
y) and (p, q) in the ratio m : nare: in the ratio m1 : m2 are:
𝑛𝑥+𝑝𝑚 𝑛𝑦+𝑚𝑞
( , )
𝑚+𝑛 𝑚+𝑛

MID-POINT The co-ordinates of the midpoint, of The co-ordinates of the mid-point of the line
FORMULA the line joining the points (x, y) and joining the points (x1, y1, z1) and (x2, y2, z2)are:
(p, q) are:
𝑥+𝑝 𝑦+𝑞
( , )
2 2

CENTROID OF The coordinate of the centroid of The co-ordinates of the centroid of the triangle
TRIANGLE the triangle, whose vertices are having vertices; (x1,y1,z1) and (x2,y2,z2) and
(x,y),(p,q)and(r,s)are (x3,y3,z3) are:
𝑥+𝑝+𝑟 𝑦+𝑞+𝑟
( , )
3 3
.

196
EXEMPLAR QUESTIONS
TYPE-1 (MCQ)
1 .What is the perpendicular distance of the point P (6, 7, 8) from xy-plane?
(A) 8 (B) 7 (C) 6 (D) None of these
Solution:-
(A)Let L be the foot of perpendicular drawn from the point P(6, 7, 8) to the xy-
plane and the distance of this foot L from P is z-coordinate of P, i.e., 8 units.
2.What is the locus of a point for which y = 0, z = 0?
(A) equation of x-axis (B) equation of y-axis
(C) equation of z-axis (D) none of these
Solution:-
(A) Locus of the point y = 0, z = 0 is x-axis, since on x-axis both y = 0 and z = 0.
3.L, is the foot of the perpendicular drawn from a point P (3, 4, 5) on the
xz plane. What are the coordinates of point L?
(A) (3, 0, 0) (B) (0, 4, 5) (C) (3, 0, 5) (D) (3, 4, 0)
Solution:-
(D) Since L is the foot of perpendicular segment drawn from the point P (3, 4, 5)
on the xz-plane. Since the y-coordinates of all points in the xz-plane are zero, coordinate of the foot of
perpendicular are (3, 0, 5).
4.The length of the foot of perpendicular drawn from the point P (3, 4, 5)
on y-axis is
(A) 10 (B) (34)1/2 (C) (113)1/2 (D) 52
Solution:-
Let l be the foot of perpendicular from point P on the y-axis. Therefore, its x and z-coordinates are zero, i.e.,
(0,4,0). Therefore, distance between the points (0,4,0) and (3,4,5) is (34)1/2 .
Fill in the blanks
5. A line is parallel to xy-plane if all the points on the line have equal _____.
Solution:-A line parallel to xy-plane if all the points on the line have equal z-coordinates.
6. The equation x = b represents a plane parallel to _____ plane.
Solution:-
Since x = 0 represent yz-plane, therefore x = b represent a plane parallel to
197
yz -plane at a unit distance b from the origin.
7.L is the foot of perpendicular drawn from the point P (3, 4, 5) on zx-planes. The coordinates of L are ________.
Solution:-
Since L is the foot of perpendicular from P on the zx-plane, y-coordinate of
every point is zero in the zx-plane. Hence, coordinate of L are (3, 0, 5).
Check whether the statements are True or False.
8. The y-axis and z-axis, together determine a plane known as yz-plane.
Solution:- True
9. The point (4, 5, – 6) lies in the VIth octant.
Solution:- False, the point (4, 5, – 6) lies in the Vth octant,
10. The x-axis is the intersection of two planes xy-plane and xz plane.
Solution:- True.
11. Three mutually perpendicular planes divide the space into 8 octants.
Solution:- True.
12. The equation of the plane z = 6 represent a plane parallel to the xy-plane,
having a z-intercept of 6 units.
Solution:- True.
13.The equation of the plane x = 0 represent the yz-plane.
Solution :- True.
14. The point on the x-axis with x-coordinate equal to x0 is written as (x0,0, 0).
Solution :- True
TYPE – (02) ASSERTION TYPE QUESTION
In the following question a statement of assertion (A) is followed by a statement of Reason (R). Pick the correct
option :
(a)Both (A) and (R) are true and (R) is the correct explanation of (A).
(b) Both (A) and (R) are true and (R) is not the correct explanation of (A).
(c)(A) is true But (R) is false.
(d)(A) is false But (R) is true.
1.ASSERTION-Vertices of triangle are A(2,0,1),B(1,-1,1),C(2,1,-1).Then side AB is (2)1/2.
REASON- The distance between the points (x1, y1, z1) and (x2, y2,z2) and is given by:

198
.
Solution:-
(a) Using distance formula we are able to find side AB so it is correct explanation.

2.ASSERTION-If the distance between the points (a,2,1)and (1,-1,1) is 5, then a is

√16
REASON - The distance between the points (x1,y1,z1) and (x2,y2,z2) and is given by:

Solution:-
(d) Assertion is false and reason is true.
3.ASSERTION- The locus of a point for which x=0 is yz plane.
REASON – The co-ordinate of a point in yz plane is (x,y,0).
Solution:-
(c)Assertion is true and reason is false. As co-ordinate of point in YZ plane is (0,y,z).

ASSERTION – The point(-2,3,-3) lies in the seventh octant .

REASON – Any arbitrary point of seventh octant have all the coordinate negative.
Solution:-
(b)All the coordinate of seventh octant will be negative , so given co-ordinate will not lies in seventh co-ordinates
.Hence it is not the correct explanation of given statement.
5. ASSERTION – Equation of y axis is considered as x=0,z=0.
REASON – Equation of x axis is considered as x=a,y=0,z=2.
Solution:-
(c)Equation of x-axis is x=a, and all other co-ordinates are zero.
TYPE – (03) SHORT ANSWER TYPE
1.Locate the points (i) (2, 3, 4) (ii) (–2, –2, 3) in space.
Solution:-

199
(i) To locate the point (2, 3, 4) in space, we move 2 units from O along the positive direction of x-axis. Let this
point be A(2, 0, 0). From the point A moves 3 units parallel to +ve direction of y-axis. Let this point be B (2, 3, 0).
From the point B moves 4 units along positive direction of z-axis. This is our required point be P (2, 3, 4).
(ii)From the origin, move 2 units along the negative direction of x-axis. Let this point be A (–2, 0, 0). From the
point A move 2 units parallel to negative direction of y-axis. Let this point be B (–2, –2, 0). From B move 3 units
parallel to positive direction of z - axis. This is our required point Q (–2, –2, 3).

2.Let L, M, N be the feet of the perpendiculars drawn from a point P (3, 4, 5)


On the x, y and z-axes respectively. Find the coordinates of L, M and N.
Solution:-
Since L is the foot of perpendicular from P on the x-axis, its y and z co-
ordinates are zero. The coordinates of L is (3, 0, 0). Similarly, the coordinates of M and N are (0, 4, 0) and (0, 0,
5), respectively.

3.How far apart are the points (2,0,0)and (-3,0,0)?


Solution :-
We know that to find distance between two points p(x1,y1,z1) and q(x2,y2,z2) is

. So, putting values of x1=2,y1=0,z1=0 and x2 =-3,y2=0,z2=0 we get


(52)1/2 i.e 5.

4. Write co-ordinate of foot of perpendicular from (3,7,9) on x-axis, y-axis.

Solution:-

On 𝑋 − 𝑎𝑥𝑖𝑠, the co-ordinates are (3, 0, 0)

On 𝑌 − 𝑎𝑥𝑖𝑠, the co-ordinates are (0, 7, 0)

On 𝑍 − 𝑎𝑥𝑖𝑠, the co-ordinates are (0, 0, 9)

5. If a parallelo piped is formed by planes drawn through the points (2, 3, 5)


and (5, 9, 7) parallel to the coordinate planes, then find the length of edges of a
Parallele piped.

200
Solution:-

Length of edges of the parallelepiped are √(5 – 2)2 + (9 − 3)2 + ( 7 – 5 )2 .

√32 + 62 + 22 =√49=7
6. Show that the points (5, –1, 1), (7, – 4, 7), (1 – 6, 10) and (–1, – 3, 4) are
the vertices of a rhombus.
Solution:-
Let A (5, – 1, 1), B (7, – 4, 7), C(1, – 6, 10) and D (– 1, – 3, 4) be the four
points of a quadrilateral. Here

AB = √(5 − 7)2 + (−1 + 4)2 + (1 − 7)2=√4 + 9 + 36 =√49 = 7 ,

BC = √(7 − 1)2 + (−4 + 6)2 + (7 − 10)2 =√36 + 4 + 9= 7,

CD = √(1 + 1)2 + (−6 + 3)2 + (10 − 4)2 =√4 + 9 + 36= 7

AD=√(5 + 1)2 + (−1 + 3)2 + (1 − 4)2 =√36 + 4 + 9=7


AB= BC = CD = DA. Therefore, ABCD is a rhombus.
7. Three consecutive vertices of a parallelogram are 𝑨(−𝟔, 𝟐, 𝟒), 𝑩(𝟐, 𝟒, −𝟖) and𝑪(−𝟐, 𝟐, 𝟒). Find the co-
ordinates of the fourth vertex.
Solution:-
Let the co-ordinates of the fourth vertex be 𝐷(𝑥, 𝑦, 𝑧) we know that the diagonals of a parallelogram bisect
each other
∴ Midpoint of 𝐴𝐶 = 𝑀𝑖𝑑𝑝𝑜𝑖𝑛𝑡𝑜𝑓𝐵𝐷
−6−2 2+2 4+4 2+𝑥 4+𝑦 −8+𝑧
⇒( , , )=( , , )
2 2 2 2 2 2
2+𝑥 4+𝑦 −8+𝑧
⇒ (−4, 2, 4) = ( , , )
2 2 2
2+𝑥 4+𝑦 −8+𝑧
⇒ =-4, =2, =4
2 2 2

⇒𝑥 = −10 , 𝑦 = 0 ,𝑧 = 16.
8. L is the foot of the perpendicular drawn from a point P (3, 4, 5) on the
xz plane. What are the coordinates of point L.
Solution:- Since L is the foot of perpendicular segment drawn from the point P (3, 4, 5) on the xz-plane. Since
the y-coordinates of all points in the xz-plane are zero, coordinate of the foot of perpendicular are (3, 0, 5).
9.Calculate the perpendicular distance of the point P(6, 7, 8) from the XY – Plane.

201
Solution :-Assume that A be the foot of perpendicular drawn from the point P (6, 7, 8) to the XY plane and the
distance of this foot A from P is the z-coordinate of P, i.e., 8 units.
10.Find the image of:
(i) (-2, 3, 4) in the yz-plane
(ii) (-5, 4, -3) in the xz-plane
(iii) (5, 2, -7) in the xy-plane
(iv) (-5, 0, 3) in the xz-plane
Solution:-
(i) Given: Point is (-2, 3, 4)
To find: the image of the point in yz-plane
Since we need to find its image in yz-plane, a sign of its x-coordinate will change
So, Image of point (-2, 3, 4) is (2, 3, 4)
(ii) Given: Point is (-5, 4, -3)
To find: image of the point in xz-plane
Since we need to find its image in xz-plane, sign of its y-coordinate will change
So, Image of point (-5, 4, -3) is (-5, -4, -3)
(iii) Given: Point is (5, 2, -7)
To find: the image of the point in xy-plane
Since we need to find its image in xy-plane, a sign of its z-coordinate will change
So, Image of point (5, 2, -7) is (5, 2, 7)
(iv) Given: Point is (-5, 0, 3)
To find: image of the point in xz-plane
Since we need to find its image in xz-plane, sign of its y-coordinate will change
So, Image of point (-5, 0, 3) is (-5, 0, 3)
11. Find the ratio in which the line joining (2, 4, 5) and (3, 5, -9) is divided by the yz-plane.
Solution:-
We have points A(2, 4, 5) and B(3, 5, -9)
To find the ratio in which the line joining given points is divided by the yz-plane
line AB is divided by C in m:n where A(x, y, z) and B(a, b, c).

The coordinates of C is given by,


202
x coordinate is always 0 on yz-plane.
Let Point C(0, y, z) and C divides AB in ratio k : 1
Therefore, m1 = k and m2 = 1
A(2, 4, 5) and B(3, 5, -9), Coordinates of C using section formula:
3𝑘+2 5𝑘+4 −9𝑘+5
( 𝑘+1 , , )
𝑘+1 𝑘+1

On comparing:
⇒ 3k + 2 = 0(k + 1)
⇒ 3k + 2 = 0
⇒ 3k = –2
Hence, C divides AB externally in ratio 2 : 3.

TYPE (04)- LONG ANSWER QUESTION


1. Using distance formula show that points A(1, −1, 3), B(2, -4, 5) and
C(5, −13, 11) are collinear.

Sol. AB = √1 – 2)^2 + (−1 + 4)^2 + (3 – 5)^2

= √1 + 9 + 4 =√14

BC = √(2 − 5)2 + (−4 + 13)2 + (5 − 11)2


= √9 + 81 + 36 = √126= 3√14

AC =√(5 − 1)2 + (−13 + 1)2 + (11 − 3)2


= √16 + 144 + 64 = √214= 4√14
Since AB+ BC = AC
∴ A, B, C are collinear.

2. A point C with z-coordinate 8 lies on the line segment joining the points A(2,-3,0) and B(2, -3, 10). Find
the co-ordinates of C..

Solution:-
Let the co-ordinates of C be (x, y, 8) that divides AB in the ratio m: n
10𝑚+4𝑛
∴ 𝑚+𝑛
=8

⇒ 10m + 4n = 8m + 8n

203
⇒ 2m = 4n
⇒ m = 2n
⇒ m: n = 2: 1
2∗8+1∗2 18
x= = = 6 And
3 3
2∗0+1∗−3 −3
y= = =-1
3 3

∴ Co-ordinate of C are (6, −1, 8)


3. What are the coordinates of the vertices of a cube whose edge is 5 units, one of whose vertices coincides
with the origin and three edges passing through the origin and coincides with the positive direction of
the axes through the origin? .

Solution:- Given, edge of a cube is5unit


∴ Co-ordinates of O = (0, 0, 0)
∴ Co-ordinates of A = (5, 0, 0)
∴ Co-ordinates of G = (0, 5, 0)
∴ Co-ordinates of D = (0, 0, 5)
∴ Co-ordinates of B = (5, 5, 0)
∴ Co-ordinates of F = (5, 5, 5)
∴ Co-ordinates of E = (5, 0, 5)
∴ Co-ordinates of C = (0, 5, 5).
4. Show that the points A(1,3,0), B(−5,5,2), C(−9, −1, 2) and D(−3, −3, 0) are the vertices of a parallelogram
ABCD, but it is not a rectangle.

Solution:-

AB = √(1 + 5)2 + (3 − 5)2 + (0 − 2)2 = √36 + 4 + 4 =√44 = 2√11

BC =√(−5 + 9)2 + (5 + 1)2 + (2 − 2)2 =√16 + 36 + 0= √52 = 2√13

CD =√(−9 + 3)2 + (−1 + 3)2 + (2 − 0)2 = √36 + 4 + 4 = √44 = 2√11

AD= √(−3 − 1)2 + (−3 − 3)2 + (0 − 0)2 = √16 + 81 = 2√13

Also, AC = √(1 + 9)2 + (3 + 1)2 + (0 − 2)2 =√100 + 16 + 4 =√120 = 2√30

BD =√(−5 + 3)2 + (5 + 3)2 + (2 − 0)2 =√4 + 64 + 4 = √72 = 6√2 Since AB = CD, BC = AD and AC ≠ BD
∴ The opposite sides are equal and diagonals are unequal, so the given points are the vertices of a parallelogram
not rectangle.
204
5.Determine the point in yz-plane which is equidistant from three points A (2, 0 3) B (0, 3, 2) and C (0, 0, 1).
Solution:-
Since x-coordinate of every point in yz-plane is zero. Let P (0, y,z) be a point on the yz-plane such that PA = PB
= PC. Now PA = PB

⇒√(0 − 2)2 + (𝑦 − 0)2 + (𝑧 − 3)2 =√(0 − 0)2 + (𝑦 − 3)2 + (𝑧 − 2)2


4+y2+z2+9-6z =y2+9-6y+z2+4-4z
i.e. z – 3y = 0 and PB = PC

√(0 − 0)2 + (𝑦 − 3)2 + (𝑧 − 2)2 =√(0 − 0)2 + (𝑦 − 0)2 + (𝑧 − 1)2


⇒ y2 + 9 – 6y + z2 + 4 – 4z = y2 + z2 + 1 – 2z ,
i.e. 3y + z = 6 Simplifying the two equating, we get y = 1, z = 3 Here, the coordinate of the point P are (0, 1, 3).
TOPIC (05) – CASE BASED QUESTIONS
1) The Indian coast Guards while patrolling saw a suspicious boat with four men. They were nowhere looking
like fisher men the soldiers were closely observing the movement of the boat for an opportunity to seize
the boat. They observe the boat is moving along a planar surface . At instant of time theco-ordinate of the
position of coast guard helicopter and boat are (2,3,5) and (1,4,2) respectively.

Based on the above information, answer the following:


i)What is the distance between helicopter and boat?
a)√11 b)√10 c)√12 d)√1
ii) what is the co-ordinate of (2,3,5) on x axis ?
a)(2,0,0) b)(2,3,0)
c)(0,3,0) d)(0,0,5)
iii) what is the co-ordinate of midpoint of helicopter and boat ?
3 3 7 7 7 7
a)( 2 , 2 , 2) b)( 2 , 2 , 2)
3 7 7 3 5 7
c)( 2 , 2 , 2) d )( 2 , 2 , 2)

iv) what is the distance of (1,4,2)from origin?


a)√12 b)√21 𝑐)√17 d)√22

Solution:-
We know that to find distance between two points p(x1,y1,z1) and q(x2,y2,z2) is

205
.

i)(a)Putting the value (x1,y1,z1) as (2,3,5) and (x2,y2,z2) as (1,4,2) then we get √ (3-4)2+(5-
2)2+(2 − 1) =√11
ii)(a)co-ordinate on x axis is (2,0,0)

2+1 3+4 5+2


iii)(c) co-ordinates of mid point is ( , , )
2 2 2
3 7 7
= (2 , 2 , 2)
2
iv)(b) Putting the value (x1,y1,z1) as (0,0,0) and (x2,y2,z2) as (1,4,2) then we have √12 + 42 + 2 =√21

2)In a diamond exhibition , a diamond is covered in cubical glass box having co-ordinate
A(0,0,0,),B(1,0,0),C(1,2,0),D(0,2,0),E(0,0,3),F(1,0,3),G(1,2,3)and H(0,2,3).
From the above information, answer the following:
i)(0,0,3) lies on which axis –
a)x-axis b)y-axis
c)z- axis d)origin
ii)what is the length of side AB?
a)√2 b)√3
𝑐)√4 d)√21
𝑖𝑖𝑖)What will be the length of diagonals joining A and C?
𝑎)√5 b)√1
𝑐)√12 d)√2
𝑖𝑣)In which quadrant the value (1,2,3) lies?
a) I octant b)II octant
c) IV octant d) VI octant

Solution:-
i)© Given point will lie on z axis.
ii)© Using distance formula we get √4

206
iii)(a) Using diatance formula we get √5
iv)(a)As all the co-ordinates are positive so it will lie in 1st quadrant.

3)A mobile tower stands at the top of a hill, consider the surface on which tower stands as a plane having points
A(0,1,2),B(3,4,5),C(2,4,2) on it. The tower is tied with 3 cables from the points A,C, and C such that it stands
vertically on the ground. The peak of the tower is at the points (6,5,9) as shown in figure:
From the above information, answer the following:

i) What is the length of cable A from peak?


a)√101 b)√103 c)√102
d)√106
ii)In which quadrant will (3,4,-1) will lie?
a) I octant b)II octant
c) V octant d) VI octant
iii)If base of tower is at origin, then the length of tower will be-
a)√142 b)√123
c)√161 d)√141
iv) How far is cable A from cable B?
a)√207 b)√72
c)√27 d)√702

Solution:-
i)(a)Using distance formula , we have √101
ii)©According to the sign of co-ordinate in octant then this will lie in v octant .
iii)(a)Using distance formula , we have √142
1
iv)©Using distance formula, we have (27)2

4)Ramesh was playing in triangular garden with is two friend, Vivek and Kundan at the corner of field, having
coordinates Ramesh( 0,0,0)Vivek (2,4,6),Kundan(0,-2,5).
207
From the above information, answer the following:
i)What is co-ordinate of mid point of line joining Ramesh and Vivek?
a) (1,2,1)b) (1,2,2)
c)(1,2,4) d)(1,2,3)
ii)The co-ordinate of centriod of the triangle-
2 2 1 2 7 1
a)(3 , 3 , 3) c)(3 , 3 , 3)
1 2 1 2 1 1
b)(3 , 3 , 3) d)(3 , 3 , 3)

iii)Let D be the point of line joining vivek and ramesh such that it divides them in ratio 2:1, the co-ordinate of
the point D is
2 4 6 2 2 1
a)(3 , 3 , 3) b)(3 , 3 , 3)
2 5 1 2 0 −4
c)(3 , 3 , 3) d)(3 , 3 , )
3

iv)What is the length of line joining vivek and kundan ?


a)√41 b)√72
c)√160 d)√102

Solution:-
2+0 4+(−2) 6+(−5) 2 4 6
i) © Using midpoint formula we have ( , , )=(2 , 2 , 2)=(1,2,3).
2 2 2
0+2+0 0+4+(−2) 0+6+(−5) 2 2 1
ii)(a)Coordinate of centroid is ( , , )=(3 , 3 , 3)
3 3 3
0+2 4−4 6−10 2 0 −4
iii)(d) Coordinate of point D is ( , , )=(3, 3 , )
3 3 3 3

iv)(a)Using distance formula we have √41

5) Kajal is eating with her parents on rectangular table ABCD having co-ordinate A(4,7,8),B(2,3,4),C(-1,-
2,1),D(1,2,5) respectively. She has four family member in her family including her and all are sitting at the corner
of the table .
From the above information, answer the following:
i)What is the length of the table?
a)6 b)5
c)4 d)7

208
ii)what is the breadth of the table?
a)√46 b)√42
c)√40 d)√43
iii)what is the area of the table?
a)√1588 b)√1548
c)√1648 d)√1528
iv)what is the co-ordinate midpoint of AC?
2 5 6 3 4 6
a)(3 , 2 , 2) b)(2 , 2 , 2)
2 5 1 3 5 7
c)( , , ) d)( , , )
2 2 2 2 2 2

Solution:-
i)(a)Using distance formula , the length of table is 6.
ii)(d)Using distance formula ,the breadth of table is √43 .
iii)(b)Area of rectangle is √43×√36= √1548.
3 5 7
iv)(d) Midpoint of AC is( 2 , 2 , 2)

SELF PRACTICE QUESTIONS

TYPE (01) – MULTIPLE CHOICE QUESTIONS


1. Write the co-ordinates of the feet of perpendicular from the point (a, b, c) on the co-ordinate axes
2. Find the perpendicular distances of the point P(a, b, c) from the co-ordinate axes.
3. Find the distance between the points A(-1, 3, -4) and B(1, -3, 4).
4. Find the locus of a point, which is equidistant from the point (-1, 2, 3) and (3, 2, 1).
5. Find ‘k’ so that the distance between the points (7, 1, -3) and (3, 2, 1)be 13 units.

6. Match each item given under the column C1 to its correct answer given under column C2 .

Column C1 Column C2
(a) In xy-plane (i) Ist octant
(b) Point (2, 3,4) lies in the (ii) yz-plane

209
(c) Locus of the points having x coordinate (iii) z-coordinate is zero
0 is
(d) A line is parallel to x-axis if and only (iv) z-axis
(e) If x = 0, y = 0 taken together will (v) plane parallel to xy-plane
Represent the
(f) z = c represent the plane vi)) if all the points on the line have

equal y and z-coordinates.

(g) Planes x = a,y = b represent the line (vii) from the point on the respective.
(h) Coordinates of a point are the distances (viii) parallel to z - axis
from the origin to the feet of
perpendiculars.

(i) A ball is the solid region in the space (ix) disc


enclosed by a
(j) Region in the plane enclosed by a circle is (x) sphere
known as a

7.Find the co-ordinates of the points, which trisect AB given that A is (4, 2, -6) and B is (10, -16, 6).
8. Square of distance of the point (3, 4, 5) from the origin (0, 0, 0) is
(A) 50 (B) 3 (C) 4 (D) 5 .
9. If the distance between the points (a, 0, 1) and (0, 1, 2) is 27 , then the value of a is
(A) 5 (B) ± 5 (C) – 5 (D) none of these
10. x-axis is the intersection of two planes
(A) xy and xz (B) yz and zx (C) xy and yz
(D) none of these

210
ANSWER
1.(a,0,0),(0,b,0),(0,0,c)

2.√𝑏 2 + 𝑐 2 √𝑎2 + 𝑐 2 ,√𝑏 2 + 𝑎2 √𝑏 2 + 𝑐 2 ,√𝑎2 + 𝑐 2 √𝑏 2 + 𝑎2


3.√104
4.2x-z=0
5.9,-15
6.a-iii
b-i
c-ii
d-vi
e-iv
f-v
g-viii
h-vii
i-x
j-ix
7. (6,-4,-2) (8,-10,2)
8.a
9.b
10.a
TYPE (02) SHORT ANSWER TYPE QUESTION
1. If the origin is the centroid of a triangle ABC having vertices A(a, 1, 3), B(-2, b, -5) and C(4, 7, c), Find the
values of a, b and c.
2. Find the co-ordinates of the mid-point of the join of the points A(3, 5, 7) and B(-3, -3, 1).
3. Find the points on the X-axis, which are at a distance of 2√6units from the point (1, -2, 3).
4. Find the point on the Y-axis, which are at a distance of 5√2units from the point (3, -2, 5).
5. Write the perpendicular distance of the point (x, y, z) from the three co-ordinate planes.
6. Using distance formula, show that the points A(-3, 2, 4); B((-1, 5, 9) and C(1, 8, 14) are collinear.
7. Show that the points A(0, 1, 2); B(2, -1, 3) and C(1, -3, 1) are vertices of an isosceles triangle.

211
8. Find the ratio in which yz plane divides the line segment formed by joining the points (-2, 4, 7) and (3, -
5, 8).
9. The distance between the points (a, 0, 1) and (0, 1, 2) is √25. Find the value of A.
10. Find x so that the point (6, 5, -3) is at a distance of 13 unit from the point
(x, -7, 0).
ANSWER
(1)a=2,b=-8,c=2
(2)(0,1,3)
(3)(12,0,0)
(4)(0,2,0) (0,-6,0)
(5) x from YOZ; y from ZOX and z from XOY plane
(8)3/2
(9)25
(10)(2-7,0) (10,-7,0)

TYPE-(03) CASE STUDY


1.The points A(3, 2, 0), B(5, 3, 2) and C(0, 2, 4) are the vertices of triangle ABC as given in the figure :

Based on the above information, answer the following:

1. The length of AB is:


(a) 3 unit
(b) 4 unit
(c) 5 unit
(d) none of these
212
2. The length of AC is:
(a) 4 unit
(b) 5 unit
(c) 6 unit
(d) none of these
3. If AD is the angle bisector ofA, the D divides BC in:
(a) 1:2
(b) 2:3
(c) 3:4
(d) 4:5
4. The co-ordinates of point D are:

(a)

(b)
(c) (2, 3, 4)
(d) none of these
5. The length of angle bisector AD is:

(a)

(b)

(c)
(d) none of these

2. Raj and his father were walking in a large park. They saw a kite flying in the sky. The position of kite, Raj and
Raj’s father are at (20, 30, 10), (4, 3, 7) and (5, 3, 7) respectively.

213
On the basis of above information, answer the following:
(i) Find the distance between Raj and kite.
(ii) Find the form of the co-ordinates of points in the XY-plane.
(iii) If co-ordinates of kite, Raj and Raj’s father form a triangle, then find the centroid of it.
3.Deepak and his friends went for camping for 2 or 3 days. There they set up a tent which is triangular in shape.
The vertices of the tent are A(4, 5, 9), B(3, 2, 5), C(5, 2, 5), D(–3, 2, –5) and E(–4, 5, –9) respectively. The vertex
A is tied up by the rope at the ends F and G and the vertex E is tied up at the ends I and H.

On the basis of above information, answer the following:


(i) If M denotes the position of their bags inside the tent and it is just in middle of the vertices B and D, then find
the coordinates of M. (1)
(ii) Find the length AE. (1)
(iii) What is the length of the rope by which E is tied up with D ?

4.Consider a ABC with vertices A(x, y, z), B(a, b, c) and


214
C(p,q,r). AD, BE and CF are medians of ABC. G is the point of intersection.

Based on the above information, answer the following questions:-


i.) Coordinates of Point D are ?
𝑎+𝑝 𝑏+𝑞 𝑐+𝑟
(a)( , , )
2 2 2
𝑎+2𝑝 𝑏+2𝑞 𝑐+2𝑟
(b)( , , )
2 2 2
𝑥+𝑎+𝑝 𝑦+𝑏+𝑞 𝑧+𝑐+𝑟
(c)( , , )
3 3 3

d) None of these

ii)For ABC, G is
(a) Incentre (b) Circumcentre
(c) Centroid (d) Orthocentre
iii)G divides BE in ratio
(a) 1 : 2 (b) 2 : 1
(c) 3 : 1 (d) 1 : 3
iv)A point G divides AD in 2 : 1, the coordinates of G are
𝑎+𝑝 𝑏+𝑞 𝑐+𝑟
(a)( , , )
2 2 2
𝑎+2𝑝 𝑏+2𝑞 𝑐+2𝑟
(b)( , , )
2 2 2
𝑥+𝑎+𝑝 𝑦+𝑏+𝑞 𝑧+𝑐+𝑟
(c)( , , )
3 3 3
𝑎+2𝑝 𝑏+2𝑞 𝑐+2𝑟
(d)( , , ).
2 2 2

SOLUTION :-
1)(i)a
(ii)b
(iii)d
(iv)a
(v)b

2)(i)√944
215
(ii)(5,3,0),(4,3,0),(20,30,0)
29 36 24
(iii)( 3 , 3 , 3 )

3)(i)(3,0,5)
(ii)√388
(iii)√26
4)(i)a
(ii)c
(iii)b
(iv)c
TYPE (04) – LONG ANSWER QUESTION
1. Find the co-ordinates of the point R, which divides the join of P (0, 0, 0) and Q (4, -1, -2) in the ratio 1 : 2
externally and verify that P is the mid-point of RQ.
2. If A and B are the points (1, 2, 3) and (0, -1, 2) and P is a point such that AP2 –BP2= 10. Find the equation
of locus of P.
3. The diagonals of a parallelogram meets in the point (1, -2, 3) and the ends of a side are (0, 0, 0) and (2,
4, -3). Find the remaining two vertices of the parallelogram.
4. Find x so that the point (6, 5, -3) is at a distance of 13 unit from the point (x, -7, 0).
5. The mid-point of the sides of a triangle are (1, 5, -1), (0, 4, -2) and (2, 3, 4). Find the vertices of triangle and
also find the centroid.

Answers
1) (-4,1,2)
2) 2X+6Y+2Z=0
3) (2,-4,6) (0,-8,9)
4) X=2,10
5) (3,4,5),(-1,6,-7),(1,2,3),(1,4,1/3)
TYPE (05) – ASSERSTION REASONS BASED QUESTION
In the following questions, a statement of assertion (A) is followed by a statement of Reason (R).
Choose the correct answer out of the following choices.
1). Assertion (A) : If a parallelopiped is formed by planes drawn through the points (5, 8, 10) and (3, 6, 8) parallel
to the coordinate planes, then the length of diagonal of the parallelopiped is 23.

216
Reason(R) : If a parallelopiped is formed by planes drawn through the point p(x,y,z) and Q(a,b,c) parallel

to the coordinate planes, then the length of the diagonal = √(𝑥 − 𝑎)2 + (𝑦 − 𝑏)2 + (𝑧 − 𝑐)2.
(A) Both A and R are true and R is the correct explanation of A.
(B) Both A and R are true but R is not the correct explanation of A.
(C) A is true but R is false.
(D) A is false but R is true.
2). Assertion (A) : The point on x – axis which is equidistant from the points A(3,2,2) and B(5,5,4) is (4,0,0)
Reason (R) : Any point on the x – axis is of the form (x,0,0).
(A) Both A and R are true and R is the correct explanation of A.
(B) Both A and R are true but R is not the correct explanation of A.
(C) A is true but R is false.
(D) A is false but R is true.
3). Assertion (A) : The point (– 4, 5, – 6) lies in the VI octant.
Reason (R) : The three coordinate planes divide the space into eight equal parts known as octants.
(A) Both A and R are true and R is the correct explanation of A.
(B) Both A and R are true but R is not the correct explanation of A.
(C) A is true but R is false.
(D) A is false but R is true.
4). Assertion (A) : The points (3, – 1, – 1), (5, – 4, 0), (2, 3, – 2) and (0, 6, – 3) are the vertices of a parallelogram.
Reason (R) : In the parallelogram, both the pairs of opposite sides are equal and diagonals are also
same.
(A) Both A and R are true and R is the correct explanation of A.
(B) Both A and R are true but R is not the correct explanation of A.
(C) A is true but R is false.
(D) A is false but R is true.
ANSWER
1)(A) Both A and R are true and R is the correct explanation of A.
2)(D) A is false but R is true.
3)(B) Both A and R are true but R is not the correct explanation of A.
4)(C) A is true but R is false.

217
LIMITS AND DERIVATIVES
CONCEPTS:
Let y = f(x) be a function of x. If at x = a, f(x) takes indeterminate form, then we consider the values of the
function which is very near to a. If these values tend to a definite unique number as x tends to a, then the unique
number so obtained is called the limit of f(x) at x = a and we write it as lim 𝑓(𝑥) = 𝑓(𝑎).
𝑥→𝑎

Left Hand and Right-Hand Limits


If values of the function at the point which are very near to a on the left tends to a definite unique number as
x tends to a, then the unique number so obtained is called the left-hand limit of f(x) at x = a, we write it as
𝐿𝐻𝐿 ∶ lim− 𝑓(𝑥) = lim 𝑓(𝑎 − ℎ)
𝑥→𝑎 ℎ→0

Similarly, right hand limit is


𝑅𝐻𝐿 ∶ lim+ 𝑓(𝑥) = lim 𝑓(𝑎 + ℎ)
𝑥→𝑎 ℎ→0

Existence of Limit
Limit of 𝑓(𝑥) exists, if 𝑳𝑯𝑳 = 𝑹𝑯𝑳

Some Properties of Limits

Let f and g be two functions such that both 𝑓(𝑥)and 𝑔(𝑥)exists, then

(i) lim {(𝑓 + 𝑔)(𝑥)} = lim 𝑓(𝑥) + lim 𝑔(𝑥)


𝑥→𝑎 𝑥→𝑎 𝑥→𝑎

(ii) lim (𝑘𝑓(𝑥)) = 𝑘 lim 𝑓(𝑥)


𝑥→𝑎 𝑥→𝑎

(iii) lim [𝑓(𝑥) . 𝑔(𝑥)] = lim 𝑓(𝑥)x lim 𝑔(𝑥)


𝑥→𝑎 𝑥→𝑎 𝑥→𝑎

𝑓(𝑥) lim 𝑓(𝑥)


𝑥→𝑎
(iv) lim (𝑔(𝑥)) = , where g(x) ≠ 0
𝑥→𝑎 lim 𝑔(𝑥)
𝑥→𝑎

Some Standard Limits

𝑥 𝑛 −𝑎𝑛
(i) lim = 𝑛𝑎𝑛−1
𝑥→𝑎 𝑥−𝑎

sin 𝑥
(ii) lim =1
𝑥→0 𝑥

tan 𝑥
(iii) lim =1
𝑥→0 𝑥

𝑎𝑥 −1
(iv) lim = ln 𝑎
𝑥→0 𝑥

218
𝑒 𝑥 −1
(v) lim =1
𝑥→0 𝑥

𝑙𝑜𝑔(1+𝑥)
(vi) lim =1
𝑥→𝑎 𝑥

Derivatives
Suppose f is a real-valued function, then
𝑓(𝑥+ℎ)−𝑓(𝑥) 𝑓(𝑥+ℎ)−𝑓(𝑥)
𝑓 ′(𝑥) = lim is called the derivatives f at x iff exists finitely.
ℎ→0 ℎ ℎ

Fundamental Derivative Rules of Function


𝑑 𝑑 𝑑
(i) [𝑓(𝑥) + 𝑔(𝑥)] = [𝑓(𝑥)] + [𝑔(𝑥)]
𝑑𝑥 𝑑𝑥 𝑑𝑥

𝑑 𝑑 𝑑
(ii) [𝑓(𝑥) − 𝑔(𝑥)] = [𝑓(𝑥)] − [𝑔(𝑥)]
𝑑𝑥 𝑑𝑥 𝑑𝑥

𝑑 𝑑 𝑑
(iii) [𝑓(𝑥). 𝑔(𝑥)] = [ 𝑓(𝑥)] . 𝑔(𝑥) + 𝑓(𝑥) [𝑑𝑥 𝑔(𝑥)]
𝑑𝑥 𝑑𝑥

𝑑 𝑑
𝑑 𝑓(𝑥) [ 𝑓(𝑥)].𝑔(𝑥)−𝑓(𝑥)[ 𝑔(𝑥)]
𝑑𝑥 𝑑𝑥
(iv) [ ]= [𝑔(𝑥)]2
𝑑𝑥 𝑔(𝑥)

𝑑
Some Standard Derivatives𝑑𝑥 (𝑥 𝑛 ) = 𝑛𝑥 𝑛−1

𝑑
(i) (𝑠𝑖𝑛𝑥) = 𝑐𝑜𝑠𝑥
𝑑𝑥

𝑑
(ii) (cos 𝑥) = −𝑠𝑖𝑛𝑥
𝑑𝑥

𝑑
(iii) (tan 𝑥) = 𝑠𝑒𝑐 2 𝑥
𝑑𝑥

𝑑
(iv) (cot 𝑥) = −𝑐𝑜𝑠𝑒𝑐 2 𝑥
𝑑𝑥

𝑑
(v) (sec 𝑥) = sec 𝑥 tan 𝑥
𝑑𝑥

𝑑
(vi) (𝑐𝑜𝑠𝑒𝑐 𝑥) = −𝑐𝑜𝑠𝑒𝑐 𝑥 cot 𝑥
𝑑𝑥

𝑑
(vii) (𝑎 𝑥 ) = 𝑎 𝑥 ln 𝑎
𝑑𝑥

𝑑
(viii) (𝑒 𝑥 ) = 𝑒 𝑥
𝑑𝑥

𝑑 1
(ix) (𝑙𝑜𝑔𝑥) =
𝑑𝑥 𝑥

219
MCQs

1−cos 𝑥
1. lim is equal to:
𝑥→0 sin 𝑥

2 3
(a) 0 (b) 3 (c) -2 (d) 1

2. Find the derivative of sin n x with respect to x.

(a) n sin n – 1x. sin x (b) n cos n – 1x. sin x

(c) (n-1) sin n x. cos x (d) n sin n – 1x. cos x

3. (𝑥+1)5 −1
Evaluate: lim
𝑥→0 𝑥

(a) 2 (b) 5 (c) 0 (d) 1

4. 𝑠𝑖𝑛𝑎𝑥
Evaluate: lim
𝑥→𝑎 𝑏𝑥

𝑎 𝑏 𝑎
(a) (b) (c) - 𝑏 (d) ) 0
𝑏 𝑎

5. sin(𝜋−𝑥)
Evaluate: lim
𝑥→𝜋 𝜋(𝜋−𝑥)

1
(a) π (b) 𝜋 (c) 0 (d) 1
𝑡𝑎𝑛2𝑥
6. Evaluate: lim𝜋 𝜋
𝑥→ (𝑥− 2 )
2

(a) 2 (b) 5 (c) 0 (d) π

7. Find the derivative of (sin x. cos x) with respect to x.

a) cos 2x (b) cos x (c) sin 2x (d) sin x

8. Find the derivative of [ x -4. (3 – 4 x -7)] with respect to x.

(a) 12 x -5 + 44 x – 12 (b) -12 x - 5 + 44 x 12

(c) 12 x - 5 + 44 x 12 (d) -12 x - 5 + 44 x – 12

9. Find the derivative of (tan x – sec x) with respect to x.

a) Sec2x + sec x tan x (b) Sec2x – sec x cot x

(c) Sec2x – sec x tan x (d) Sec x – sec x tan x

220
10. Find the derivative of (5 sin x – 7 sec x) with respect to x.

a) 5 cos x – 7 sec x (b) 7 cos x – 5 sec x

(c) 5 cos x +7 sec x tan x (d) 5 cos x – 7 sec x tan x

ASSERTION REASONING QUESTIONS


1
1. Statement: ƒ(𝑥) = 𝑠𝑖𝑛2 𝑥 + 2 𝑐𝑜𝑠2𝑥 + 𝑐𝑜𝑡𝛼, 𝑡ℎ𝑒𝑛 ƒ′(𝑥) = 0

Reason: Derivative of a constant function is always zero.

(a) Both A and R are true and R is the correct explanation of A

(b) Both A and R are true but R is NOT the correct explanation of A.

(c) A is true but R is false.

(d) A is false but R is true .

(e) Both A and R are false.

3
2. Statement: The derivative of 𝑦 = 2𝑥 – 4 w. r .t. x is 2.

Reason: The derivative of y=cx w. r .t. x is c.

(a) Both A and R are true and R is the correct explanation of A .

(b) Both A and R are true but R is NOT the correct explanation of A.

(c) A is true but R is false.

(d) A is false but R is true.

(e) Both A and R are false.

221
3. Statement: The derivative of f(𝑥) = 𝑥 3 𝑤. 𝑟 . 𝑡. 𝑥 i𝑠 𝑥 2 .

Reason: The derivative of f(𝑥) = 𝑥 𝑛 𝑤. 𝑟 . 𝑡. 𝑥 i𝑠 𝑥 𝑛−1 .

(a) Both A and R are true and R is the correct explanation of A.

(b) Both A and R are true but R is NOT the correct explanation of A.

(c) A is true but R is false.

(d) A is false but R is true.

(e) Both A and R are false.

𝑠𝑖𝑛𝑎𝑥 𝑎
4. Statement: lim i𝑠 𝑒𝑞𝑢𝑎𝑙 𝑡o 𝑏.
𝑥→0 𝑏𝑥

𝑠𝑖𝑛𝑥
Reason: lim = 1.
𝑥→0 𝑥

(a) Both A and R are true and R is the correct explanation of A.

(b) Both A and R are true but R is NOT the correct explanation of A.

(c) A is true but R is false.

(d) A is false but R is true.

(e) Both A and R are false.

𝑐𝑜𝑠2𝑥−1
5. Statement: : lim i𝑠 𝑒𝑞𝑢𝑎𝑙 𝑡𝑜 4.
𝑥→0 𝑐𝑜𝑠𝑥−1

𝑡𝑎𝑛𝑥
Reason: : lim =1
𝑥→0 𝑥

(a) Both A and R are true and R is the correct explanation of A.

(b) Both A and R are true but R is NOT the correct explanation of A.

(c) A is true but R is false.

(d) A is false but R is true.

(e) Both A and R are false.

222
SHORT ANSWER TYPE QUESTIONS

1
1. Find the derivative of with respect to x.
𝑥

2. Find the derivative of x3 – 3 w. r .t. x at x = 10.

3. Find the derivative of (5 sec x + 7cos x) with respect to x.

4. Find 𝑓(𝑥), where f(x) = |𝑥| – 5.

5. 𝑎𝑥+𝑏
Evaluate : lim
𝑥→0 𝑐𝑥+1

𝑥+1
6. Find the derivative of from first principle.
𝑥−1

7. 𝑓(𝑥)−2
If the function f(x) satisfies = 𝜋.Evaluate: 𝑓(𝑥)
𝑥 2 −1

8. Find the derivative of (x3 + x2 + 3)(x – 5) with respect to x.

9. 𝑥 2 − 1, 𝑥≤1
Find 𝑓(𝑥) , where f(x) = { at x=1
−𝑥 2 − 1, 𝑥>1

10. 𝑥 100 𝑥 99 𝑥2
For the function f(x) = + +……….+ + x + 1. Prove that: f ‘(1) = 100 f ‘(0).
100 99 2

223
CASE STUDY BASED QUESTIONS

1. The shape of a Syntex water tank of a house as shown in the figure.

Radius of its base of tank is ‘x’ m and its height is ‘2x’ m. If total surface area and
volume of tank is denoted by A and V respectively then:

(i) What is the derivative of circumference of its base?

(a) π (b) 2π (c) 3π (d) 4π

(ii) 𝑑𝐴
Find :
𝑑𝑥

(a) 12π x (b) 24πx (c) 36πx (d) 48πx

(iii) 𝑑𝑉
Find :
𝑑𝑥

(a) 2π (b) 4π (c) 6πx2 (d) 8π

(iv) What is the derivative of total surface area of the tank at x = 3:

(a) 12π (b) 24π (c) 36π (d) 48π

(v) Find the derivative of its volume at x = 2:

(a) 6π (b) 2π (c) 18π (d) 24π

2. Indeterminate forms of limits.


0 ∞
On direct evaluation, if a limit takes the form 0 , ∞ , 0 × ∞, …, we use standard results
0 ∞
for evaluating the limits. The below figure shows a few indeterminate forms. ,∞0 , ∞
0
, 00 , 10 ,0 × ∞, ∞ - ∞

Based on the above data, answer any four of the following questions

224
(i) 𝑥 6 −64
Evaluate:lim
𝑥→2 𝑥−2

(a) 0 (b) 80 (c) 192 (d) ∞

(ii) 𝑥 15 −1
Evaluate:lim 𝑥 10 −1
𝑥→1
3
(a) 0 (b) 2 (c) ∞ (d) 15

(iii) Evaluate:lim
√1+3𝑥−√1−3𝑥
𝑥→0 𝑥

(a) 0 (b) 1 (c) 3 (d) 6

(iv) 8𝑥 −2𝑥
Evaluate:lim
𝑥→0 𝑥

(a) 0 (b) log2 (c) log4 (d) log8

(v) 𝑐𝑜𝑠5𝑥−𝑐𝑜𝑠𝑥
Evaluate:lim
𝑥→0 𝑥2

(a) 0 (b) -12 (c) 1 (d) 12

3. Let f(x) be a real function defined as

𝑙𝑜𝑔(1+𝑎𝑥)−log(1−𝑏𝑥)
𝑓(𝑥) = , 𝑥< 0
𝑥

𝑓(𝑥) = 5,𝑥 = 0

√1+𝑏𝑥−1
𝑓(𝑥) = , 𝑥> 0
𝑥

Based on the above information, answer the following questions:

(i) 𝑓(𝑥) i𝑠

(a) a + b (b) a - b (c) b - a (d) –a – b

(ii) 𝑓(𝑥)i𝑠
(a) b (b) 𝑏/2 (c) 𝑏/3 (d) 𝑏/4

225
(iii) 𝑓(𝑥) = 𝑓(𝑥), then a relation between a and b is
(a) a+2b=0 (b) 2a-b=0 (c) 2a+b=0 (d) 3a+2b=0

(iv) The value of b, if 𝑓(𝑥) = ƒ (0) i𝑠


(a) 5 (b) 15 (c) 20 (d) 10

(v) The values of a and b if 𝑓(𝑥) = ƒ (0) = 𝑓(𝑥) are


(a) -5,5 (b) -5,10 (c) 5,10 (d) 10,15

4. The derivative of y with respect to x is the change in y with respect to a change in x. The
derivative of f(x) at 𝑥ₒ is given by

∆𝑦 𝑓(𝑥ₒ+∆𝑥)−𝑓(𝑥ₒ)
ƒ′(𝑥ₒ) = ∆𝑥 = 𝑙𝑖𝑚
∆𝑥→0 ∆𝑥

Based on the above information, answer any four of the following questions:

(i) Derivative of sin x with respect to x is:

(a) sin x (b) cos x (c) –sin x (d) –cos x

(ii) Derivative of cos x with respect to x is:

(a) sin x (b) cos x (c) –sin x (d) –cos x

(iii) Derivative of tan x with respect to x is :

(a)𝑠𝑒𝑐 2 𝑥 (b)− 𝑠𝑒𝑐 2 𝑥 (c) 𝑐𝑜𝑠𝑒𝑐 2 𝑥 (d) −𝑐𝑜𝑠𝑒𝑐 2 𝑥

(iv) If ƒ(𝑥) = 𝑥100 – 𝑥 80 , ƒ′(1) is………..

(a) 0 (b) 20 (c) 51 (d) 101100

(v) 𝑥 𝑑𝑦
𝑦 = 𝑡𝑎𝑛𝑥 , 𝑡ℎ𝑒𝑛𝑑𝑥 =…………..

𝑡𝑎𝑛𝑥−𝑠𝑒𝑐𝑥 𝑡𝑎𝑛𝑥−𝑥 𝑠𝑒𝑐 2 𝑥


(a) 𝑐𝑜𝑠 2 𝑥 (b) 𝑠𝑒𝑐 2 𝑥 (c) (d)
𝑡𝑎𝑛2 𝑥 𝑡𝑎𝑛2 𝑥

226
LONG ANSWER TYPE QUESTIONS

1. Find the derivative of cos (x +1) from first principle.

2. Suppose f(x) = {𝑎 + 𝑏𝑥, 𝑥 < 14, 𝑥 = 1𝑏 − 𝑎𝑥, 𝑥 > 1 and if 𝑓(𝑥) =


𝑓(1).
What are the possible values of a and b?

3. 𝑎 𝑏 𝑐
Find the derivative of
𝑥 4 − 𝑥 3 + 𝑥2
+ 𝑐𝑜𝑠𝑥 w.r.to x.
4. Find the derivative of (x cos x) from first principle.

5. 𝑚𝑥 2 + 𝑛, 𝑥<0
If f(x) = {𝑛𝑥 + 𝑚, 0≤𝑥≤1 for what integers m and n does both 𝑓(𝑥)
𝑛𝑥 3 + 𝑚, 𝑥>1
and 𝑓(𝑥) exist ?

ANSWER KEY AND SOLUTIONS


MCQs

1. a 2. d 3. b 4. a 5. b

6. a 7. a 8. d 9. c 10. d

ASSERTION REASONING QUESTIONS

1. (b) Both A and R are true but R is NOT the correct explanation of A.

2. (a) Both A and R are true and R is the correct explanation of A

3. (d) A is false but R is true

4. (a) Both A and R are true and R is the correct explanation of A

5. (b) Both A and R are true but R is NOT the correct explanation of A

227
SHORT ANSWER TYPE QUESTIONS

1. 1

𝑥2
2. 300
3. 5 sec x tan x – 7 sin x
4. 0

5. b

6. −2
(𝑥 − 1)2

7. 2

8. 4x3 – 12x2 – 10x + 3

9. Limit does not exist at x = 1.

10. Prove that f ‘(1) = 100 f ‘(0)

CASE STUDY BASED QUESTIONS

1. (i) 2π (ii) 12πx (i) 6π𝑥 2 (iv) 36π (v) 24π

2. (i) c (ii) b (iii)c (iv) c (v)b

3. (i) a (ii) b (iii) c (iv) d (v)b

4. (i) b (ii) c (iii) a (iv) b (v)b

228
LONG ANSWER TYPE QUESTIONS

1. -Sin (x +1)

2. a=0, b=4.

4𝑎 3𝑏 2𝑐
3. − + - - sinx
𝑥5 𝑥4 𝑥3

4. Cos x – x sin x

5. For 𝑓(𝑥), m = n and for 𝑓(𝑥), m and n may have any integral value.

229
STATISTICS
CONCEPTUAL NOTES:
∑ 𝑥𝑖
** Mean ,𝑋̅ = 𝑛

∑ 𝑓𝑖 𝑥𝑖
=
∑ 𝑓𝑖

∑ 𝑓𝑖 𝑢𝑖 𝑥𝑖−𝑎
= a +( ) h ,where a is the assumed mean, h is the class width and 𝑢𝑖 =
∑ 𝑓𝑖 ℎ

𝑛+1
** Median = ( 2
)𝑡ℎ observations (arranged in ascending or descending order) and the number of
observations is odd.
𝑛 𝑛
= mean of ( 2)𝑡ℎ 𝑎𝑛𝑑 ( 2 + 1) 𝑡ℎ observations arranged in ascending or descending order & the number of
observations is odd.
𝑛
−𝑐𝑓
= l + ( 2 𝑓 )xh , where, l lower limit of median class, n number of observations, cf cumulative frequency of class
preceding the median class, f frequency of median class, h class size.
** Measures of Dispersion: The dispersion or scatter in a data is measured on the basis of the observations
and the types of the measure of central tendency, used there. There are following measures of dispersion:
(i) Range, (ii) Quartile deviation, (iii) Mean deviation, (iv) Standard deviation.

** Range: Range of a series = Maximum value – Minimum value.


** Mean Deviation : The mean deviation about a central value „a‟ is the mean of the absolute values of
the deviations of the observations from „a‟. The mean deviation from „a‟ is denoted as M.D. (a).
1
(i)For ungrouped data: M.D about mean=𝑛 ∑𝑛𝑖=1|𝑥𝑖 − x̅| ,where 𝑥̅ is the mean .
1
M.D about median =𝑛 ∑𝑛𝑖=1|𝑥𝑖 − 𝑀| ,where M is the median.

(ii) For grouped frequency distribution:


(a) Discrete frequency distribution:
∑𝑛
𝑖=1 𝑓𝑖 |𝑥𝑖 −𝑥̅ | 1
MD (𝑥̅ ) = ∑𝑛
=𝑛 ∑𝑛𝑖=1 𝑓𝑖 |𝑥𝑖 − x̅|
𝑖=1 𝑓𝑖
1
MD (𝑀) = 𝑛 ∑𝑛𝑖=1 𝑓𝑖 |𝑥𝑖 − 𝑀|
(b) Continuous frequency distribution:
1 ∑ 𝑓𝑖 𝑢𝑖
MD (𝑥̅ ) = 𝑛 ∑𝑛𝑖=1 𝑓𝑖 |𝑥𝑖 − 𝑥̅ | ,𝑋̅ = a +( )h,
∑ 𝑓𝑖

230
𝑛
1 −𝑐𝑓
MD (𝑀) = ∑𝑛 𝑓 |𝑥 − 𝑀| ,M= l + ( 2
)xh
𝑛 𝑖=1 𝑖 𝑖 𝑓

1
(iii)Variance : Let x1, x2, ..., xn be n observations with 𝑥̅ is the mean. The variance, denoted σ2 =𝑛 (𝑥𝑖 − 𝑥̅ )2

(iv)Standard Deviation: If σ2 is the variance, then σ, is called the standard deviation, is given by 𝜎 =
1
√ (𝑥𝑖 − 𝑥̅ )2
𝑛

(v)Standard deviation for a discrete frequency distribution is given by

1
𝜎 = √ 𝑓𝑖 (𝑥𝑖 − 𝑥̅ )2
𝑁

where fi’s are the frequencies of xi’ s and N = ∑𝑛𝑖=1 𝑓𝑖


(vi) Standard deviation of a continuous frequency distribution (grouped data) is given by

1
𝜎 = √ 𝑓𝑖 (𝑥𝑖 − 𝑥̅ )2
𝑁

where xi are the midpoints of the classes and fi their respective frequencies.
MCQs
1. The mean deviation of the data 2, 9, 9, 3, 6, 9, 4 from the mean is
(A) 2.23 (B) 2.57 (C) 3.23 (D) 3.57
Solution (B) is the correct answer
M.D. ( x ) = (4+ 3+ 3+ 3+ 0+ 3+ 2)/7 =2.57
2. Variance of the data 2, 4, 5, 6, 8, 17 is 23.33. Then variance of 4, 8, 10, 12, 16, 34 will be
(A) 23.23 (B) 25.33 (C) 46.66 (D) 48.66
Solution (C) is the correct answer. When each observation is multiplied by 2, then variance is also
multiplied by 2.
1. A set of n values x1, x2, ..., xn has standard deviation σ . The standard deviation of n values x1 + k, x2
+ k, ..., xn + k will be
(A) σ (B) σ + k (C) σ – k (D) kσ
Solution (A) is correct answer. If observation is increased by a constant k, , then standard deviation is
unchanged.
2. A quality control team tested the lifetimes of a set of light bulbs to ensure their reliability. The
recorded lifetimes (in ours) for 5 bulbs were: 1357, 1090, 1666, 1494, 1623

231
Calculate the mean deviation (in hours) from the mean lifetime for these light bulbs.

A) 178 B) 179 C) 220 D) 356


Ans. A)178
3. The following information relates to a sample of size 60:
X2 = 18000, x = 960. The variance is:
(A) 6.63 (B) 16 (C) 22 (D) 44
Ans: 44
6. The average marks obtained by the students in a class are 43. If the average marks obtained by 25 boys
are 40 and the average marks obtained by the girl students are 48, then what is the number of girl students in
the class?

a) 18 b) 17 c) 20 d) 15

Ans. d)15

7. The sum of 10 items is 12 and the sum of their squares is 18. The standard deviation is
(a)1/5 (b) 2/5 (c) 3/5 (d) 4/5

Ans: (c) 3/5

8. Find the mean deviation about the median for the following data :
3, 9, 5, 3, 12, 10, 18, 4, 7, 19, 21.
( a) 5.5 (b) 5.27 (c) 6.27 (d) 6.5

Ans: (b) 5.27

9. When tested the lives (in hours) of 5 bulbs were noted as follows : 1357, 1090, 1666, 1494, 1623. The mean
of the lives of 5 bulbs is

(a) 1445 (b) 1446 (c) 1447 (d) 1448

Ans: (b) 1446

10. Compute the variance and standard deviation of the following observations of marks of 5 students of a
tutorial group :
Marks out of 25 : 8 , 12, 13 , 15 , 22 .
(a) 13 (b) 15 (c) 15.2 (d) 21.2
232
Ans: (d) 21.2

Assertion Reasoning
DIRECTION : In each of the following questions, a statement of Assertion is given followed by a

corresponding statement of Reason just below it. Of the statements, mark the correct answer as

(a) Both assertion and reason are true and reason is the correct explanation of assertion.

(b) Both assertion and reason are true but reason is not the correct explanation of assertion.

(c) Assertion is true but reason is false.

(d) Assertion is false but reason is true

𝐷𝑒𝑣𝑖𝑎𝑡𝑖𝑜𝑛𝑠
11. Assertion (A): Sum of absolute values of Mean of deviations = 𝑁𝑢,𝑏𝑒𝑟 𝑜𝑓 𝑜𝑏𝑠𝑒𝑟𝑣𝑎𝑡𝑖𝑜𝑛𝑠

Reason(R) : Sum of deviations from mean is 1.

Soln: Answer is ( C ) A is correct and R is incorrect as the sum is zero.


𝑝𝑟𝑜𝑑𝑢𝑐𝑡 𝑜𝑓 𝐷𝑒𝑣𝑖𝑎𝑡𝑖𝑜𝑛𝑠
12. Assertion (A):Mean of deviations = 𝑁𝑢,𝑏𝑒𝑟 𝑜𝑓 𝑜𝑏𝑠𝑒𝑟𝑣𝑎𝑡𝑖𝑜𝑛𝑠

Reason(R) :To find the dispersion of values of x from mean ,we take absolute measure of dispersion.

Soln: Answer is (d)

13. Let 𝑥1 , 𝑥2 , 𝑥3 … 𝑥𝑛 𝑏𝑒 𝑛 𝑜𝑏𝑠𝑒𝑟𝑣𝑎𝑡𝑖𝑜𝑛𝑠 𝑎𝑛𝑑 𝑙𝑒𝑡 𝑥̅ be the mean and 𝜎2 be the variance .

Assertion (A): Variance of 2𝑥1 , 2𝑥2 , 2 𝑥3 … 2𝑥𝑛 is 4𝜎2

Reason(R) : Arithmetic mean of 2𝑥1 , 2𝑥2 , 2 𝑥3 … 2𝑥𝑛 is 4𝑥̅ .

Soln: Answer is ( C ) If each observation is multiplied by k, mean gets multiplied by k and variance gets
multiply by square of k.

Hence the new mean = 2x ̅ and new variance =𝑘 2 𝜎 2 ..

So Assertion is true and reason is false.

14. Assertion (A): The range is the difference between two extreme observations of the distribution.

Reason(R) : The variance of a variate X is the arithmetic mean of the squares of all k deviations of X from
the arithmetic mean of the observations.

Soln : Answer is ( b)

15. Assertion (A):The mean deviation of the data 2, 9, 9, 3, 6, 9,4 from the mean is 2,57 .

233
1
Reason(R) : For individual observation Mean Deviation about mean is 𝑛 ∑𝑛𝑖=1|𝑥𝑖 − x̅|

Soln: Answer is ( a)

Mean =n(2+3+9+9+3+6+9+4)/7 =6

MD= (4+3+3+3+0+3+2)/7 = 2.57

SHORT ANSWER QUESTIONS


16. A fitness trainer is conducting a study on the performance of two different workout routines. The trainer
tracks the number of repetitions completed by participants in each routine. There are two sets of
observations, each containing 20 participants. The first set has a mean of 17 repetitions, and the second set
has a mean of 22 repetitions. Surprisingly, both sets have the same standard deviation of 5 repetitions.

What would be the standard deviation of the combined set obtained by merging the two sets of observations?

Ans: To determine the standard deviation of the combined set, we need to consider the concept of weighted
averages and their effect on standard deviation.

Calculate the weighted average of the means:

Weighted Mean = (Number of observations in Set 1 * Mean of Set 1 + Number of observations in Set 2 *
Mean of Set 2) / Total Number of Observations

Weighted Mean = (20 * 17 + 20 * 22) / 40 = 19.5.

Calculate the variance of the combined set using the formula:

Variance = (Number of observations in Set 1 * Variance of Set 1 + Number of observations in Set 2 *


Variance of Set 2) / Total Number of Observations

Variance = (20 * 52 + 20 * 52) / 40 = 25.

Calculate the standard deviation of the combined set:

Standard Deviation = √Variance = √25 = 5.

Therefore, the standard deviation of the combined set obtained by merging the two sets of observations
would be 5.

17.A collection of 100 items was analyzed, and the statistical properties of the data were observed. The mean
of the items is 50, and the standard deviation is 4.

234
Calculate the sum of all the items and the sum of the squares of the items.

Ans: To solve this, we can use the formulas for the mean and standard deviation:

Sum of all items = Mean × Number of items = 50 × 100 = 5000.

Sum of squares of items = Variance × (Number of items - 1) + Mean2 × Number of items

= (42) × (100 - 1) + 502 × 100 = 159600.

So, the sum of all the items is 5000, and the sum of the squares of the items is 159600.

18. A set of data points were collected and analyzed. For this distribution, two pieces of information were
obtained: (x – 5) = 3 and (x – 5)2 = 43. It is also known that the total number of items in the dataset is 18.

Calculate the mean and standard deviation for this distribution.

Ans: Given that (x - 5) = 3, we can solve for x:

x = 3 + 5 = 8.

Now, let's calculate the mean and standard deviation:

Mean:

The sum of all values (x) can be calculated by multiplying the mean by the number of items: Sum = Mean ×
Number of items = 8 × 18 = 144.

Variance:

Variance = [(Sum of squares of all values) - (Sum of all values)2 / Number of items] / (Number of items - 1)

Plugging in the values:

Variance = [(43) - (1442 / 18)] / 17 ≈ 9.

Standard Deviation:

Standard Deviation = √Variance = √9 = 3.

So, the mean of the distribution is 8 and the standard deviation is 3.

235
19. If the mean and standard deviation of 100 observations are 50 and 4 respectively. Find the sum of all the
observations and the sum of their squares.
Ans: Let 𝑥1 , 𝑥2 , ……..𝑥100 be 100 observations and their mean = 𝑥̅ and standard deviation = σ

𝛴𝑥 𝛴𝑥 2
Mean 𝑥̅ = 𝜎2 = − x̅ 2
𝑛 𝑛
𝛴𝑥 𝛴𝑥 2
50 = 100 42 = 100 − 50 2

1600 = 𝛴𝑥2 − 250000


Sum of all observations 𝛴𝑥 = 5000 Sum of their squares 𝛴 𝑥2 = 251600

20. Calculate the mean deviation about the median of the following observations:

38, 70, 48, 34, 63, 42, 55, 44, 53, 47

Ans: Arranging the observations in ascending order :34 , 38 , 42 , 44 , 47 , 48 , 53 , 55 , 63 , 70


47+48
Median M = = 47.5
2

Calculation of mean deviation about the median .

𝑥 |𝑑|=|𝑥 − 𝑀|

34 13.5

38 9.5

42 5.5

44 3.5

47 0.5

48 0.5

53 5.5

55 7.5

63 15.5

70 22.5
𝛴|𝑥 − 𝑀| = 84
𝛴|𝑥−𝑀| 84
Mean Deviation = = 10 = 8.4
𝑛

236
21. Find the mean and variance of first n natural numbers.
Ans: First n natural numbers are = 1 , 2 , 3 , …………. , n
(1+2+3+⋯+𝑛) 1 𝑛(𝑛+1) (𝑛+1)
Mean 𝑥 = =𝑛. =
𝑛 2 2

𝛴𝑥 2 𝛴𝑛 2 (𝑛+1) 2 𝑛(𝑛+2)(2𝑛+1) (𝑛+1)2 (𝑛2 −1)


Variance 𝜎2 = − 𝑥2 = − { } = − =
𝑛 𝑛 2 6𝑛 4 12

22. Find the variance and standard deviation for the following data: 57, 64, 43, 67, 49, 59, 44, 47, 61, 59.
57+64+43+67+49+59+61+59+44+47 550
Ans: Mean(𝑥)= = 10 = 55
10

Variance (𝜎2 )
2
∑ (𝑥𝑖 −𝑥)
= 𝑛

22 + 92 + 122 + 122 + 62 + 42 + 62 + 42 + 112 + 82


=
10
662
=
10
=66.2

Standard deviation (𝜎)=√𝜎 2 = √66.2 = 8.13

23. The mean weight of 150 students in a certain class is 60 kilograms. The mean weight of boys in the class is
70 kilograms and that of the girls is 55 kilograms, then find the number of boys and girls of the class.
Ans: Total students in class =150

mean weight=60kg
total weight =150×60=9000kg
Let the total number of boys =x
mean weight of boys =70kg
total weight of boys=70xkg
total number of girls = total students - no. of boys =150−x
mean weight of girls=55kg
total weight of girls =55×(150−x)=55×150−55x=(8250−55x)kg
Total weight = weight of boys + weight of girls
9000=70x+(8250−55x)
9000=70x+8250−55x
9000−8250=70x−55x

237
750=15x
x=15750
x=50
So number of boys =50
number of girls =150−50=100
24. The mean of 100 observations is 50 and their standard deviation is 5. Then find the sum of squares of all
observations.
Ans: ∑ 𝑥𝑖 2 = n { 𝜎 2 + ( 𝑥̅ )2 } = 100 (502 + 52 ) =252500
25. Mean of 10 items is 17. If an observation 21 is replaced with 12, then what will be the new mean.
Ans: Original sum of all the 10 items

=(mean x number of items)


=17 x 10
=170
New sum=170 – 21 + 12 =161
New mean = 161/10 =16.1
CASE BASED QUESTIONS
1. City A's daily average PM10 (particulate matter with a diameter of 10 micrometres or smaller) levels
for a week were: 50, 60, 45, 70, 55, 65, 75 (in µg/m³). City B's corresponding PM10 levels were: 40, 80,
50, 60, 55, 85, 65 (in µg/m³).

a) What is the range of City A ?


b) What is the range of City B ?
c) Which city had greater variability in PM10 levels?
Soln:

Variability in data can be measured using measures of dispersion such as the range, variance, and standard
deviation. In this case, we need to compare the variability of PM10 levels in City A and City B.

City A's PM10 levels: 50, 60, 45, 70, 55, 65, 75
City B's PM10 levels: 40, 80, 50, 60, 55, 85, 65

238
a) The range is the difference between the maximum and minimum values.
For City A:
Range = 75 (max) - 45 (min) = 30
b) For City B:
Range = 85 (max) - 40 (min) = 45
c) City B has a larger range of PM10 levels, indicating greater variability in its data. Therefore, the correct
answer is City B.

2. A fitness trainer is conducting a study on the performance of two different workout routines. The
trainer tracks the number of repetitions completed by participants in each routine. There are two sets
of observations, each containing 20 participants. The first set has a mean of 17 repetitions, and the
second set has a mean of 22 repetitions. Surprisingly, both sets have the same standard deviation of 5
repetitions.

a) Calculate the weighted average of the means.

b) Calculate the variance of the combined set.

c) Calculate the standard deviation of the combined set.

Soln:
a) Weighted Mean = (Number of observations in Set 1 * Mean of Set 1 + Number of observations in Set 2 *
Mean of Set 2) / Total Number of Observations
Weighted Mean = (20 * 17 + 20 * 22) / 40 = 19.5.
b) Variance = (Number of observations in Set 1 * Variance of Set 1 + Number of observations in Set 2 *
Variance of Set 2) / Total Number of Observations
Variance = (20 * 52 + 20 * 52) / 40 = 25.
C ) Standard Deviation = √Variance = √25 = 5.

239
3. A group of athletes participated in a practice session for a particular exercise routine over the course of
several days. The recorded practice times (in minutes) for each athlete were as follows: 38, 70, 48, 40,
42, 55, 63, 46, 54, 44.
a) Calculate the mean for the athlete’s practice time.

a) Calculate the mean deviation about the mean for the athletes' practice times.

b) Calculate the deviations from the mean for each practice time.

c) Calculate the mean of the absolute deviations

Soln: :
a) mean practice time: (38 + 70 + 48 + 40 + 42 + 55 + 63 + 46 + 54 + 44) / 10 = 50.
b) deviations from the mean for each practice time:
Deviations: -12, 20, -2, -10, -8, 5, 13, -4, 4, -6.
c) the absolute values of the deviations: 12, 20, 2, 10, 8, 5, 13, 4, 4, 6.
the mean of the absolute deviations:
= (12 + 20 + 2 + 10 + 8 + 5 + 13 + 4 + 4 + 6) / 10 = 8.4.
The mean deviation about the mean for the athletes' practice times is indeed 8.4.

4. A group of cars went through a series of servicing sessions at a garage. The recorded mileage before each
servicing session (in thousands of kilometres) for each car were as follows: 36, 72, 46, 42, 60, 45, 53, 46, 51,
49.

a) Find the median of the observations.

b) Calculate the absolute values of deviations of each mileage


from the median.

c)Calculate the mean of the absolute deviations.

Calculate the mean deviation about the median for the


recorded mileages of the cars before their servicing sessions.

Soln: a) Arrange the mileages in ascending order: 36, 42, 45,


46, 46, 49, 51, 53, 60, 72.

the median is the average of the fifth sixth values, which is (46 + 49) / 2 = 47.5.

b) deviations from the median for each mileage:

240
Deviations: -11.5, -5.5, -2.5, -1.5, -1.5, 1.5, 3.5, 5.5, 12.5, 24.5.

absolute values of the deviations: 11.5, 5.5, 2.5, 1.5, 1.5, 1.5, 3.5, 5.5, 12.5, 24.5.

c)Calculate the mean of the absolute deviations:

= (11.5 + 5.5 + 2.5 + 1.5 + 1.5 + 1.5 + 3.5 + 5.5 + 12.5 + 24.5) / 10 = 10.5.

5. A student collected 10 readings and attempted to calculate the mean and variance. Unfortunately, the
student mistakenly used a reading of 52 instead of the correct reading 25. The student calculated the mean
and variance as 45 and 16 respectively.

a) Determine the correct mean by considering the mistaken reading of 52 instead of the actual reading
25.
b) Determine the correct variance by considering the mistaken reading of 52 instead of the actual reading
25.

Soln: a) while calculating the mean and variance of 10 readings, a student wrongly used the reading 52 for
the correct reading 25. He obtained the mean and variance 45 and 16 respectively.

To find correct mean and variance

N=10

Mean before correction (𝑥)=45


10
∑10
i=1 𝑥𝑖
𝑥= ⇒ ∑ 𝑥𝑖 = 𝑥 × 𝑁 = 45 × 10 = 450
𝑁
i=1

Correct ∑10
i=1 𝑥𝑖 = 450 − 52 + 25 = 423

𝐶𝑜𝑟𝑟𝑒𝑐𝑡 ∑ 𝑥𝑖 423
Correct mean= = = 42.3
𝑁 10

b. given 𝜎2 = 16
∑10
i=1 𝑥𝑖
2 ∑10
i=1 𝑥𝑖 2
but 𝜎2 = −( )
𝑁 𝑁
∑10
i=1 𝑥𝑖
2 ∑10
i=1 𝑥𝑖
2
⇒ 16 = − (45)2 ⇒ = 16 + 2025 = 2041
𝑁 10

241
10

⇒ ∑ 𝑥𝑖 2 = 20410
i=1
One reading 25 was wrongly taken as 52, so
correct ∑ 𝑥𝑖 2 = 20410 − 522 + 252 = 20410 − 2704 + 625 = 18331
𝑐𝑜𝑟𝑟𝑒𝑐𝑡 ∑ 𝑥𝑖 2 18331
correct 𝜎2 = − (𝑐𝑜𝑟𝑟𝑒𝑐𝑡 𝑚𝑒𝑎𝑛)2 = − (42.3)2 = 1833.1 − 1769.29 = 43.81
𝑁 10

LONG ANSWER TYPE


1. A group of individuals' heights were measured and categorized into different height ranges (in centimetres).
Find the variance and standard deviation, if the number of individuals falling into each height range is
recorded as follows:

Height 70-75 75 - 80 - 85 - 90 - 95-100 100-105 105-110 110-115


80 85 90 95

frequency 3 4 7 7 15 9 6 6 3

Ans:

CI 𝑓𝑖 𝑥𝑖 𝑥𝑖 − 92.5 𝑦𝑖2 𝑓𝑖 𝑦𝑖 𝑓𝑖 𝑦𝑖2


𝑦𝑖 =
5
70-75 3 72.5 −4 16 −12 48

75-80 4 77.5 −3 9 −12 36

80-85 7 82.5 −2 4 −14 28

85-90 7 87.5 −1 1 −7 7

90-95 15 92.5 0 0 0 0

95-100 9 97.5 1 1 9 9

100-105 6 102.5 2 4 12 24

105-110 6 107.5 3 9 18 54

110-115 3 112.5 4 16 12 48
n n n

∑ 𝑓𝑖 = 60 ∑ 𝑓𝑖 𝑦𝑖 = 6 ∑ 𝑓𝑖 𝑦2𝑖 = 254
1=1 i=1 i=1

∑n
i 𝑓 𝑖 𝑦𝑖 6
Mean =𝑥 = 𝐴 + × ℎ = 92.5 + 60 × 5 = 92.5 + 0.5 = 93
𝑁

242
ℎ2 2
Variance (𝜎 2 ) = {𝑁 ∑ni=1 𝑓𝑖 𝑦2𝑖 − (∑ni=1 𝑓𝑖 𝑦𝑖 ) }
𝑁2

52
= 2 {60 × 254 − (6)2 }
60
25
= {15204} = 105.58
3600
Standard deviation (𝜎)=√105.58 = 10.27
2. An age distribution of 100 persons is available, and their ages are categorized into different age ranges. The
number of persons falling into each age range is recorded as follows:

Find the mean deviation about the median of given data.

age 16-20 21-25 26-30 31-35 36-40 41-45 46-50 51-55

number 5 6 12 14 26 12 16 9

Ans: The given data is not continuous. Therefore, it has to be converted into continuous frequency
distribution.

age Number(𝑓𝑖 ) c.f Mid-point( 𝑥𝑖 ) |𝑥𝑖 − 𝑀𝑒𝑑| 𝑓𝑖 |𝑥𝑖 − 𝑚𝑒𝑑|

15.5-20.5 5 5 18 20 100

20.5-25.5 6 11 23 15 90

25.5-30.5 12 23 28 10 120

30.5-35.5 14 37 33 5 70

35.5-40.5 26 63 38 0 0

40.5-45.5 12 75 43 5 60

45.5-50.5 16 91 48 10 160

50.5-55.5 9 100 53 15 135


n n

∑ 𝑓𝑖 = 100 ∑ 𝑓𝑖 |𝑥𝑖 − 𝑚𝑒𝑑|


i=1 i=1
= 735
𝑁
Here, 2 = 50, median class is 35.5-40.5

243
𝑁
−𝐶
Median =𝑙 + 2 𝑓 × ℎ

𝑙 = 35.5, 𝐶 = 37, 𝑓 = 26, ℎ = 5 𝑎𝑛𝑑 𝑁 = 50


50−37
Median =35.5 + × 5 = 38
26

∑ni 𝑓𝑖 |𝑥𝑖 −𝑚𝑒𝑑| 735


MD(M)= = 100 = 7.35
𝑁

3. In a survey of 44 villages of a state, about the use of LPG as a cooking mode, the following information
about the families using LPG was obtained.

Number of families 0-10 10-20 20-30 30-40 40-50 50-60

Number of villages 6 8 16 8 4 2

i. Find the mean deviation about median for the following data.

Do you think more awareness is needed for the villagers to use LPG as a mode of cooking?

Ans:

Number of Mid value Number of 𝑐𝑓 |𝑥𝑖 − 𝑀| 𝑓𝑖 |𝑥𝑖 − 𝑀|


families (𝑥𝑖 ) villages (𝑓𝑖 )
0 − 10 5 6 6 20 120
10 − 20 15 8 14 10 80
20 − 30 25 16 30 0 0
30 − 40 35 8 38 10 80
40 − 50 45 4 42 20 80
50 − 60 55 2 44 30 60
Here, 𝑁 = 44
𝑁 44
Now, 2 = = 22, which, lies in the cumulative frequency of 30, therefore median class is 20-30
2

𝑙 = 20, 𝑓 = 16, 𝑐𝑓 = 14 𝑎𝑛𝑑 ℎ = 10


𝑁
−𝑐𝑓 22−14
2
Median =𝑙 + × ℎ = 20 + × 10 = 20 + 5 = 25
𝑓 16

∑ni=1 𝑓𝑖 |𝑥𝑖 −𝑚𝑒𝑑| 420


MD(M)= = = 9.55
𝑁 44
ii. There is a need for awareness among villagers for using LPG as a mode of cooking. Because it will help in
keeping the environment clean and will also help in saving of forests.

244
4. From the frequency distribution consisting of 18 observations, the mean and the standard deviation were
found to be 7 and 4, respectively. But on comparison with the original data, it was found that a figure 12 was
miscopied as 21 in calculations. Calculate the correct mean and standard deviation.
∑18
i=1 𝑥𝑖
Ans: Mean =7, (𝑥 = )
𝑁

18
∑18
i=1 𝑥𝑖
7= => ∑ 𝑥𝑖 = 126
18
i=1

Since, an observation 12 was miscopied as 21

So, correct ∑ 𝑥𝑖 = 126 − 21 + 12 = 117


𝑐𝑜𝑟𝑟𝑒𝑐𝑡 ∑18
i=1 𝑥𝑖 117
Hence , true mean= = = 6.5
18 18

Also, given variance =42 = 16


18
∑18
1=1 𝑥
2
2
∑18
i=1 𝑥
2
− (𝑚𝑒𝑎𝑛) = 16 ⟹ = 16 + 72 ⟹ ∑ 𝑥 2 = 18(16 + 49) = 1170
18 18
i=1

But one observation 12 was miscopied as 21

So, correct ∑ 𝑥𝑖 2 = 1170 − 212 + 122 = 1170 − 441 − 144 = 873

𝑐𝑜𝑟𝑟𝑒𝑐𝑡 ∑ 𝑥𝑖 2 873
Hence ,true variance = − (𝑡𝑟𝑢𝑒 𝑚𝑒𝑎𝑛)2 = − (6.5)2 = 48.5 − 42.25 = 6.25
18 18

True S.D=√𝑡𝑟𝑢𝑒 𝑣𝑎𝑟𝑖𝑎𝑛𝑐𝑒 = √6.25 = 2.5

5. The mean and standard deviation of 20 observations are found to be 10 and 2 respectively. On checking, it
was found that an observation 8 was incorrect. Calculate the correct mean and correct standard deviation in
each of the following cases:
(a) If the wrong observation is omitted.
(b) If it is replaced by 12.
Ans: (i) Given, number of observations n=20
Incorrect mean =10
Incorrect standard deviation =2
20
1
𝑥 = ∑ 𝑥𝑖
𝑛
𝑖=1
20
1
10 = ∑ 𝑥𝑖
20
𝑖=1

245
20

∑ 𝑥𝑖 = 200
𝑖=1

So, the incorrect sum of observations =200


Correct sum of observation =200−8=192
⇒ Correct mean =Correct sum/19 =192/ 19=10.1

1 1 2 1 2
S.D (σ) = √𝑛 ∑𝑛𝑖=1 𝑥𝑖 2 − (𝑛 ∑𝑛𝑖=1 𝑥𝑖 ) = √𝑛 ∑𝑛𝑖=1 𝑥𝑖 2 − (𝑥)

1
2 = √20 𝑖𝑛𝑐𝑜𝑟𝑟𝑒𝑐𝑡 ∑𝑛𝑖=1 𝑥𝑖 2 − (10)2

1
4 = 20 𝑖𝑛𝑐𝑜𝑟𝑟𝑒𝑐𝑡 ∑𝑛𝑖=1 𝑥𝑖 2 − 100
𝑛

𝑖𝑛𝑐𝑜𝑟𝑟𝑒𝑐𝑡 ∑ 𝑥𝑖 2 = 2080
𝑖=1
𝑛 𝑛

𝑐𝑜𝑟𝑟𝑒𝑐𝑡 ∑ 𝑥𝑖 = 𝑖𝑛𝑐𝑜𝑟𝑟𝑒𝑐𝑡 ∑ 𝑥𝑖 2 − (8)2 = 2080 − 64 = 2016


2

𝑖=1 𝑖=1

1
∴ Correct Standard deviation = √𝑛 𝑐𝑜𝑟𝑟𝑒𝑐𝑡 ∑𝑛𝑖=1 𝑥𝑖 2 − (𝑐𝑜𝑟𝑟𝑒𝑐𝑡 𝑚𝑒𝑎𝑛)2

2016
=√ − (10.1)2 = √106.1 − 102.01 = √4.09 = 2.02
19

(ii)When 8 is replaced by 12
Incorrect sum of observation =200
∴ Correct sum of observations =200−8+12=204
∴ Correct mean =Correct sum /20 =204/20 =10.2

1 1 2 1 2
S.D (σ) = √𝑛 ∑𝑛𝑖=1 𝑥𝑖 2 − (𝑛 ∑𝑛𝑖=1 𝑥𝑖 ) = √𝑛 ∑𝑛𝑖=1 𝑥𝑖 2 − (𝑥)

1
2 = √20 𝑖𝑛𝑐𝑜𝑟𝑟𝑒𝑐𝑡 ∑𝑛𝑖=1 𝑥𝑖 2 − (10)2

1
4 = 20 𝑖𝑛𝑐𝑜𝑟𝑟𝑒𝑐𝑡 ∑𝑛𝑖=1 𝑥𝑖 2 − 100, 𝑖𝑛𝑐𝑜𝑟𝑟𝑒𝑐𝑡 ∑𝑛𝑖=1 𝑥𝑖 2 = 2080
𝑛 𝑛

𝑐𝑜𝑟𝑟𝑒𝑐𝑡 ∑ 𝑥𝑖 = 𝑖𝑛𝑐𝑜𝑟𝑟𝑒𝑐𝑡 ∑ 𝑥𝑖 2 − (8)2 + 122


2

𝑖=1 𝑖=1

= 2080 − 64 + 144 = 2160

1
∴ Correct Standard deviation = √𝑛 𝑐𝑜𝑟𝑟𝑒𝑐𝑡 ∑𝑛𝑖=1 𝑥𝑖 2 − (𝑐𝑜𝑟𝑟𝑒𝑐𝑡 𝑚𝑒𝑎𝑛)2

246
2160
=√ − (10.2)2 = √108 − 104.04 = √3.96 = 1.98
20

SELF PRACTICE QUESTIONS


1. A group of students participated in a science experiment, and their recorded results (in arbitrary units) were
as follows: 6, 5, 9, 13, 12, 8, 10.

Calculate the standard deviation of the recorded results.

(A) 52 B) √ (52/7) C) √ 5 D) 4

2. A group of researchers conducted a study on the growth of a certain type of plant. They collected 100
observations of the plant’s height (in centimetres) and found that the mean height was 50 centimetres, and
the standard deviation was 5 centimetres.

Calculate the sum of the squares of all the observations.

(A) 50000 B) 250000 C) 252500 D) 255000

3. A meteorological study collected temperature data in degrees Celsius (°C) to analyse variations. The
standard deviation of the collected data was found to be 5 °C.

If the temperature data were converted into degrees Fahrenheit (ºF), what would be the variance of the data?

(A) 81 B) 57 C) 36 D) 25

4. A man travels at a speed of 20 km/hour and then returns at a speed of 30 km/hour. His average speed of
the whole journey is

a) 25 km/hour b) 24 km/hour c) 24.5 km/hour d) 26 km/hour

5. The mean and standard deviation of 100 items are 50, 5 and that of 150 items are 40, 6 respectively. What
is the combined standard deviation of all 250 items?

a) 7.5 b) 7.7 c) 7.3 d) 7.1

247
6. If the mean of the squares of first n natural numbers is 11, then n is equal to

a) 5 b) 14 c) 11 d) 13

7. Consider the numbers 1, 2, 3, 4, 5, 6, 7, 8, 9, 10. If 1 is added to each number, the variance of the numbers
so obtained is

a) 3.87 b) 8.25 c) 2.87 d) 6.5


8. Variance of the data 2, 4, 5, 6, 8, 17 is 23.33. Then variance of 4, 8, 10, 12, 16, 34 will be:
(A) 23.23 (B) 25.33 (C) 46.66 (D) 48.66

9. Coefficient of variation of two distributions are 50 and 60, and their arithmetic means are 30 and 25
respectively. Difference of their standard deviation is:
(A) 0 (B) 1 (C) 1.5 (D) 2.5

10. Let a, b, c, d, e be the observations with mean m and standard deviation s. The standard deviation of the
observations a + k, b + k, c + k, d + k, e + k is:
(A) s (B) k s (C) s + k (D) s/k

Assertion Reasoning
DIRECTION : In each of the following questions, a statement of Assertion is given followed by a

corresponding statement of Reason just below it. Of the statements, mark the correct answer as

(a) Both assertion and reason are true and reason is the correct explanation of assertion.

(b) Both assertion and reason are true but reason is not the correct explanation of assertion.

(c) Assertion is true but reason is false.

(d) Assertion is false but reason is true

11.Assertion(A) : In order to find the dispersion of values of x from the mean 𝑥, we take absolute measure of
dispersion .

Reason( R ) :Sum of the deviations from mean is zero.

12. Assertion(A) : The mean deviation about mean for the data 4, 7, 8, 9, 10, 12, 13, 17 is 3.

Reason( R ) : The mean deviation about mean for the data38, 70, 48, 40, 42, 55, 63, 46, 54, 44 is 8.5.

13. Assertion(A) :Consider the following data .

𝑥𝑖 5 10 15 20 25

𝑓𝑖 7 4 6 3 5

Then the mean deviation about the mean is 6.32 .

248
Reason( R ) : Consider the following data .

𝑥𝑖 10 30 50 70 90

𝑓𝑖 4 24 28 16 8

Then the mean deviation about the mean is 15.

14.Assertion(A): The mean deviation about median calculated for series, where variability is very high, cannot
be fully relied.

Reason(R): The median is not a representative of central tendency for the series where degree of variability is
very high.

15. Assertion(A): The average marks of boys in a class is 52 and that of girls is 42. The average marks of boys
and girls combined is 50. The percentage of boys in the class is 80%.

Reason(R): Mean marks scored by the students of a class is 53. The mean marks of the girls is 55 and the mean
marks of the boys is 50. The percentage of girls in the class is 64%.

SHORT ANSWER QUESTIONS


16. The annual incidence rates of Huntington’s Disease per 100,000 individuals were recorded over a span of
five years. The data is as follows: 4, 7, 8, 9, 10. Calculate the mean deviation about the mean for these rare
disease rates

17. A group of athletes participated in a practice session for a particular exercise routine over the course of
several days. The recorded practice times (in minutes) for each athlete were as follows: 38, 70, 48, 40, 42, 55,
63, 46, 54, 44.

Calculate the mean deviation about the mean for the athletes' practice times.

18. A group of cars went through a series of servicing sessions at a garage. The recorded mileage before each
servicing session (in thousands of kilometres) for each car were as follows: 36, 72, 46, 42, 60, 45, 53, 46, 51,
49.

249
Calculate the mean deviation about the median for the recorded mileages of the cars before their servicing
sessions.
19. The mean and standard deviation of 20 observation is found to be 10 and 2, respectively. On rechecking, it
was found that an observation 8 was incorrect. Calculate the correct mean and standard deviation in case of
the wrong item is omitted.
20. Find the mean and standard deviation of first n terms of an A.P. whose first term is a and the common
difference is d.
21. Calculate the mean deviation about median for the daily wages of 12 labours getting wages (in Rs.) 48, 45,
60, 50, 46, 48, 50, 45, 70, 65,47,50
22. The mean life of a sample of 60 bulbs was 650 hours and the standard deviation was 8 hours. A second
sample of 80 bulbs has a mean life of 660 hours and standard deviation 7 hours. Find the overall standard
deviation.
23. Mean and standard deviation of 100 items are 50 and 4, respectively. Find the sum of all the item and the
sum of the squares of the items.
24. The mean and standard deviation of a group of 100 observations were found to be 20 and 3, respectively.
Later on it was found that three observations were incorrect, which were recorded as 21, 21 and 18. Find the
mean and standard deviation if the incorrect observations are omitted.
25. If 𝑎 is a positive integer and the frequency distribution has a variance of 160 . Determine the value of .

𝑥 𝑎 2𝑎 3𝑎 4𝑎 5𝑎 6𝑎
𝑓 2 1 1 1 1 1

CASE BASED QUESTIONS


26. A set of data points were collected and analysed. For this distribution, two pieces of information were
obtained: (x – 5) = 3 and (x – 5)2 = 43. It is also known that the total number of items in the dataset is 18.

a) Calculate the mean.

b) Calculate standard deviation for this distribution.

c) Calculate Variance.

250
27.The weights of coffee in 70 jars is shown in the following table:

Weight Range (grams) Frequency Weight Range (grams) frequency

fre200 – 201 13 203 - 204 10

201 – 202 27 204 - 205 1

202 – 203 18 205 - 206 1

a) Determine the mean.


b) Determine the standard deviation.
c) Find variance.

28.An analysis of monthly wages paid to workers in two firms A and B, belonging to the same industry, gives
the following results:

Firm A Firm B

Number of workers 897 468

Mean amount of wages(INR) 6345 6345

Variance of distribution of wages 100 169

a) What is the standard deviation of Firm A?

251
b) What is the standard deviation of Firm B?
c) Which firm, A or B, shows greater variability in individual wages?
29.A meteorological study collected temperature data in degrees Celsius (°C) to analyse variations. The
standard deviation of the collected data was found to be 5 °C.

a) If the temperature data were converted into degrees Fahrenheit (ºF), what would be the variance of the
data?

(A) 81 B) 57 C) 36 D) 25

30. A collection of 100 items was analysed, and the statistical properties of the data were observed. The mean
of the items is 50, and the standard deviation is 4.

a) Calculate the sum of all the items.

b) Calculate the sum of the squares of the items.

c) What is the variance?

To solve this, we can use the formulas for the mean and standard deviation:
Sum of all items = Mean × Number of items = 50 × 100 = 5000.
Sum of squares of items = Variance × (Number of items - 1) + Mean2 × Number of items = (42) × (100 - 1) +
502 × 100 = 159600.
So, the sum of all the items is 5000, and the sum of the squares of the items is 159600.

LONG ANSWER TYPE

252
31 Find the mean , variance and standard deviation using short-cut method.

Height No.of
(in cm) children

70 – 75 3

75 – 80 4

80 – 85 7

85 – 90 7

90 – 95 15

95 – 100 9

100 – 105 6

105 – 110 6

110 – 115 3

32 The diameters of circles (in mm) drawn in a design are given below. Calculate the standard
deviation and mean diameter of the circles.
Diameters 33 – 36 37 – 40 41 – 44 45 – 48 49 – 52

No.of 15 17 21 22 25
circles

33

A student collected 10 readings and attempted to calculate the mean and variance.
Unfortunately, the student mistakenly used a reading of 52 instead of the correct reading 25.
The student calculated the mean and variance as 45 and 16 respectively.

Determine the correct mean and variance by considering the mistaken reading of 52 instead of
the actual reading 25.

34 The mean and standard deviation of 6 observations are 8 and 4 respectively. If each observation
is multiplied by 3, then:

(a) Find the new mean.

(b) Find the new standard deviation of the resulting observations.

253
35 Mean and standard deviation of 100 observations were found to be 40 and 10, respectively. If at
the time of calculation two observations were wrongly taken as 30 and 70 in place of 3 and 27
respectively, find the correct standard deviation.

KEY FOR SELF PRACTICE QUESTIONS


1 B 11 a 21 5.5 31 93, 105.58 and 10.27

2 C 12 c 22 8.9 32 43.5, 30.84, 5.55

3 A 13 c 23 251600 33 42.3 and 43.81

4 B 14 a 24 3.036 34 24 and 12

5 A 15 c 25 7 35 10.241

6 A 16 1.64 26 3,3,9

7 B 17 8.4 27 201.9,1.166
,1.08

8 C 18 10.5 28 10,13,B

9 A 19 1.997 29 a

10 A 20 30 5000,15960
𝑛2 − 1
𝜎 = 𝑑√ 0.
12
971

254
PROBABILITY
CONCEPTS:

* Random Experiments: An experiment is called random experiment if it satisfies the following two
conditions:

(i) It has more than one possible outcome.

(ii) It is not possible to predict the outcome in advance.

** Outcomes and sample space: A possible result of a random experiment is called its outcome. The set of all
possible outcomes of a random experiment is called the sample space associated with the experiment. Each
element of the sample space is called a sample point. Any subset E of a sample space S is called an event.

** Impossible and Sure Events: The empty set φ and the sample space S describe events. φ is called an
impossible event and S, i.e., the whole sample space is called the sure event.

** Compound Event: If an event has more than one sample point, it is called a Compound event.

** Complementary Event: For every event A, there corresponds another event A′ called the complementary
event to A. It is also called the event ‘not A’.

** The Event ‘A or B’: When the sets A and B are two events associated with a sample space, then ‘A ∪ B’ is
the event ‘either A or B or both’. This event ‘A ∪ B’ is also called ‘A or B’.

** The Event ‘A and B’: If A and B are two events, then the set A ∩ B denotes the event ‘A and B’.

** The Event ‘A but not B’: the set A – B denotes the event ‘A but not B’.

A – B = A ∩ B´

** Mutually exclusive events: two events A and B are called mutually exclusive events if the occurrence of
any one of them excludes the occurrence of the other event, i.e., if they cannot occur simultaneously. In this
case the sets A and B are disjoint i.e. A ∩ B = φ.

Exhaustive evnts: if 𝐸1 , 𝐸2 , 𝐸3 … 𝐸𝑛 are n events of a sample space S and if 𝐸1 ∪ 𝐸2 ∪ 𝐸3 ∪ … ∪ 𝐸𝑛 = 𝑆, then


𝐸1 , 𝐸2 , 𝐸3 … 𝐸𝑛 are called exhaustive events.

255
Mutually exclusive and exhaustive events: If 𝐸𝐼 ∩ 𝐸𝐽 = ∅ for 𝑖 ≠ 𝑗 and

𝐸1 ∪ 𝐸2 ∪ 𝐸3 ∪ … ∪ 𝐸𝑛 = 𝑆, then 𝐸1 , 𝐸2 , 𝐸3 … 𝐸𝑛 are called Mutually exclusive and exhaustive events

** Axiomatic Approach to Probability: Let S be the sample space of a random experiment. The probability P is
a real valued function whose domain is the power set of S and range is the interval [0,1] satisfying the
following axioms

(i) For any event E, P (E) ≥ 0


(ii) P (S) = 1
(iii) If E and F are mutually exclusive events, then P(E ∪ F) = P(E) + P(F).

From the axiomatic definition of probability, it follows that

(i)0 ≤ P (ωi) ≤ 1 for each ωi ∈ S

(ii) P (ω1) + P (ω2) + ... + P (ωn) = 1

(iii) For any event A, P(A) = Σ P(ωi ), ωi ∈ A.

** Equally likely outcomes: All outcomes with equal probability.

** Probability of an event: For a finite sample space with equally likely outcomes

Probability of an event P(A) =n (A)/ n(S)

where n(A) = number of elements in the set A,

n(S) = number of elements in the set S.

** Probability of the event ‘A or B’ :

P(A or B) = P(A ∪ B) = P(A) + P(B) – P(A∩B).

For mutually exclusive events A and B, we have P(A ∪ B)= P(A) +P(B)

** Probability of event ‘not A’ = P( A′ ) = P(not A) = 1 – P(A).

Example 1: What is the total number of sample spaces when a die is thrown 2 times?

A. 6 B. 12 C. 18 D. 36

Solution: The possible outcomes when a die is thrown are 1, 2, 3, 4, 5, and 6.


256
Given, a die is thrown two times.

Then, the total number of sample spaces = (6 × 6)=36

Example2: What is the total number of sample spaces when a coin is tossed and a die is thrown?

A. 6 B. 12 C. 8 D.16

Solution: The possible outcomes when a coin is tossed are Head (H) or Tail (T).

The possible outcomes when a die is thrown are 1, 2, 3, 4, 5, and 6.

Then, total number of space = (2 × 6) = 12

Example3: Three identical dice are rolled. What is the probability that the same number will appear on each
of them?

A. 1/6 B. 1/36 C. 1/18 D. 3/28

Solution:

Total number of cases = 6³ = 216

The same number can appear on each of the dice in the following ways:

(1, 1, 1), (2, 2, 2), ………….(3, 3, 3)

So, favourable number of cases = 6

Hence, required probability = 6/216 = 1/36

Example4: A bag contains 5 brown and 4 white socks. Ram pulls out two socks. What is the probability that
both the socks are of the same colour?

A. 9/20 B. 2/9 C. 3/20 D. 4/9

Solution: Total number of socks = 5 + 4 = 9

Two socks are pulled.


257
Now, P(Both are same colour) = (5C2 + 4C2)/9C2

= {(5×4)/(2×1) + (4×3)/(2×1)}/{(9×8)/(2×1)}

= {(5×4) + (4×3)/}/{(9×8)

= (5 + 3)/(9×2)

= 8/18

= 4/9

Example5:What is the probability of getting the number 6 at least once in a regular die if it can roll it 6 times?

A. 1 – (5/6)6 B. 1 – (1/6)6 C. (5/6)6 D. (1/6)6

Solution:

Let A be the event that 6 does not occur at all.

Now, the probability of at least one 6 occurs = 1 – P(A.)

= 1 – (5/6)6

Example6: Events A and B are said to be mutually exclusive if:

A.. P (A U B) = P(A) + P(B) B. P (A ∩ B) = P(A) × P(B) C. P(A U B) = 0

D. None of these

Solution: If A and B are mutually exclusive events,

Then P(A ∩ B) = 0

Now, by the addition theorem,

P(A U B) = P(A) + P(B) – P(A ∩ B)

⇒ P(A U B) = P(A) + P(B)

258
Example7:A die is rolled. What is the probability that an even number is obtained?

A. ½ B. 2/3 C. ¼ D. 3/4

Solution: When a die is rolled,

total number of outcomes = 6 (1, 2, 3, 4, 5, 6)

Total even number = 3 (2, 4, 6)

So, the probability that an even number is obtained = 3/6 = ½

Example8:What is the probability of selecting a vowel in the word “PROBABILITY”?

A. 2/11 B. 3/11 C. 4/11 D. 5/11

Solution:

In the word "PROBABILITY"

Total no. of letters =11

Number of vowels =4

We can select any of the 4 vowels at random

So, Favourable outcomes =4

∴ Selecting a vowel at random =Favourable outcomes /Total outcomes=4/11

Example9:An urn contains 6 balls of which two are red and four are black. Two balls are drawn at random.
What is the probability that they are of different colours?

A.⅖ B.1/15 C.8/15 D.4/15

Solution: Given that, the total number of balls = 6 balls

Let A and B be the red and black balls, respectively,

259
The probability that two balls are drawn are different = P (the first ball drawn is red)(the second ball
drawn is black)+ P (the first ball drawn is black)P(the second ball drawn is red)

= (2/6)(4/5) + (4/6)(2/5)

=(8/30)+ (8/30)

= 16/30

= 8/15

Example10:20 cards are numbered from 1 to 20. If one card is drawn at random, what is the probability that
the number on the card is a prime number?

A. ⅕ B. ⅖ C. ⅗ D. 5

Solution: Let E be the event of getting a prime number.

E = {2, 3, 5, 7, 11, 13, 17, 19}

Hence, P(E) = 8/20 = 2/5

Assertion –Reason based questions:

In the following questions a statement of assertion (A) is followed by a statement of Reason (R). Choose the
correct answer out of the following choices

a). Both A and R are true and R is correct explanation of A.

b). Both A and R are true but R is not the correct explanation of A.

c). A is true but R is false.

d). A is false but R is true

Example1: Assertion : Two dice are thrown simultaneously. There are 11 possible outcomes and each of them
has a probability 1/11.
Reason : Probability of an event (E) is defined as

P(E)=Number of favourable outcomes /Total number of possible outcomes


260
Solution: The correct option is D.

Assertion is false but reason is true.

If two dice will be thrown simultaneously then the total number of possible outcomes is 36 and probability of
each outcome is not equal to 1/11.
Probability of an event (E) is defined as

P(E)=Number of outcomes favourable to E /Total number of possible outcomes.


For example, probability of getting an even number on a die when a die is thrown
=Number of outcomes favourable to E /Total number of possible outcomes
=3/6
[∵ Here, the number of favourable outcomes will be 3 as 2, 4 and 6 are the possible even numbers]
⇒Assertion: False
⇒Reason : True

Example2: Assertion: The probability of a sure event is 1.


Reason: Let E be an event. Then, 0≤𝑃(𝐸)≤1.

Solution:(b) Both Assertion and Reason are true and Reason is not a correct explanation of Assertion.

Explanation:

Clearly, Assertion (A) and Reason ( R) are both true. But, Reason (R) is not a correct explanation of Assertion
(A).
Hence, the correct answer is (b).

Example3: Assertion: In rolling a dice, event A = {1, 3, 5} and event B = {2, 4} are mutually exclusive events.
Reason: In a sample space, two events are mutually exclusive if they do not occur at the same time.

Solution: The correct option is A

Both assertion and reason are true and reason is the correct explanation of assertion.

A={1,3,5} and B={2,4}


Here, A∩B=Φ
∴ A and B are mutually exclusive events.

261
Assertion: True
Reason: True and is the correct explanation of assertion.

Example4 Assertion: Two events A and B are such that P(A)=1/3 and P(B)=2/3 then P(A∪B)≥2/3
Reason : If P(B)>P(A), then P(A∪B)≥P(B)

Solution: the correct option is A Both assertion and reason are true and reason is the correct explanation of
assertion.
We know that, P(B) = P(A), then,
P(B)≤P(A∪B)≤1 and 0≤P(A∩B)≤P(A)
Now P(B)=2/3,P(A)=1/3
∴P(B)>P(A)⇒P(A∪B)≥P(B)
⇒P(A∪B)≥2/3
So, assertion and reason both are true and reason is the correct explanation of assertion.

Example5: Assertion(A) : Two coins are tossed simultaneously. The probability of getting two heads, if it is
1
known that at least one head comes up is 3

𝐸 𝑃(𝐸∩𝐹)
Reason ( R ): let E and F be two events with a random experiment, then 𝑃 (𝐹 ) = 𝑃(𝐹)

Solution: Both A and R are true and R is correct explanation of A

Short Answer Type Questions:

Example 1: If P(A) = ⅗. Find P (not A)

Solution: Given that: P(A) = ⅗

P(not A) = 1 – P(A)

P (not A) = 1- ⅗

= (5-3)/5

=⅖

Therefore, P(not A) = ⅖.

262
Example2: An urn contains 6 balls of which two are red and four are black. Two balls

are drawn at random. The probability that they are of different colours is

(i) ⅖ (ii) 1/15 (iii) 8/15 (iv) 4/15

Solution: Given that, the total number of balls = 6 balls

Let A and B be the red and black balls respectively,

The probability that two balls drawn, are different = P(the first ball drawn is red)(the second ball drawn is
black)+ P(the first ball drawn is black)P(the second ball drawn is red)

= (2/6)(4/5) + (4/6)(2/5)

=(8/30)+ (8/30)

= 16/30

= 8/15

Example3: One die of red colour, one of white colour and one of blue colour are placed in a bag. One die is
selected at random and rolled, its colour and the number on its uppermost face is noted. Describe the sample
space.

Solution: Let us assume that 1, 2, 3, 4, 5 and 6 are the possible numbers that come when the die is thrown.

And also, assume die of red colour be ‘R’, die of white colour be ‘W’, die of blue colour be ‘B’.

So, the total number of sample space = (6 × 3) = 18

The sample space of the event is

S={(R,1),(R,2),(R,3),(R,4),(R,5),(R,6),(W,1),(W,2),(W,3),(W,4),(W,5),(W,6) (B,1),(B,2),(B,3),(B,4),(B,5),(B,6)}

Example 4: Three coins are tossed once. Find the probability at most two heads.

Solution: S={ HHH,HHT,HTH,HTT,THH,THT,TTH,TTT}

263
E={HHT,THH,HTH,HTT,THT,TTH,TTT}

P(E)=7/8

Example 5: Suppose 3 bulbs are selected at random from a lot. Each bulb is tested and classified as defective
(D) or non-defective (N). Write the sample space of this experiment.

Solution: From the question,

‘D’ denotes the event that the bulb is defective, and ‘N’ denotes the event of non-defective bulbs.

Then, Total number of Sample space = 2 × 2 × 2 = 8

Thus, Sample space S = {DDD, DDN, DND, NDD, DNN, NDN, NND, NNN}

Example 6: A die is rolled. Let, E be the event “die shows 4” and F be the event “die shows even number”. Are
E and F mutually exclusive?

Solution:1,2,3,4,5and 6 are the possible outcomes when a die is thrown. So,

S = {1, 2, 3, 4, 5, 6}

As per the conditions given the question

E be the event “die shows 4”

E = {4}

F be the event “die shows even number”

F = {2, 4, 6}

E ∩ F = {4}

∴ E ∩ F≠φ …[because there is a common element in E and F]

Therefore, E and F are not mutually exclusive event

264
Example 7: Three letters are dictated to three persons and an envelope is addressed to each of them, the
letters are inserted into the envelopes at random so that each envelope contains exactly one letter. Find the
probability that at least one letter is in its proper envelope.

Solution: Total number of ways by which three letters can be put into three envelopes=

3! = 6

1 1 1 1
Derangement of n objects = 𝑛! [1 − 1! + 2! − 3! + ⋯ + (−1)𝑛 𝑛!]

1 1 1 1 1
Derangement of 3 objects=3! [1 − + − ]=6 (1 − 1 + − ) = 2
1! 2! 3! 2 6

2 1
P( no letter is in the correct envelope)= 6=3

1 2
P( atleast one is in the correct envelope)= 1 − 3 = 3

Example 8:If 4-digit numbers greater than 5,000 are randomly formed from the digits 0, 1, 3, 5 and 7, what is
the probability of forming a number divisible by 5 when the digits are repeated?

Solution:Since 4 digit numbers greater than 5000 are formed. The thousand’s place digit is either 7 or 5.

The total number of 4 digits number greater than 5000 = 2 × 5 × 5 × 5 − 1 = 249

A number is divisible by 5, if the digit at it’s unit place is either 0 or 5.

∴ The total number of 4 − digits numbers greater than 5000 and divsible by 5 = 2 × 5 × 5 × 2 − 1 = 99

∴ P(forming a number which is greater than 5000 and divisible by 5) = 99/249 = 33/83

Example 9: If 4-digit numbers greater than 5,000 are randomly formed from the digits 0, 1, 3, 5 and 7, what is
the probability of forming a number divisible by 5 when the repetitions of digits are not allowed?

Solution: Since 4-digit numbers greater than 5000 are formed. The thousand’s place digit is either 7 or 5
The total number of 4-digit number greater than 5000=2X4X3X2=48
A number is divisible by 5, if the digit at its unit place is either 0 or 5
The total number of 4-digits numbers starting with 5 and divisible by 5=1X3X2X1=6

265
The total number of 4-digits numbers starting with 7 and divisible by 5=1X3X2X2=12
The total number of 4-digits numbers greater than 5000 and divisible by 5=6+12=18
18 3
P(forming a number which is greater than 5000 and divisible by 5)= 48 = 8

Example 10: Find the probability that win a hand of 7 cards is drawn from a well shuffled deck of 52 cards, it
contains I. All kings. II. 3 kings. III. At least 3 kings

Solution: The total number of possible hands =C(52,7)


i. Number of hands with 4 kings=C(4,4) XC(48,3)
𝐶(4,4)×𝐶(48,3) 1
P(a hand will have 4 king)= = 7735
𝐶(52,7)

ii. Number of hands with 3 kings and 4 non-kings= C(4,3)XC(48,4)


𝐶(4,3)×𝐶(48,4) 9
P( a hand with 3 kings) = = 1547
𝐶(52,7)
9 1 46
iii. P( atleast 3 king)=P( 3 kings or 4 kings)=P( 3 kings)+ P( 4 kings)= + 7735 = 7735
1547

Case Study based question:

Example 1: Rahul went to a fair. There he saw in a shop a lottery seller was selling lotteries. He asked the
shopkeeper about this lottery game and he got the information that among these 10000 tickets, there are 10
prizes will be awarded. He is willing to know

I.What is the probability of not getting a prize, if he buys one ticket?

II.What is the probability of not getting a prize if he buys 2 tickets?

III. What is the probability of not getting a prize if he buys 10 tickets?

Solution:

i. out of 10000 tickets, one ticket can be chosen in C(1000,1)=10000 ways

266
there are 9990 tickets not containing a prize. Out of these 9990 tickets one can be chosen in
C(9990,1)=9990 ways
9990 999
P( not getting prize) = 10000 = 1000

ii. out of 10000 tickets, two tickets can be chosen in C(10000,2) ways
as there are 9990 tickets not containing a prize. Out of these 9990 tickets two can be chosen in
C(9990,2) ways
𝐶(9990,2)
P(not getting prize) =𝐶(10000,2)

iii. out of 10000 tickets, ten tickets can be chosen in C(10000,10) ways
as there are 9990 tickets not containing a prize. Out of these 9990 tickets two can be chosen in
C(9990,10) ways
𝐶(9990,10)
P(not getting prize) =𝐶(10000,10)

Example 2: Shivnath went to a fair. In the fare he saw a shopkeeper was mixing tickets numbered 1 to 20
thoroughly and asking customers to take out a ticket randomly. He is willing to know

I.What is the probability that drawn ticket number is a multiple of 3?

II. What is the probability that drawn ticket number is a multiple of 7?

III. If drawn ticket number is a multiple of 3 or 7 then he wins. What is the probability of his winning?

Solution: Let S be the sample space associated with the given random experiment. A and B denotes the events
getting a ticket bearing a number which is a multiple of 3 and 7 respectively, then 𝑆 = {1,2,3, … 20} 𝐴 =
{3,6,9, … 18}, 𝐵 = {7,14}

6 3
i. P( ticket number is a multiple of 3)=20 = 10
2 1
ii. P( ticket number is a multiple of 7)=20 = 10

267
3 1 4 2
iii. P( winning)= 10 + 10 = 10 = 5

Example 3:In a relay race, there are 5 teams A, B, C, D and E.

I. What is the probability that A, B& C finish first, second and third respectively.

II. What is the probability that A, B & C are first three to finish (in any order).

Solution: If we consider the sample space consisting of all finishing orders in the first three places, we will
1
have 𝑃(5,3) = 60 sample points, each with probability of 60

i. A,B and C finish first ,second and third respectively. There is only one finishing order for this, i.e ABC
1
P( A,B and C finish first ,second and third respectively)= 60

ii. A, B and C are the first three finishers. There will be 3! = 6 arrangements for A, B and C
6 1
P( A, B and C are the first three to finish) = 60 = 10

Example 4:One card is drawn from a well-shuffled pack of 52 cards. What is the probability that a card will be

(i) a diamond (ii) Not an ace (iii) a black card (iv) not a diamond

Solution:(i) the probability that a card is a diamond

We know that there are 13 diamond cards in a deck. Therefore, the required probability is:

P( getting a diamond card) = 13/52 = ¼

(ii) the probability that a card is not an ace

268
We know that there are 4 ace cards in a deck.

Therefore, the required probability is:

P(not getting an ace card) = 1-( 4/52)

= 1- (1/13)

=(13-1)/13

= 12/13

(iii) the probability that a card is a black card

We know that there are26 black cards in a deck.

Therefore, the required probability is:

P(getting a black card) = 26/52 = ½

(iv) the probability that a card is not a diamond

We know that there are 13 diamond cards in a deck.

We know that the probability of getting a diamond card is 1/4

Therefore, the required probability is:

P( not getting a diamond card) = 1- (1/4)

= (4-1)/4

Example 5: Aditi runs a handicraft shop in Bapu bazar in Jaipur. She makes beautiful necklaces using colourful
beads which she keeps in potli. Today she prepared 19 necklaces but could not make the 20 th necklace as she
had no yellow beads left. She counted the beads and found that there were 8 red, 6 green and 14 blue beads

269
remaining in her potli. Her little daughter Dulari requested for a bead. Aditi decides to take out one bead from
her potli for Dulari.

A. Find the probability that she draws a green bead.

3 3 11 3
(I ) 11 ( ii ) 7 ( iii ) ( iv) 14
14

B. Find the probability that the bead drawn by her s not green

3 3 11 3
(I ) 11 ( ii ) 7 ( iii ) ( iv) 14
14

C. Find the probability that she draws either a green or a blue bead.

5 5 7 3
(I ) 7 ( ii ) 12 ( iii ) ( iv) 14
12

D, Find the probability that she draws neither a red nor a green bead.

3 1 3 1
(I ) 14 ( ii ) 3 ( iii ) ( iv) 2
7

Solution:(a) Total number of beads in the Potli = 8+6+14=28

Number of green beads in the Potli= 6

Required probability= 6/28=3/14

(b) Number of beads not green in the Potli= 28-6=22

Required probability= 22/28=11/14

(c) Number of blue and green beads in the Potli= 14+ 6=20

270
Required probability= 20/28=5/7

(d) Number of beads neither green nor red in the Potli =

Number of blue beads in the Potli=14

Required probability= 14/28=1/2

Long answer type question:

Example1:A couple has two children,

(i) Find the probability that both children are males if it is known that at least one of the children is male.

(ii) Find the probability that both children are females if it is known that the elder child is a female.

Solution: A couple has two children,

Let, the boy be denoted by b & girl be denoted by g

So, S = {(b, b) ,(b, g),(g, b), (g, g)}

To find probability that both children are males, if known that at least one of children is male

Let E : Both children are males


F : At least one child is male

To find P(E|F)

E : Both children are males

E = {(b, b)}

P(E) = 1/4

F : At least one child is male

F = {(b, g), (g, b), (b, b)}

P(F) = 3/4

271
E ∩ F = {(b, b)}

P(E) = 1/4

F : At least one child is male

F = {(b, g), (g, b), (b, b)}

P(F) = 3/4

E ∩ F = {(b, b)}

P(E ∩ F ) = 1/4

P(E|F) = (𝑃(𝐸 ∩ 𝐹))/(𝑃(𝐹))

= (1/4)/(3/4)

= 1/3

∴ Required Probability is 𝟏/𝟑

(ii) Find the probability that both children are females if it is known that the elder child is a female.

S = {(b, b) ,(b, g),(g, b), (g, g)}

To find the probability that both children are females, if from that the elder child is a female.

Let E : both children are females


F : elder child is a female

To find P(E|F)

E : both children are females

E = {(g, g)}

P(E) = 1/4

272
F : elder child is a female

F = {(g, b), (g, g)}

P(F) = 2/4=1/2

Also, E ∩ F = {(g, g)}

So, P(E ∩ F) = 1/4

P(E|F) = (𝑃(𝐸 ∩ 𝐹))/(𝑃(𝐹))

= (1/4)/(1/2)

Example2::An experiment involves rolling a pair of dice and recording the numbers that come up. Describe
the following events:

A: the sum is greater than 8.

B: 2 occurs on either die

C: the sum is at least 7 and a multiple of 3.

Which pairs of these events are mutually exclusive?

Solution:Given that a pair of dice rolled.

Sample space = S = {(1, 1), (1, 2), (1, 3), (1, 4), (1, 5), (1, 6), (2, 1), (2, 2), (2, 3), (2, 4), (2, 5), (2, 6) (3, 1), (3, 2),
(3, 3), (3, 4), (3, 5), (3, 6), (4, 1), (4, 2), (4, 3), (4, 4), (4, 5), (4, 6), (5, 1), (5, 2), (5, 3), (5, 4), (5, 5), (5, 6), (6, 1), (6,
2), (6, 3), (6, 4), (6, 5), (6, 6)}

n(S) = 36

Event A: The sum is greater than 8

A = {(3, 6), (4, 5), (4, 6), (5, 4), (5, 5), (5, 6), (6, 3), (6, 4), (6, 5), (6, 6)}

Event B: 2 occurs on either die

273
B = {(1, 2), (2, 1), (2, 2), (2, 3), (2, 4), (2, 5), (2, 6), (3, 2), (4, 2), (5, 2), (6, 2)}

Event C: The sum is at least 7 and a multiple of 3

C = {(3, 6), (4, 5), (5, 4), (6, 3), (6, 6)}

Here,

A∩B=Φ

B∩C=Φ

A∩C≠Φ

Therefore, the pair of events A, B and B, C are mutually exclusive.

Example3:A pack of 50 tickets is numbered from 1 to 50 and is shuffled. Two tickets are drawn at random.
Find the probability that (i) both the tickets drawn bear prime numbers (ii) Neither of the tickets drawn bear
prime numbers.

Solution:The total number of tickets = 50

Prime numbers from 1 to 50 are 2, 3, 5, 7, 11, 13, 17, 19, 23, 29, 31, 37, 41, 43, and 47.

The total number of prime numbers between 1 and 50 is 15.

(i) Probability that both tickets are drawn bears prime numbers:

P(Both tickets bearing prime numbers ) = 15C2 / 50 C2= 3/35

Hence, the probability that both tickets are drawn bear prime numbers is 3/35.

(ii) Probability that neither of the tickets drawn bears prime numbers:

P ( Neither of the tickets bearing prime numbers) = 35C2 / 50C2= 17/35.

Therefore, the probability that neither of the tickets drawn bears a prime number is 17/35.

Example4:20 cards are numbered from 1 to 20. If one card is drawn at random, what is the probability that
the number on the card is:

274
1. Prime number

2. Odd Number

3. A multiple of 5

4. Not divisible by 3

Solution:Let S be the sample space.

S = {1, 2, 3, 4, 5, 6, 7, 8, 9, 10, 11, 12, 13, 14, 15, 16, 17, 18, 19, 20}

(1) Probability that the card drawn is a prime number:

Let E1 be the event of getting a prime number.

E1 = {2, 3, 5, 7, 11, 13, 17, 19}

Hence, P(E1) = 8/20 = 2/5.

(2) Probability that the card drawn is an odd number:

Let E2 be the event of getting an odd number.

E2 = {1, 3, 5, 7, 9, 11, 13, 15, 17, 19}

Hence, P(E2) = 10/20 = 1/2.

(3) Probability that the card drawn is a multiple of 5

Let E3 be the event of getting a multiple of 5

E3 = {5, 10, 15, 20}

Hence, P(E3) = 4/20 = 1/5.

(4) Probability that the card drawn is not divisible by 3:

Let E4 be the event of getting a number that is not divisible by 3.

E4 = {1, 2, 4, 5, 7, 8, 10, 11, 13, 14, 16, 17, 19, 20}

275
Hence, P(E4) = 14/20 = 7/10.

Example5: Find the probability that when a hand of 7 cards are drawn from the well-shuffled deck of 52 cards,
it contains

(i) all kings and (ii) 3 kings

Solution:

(i) To find the probability that all the cards are kings:

If 7 cards are chosen from the pack of 52 cards

Then the total number of combinations possible is: 52C7

= 52!/[7! (52-7)!]

= 52!/ (7! 45!)

Assume that A be the event that all the kings are selected

We know that there are only 4 kings in the pack of 52 cards

Thus, if 7 cards are chosen, 4 kings are chosen out of 4, and 3 should be chosen from the 48 remaining cards.

Therefore, the total number of combinations is:

n(A) = 4C4 x48C3

= [4!/4!0! ] x [48!/3!(48-3)!]

= 1 x [48!/3! 45!]

= 48!/3! 45!

Therefore, P(A) = n(A)/n(S)

= [48!/3! 45!] ÷[52!/ (7! 45!]

276
= [48! x 7!] ÷ [3!x 52!]= 1/7735

Therefore, the probability of getting all the 7 cards are kings is 1/7735

(ii) To find the probability that 3 cards are kings:

Assume that B be the event that 3 kings are selected.

Thus, if 7 cards are chosen, 3 kings are chosen out of 4, and 4 cards should be chosen from the 48 remaining
cards.

Therefore, the total number of combinations is:

n(B) = 4C3 x48C4

= [4!/3!(4-3)! ] x [48!/4!(48-4)!]

= 4 x [48!/4! 44!]

= 48!/3! 45!

Therefore, P(B) = n(B)/n(S)

= [4 x48!/4! 44!] ÷[52!/ (7! 45!]

= 9/1547

Therefore, the probability of getting 3 kings is 9/1547

Practice Questions

Multiple choice questions:

Q.1 What is thetotal number of elements in sample spaces when a coin is tossed and a die is thrown?

a)12 b)10 c)11 d)13

Q.2 A bag contains 5 brown and 4 white socks. Ram pulls out two socks. What is the probability that both the
socks are of the same colour?

277
a)4/5 b)4/9 c)4/3 d)4/7

Q.3: What is the probability of selecting a vowel in the word “ZIET”?

a)1 b)2 c) 0.5 d) None of the above

Q.4 An urn contains 6 balls of which two are red and four are black. Two balls are drawn at random. What is
the probability that they are of different colours?

a)4/15 b)2/15 c)1/15 d)8/15

Q.5What is the total number ofsamplespaceswhenadieisthrown2 times?

a)6 b)12 c)18 d)36

Q.6Three identical dice are rolled. What is the probability that the same number will appear on each of them?

a)1/6 b)1/36 c)1/18 d)3/28

Q.7An urn contains 6 balls of which two are red and four are black. Two balls are drawn at random. What is
the probability that they are of different colour?

a)2/15 b)1/15 c)8/15 d)4/15

Q.8 If A and B are two mutually exclusive and exhaustive events,

then P(A) + P(B) =

a) 0 b) 0.5 c) 0.25 d)1

Q.9 If A, B and C are three mutually exclusive and exhaustive events of an

experiment such that 3P(A) = 2P(B) = P(C), then P(A) equals to:

a)2/11 b) 3/11 c) 4/11 d)6/11

Q.10 If P(A) = 0.2,P(B) = 0.3, and P(A ∩ B) = 0.1. Then P(A ∪ B)

equal to:

a)1/2 b) 2/5 c) 5/6 d)4/5

278
Assertion –Reason based questions:

In the following questions a statement of assertion (A) is followed by a statement of Reason (R). Choose the
correct answer out of the following choices

a). Both A and R are true and R is correct explanation of A.

b). Both A and R are true but R is not the correct explanation of A.

c). A is true but R is false.

d). A is false but R is true.

Q.1 Assertion (A): Probability of getting a head in a toss of an unbiased coin is 1/2.

Reason (R): In a simultaneous toss of two coins, the probability of getting ‘no tails’ is 1/4.

Q.2Assertion (A): In tossing a coin, the exhaustive number of cases is 2.

Reason (R): If a pair of dice is thrown, then the exhaustive number of cases is 6x6

Q.3Assertion: If a box contains 𝟓 white, 𝟐 red and 4 black marbles, then the probability drawing a white
marble from the box is 𝟓/𝟏𝟏.
Reason: 𝟎≤𝑷(𝑬)≤𝟏, where 𝐸 is any event.

Q.4Assertion : Two dice are thrown simultaneously. There are 11 possible outcomes and each of them has a
probability 1/11

Reason : Probability of an event E is defined as P(E) =Number of favourable outcomes/total number of


outcomes

Q.5 Assertion: If a die is thrown, the probability of getting a number less than 3 and greater than 2 is zero.

Reason: Probability of an impossible event is zero.

Short Answer Type Questions:

Q.1 A young man visits a hospital for medical check-up. The probability that he has lungs problem is 0.45,
heart problem is 0.29 and either lungs or heart problem is 0.47. What is the probability that he has both types
of problems?

279
lungs as well as heart? Out of 1000 persons how many are expected to have both types of problems?

Q.2The probability that a student will pass the final examination in both English and Hindi is 0.5 and the
probability of passing neither is 0.1. If the probability of passing the English examination is 0.75, what is the

probability of passing the Hindi examination?

Q.3What is the probability that a leap year selected at random will contain 53 Sundays?

Q.4A bag contains 5 green and 7 red balls. Two balls are drawn at random. What is the probability that one is
green and the other is red?

Q.5 4 cards are drawn from a well-shuffled deck 52 cards. What is the probability of obtaining 1 diamond and
3 spades?

Q.6 A card drawn from a pack of 52 cards.

(i) How many points are there in the sample space?

(ii) Calculate the probability that card is an ace of spade.

(iii) Calculate the probability that the card is

(a) an ace (b) a black card

Q.7Given P(A) = 3/5 and P(B) = 1/5, find P(A or B) if A and B are mutually exclusive events.

Q.8An experiment involves rolling a pair of dice and recording the number that comes up. Describe the
following events:

A : the sum is greater than 8,

B : 2 occurs on either die,

C : the sum is at least 7 and a multiple of 3; which

pairs of these events are mutually exclusive?

Q.9The probability that a student will pass the final examination in both English and Hindi is 0.5 and the
probability of passing neither is 0.1. If the probability of passing the English Examination is 0.75. What is the

probability of passing Hindi Examination?

280
Q.10A and B are two events such that P(A) = 0.54, P(B) = 0.69 and P(A ∩ B) = 0.35. Find

i. P(A ∪ B) II. P(A ′ ) III. P(A ′ ∩ B ′ ) IV. P(A ∩ B ′ )

i.22/25 ii.0.46 iii.3/25 iv. 19/100

Case Study based question:

Read the following passage and answer the questions given below

Q.1 In a class of 60 students, 30 opted for NCC, 32 opted for NSS and 24 opted for both NCC and NSS. One of
these students is selected at random.

I. Find the probability that the student opted for NCC or NSS

ii. Find the probability that the student has opted NSS but not NCC.

Q.2 One school decided to organize the sports day. In one of the events of relay

race students of class 11 participated.

In the relay race there are five teams A, B, C, D and E.

281
On the basis of above information answer the following:

(a) What is the probability that A, B and C finish first, second and

third respectively. Answer 1/60

(b) What is the probability that A, B and C are first three to finish (in

any order) 1/10

(Assume that all finishing orders are equally likely).

Q.3Two students Rohan and Soham appeared in an examination. The probability that Rohan will qualify the
examination is 0.05 and that Soham will qualify the examination is 0.10. The 97 probabilities that both will
qualify the examination is 0.02. Based on the above information, answer the following questions:

(i) Find the probability that both Rohan and Soham will not qualify the examination.

(ii) Find the probability that at least one of them will not qualify the examination.

(iii) only one of them will qualify the exam

Q.4 20 cards are numbered from 1 to 20. One card is drawn at random what is the prob. that the number on
the card drawn is (i) A prime Number (ii) An odd Number (iii) A multiple of 5 OR (iii) Not divisible by 3

Q.5 Out of 100 students, two sections of 40 and 60 are formed. If you and your friend are among the 100
students what is the probability that

(a) You both enter the same section

(b) You both enter the different section.

Long answer type question:

Q.1. A fair coin is tossed four times, and a person win ₹ 1 for each head and lose ₹ 1.50 for each tail that turns
up. From the sample space calculate how many different amounts of money you can have after four tosses
and the probability of having each of these amounts.

Q.2.In a town, there are 6000 people of which 1200 are over 50 years old and 2000 are females. It is said that
30% of females are over 50 years. Find the probability that an individual chosen randomly from the town is
either female or over 50 years.
282
Q.3. There are 20 cards that are numbered from 1 to 20. If a card is withdrawn randomly, then find the
probability that a number on the card will be: (i) Multiple of 4 (ii) Even number (iii) Not divided by 5 (iv) Prime
Number

Q.4. If an entrance exam that is graded based on two exams, the probability of chosen at random, students
clearing the 1st exam is 0.8 and the probability of passing the 2nd exam is 0.7. The probability of clearing at
least one of them is 0. 95. Find the probability of clearing both.

Q.5. A card has been drawn from a well-shuffled deck of 52 cards. What will be the probability that a card will
be an (i) Diamond (ii) Black card (iii) Not an ace (iv) Not a diamond

ANSWERS

Multiple choice questions:

1.(a) 2.(b) 3.(c) 4.(d) 5.(d) 6.(b) 7.(c) 8.(d) 9.(a) 10.(b)

Short Answer Type Questions:

1. 0.27, 270
2. 0.65
3. 2/7
4. 35/66
5. 286/20825
6. (i)52,(ii)1/52,(iii) a)1/13, b)1/2
7. 4/5
8. A & B, A &C
9. 0.65
10. (i)22/25(ii)0.46(iii)3/25(iv) 19/100

Assertion –Reason based questions:1.(a) 2.(b) 3.(b) 4.(d) 5.(a)

Case Study based question:1. i) 19/30 ii)4/30

2. i) 1/60 ii) 1/10

3. i) 0.87 ii)0.98 iii)0.11

283
4.i) 2/5 ii)1/2 iii)1/5 OR 7/10

5.i) 17/33 (ii) 16/33

LONG ANSWER TYPE QUESTION

1. 1/16

2. 13/20

3. (i) ¼ (ii) ½ (iii) 4/5 (iv) 2/5

4. 0.55

5.(i) ¼ (ii) ½ (iii) 12/13 (iv) 3/4

284
Class Xl – Mathematics
SAMPLE PAPER -I[BLUE PRINT]

2M 3M 5M 4M
1M Marks
CHAPTERS for each
unit

Sets 3 1

Relations & Functions 2 1 1 23M


Trigonometric Functions 4 1 1

Complex Numbers and 1


Quadratic Equations

Linear Inequalities 1

Permutations and 1 25M


Combinations 1

Binomial Theorem 1 1

Sequence and Series 1 1

Straight Lines 1 1

1 12M
Conic Sections 1

Introduction to Three- 1 1
dimensional Geometry

1 1 08M
Limits and Derivatives 2

Statistics 1

1 12M
Probability 3 1

Total Marks 20 M 10 M 18 M 20 M 12 M 80 M

285
SAMPLE PAPER -I

Maximum Marks: 80 Time Allowed: 3 hours

General Instructions:

1. This Question paper contains - five sections A, B, C, D and E. Each section is compulsory. However,
there are internal choices in some questions.
2. Section A has 18 MCQ’s and 02 Assertion-Reason based questions of 1 mark each.
3. Section B has 5 Very Short Answer (VSA)-type questions of 2 marks each.
4. Section C has 6 Short Answer (SA)-type questions of 3 marks each.
5. Section D has 4 Long Answer (LA)-type questions of 5 marks each.
6. Section E has 3 source based/case based/passage based/integrated units of assessment (4
marks each) with sub parts.

SECTION A

Q.1) Derivative of the function 𝑓(𝑥) = 𝑠𝑖𝑛2 𝑥 is

a) 𝑐𝑜𝑠 2 𝑥 b) 𝑐𝑜𝑠2𝑥 c) −𝑐𝑜𝑠2𝑥 d) 𝑠𝑖𝑛2𝑥

Q.2) S= {x:x is a positive multiple of 3 less than 100}


P= {x: x is a prime no. less than 20},
then n(s)+n(p) is
a) 34 b)41 c)33 d)30
Q.3) Two finite sets A and B are such that A ⊂B, then which of the following is not correct?
(a) A ∪ B = B (b) A ∩ B = A (c) A – B = ∅ (d) B – A = ∅
Q.4) Which of the following is not equal to (a) 𝑐𝑜𝑠 2 𝑥 − 𝑠𝑖𝑛2 𝑥
(b) 1 − 2𝑠𝑖𝑛2 𝑥
(c) 1 − 2𝑐𝑜𝑠 2 𝑥
1−𝑡𝑎𝑛2 𝑥
(d) 1+𝑡𝑎𝑛2 𝑥Q.5) . the locus of a point for which y = 0, z = 0?
a) equation of z-axis
b) equation of y-axis
c) equation of x-axis
d) All of these
𝑥 2 +2𝑥+1
Q.6) The domain of real function f(x) = 𝑥 2 −𝑥−6 is given by
a) 𝑅 − {3, −2} b)𝑅 − {−3,2} c)𝑅 − {−3, −2} d)𝑅 − (3, −2)
Q.7) Two dice are thrown simultaneously. The probability of obtaining total score of seven is
5 6 7 8
a) 36 b)36 c)36 d)36

Q.8) . If – 3x + 17 < – 13, then


(a) x ∈ (10, ∞)

286
(b) x ∈ [10, ∞)
(c) x ∈ (– ∞, 10]
(d) x ∈ [– 10, 10)

Q.9) If 𝑠𝑖𝑛 sin 𝜃 + 𝜃 = 2 then 𝑠𝑖𝑛2 𝜃 + 𝑐𝑜𝑠𝑒𝑐 2 𝜃 is equal to …


(a) 1 b) -1 c) 2 d) -2
Q.10) Range of real function f(x) = −|x| is
(a) (0,∞) (b) (-∞,0) (c) R (d) None of these
Q.11) Number of proper subset of a set A of order n is equal to:
(a) n (b) 2𝑛 − 1 (c) 2n (d) n2
3
Q.12) A line cutting of intercepts −3 on Y-axis and slope of the line is , then equation of line is
5
(a) 5y−3x+15=0 (b) 3y−5x+15=0 (c) 5y−3x−15=0 (d) None of these
Q.13. The value of sin (45° + θ) – cos (45° – θ) is
(a) 2 cosθ
(b) 2 sinθ
(c) 1
(d) 0

Q.14) Radius of circle x2 + y2− 4x−4y −5 = 0, is

(a) √13 (b) 7 (c) 4 (d) 5

Q.15) In a class of 60 students, 30 opted for NCC, 32 opted for NSS and 24 opted for both NCC and NSS. If one
of these students is selected at random, the probability of the student has opted NSS but not NCC
19 11 2
(a) 30 (b)30 (c)15 (d) None of these
𝑦 2 −4
Q.16) . What is the value of 𝑦−2
(a) 2
(b) 4
(c) 0
(d) cannot be evaluated

Q.17) One card is drawn from a pack of 52 cards. The probability that it is the card of a King or spade is

1 3 4 3
(a) (b) 26 (c) 13 (d)13
26

Q.18) Value of cosec (−14100) is

(a) 2 (b) 1 (c) −2 (d) −1


ASSERTION-REASON BASED QUESTIONS

In the following questions, a statement of assertion (A) is followed by a statement of Reason (R). Choose the
correct answer out of the following choices.
(a) Both A and R are true and R is the correct explanation of A.
287
(b) Both A and R are true but R is not the correct explanation of A.
(c) A is true but R is false.
(d) A is false but R is true.

Q19 If 𝑎, 𝑏 ∈ R and 𝑛 ∈N, then

(𝑎 + 𝑏)𝑛 =n C0 𝑎𝑛 + n C1 𝑎𝑛−1 𝑏+ n C2 𝑎𝑛−2 𝑏 2 +………. +n Cn𝑏 𝑛

Q.20) Assertion (A): If 16 and 9 be A.M and G.M respectively between two +ve numbers, then numbers are
16 ± 5√7

Reason ( R): If A and G A.M and G.M respectively between two +ve numbers, then numbers are
𝑨 ± √(𝑨 + 𝑮)(𝑨 − 𝑮)

SECTION B

1 3 1
Q.21) If 𝑓(𝑥) = 𝑥 − , prove that (𝑓(𝑥)) = 𝑓(𝑥 3 ) + 3𝑓 (𝑥)
𝑥

OR

Let A = {9,10,11,12,13} and let f: A→N be defined by f (n) = the highest prime factor of n. Find the
range of f.

Q.22) If 𝑥 = −1 + 𝑖 , then find the value of 𝑥 2 + 2𝑥 − 1

OR

Express the following in 𝑎 + 𝑖𝑏 form:

(𝑖 18 + 𝑖 −25 )3

Q.23) Evaluate the following Limit:

1−𝑐𝑜𝑠2𝑥
𝑥2

Q.24) Show that the points (–2, 3, 5), (1, 2, 3) and (7, 0, –1) are collinear.

Q.25) If A = {1,2,3,4,5,6} and B = {2,4,6,8} then find that (i) AUB (ii) A-B.

SECTION C

Q.26) Find the derivative of 𝑠𝑖𝑛𝑥 by first principle.

288
OR

Find derivative of following function w.r.t. x

3𝑥+4
𝑓(𝑥) =
5𝑥 2 −7𝑥+9

𝑥2
Q.27) Let 𝑓 = {(𝑥 , 1+𝑥2) : 𝑥 𝜖 𝑅} be a function from R into R. Determine the range of f.
OR
𝑥 2 +2𝑥+1
Find the domain of the function f(x) = 𝑥 2−8𝑥+12
Q.28) In how many ways can one select a cricket team of eleven from 17 players in which only 5 players can bowl if
each cricket team of 11 must include exactly 4 bowlers.
OR
In how many ways can the letters of the word “ASSASSINATION” be arranged so that all the S’s are together?

Q.29) How many litres of water will have to be added to 1125 litres of the 45% solution of acid so that the
resulting mixture will contain more than 25% but less than 30% acid content?

Q.30) If p and q are the lengths of perpendiculars from the origin to the lines 𝑥𝑐𝑜𝑠𝜃 − 𝑦𝑠𝑖𝑛𝜃 = 𝑘𝑐𝑜𝑠2𝜃
and 𝑥𝑠𝑒𝑐𝜃 + 𝑦𝑐𝑜𝑠𝑒𝑐𝜃 = 𝑘 , respectively, prove that p2 + 4q2 = k2.
𝜋
Q.31 Find the value of 𝑡𝑎𝑛 𝑡𝑎𝑛 8 .

SECTION D

Q.32) Find the mean and the standard deviation for the following data:

classes 30 − 40 40 − 50 50 − 60 60 − 70 70 − 80 80 − 90 90 − 100
Frequency 3 7 12 15 8 3 2

Q.33) Prove that

3 3𝜋 𝑥 𝑥 𝑥
If 𝑡𝑎𝑛 tan 𝑥 = 4 , 𝜋 ≤ 𝑥 ≤ then find the values of 2 ,𝑐𝑜𝑠 cos 2 ,𝑡𝑎𝑛 tan 2
2

Q.34) If S be the sum, P be the product and R be the sum of reciprocals of n terms of a G.P. Prove that:
𝑃2 𝑅 𝑛 = 𝑆 𝑛 .

OR

Sum of two numbers is six times their geometric mean, show that the numbers are in the ratio (3+2√2):
(3−2√2).

289
6
Q.35) Using binomial theorem, find(𝑥 + 1)6 + (𝑥 − 1)6 . Hence, by using it find the value of (√2 + 1) +
6
(√2 − 1) .

SECTION E

(CASE STUDY BASED QUESTIONS)

This section comprises of 3 case-study/passage-based question of 4 marks each. First case study problems
have 3 sub parts (i), (ii) and (iii) of marks 1, 1 and 2 respectively. Second case-study has four sub parts (i),
(ii), (iii), (iv) each carrying 1 mark. Third case-study problem has two sub parts (i) and (ii) of two marks each.

Q.36) During the Mathematics class, A teacher clears the concept of permutations and combinations to
the 11thclass students. After the class was over he asks the students some more questions.

On the basis of the information given above answer the following:-


(a) Find the number of arrangements of the letters of the word INDEPENDENCE.
(b) In How many of these do the words begin with I and end in P.
(c) In How many of these do all the vowels never occur together.
OR
In How many of these do all the four E’s do not occur together

Q.37) On her winter vacations, Ayesha visits four cities (Delhi, Mumbai, Goa and Bangalore ) in random
order.

On the basis of the information given above answer the following

290
(a) What is the probability that she visits Delhi before Goa and Goa before Mumbai?
OR
What is the probability that she visits Delhi First and Mumbai last?

(b) What is the probability that she visits Delhi just before Mumbai?

Q.38) A colonel running a race course observes that sum of the distances of two flag posts A and B from him is
always 10 m and distance between the flag posts is 8 m.

Now answer the following questions:

i. Find the value of b?


ii. Find the locus (equation) of the path traced out by colonel?

291
MARKING SCHEME
Class XI
Mathematics (Code – 041)

Q.NO Answer Marks


1. D 1

2. B 1

3. D 1

4. C 1

5. C 1

6. A 1

7. B 1

8. A 1

9. C 1

10. D 1

11. B 1

12. A 1

13. D 1

14. A 1

15. C 1

16. B 1

17. C 1

18. A 1

19. C 1

20. C 1

21. 1 1 1 ½
To write 𝑓 (𝑥) = 𝑥 − 𝑥, and 𝑓(𝑥 3 ) = 𝑥 3 – ,
𝑥3
3 1
and for proving (𝑓(𝑥)) = 𝑓(𝑥 3 ) + 3𝑓 (𝑥) 1½
OR

To write f = {(9,3), (10,5)(11,11)(12,3)(13,13)}, 1½


292
And to write range of f = {3,5,11,13} ½
22. Simplifying and getting answer as −3 2
OR
3
Getting −(1 + 𝑖)
And to get the correct solution as 2-2i. 1
1
23. 2𝑠𝑖𝑛2 𝑥 1
𝑥2
𝑠𝑖𝑛𝑥 𝑠𝑖𝑛𝑥 ½
. )
𝑥 𝑥
2.1.1=2 ½
24. Find PQ=√14, 1
QR = 2√14,
PR = 3√14
PQ+QR = PR 1
Hence, points P,Q,R are collinear
25. 𝐴 = {0, −1,1} 𝑎𝑛𝑑 𝐵 = {−2, −1,1} 1
(i) 𝐴 ∪ 𝐵 = {0, −1,1, −2} ½
(ii) 𝐴 ∩ 𝐵 = {−1,1} ½

26. 𝑠𝑖𝑛(𝑥+ℎ)−sin(𝑥) 1
Let f(x) = 𝑠𝑖𝑛 𝑠𝑖𝑛𝑥=> f`(x) = (by using first principle)

2𝑥+ℎ ℎ
2𝑐𝑜𝑠( ).sin( )
= 2 2 ½


2𝑥+ℎ sin( )
2
= 𝑐𝑜𝑠 ( )X ℎ
2
2
1
= 𝑐𝑜𝑠(𝑥) X 1 = 𝑐𝑜𝑠(𝑥)
OR
½
For applying quotient rule of derivative correctly.
And for getting the correct answer


27.
1

OR 1

293
3

28. Total number of players =17 1½


It is given that only 5 players can bowl,
∴ Number of bowlers =5
∴ Number of other players =17-5 =12 ½
1
A team of 11 players can consist of exactly 4 bowlers and 7 other players.
Exactly 4 bowlers can be selected out of 5 bowlers 5𝑐4 ways, 7 other
players can be selected out of 12 other players in 12𝑐7 .
1
Hence , a team of 11 players consisting if exactly 4 bowlers and 7 other
players can be selected out of 5 bowlers and 12 other players in
5𝑐4 × 12𝑐7 ways
=5𝑐1 × 12𝑐5 ways=3960 ways.
OR
Find total no of letters taking all S together
Use correct formula 1
Calculate correct answer = 151200 1
1

29. Let x litres of water can be added to 1125 litres of 45% of boric acid
solution, then A.P.Q.,
25% of (x+1125) < 45% of 1125 < 30% of (x+1125). 1½
On solving the above inequations and to get the answer as
More than 562.5 litres but less than 900 litres. 1½
30. 31. 𝑝 = length of perpendicular from origin to the line x cosθ -y sin θ =
k cos2θ 1
|0−0−𝑘𝑐𝑜𝑠𝑐𝑜𝑠2𝜃|
𝑝= = |𝑘 𝑐𝑜𝑠 𝑐𝑜𝑠2𝜃|
1

𝑞 =length of perpendicular from origin to the line x secθ + y


cosecθ= k 1
=length of perpendicular from origin to the line 𝑥 𝑠𝑖𝑛 𝑠𝑖𝑛𝜃 + 𝑦
𝑐𝑜𝑠 𝑐𝑜𝑠𝜃 = 𝑘 𝑠𝑖𝑛 sin 𝜃 𝑐𝑜𝑠 cos 𝜃
𝑘
|0−0− 𝑠𝑖𝑛𝑠𝑖𝑛2𝜃|
2 𝑘 1
𝑞= = |2 𝑠𝑖𝑛 𝑠𝑖𝑛2𝜃|
1
so 𝑝2 + 4𝑞 2 = 𝑘 2 (1) = 𝑘 2

294
31. (𝜃)
𝑡𝑎𝑛 tan(2𝜃) = 1− 𝜃
𝜋
𝜋 ( ) 𝜋
8
tan ( 4 ) = 𝜋 ( 𝜃 = 8)
1−
8
2𝑥 𝜋
1 = 1−𝑥 2 , (x = 𝑡𝑎𝑛 tan ( 8 ))
2
So, x + 2x – 1 = 0
−2±√4+4 −2±2√2
So, x = =
2 2
𝜋
So, x =−1 ± √2 but 8 ∈ I quadrant
𝜋
So, x = 𝑡𝑎𝑛 tan ( 8 ) = √2 − 1

32..
CLASS FREQUEN MID- f i . xi ( xi - x̄)2 fi( xi - x̄)2
CY (fi) POINT(xi
)

30-40 3 35 105 729 2187

40-50 7 45 315 289 2023

50-60 12 55 660 49 588

60-70 15 65 975 9 135


.
70-80 8 75 600 169 1352

80-90 3 85 255 529 1587

90-100 2 95 190 1089 2178

TOTAL 50 3100 10050

∑ 𝑓𝑖 𝑥𝑖 3100
MEAN = √ = = 62
∑ 𝑓𝑖 50
2
𝑓𝑖 (𝑥𝑖− 𝑥)2
= √10050

S.D.= √ = 14.18
∑ 𝑓𝑖 50 3

295
1 4 3𝜋
33. cos 𝑥 = − √1+𝑡𝑎𝑛2 = − 5 𝑎𝑠 𝜋 < 𝑥 <
𝑥 2
1
3𝜋 𝜋 𝑥 3𝜋
Again 𝜋 < 𝑥 < ⇒ <2< ⇒ cos 𝑥 < 0 𝑎𝑛𝑑 sin 𝑥 > 0
2 2 4
1
𝑥 1+ 𝑐𝑜𝑠 𝑥 1
cos 2 = −√ = − ,
2 √10

1
1 − 𝑐𝑜𝑠 𝑥 3
=√ = ,
2 √10

𝑥 1
𝑠𝑖𝑛 2 𝑥
tan =
𝑥 = −3
𝑐𝑜𝑠 2 2 1
34. Let the terms of G.P. are a, ar, ar2,… arn-1
𝑎(𝑟 𝑛 −1) 1
So, S = ………(1)
𝑟−1
𝑛(𝑛−1)
& P= a . ar.ar2.… arn-1 = an . r1+2+…(n-1) = an . 𝑟 2 …..(2) 1
1 1
1 1 1 1 (1− 𝑛 )
𝑎 𝑟
R=𝑎+ + 𝑎𝑟 2 + ⋯ 𝑎𝑟 𝑛−1 = 1 ……………(3)
𝑎𝑟 1−
𝑟
1
For reaching the answer as P2 . Rn = Sn
2
OR
Let the two numbers are a and b.
So, a + b = 6√𝑎𝑏 1

𝑎+𝑏 3 𝑎+𝑏+2√𝑎𝑏 3+1


= 1=>𝑎+𝑏−2√𝑎𝑏 = 3−1 (By C and D)
2√𝑎𝑏

2
(√𝑎+√𝑏) 2
⇨ 2 =1
(√𝑎−√𝑏) 2
(√𝑎+√𝑏) √2
⇨ =
(√𝑎−√𝑏) 1

√𝑎 (√2+1)
Again applying C & D to get =(
√𝑏 √2−1)
𝑎 3+2√2
Finally, for getting the answer as 𝑏 = 3−2√2
2

296
35. Using binomial theorem, the expressions (x + 1)6 and (x – 1)6 can be
expressed as

(x + 1)6 = 6C0 x6 + 6C1 x5 + 6C2 x4 + 6C3 x3 + 6C4 x2 + 6C5 x + 6C6


1
(x – 1)6 = 6C0 x6 – 6C1 x5 + 6C2 x4 – 6C3 x3 + 6C4 x2 – 6C5 x + 6C6

Now, (x + 1)6 – (x – 1)6 = 6C0 x6 + 6C1 x5 + 6C2 x4 + 6C3 x3 + 6C4 x2 + 6C5 x + 6C6 – 1
[6C0 x6 – 6C1 x5 + 6C2 x4 – 6C3 x3 + 6C4 x2 – 6C5 x + 6C6]

= 2 [6C0 x6 + 6C2 x4 + 6C4 x2 + 6C6]

= 2 [x6 + 15x4 + 15x2 + 1]

Now by substituting x = √2, we get


1
(√2 + 1)6 – (√2 – 1)6 = 2 [(√2)6 + 15(√2)4 + 15(√2)2 + 1]

= 2 (8 + 15 × 4 + 15 × 2 + 1)

= 2 (8 + 60 + 30 + 1)

= 2 (99) 1

= 198 1

36 1. In the word INDEPENDENCE there are 12 letters of which 3 are N’s,


4 are E’s and 2 are D’s
(i)
12!
Total number of arrangements = 3!4!2! = 1663200 1

10!
(ii) Number of words begin with I an end with P = 3!4!2! = 1
12600
(iii) Number of Words in which vowels never occur together 1
= Total number of arrangements – Number of
arrangements in which vowels occur together
8! 5!
= 1663200 –3!2! × 4!1!

= 1663200 − 16800 = 1646400 1


OR
Number of words in which all the four E’s do not occur
together

1
297
= Total number of arrangements – Number of
arrangements in which all the four E’s occur together

9! 1
= 1663200 − 3!2! = 1663200 − 30240 = 1632960

37 1. Here n(s)= 24 1
(i) 𝐸1 = {𝐷𝐵𝐺𝑀, 𝐷𝐺𝐵𝑀, 𝐷𝐺𝑀𝐵, 𝐵𝐷𝐺𝑀} 1
1
4 1
𝑃(𝐸1 ) = 24 = 6 1
OR
𝐸2 = {𝐷𝐵𝐺𝑀, 𝐷𝐺𝐵𝑀}

2 1
𝑃(𝐸2 ) = =
24 12

(ii) 𝐸3 = {𝐷𝑀𝐺𝐵, 𝐷𝑀𝐵𝐺, 𝐺𝐵𝐷𝑀, 𝐺𝐷𝑀𝐵, 𝐵𝐺𝐷𝑀, 𝐵𝐷𝑀𝐺}


6 1
𝑃(𝐸3 ) = =
24 4
1
38 (I) PA + PB = 2a = 10 (Hence, locus is ellipse) 2
=> a = 5cm
Now, 2c = 8 => c = 4cm
But c2 = a2 – b2
⇨ b2 = a2 – c2 = 25 -16 = 9
⇨ b = 3cm
(II) Equation of required locus is 2
𝑥2 𝑦2 𝑥2 𝑦2
+𝑏2 = 1 =>25 + 9 = 1
𝑎2

298
SAMPLE PAPER –II

BLUE PRINT
SNO UNIT/TOPIC SECTION‘A’ SECTION‘B’ SECTIO SECTIO SECTI TOTAL UNIT
N‘C’ N’D’ ON‘E’ WISE
(LA) (CBQ) MAR
KING

1M 1M 2M 3 5 4
M M M

1 SETS 2(2) 1(2) 1(3) 3(4) 19 M

2 RELATIONS AND 2(2) 1(5) 3(7)


FUNCTIONS
3 TRIGONOMETRIC 2(1) 1(2) 1(5) 3(8)
FUNCTIONS
4 COMPLEX NUMBERS 2(2) 1(2) 3(4) 27 M

5 LINEAR INEQUALITIES 1(1) 1(3) 2(4)

6 1(1) 1(1) 1(4) 3(6)


PERMUTATIONS
AND
COMBINATIONS
7 BINOMIALTHEOREM 1(1) 1(3) 2(4)

8 SEQUENCE AND SERIES 1(2) 1(5) 4(9)

9 STRAIGHT LINES 1(3) 1(4) 3(8) 13 M

10 CONIC SECTION 1(1) 1(3) 2(4)

11 3-DIMENSIONAL 1(1) 1(1)


GEOMETRY
12 LIMITS AND DERIVATIVES 2(2) 1(1) 1(5) 5(11) 11M

13 STATISTICS 1(2) 1(3) 2(5) 10 M

14 PROBABILITY 3(1) 1(4) 2(5)

TOTAL 18(18) 2(2) 5(10) 6(18) 4(20) 3(12) 38(80) 80

299
SAMPLE PAPER-02
TIME= 3HRS CLASS = XI M.M 80
General Instructions-

i. the question paper contains 5 sections- A, B, C, D and E. each section is compulsoryhowever,


there are internal choices in some questions.
ii. section A has 18 MCQ and 2 A &R type questions of 1 mark each.
iii. Section B has 5 VSA type questions of 2 marks each.
iv. Section C has 6 SA type questions of 3 marks each.
v. Section D has 4 LA type questions of 5 marks each.
vi. Section E has 3 Source Based / Case based / Passage based/ integrated units of
assessments (4marks each) with sub parts.
SECTION A
1. n(A)= m, and n(B) = n. Then the total number of non-empty relations that can be defined from A
to B is
(a ) mn (b) nm (c ) 𝑚𝑛 − 1 (d )2𝑚𝑛 − 1

2. If A = {1,3,5, {2, 4 }}, then which of the following is correct ?


(a ) {2} ⊂ A (b) {2,3} ⊂ A (c ) {2, 4} ⊂ A (d ) {{2, 4 }} ⊂ A
6
3. Find the range of the relation R = {(𝑥, 𝑦): 𝑦 = 𝑥 + ; 𝑤ℎ𝑒𝑟𝑒 𝑦 ∈ 𝑁 𝑎𝑛𝑑 𝑥 < 6}
𝑥
(a ) {1,2,3,4,5,6} (b){1,2,3, } (c ) {5,7} (d ){5, 7 , 9}
4. ∅⋂A = …………………..

(a ) A (b) ∅ (c ) U (d ) ∅′
31π
5. Find the value of sin
3

√3 (c ) not defined (d ) 1
(a ) √3 (b)
2

6. 𝑖 243 𝑖𝑠 𝑒𝑞𝑢𝑙 𝑡𝑜

(a ) -1 (b) 1 (c ) i (d ) -i
7. The number of non-zero integral solutions of the equation |1 − 𝑖|𝑥 =2 is
(a ) 1 (b) 2 (c ) -1 (d ) -2
3
8. If − 4 𝑥 ≤ −3, then which of the following is correct?
(a ) 𝑥 ≥ 4 (b) 𝑥 ≥ −4 (c ) 𝑥 ≤ −4 (d ) 𝑥 ≤ 4

300
9. 6 boys and 6 girls sit in a row at random. The probability that all the girls sit together is______
( a ) 1/432 ( b ) 12/ 431 ( c ) 1/132 ( d ) none of these
Answer- c
10. The total number of terms in the expansion of (𝑥 + 𝑎 )51 − (𝑥 − 𝑎 )51 after simplification is ?

(a ) 102 (b) 26 (c ) 25 (d ) 0
11. A box contains 10 good articles and 6 with defects. One item is drawn at random. The probability
that it is either good or has a defect is__________
( a) 64/64 ( b ) 49/64 (c ) 40/64 ( d ) 24/64
12. If A.M and G.M of two positive numbers are 10 and 8 respectively., then the numbers are.(a )
20 and 64 (b) 100 and 16 ( c) 4 and 16 (d ) 10 and 64
13. If tan 𝑥 = −1/√5 and 𝑥 lies in 4th quadrant, then the value of cos 𝑥 is __________
( a ) √5/√6 ( b ) 2/√6 ( c ) 1/2 ( d ) 1/√6
14. Find the equation of ellipse with focus at (±5, 0) and length of major axis 26.

𝑥2 𝑦2 𝑥2 𝑦2 𝑦2 𝑥2 𝑥2 𝑦2
(a ) 169 + =1 (b ) 169 − =1 (c )169 + = 1 (d )144 − 169 = 1
144 144 144
15. Let L be the foot of perpendicular from a point P(3, 4, 6) on the XY plane. Then coordinates
of L are
(a ) (3, 0, 0) (b) (-3, 4, 0 ) ( c) ( 0, 0, 6) (d ) (3, 4, 0)

16. The derivative of 99x at x = 0 is


(a ) 99 (b) 0 (c ) 1 (d ) 100

17. What is the probability of drawing a red king from a pack of 52 playing cards?(a )
30/52 (b) 1/13 (c ) ½ (d ) 1/26

𝑥 2 −9𝑥+20
18. lim
𝑥→5 𝑥 2 −6𝑥+5

(a ) 1 (b) 4 (c ) not defined (d )1/4

ASSERTION-REASON BASED QUESTIONS


In Questions number 19 and 20, a statement of Assertion (A) is followed by a statement of Reason
(R). Choose the correct answer out of the following choices.
(a) If both (A) and (R) are true and (R) is the correct explanation of (A)
(b) If both (A) and (R) are true and (R) is not the correct explanation of (A)
(c) if (A ) is true and (R ) is false.
(d) If (A ) is false (R ) is true.

301
19. Assertion (A) : 7! = 42X5!
Reason (R): n! = n(n – 1 )!

𝑥 3 −27
20. Assertion (A ) : lim = 27
𝑥→3 𝑥−3
𝑥 𝑎 −𝑎𝑛
on (R ): lim = 𝑎𝑛
Res 𝑥→𝑎 𝑥−𝑎

SECTION-B

21. If𝐴={𝑥:𝑥∈𝑅,𝑥 𝑖𝑠 𝑡ℎ𝑒 𝑟𝑜𝑜𝑡 𝑜𝑓 𝑡


3
ℎ𝑒 𝑒𝑞𝑢𝑎𝑡𝑖𝑜𝑛 𝑥 −𝑥 =0},
𝐵={𝑥:𝑥∈𝑅,𝑥 𝑖𝑠 𝑡ℎ𝑒 𝑟𝑜𝑜𝑡 𝑜𝑓 𝑥 3 +2𝑥 2 - x –
2=0} ,Then find the values of ( i)𝐴∪𝐵( ii)𝐴∩𝐵

21. Provethat
𝑡𝑎𝑛 2023𝑥−𝑡𝑎𝑛 2022𝑥−𝑡𝑎𝑛𝑥=𝑡𝑎𝑛
2023𝑥 .𝑡𝑎𝑛 2022𝑥 .𝑡𝑎𝑛𝑥
OR
If tan(𝐴+𝐵)=𝑝 and 𝑡𝑎𝑛(𝐴−𝐵)=𝑞 t hen
prove that
𝑝+𝑞
tan2A =
1−𝑝𝑞

3
23.If(𝑥+𝑖𝑦) =𝑢+𝑖𝑣 then show that
𝑢 𝑣 2 2
𝑥 + 𝑦 = 4(𝑥 −𝑦 )

24.Find the sum of n terms of the following series 7 +77+777+7777+…….

25.Mean and standard deviation of 100 observations are 50 and 4 respectively. Find the sum of all the
observations and the sum of the squares of the observations.

OR
Given that 𝑥 is the mean and 𝜎 2 is the variance of n observations
𝑥1 , 𝑥2 , 𝑥3 … … . . 𝑥𝑛 . 𝑃𝑟𝑜𝑣𝑒 𝑡ℎ𝑎𝑡 𝑡ℎ𝑒 𝑚𝑒𝑎𝑛 𝑎𝑛𝑑 𝑣𝑎𝑟𝑖𝑎𝑛𝑐𝑒 𝑜𝑓 𝑡ℎ𝑒 𝑜𝑏𝑠𝑒𝑟𝑣𝑎𝑡𝑖𝑜𝑛𝑠 𝑎𝑥1 , 𝑎𝑥2 , 𝑎𝑥3 , … … 𝑎𝑥𝑛
(𝑎 ≠ 0) are a𝑥 and 𝛼 2 𝜎 2

SECTION-C

𝑀𝐴
26. IQ of a person is given the formula𝑄 = 𝑋100; where MA is Mental age and CA is Chronological
𝐶𝐴

302
age. If 80 ≤ IQ ≤ 140 for a group of 12-year children, find the range of mental age.
27.Find the coefficient of a4 in the product(1 + 2𝑎)4 (2 − 𝑎)5 using binomialtheorem.

28.If p and q are the lengths of perpendiculars from the origin to the lines

x cosθ - y sin θ = k cos2θandxsecθ+ycosecθ=k, respectively, prove that p2+4q2=k2.

29.Find the equation of the circle which passes through the point (1,1) and centre lies at the point
of intersection of lines (x+y=4) & (x-y =0)
OR
If the eccentricity of the ellipse is 5/8 and distance between its foci is 10. Find the equationof
ellipse.

30.For sets A,B and C using properties of sets prove that


𝐴 − (𝐵 ∪ 𝐶) = (𝐴 − 𝐵) ∩ (𝐴 − 𝐶)
OR
For any two sets A and B , prove that 𝐴 ∪ 𝐵 = 𝐴 ∩ 𝐵 ⟹ 𝐴 = 𝐵

31 Find the mean deviation about the median for the following data:

xi 3 6 9 12 13 15 21 22

fi 3 4 5 2 4 5 4 3

SECTION-D

32.Find the domain and range of 1 / (2−sin 𝑠𝑖𝑛3𝑥).

33.If tanx = 3/4 , π<x<3π/2 then find the values of sin(x/2) , cos(x/2) and tan(x/2).

OR
Prove that sin10osin30osin50osin70o = 1/16.

34.If a and b are the roots of x2-3x+p = 0 and c and d are roots of x2-12x+q = 0 where a, b, c and

d form a GP. Then prove that 𝑞+𝑝 17


=
𝑞−𝑝 15

303
𝑠𝑖𝑛2 𝑥
35. i. find the derivative of 1+𝑐𝑜𝑠𝑥 w.r.t x 2 marks
ii. derivative of xSinx by first principle method. 3 marks

SECTION-E

36.During the mathematics class, a teacher clears the concept of permutations and combinations to
the 11th class students. After the class was over he asks students somemore questions:

On the basis of the information given above, answer the following

i. Find the number of arrangements of the letters of the word “INDEPENDENCE”. 1mark

ii. In how many of these do the words begin with “I” and end with “P”. 1mark

iii. In how many of these do all the vowels never occur together. 2marks

OR
In how many of these do all the four E’s do not occur together. 2marks

37.For an EMC project student need rectangular sheets, therefore they made eco-friendly
rectangular sheets PQRS from the paper-waste such that on the cartesian plane. Equation of QR
is (3x+4y = 12) and point P is (2,4) while point R is (16/5,a)

304
P Q

S R

38.On her winter vacations, Aisha visits 4 cities (Delhi, Mumbai, Goa and Bangalore) in randomorder.

On the basis of above information answer the following:

iv. What is the probability that she visits Delhi before Goa and Goa before Mumbai? 2marks

OR

What is the probability that she visits Delhi first and Mumbai last? 2marks

v. What is the probability that she visits Delhi just before Mumbai? 2marks

305
MATHEMATICS (041) - Sample Paper 02
Time – 3 hours Class XI Maximum Marks - 80

MARKING SCHEME

1 d 6 D 11 a 16 a

2 d 7 B 12 c 17 d

3 c 8 A 13 a 18 d

4 b 9 C 14 a 19 a

5 b 10 B 15 d 20 c

21 21. A={0,-1,1} and B={-2,-1,1}


i. AUB={0,-1,1,-2}
1
ii. A ∩ B = {−1,1}
1

22 tan 𝑥 = tan(2023𝑥 − 2022𝑥)


𝑡𝑎𝑛2023𝑥−𝑡𝑎𝑛2022𝑥
tan(2023𝑥 − 2022𝑥) = 1+tan 2023 tan 2022𝑥
1
tan 𝑥( 1 + tan 2023𝑥. tan 2022𝑥)
= tan 2023𝑥
− tan 2022𝑥
1
tan 2023x tan2022xtan x = tan 2023x − tan2022x − tan x
OR

tan 2A=tan{(𝐴 + 𝐵) + (𝐴 − 𝐵)}𝑡𝑎𝑛[(𝐴 + 𝐵) + (𝐴 − 𝐵)] =


𝑡𝑎𝑛(𝐴+𝐵)+𝑡𝑎𝑛(𝐴+𝐵) 1
tan 2𝐴 =
1−𝑡𝑎𝑛(𝐴+𝐵)𝑡𝑎𝑛(𝐴+𝐵)

𝑝+𝑞
1
=
1−𝑝𝑞

(𝑥 + 𝑖𝑦)33 = 𝑢 + 𝑖𝑣
23
(𝑥 3 − 3𝑥𝑦 2 ) + 𝑖 (3𝑥 2 − 𝑦 3 ) = 𝑢 + 𝑖𝑣
1
𝑥(𝑥 2 − 3𝑦 2) = 𝑢 𝑎𝑛𝑑 𝑦(3𝑥 2 − 𝑦 2 ) = 𝑣
𝑢 𝑣
= 𝑥 2 − 3𝑦 2 𝑎𝑛𝑑 = 3𝑥 2 − 𝑦 2
𝑥 𝑦

306
S= 7+77+777+7777+…………n terms
24
=7(1+11+111+11111+………. n terms)
7
= (9 + 99 + 999 + 9999 + ⋯ . . … . . 𝑛 𝑡𝑒𝑟𝑚𝑠)
1
9

7
= [(10 + 102 + 103 + 104 + ⋯ … … … . 𝑛 𝑡𝑒𝑟𝑚𝑠) − (1 + 1 + 1 + ⋯ … 𝑛 𝑡𝑒𝑟𝑚𝑠)]
9
7 10(10𝑛−1) 7 10(10𝑛−1)
= [ – 𝑛] = [ – 𝑛]
9 10−1 9 9

1
∑ 𝑥𝑖
25 Mean=
100
= 50 𝑔𝑖𝑣𝑒𝑠 ∑ 𝑥𝑖 = 5000 1

∑𝑛 2
𝑖=1 𝑥 𝑖
S.D=√ − 502 = 4 ∑𝑛𝑖=1 𝑥 2 𝑖 = 251600
100
1
OR

New observation a𝑥1 , 𝑎𝑥2 , 𝑎𝑥3 , … … … … 𝑎𝑥𝑛 (𝑎 ≠ 0)

∑ 𝑦𝑖 ∑ 𝑎𝑥𝑖 ∑ 𝑥𝑖
New Mean = 𝑦 = = =𝑎 × = ax
̅̅̅
𝑛 𝑛 𝑛

(𝑥𝑖—𝑥)2 𝑛 = 𝑎2𝜎2

∑ 𝑥𝑖
26 Mean=
100
= 50 𝑔𝑖𝑣𝑒𝑠 ∑ 𝑥𝑖 = 5000 1
∑ 𝑋𝐼2
SD= √ – 502 = 4 𝑔𝑖𝑣𝑒𝑠 ∑ 𝑥𝑖2 = 251600
100

OR

∑ 𝑦𝑖 ∑ 𝑎𝑥𝑖 ∑ 𝑥𝑖
New Mean = 𝑦 = = =𝑎 × = ax
̅̅̅
𝑛 𝑛 𝑛

New Variance = 𝜎 =
2 ∑ (𝑦𝑖− 𝑦)2
=
∑ (𝑎𝑥−𝑎𝑥)2
= 𝑎2 × (𝑥𝑖—𝑥)2 𝑛 =
1
1 𝑛 𝑛
𝑎2𝜎2
𝑀𝐴 𝑀𝐴 25
27 IQ = × 100 = × 100 = 𝑀𝐴 1
𝐶𝐴 12 3

25
80≤ 3
𝑀𝐴 ≤ 140 1
3 3
80× 25 ≤ 𝑀𝐴 ≤ 140 × 25 1

9.6≤ 𝐼𝑄 ≤ 16.8
4 2 3 4
28 27. (1+2𝑎) = 1 + 8𝑎 + 24𝑎 + 32𝑎 + 16𝑎 1
(2-𝑎)5 = 32 − 80𝑎 + 80𝑎2 − 40𝑎3 + 10𝑎4 − 𝑎5 1
4 4 (2 5
=Coefficient of 𝑎 in (1 + 2𝑎) − 𝑎)
1
4 3 2 2 3 4
=(10𝑎 ) + 8a(-40𝑎 ) + 24𝑎 (80𝑎 ) + 32 𝑎 (-80 a)+16𝑎 (32)
=-438𝑎4

29 point of intersection of x + y =4 and x – y = 0 is (4,0) 1

307
Radius = √10 1
Equation of circle is (𝑥 − 4)2 + (𝑦)2 = 10 1

OR
1
5
2c = 10 gives c=5 and e=8
1
𝑐 5
= gives a=8
𝑎 8 1
2 2 2 2
Now 𝑏 + 𝑐 = 𝑎 𝑔𝑖𝑣𝑒𝑠 𝑏 = 39
𝑥2 𝑦2
So equation of ellipse is + =1
64 39

30 𝐴 − (𝐵 ∪ 𝐶) = 𝐴 ∩ (𝐵 ∪ 𝐶)′ - 1
′ ′
⟹ 𝐴 ∩ (𝐵 ∩ 𝐶 )
⟹ (𝐴 ∩ 𝐵 ′ ) ∩ (𝐴 ∩ 𝐶 ′ ) 1
⟹ (𝐴 − 𝐵) ∩ (𝐴 − 𝐶)
OR 1

Let 𝐴 = 𝐵. Then 𝐴 ∩ 𝐵 = 𝐴 𝑎𝑛𝑑 𝐴 ∪ 𝐵 = 𝐴


⟹𝐴∪𝐵 =𝐴∩𝐵 (1)
Conversely, if 𝐴 ∪ 𝐵 = 𝐴 ∩ 𝐵 1
Let , 𝑥𝜖𝐴 ⟹ 𝑥𝜖𝐴 ∪ 𝐵
⟹ 𝑥𝜖𝐴 ∩ 𝐵
⟹ 𝑥𝜖𝐴 𝑎𝑛𝑑 𝑥𝜖𝐵
⟹ 𝑥𝜖𝐵 1
So, 𝐴 ⊂ 𝐵
Let 𝑦𝜖𝐵 ⟹ 𝑦𝐴 ∪ 𝐵
⟹ 𝑦𝜖𝐴 ∩ 𝐵
⟹ 𝑦𝜖𝐴 𝑎𝑛𝑑 𝑦𝜖𝐵
⟹ 𝑦𝜖𝐴 1
So, 𝐵 ⊂ 𝐴
Therefore A=B

31 Make table of column 𝑥𝑖 , 𝑓𝑖 , 𝑐𝑖 𝑓𝑖 , |𝑥𝑖 − 𝑀| , 𝑓𝑖 |𝑥𝑖 − 𝑀| 2


15𝑡ℎ 0𝑏𝑠+16𝑡ℎ 𝑜𝑏𝑠
Median = = 13
2

∑ 𝑓𝑖 |𝑥𝑖 −𝑀|
Mean deviation about median = = 4.97(appr.). 1
∑ 𝑓𝑖

1
32 y = 2−𝑠𝑖𝑛3𝑥 1
Since -1≤ 𝑠𝑖𝑛3𝑥 ≤ 1
1≥ − 𝑠𝑖𝑛 sin 3𝑥 ≥ −1 1
1≤ 2 − 𝑠𝑖𝑛3𝑥 ≤ 3.

308
So domain of f = R 1
1≤ 2 − 𝑠𝑖𝑛3𝑥 ≤ 3.
1
1≥≥ 3 1

1
Range of f =[3 , 1] 1

1 4 3𝛱
33 . cosx = -√1+𝑡𝑎𝑛2𝑥 =- 5 as 𝛱 < 𝑥 < 2
1

Cosx< 0 , 𝑎𝑛𝑑 𝑠𝑖𝑛𝑥 > 0 1

𝑥 1 𝑥 3 𝑥
Find cos2 = - , sin2 = , tan2 = -3 1+1+1
√10 √10

OR
1
Sin 10° Sin 30° Sin 50° Sin 70° = 2 (Sin 50° Sin 10°) Sin 70°

1
= ( 2 Sin 50° Sin 10° ) Sin 70° 1
4

1
= ( cos40° - cos60° ) Sin 70° 1
4

1 1
= [sin(40° + 70° ) + sin(70° − 40°)]- sin70°. 1
8 8

1 1 1
=. ( sin110° + 2)- cos60° ) - sin (180° -110° ) 1
8 8

1 1 1
= 𝑠𝑖𝑛110° + − 8 𝑠𝑖𝑛110°
8 16

1
1
= 16

34 Let b = ar. ,c = 𝑎𝑟 2 , d = 𝑎𝑟 3 , 1

309
Given a + b = 3 , ab = p , c + d = 12 , cd = q 1

a (1 + r ) = 3 , 𝑎𝑟 2(1 + r ) = 12
1

on dividing we get r = 2 & a = 1

𝑞+𝑝 17
1
so =.
𝑞−𝑝 15

1
2 2 𝑥(−𝑠𝑖𝑛𝑥)
35 (1) 𝑑 { 𝑠𝑖𝑛 𝑥 } = (1+𝑐𝑜𝑠𝑥 )2 𝑠𝑖𝑛𝑥 𝑐𝑜𝑠𝑥−𝑠𝑖𝑛
(1+𝑐𝑜𝑠𝑥 )2
1 1
𝑑𝑥 1+𝑐𝑜𝑠𝑥

1
(1+𝑐𝑜𝑠𝑥 ) 𝑠𝑖𝑛2𝑥+𝑠𝑖𝑛3 𝑥
= (1+𝑐𝑜𝑠𝑥 )2

(2) f’(x) =
𝑓(𝑥+ℎ)−𝑓(𝑥)
=
(𝑥+ℎ)𝑠𝑖𝑛(𝑥+ℎ)−𝑥𝑠𝑖𝑛𝑥 1
ℎ ℎ

𝑥 𝑠𝑖𝑛𝑥(𝑐𝑜𝑠ℎcosh −1 ) 𝑠𝑖𝑛ℎ
+ +
ℎ ℎ 1

= x cosx + sin x
1

36 In the word INDEPENDENCE there are 12 letters of which 3 are N’s, are
4 are E’s and 2 are D’s
12!
(i) Total number of arrangements =3!4!2! = 1663200
10!
(ii) Number of words begin with I an end with P =3!4!2! = 12600 1

(iii) Number of Words in which vowels never occur together


= Total number of arrangements- Number of arrangements in which 1
vowels occur together
8! 5!
= 1663200-3!2! × 4!1!

=1663200-16800=1646400 1

1
OR

310
Number of words in which all the four E’s do not occur together= total
no. of arrangements – no. of arrangements in which all the four E’s
= 1663200- 9!/3!2! = 1663200- 30240 = 1632960 1

37 (A) Slope of PS= slope of QR= -3/4


Equation of PS is y-4= -3/4(x-2)
4y – 16 = -3x+6
3x + 4y = 22 1
(B) 3(16/5)+4a=12 gives a =3/5
1
(C) Equation of PQ is 4x-3y= -4
Coordinates of QR(4/5,12/5)
PQ=2 & QR=3 1
Area of Rectangle= 6sq.unit
OR
1
Equation of PQ is 4x-3y= -4
1
Coordinates of QR(4/5,12/5)
1
PQ=2 & QR=3
Perimeter= 10 units.

38 Here n(s) = 24 1

(A) E1= (DBGM, DGBM, DGMB, BDGM)


1

P(E1) = 4/24= 1/6


1
OR
1
E2 = (DBGM, DGBM)
P(E2) = 2/24=1/12 1

(B) E3= (DMGB, DMBG,GBDM,GDMB,BGDM,BDMG) 1


P(E3) = 6/24=1/4

311
BLUE PRINT
SAMPLE PAPER-III

Blue print-Class XIth Mathematics


Sample paper-3
Session 2024-25
UNIT TOPICS 1M A 2M 3M 5M CASE TOTAL TOTAL MAX
AND STUDY
R
Sets and Sets 1 1 1 6 23 23
Functions Relations and 1 1 1 6
Functions
Trigonometric 1 1 1 1 11
Functions
Algebra Complex 2 2 25 25
Numbers
Linear 2 2
Inequalities
Permutations 2 1 1 11
and
Combinations
Binomial 1 1
Theorem
Sequences and 2 1 1 9
Series
Coordinat Straight Lines 2 1 1 6 12 12
e Conic Sections 2 1 4
Geometry Introduction of 1 2
Three-
Dimensional
Geometry
Calculate Limits and 2 2 8 8 8
derivatives
Statics Statistics 1 5 12 12
and Probability 1 1 7
Probabilit
y
18 2 5 6 4 3 80 80 80

312
Sample Paper -3
Maximum Marks: 80 Time Allowed: 3 hours

General Instructions:

1. This Question paper contains - five sections A, B, C, D and E. Each section is compulsory.
However, there are internal choices in some questions.
2. Section A has 18 MCQ’s and 02 Assertion-Reason based questions of 1 mark each.
3. Section B has 5 Very Short Answer (VSA)-type questions of 2 marks each.
4. Section C has 6 Short Answer (SA)-type questions of 3 marks each.
5. Section D has 4 Long Answer (LA)-type questions of 5 marks each.
6. Section E has 3 source based/case based/passage based/integrated units of assessment
(4 marks each) with sub parts.

SECTION-A
1.If A and B are two sets then A ∩(A∪B) equals
(a)A (b)B (c)Ø (d) A ∩B
2 If A ={x:x2-5x+6=0} ,B = {2,4} , C ={4,5} then A X (B ∩ C) is
(a) {(2,4),(3,4)} (b) {(4,2),(4,3)} (c) {(2,4),(3,4)(4,4)} (d) {(2,2),(3,3),(4,4),(5,5)}
3.I f(x)= x3-1/x3, then f(x)+f(1/x) is equal to
(a)2x3 (b) 2/ x3 (c) 0 (d) 1
4.The conjugate of complex number is 1/i-1, then the complex number is
(a)-1/i+1 (b) 1/ i-1 (c) -1/i-1 (d) 1/i+1
5.If x,y∈R, then x+iy is not purely imaginary is
(a)x=0 (b)y=0 (c) x≠0 (d) y≠0
6. If |x+1|/x+1 >0, x ∈R then _________
(a)x∈ (-1,∞] (b) x∈ (-1,∞) (c) x∈ (-∞,-1) (d) x∈ (-∞,-1]
2𝑥−6
7. If 1/x-2 ≤ 0, x𝜖𝑅 , then ______ ≤0
5

(a) (−∞, −3) (b) x∈ (-∞,2) (c) (-∞, 3] (d) (-∞,2)∪ (3,∞)
8.The number of signals that can be sent by 6 flags of different colors of taking one or more at a time is
(a)63 (b)1956 (c)720 (d)21
9.The number of permutations of n different object taken r at a time when repetition are allowed is
(a) n P r (b)n! (c) nr (d) rn
10.The middle term in the explanation of (2x2/3+3/2x2)10 is
( a) 251 (b)252 (c) 250 (d) none of these
11. A sequence may be defined as
313
(a)relation whose range ⊂ Natural number
(b) as function whole range ⊂ N
(C) function whole domain ⊂ N
(d) progression having real values
12.The third term of G.P.is 4 If first term is 1 then the product of first three term is
(a)2 (b)4 (c)8 (d) none of these
13.If the line x/a+ y/b=1 passes through the point (2,-3)and (4,-5), then(a,b) is
(a)(1,1) (b)(-1,1) (c) (1,-1) (d) (-1,-1)
14.The inclination of the line x-y+3=0 with the positive direction of x- axis is
(a)45o (b)135o (c)-45o (d)-135o
15.The equation of the circle which passes through the point(4,5) and has it centre at (2,2) is
(a)x2+y2-4x-4y-5=0
(b) x2+y2+4x+4y+5=0
(c) x2+y2-4x+4y+5=0
(d)none of these
16.If the parabola 𝑦 2 = 4𝑥 passes through the point (3,2) then the length of its latus rectum is
(a)2/3 (b)4/3 (c)1/3 (d)4
|sin 𝑥|
17. lim is____________
𝑥→0 𝑥

(a)1 (b)-1 (c) 0 (d) does not exist

18.If F(x)=(1+x+x2/2+ x3/3+……………..+x1000 )/1000,then f(-1) is equal to


(a)1 (b)-1 (c) 0 (d) does not exist

19. Directions: Each of these questions contains two statements : Assertion (A) and Reason
(R). Each of these questions has four alternative choices, any one of which is the correct answer. You have to select one of the codes
(A), (B), (C) and (D) given below.
(A) A is true, R is true; R is correct explanation for A.
(B) A is true, R is true; R is not correct explanation for A.
(C) A is true; R is false.
(D) A is false; R is true.

A:The value of 𝑓(𝑥) = 3𝑐𝑜𝑠√𝑥 + 2 lie in the interval [−3,3]

R:The value of cos x lie in the interval [−1,1]

20.A: The tangent of the angle between the lines x/a + y/b =1 and x/a – y/b = 1 is 2ab/a2-b2

314
B: : The tangent of the angle between the lines with clops m 1 and m2 is given by
𝑚 +𝑚2
1
tan 𝜃 = |1−𝑚 |
1 𝑚2

SECTION- B
21.Are the following pair of sets equal? Give reasons.
(i) A = {2, 3}, B = {x : x is solution of x² + 5x + 6 = 0}
(ii) A = { x : x is a letter in the word FOLLOW} B = { y : y is a letter in the word WOLF}
22.Find the domain and range of the mentioned below real valued function.
1
𝑓(𝑥) = √16−𝑥 2

𝑓(𝑥) = |𝑥|
23.Find three numbers are in GP, whose sum is 13 and the sum of whose squares is 91?
24.If the vertex of a parabola is the point (-3, 0) and the directrix is the line x + 5 = 0, then find its
equation.
25.Find the coordinates of a point on y-axis which are at a distance of 5√2 units from the point P(3, -2, 5)

SECTION-C

26.Two finite sets have m and n elements. The number of subsets of the first set is 112 more than that of
the second. The values of m and n are respectively
27.The cartesian product A × A has 9 elements among which are found (-1,0) and (0, 1). Find the set A
and the remaining elements of A × A.
𝜋
28.Find equation of the line through the point (0, 2) making an angle of 3 with the positive x- axis. Also,

find the equation of line parallel to it and crossing the y-axis at a distance of 2 units below the origin.
𝑥
,𝑥 ≠ 0
29.Find lim 𝑓(𝑥), where 𝑓(𝑥) = {|𝑥|
𝑥→0 0 ,𝑥 = 0
𝑥 5 −cos 𝑥
30.Find the derivative of sin 𝑥

31. In a certain lottery 10, 000 tickets are sold and ten equal prizes are awarded. What is the probability
of not getting a prize if you buy (a) one ticket (b) two tickets (c) 10 tickets?

SECTION-D
cos 4𝑥+cos 3𝑥+cos 2𝑥
32.Prove that : = cot 3𝑥
sin 4𝑥+sin 3𝑥+sin 2𝑥

OR

315
Prove that : cot 𝑥. cot 2𝑥 − cot 2𝑥 cot 3𝑥 − cot 3𝑥 cot 𝑥 = 1

33. A group consists of 4 girls and 7 boys. In how many on a team of 5 members be selected, if the team
has
(i) No girl.
(ii) at least one boy and one girl.
(iii) at least three girls.
34.If the pth and qth terms of a G.P. are q and p, respectively, then show that (p + q)th term is
(qp/pq)1/p-q
35. Calculate the mean deviation about median age for the age distribution of 100 persons given below :
Age 16-20 21-25 26-30 31-35 36-40 41-45 46-50 51-55

Number 5 6 12 14 26 12 16 9

SECTION-E
𝑥, 𝑥 ≥ 0
36. For any real number x, (√𝑥)2 is not equal to ±x but √𝑥 2 = |𝑥| = {
−𝑥, 𝑥 < 0
Based on the above information, answer the following questions;
𝜋
i. if < 𝑥 <3π/2, then √1 − 𝑠𝑖𝑛2 x 𝑖s equal to
2

a)cosx b) –cos x c) ±cosx d) none of these

ii. if π< x< 2π, then √1 − 𝑐𝑜𝑠 2 xis equal to


a)sinx b)-sinx c) ±sinx d) none of these
1+𝐶𝑂𝑆 𝑋
iii .if π< 𝑥 < 2𝜋 then √1−𝐶𝑂𝑆 𝑋=?

sin 𝑥 −sin 𝑥 1+𝑐𝑜𝑠𝑥 1+𝑐𝑜𝑠𝑥


(a) 1−cos 𝑥 (b) 1−cos 𝑥 (c) 𝑠𝑖𝑛𝑥 𝑐𝑜𝑠𝑥

3𝜋 1+𝐶𝑂𝑆 𝑋 1−𝐶𝑂𝑆 𝑋
iv. if π< 𝑥 < ,then √1−𝐶𝑂𝑆 𝑋 + √1+𝐶𝑂𝑆 𝑋 is equal to
2

(a)2cosecx (b)-cosecx (c)cosecx (d)-2cosecx


37. Five couples were invited in a tea party. they were asked to sit on one side of a long table. based on
the above information, answer the following questions:
i. find the number of ways in which five couples can be seated.
ii. find the number of ways in which all males sit together and all females sit together.
iii.find the number of ways in which no two females sit together.
iv.find the number of ways in which all females are never together.

316
38.A team of medical students doing their internship have to assist during surgeries at a city hospital.
The probabilities of surgeries at a city hospital, the probabilities of surgeries rated as very complex,
complex, routine, simple and very simple are respectively 0.15,0.20,0.31,0.26,0.08. based on the above
information, answer the following questions;
Find the probabilities that a particular surgery will be rated
a. Complex or very complex

b. Neither very complex nor very simple


c. Routine or complex
d. Routine or simple

Answer key

1 A 6 B 11 C 16 B
2 A 7 C 12 C 17 D
3 C 8 B 12 D 18 c
4 A 9 C 14 A 19 B
5 C 10 B 15 A 20 C

21 i. No 1
ii. yes 1

22 ( I ) Domain (−4,4) , range 𝑥 ≥ 1 1


4

( ii ) domain = R ,range, 𝑥 ≥ 0 1

23 9,3,1 or 1,3,9 1+1


24 𝑥 2 = 8(𝑥 + 3) 1+1
25 (0,2,0)𝑎𝑛𝑑 (0, −6,0) 1+1
26 Correct formula 1
𝑚 = 7 𝑎𝑛𝑑 𝑛 = 4 1+1
27 . A ={-1,0,1} 1
The remaining element of set A×A are (-1,1) ,(0,-1),(0,0), 2
(1,-1),(1,0)and (1,1)

28 Calculation, 1

√3 +y-2=0 and √3+y+2=0 1+1

317
29 Using limit, finding LHL and RHL 2
Limit does not exist at x=0 1
30 Using proper derivative formula 1
Finding correct derivative of each part 1
−𝑥 5 cos 𝑥 + 5𝑥 4 sin 𝑥 + 1 1
(sin 𝑥)2
31 .(a) 999 1
1000
9990𝑐2 1
(b) 10000𝑐2
1
9990𝑐10
(c) 10000𝐶10

32 cos 4𝑥 + cos 3𝑥 + cos 2𝑥


sin 4𝑥 + sin 3𝑥 + sin 2𝑥
(cos 4𝑥+cos 2𝑥)+cos 3𝑥 2 cos 3𝑥 cos 𝑥+cos 3𝑥
= (sin 4𝑥+sin 2𝑥)+sin 3𝑥 = 1+1
2 sin 3𝑥 cos 𝑥+sin 3𝑥
cos 3𝑥(2cos 𝑥+1) 1+1
=sin 3𝑥(2 cos 𝑥+1)
1
=cot 3x
Or
1
Use of cot 3x =cot(2x+x)
1
Write the formula
2
Simplification
1
Obtaining result
33 i. 21 1
ii. 441 2
iii. 91 2

34 Obtain p th term 1
Obtaining q th term
Obtaining (p+q ) th term 2
Proving 2
35 Obtaining table 2
Getting correct values 1
Putting correct formula 1
7.35 1
36 36. (i) b 1
(iii) b 1
(iv) (iii)b

318
(v) (iv) d 1
1
37 .(i) 10 ‫יִ‬ 1
(ii)2 × 5! × 5! 1
(iii) 6! 1
(iv) 10! − 6! 1
38 (i) 0.35 1
(ii) 0.77 1
(iii) 0.51 1
(iv) 0.57
1

319
SAMPLE PAPER-4

SNO UNIT/TOPIC SECTION‘A’ SECTION SECTION‘ SECTION’ SECTION‘ TOTA UNI


(OBJECTIVET ‘B’ C’ D’ E’ L TWI
YPE) (VSA) (SA) (LA) (CBQ) SE
MA
RKI
NG

MCQ (2M (3M (5M (4M


(1MarkEach) arks arks arks arks
Each Each Each Each
) ) ) )
1 SETS 3(3) 1(2) 4(5) 23 M
2 RELATIONS AND 2(2) 1(2) 1(3) 4(7)
FUNCTIONS
3 TRIGONOMETRIC 3(3) 1(3) 1(5) 5(11)
FUNCTIONS
4 COMPLEX NUMBERS 1(1) 1(3) 2(4) 27 M
5 LINEAR INEQUALITIES 1(1) 1(5) 2(6)
6 PERMUTATI 1(2) 1(4) 3(7)
ONS AND
COMBINATIO
NS
7 BINOMIALTHEOREM 2(2) 2(2)
8 SEQUENCE AND SERIES 1(2) 1(5) 3(8)
9 STRAIGHT LINES 1(1) 1(2) 1(3) 3(6) 12M
10 CONIC SECTION 1(1) 1(4) 2(5)
11 3 DIMENSIONAL 1(1) 1(1)
GEOMETRY
12 LIMITS AND 2(2) 2(6) 4(8) 8M
DERIVATIVES
13 STATISTICS 1(5) 1(5) 10 M
14 PROBABILITY 3(1) 1(4) 2(5)

320
SAMPLE PAPER – 4

Time: 3 Hours SESSION 2024 – 25 M.M. 80


General Instructions

• This question paper contains five sections viz. Section A, Section B, Section C,
Section D and Section E.
• Section A : 20 objective type questions 1 mark each.
• Section B : 5 very short answer type questions 2 marks each
• Section C : 6 short answer type questions 3 marks each.
• Section D : 4 short answer type questions 5 marks each.
• Section E : 3 Case Based Question of 4 marks each.
• All questions are compulsory. Internal choice are given in two question of 2 marks, two
question of 3 marks and two questions of 5 marks. Attempt any one.
• Write question number carefully before attempting it.
• Attempt Objective type questions in first page of the answer script.
• The question paper has 6 printed pages.

Section A (Objective Type Questions) 1 Mark Each


Q Question Value
No point

1 Which of the following statement is false: 1


(A)A – B = A ∩ B’ (B) (A – B) = A – (A ∩ B)

(C) A – B = A – B’ (D) A – B = (A ∪ B) – B

2 Let {1, 2, {3, 4}, 5}, which of the following statement is correct: 1

(A) {3, 4} ⊂ A (B) {{3, 4}} ∈ A (C) {{3, 4}} ⊂ A (D) {1, 2, 5} ∈ A

3 Let R be the relation defined in ℕ defined by R = {(1 + 𝑥, 1 + 𝑥2): 𝑥 ≤ 5, 𝑥 ∈N }. Which of the following 1
statement is false:

(A) R = {(2, 2), (3, 5), (4, 10), (5, 17), (6, 25)} (B) Domain of R = {2, 3, 4, 5, 6}
(C) Range of R = {2, 5, 10, 17, 26} (D) (B) and (C) are true

4 If A is the set of even natural numbers less than 8 and B is the set of prime numbers less 1
than 7, then the number of relations from A to B is:

(A) 29 (B) 92 (C) 18 (D) 29 – 1

321
5 1−𝑡𝑎𝑛2 15° 1
The value of 1+𝑡𝑎𝑛2 15° is __________
√3
(A) 1 (B) √3 (C) 2
(D) 2

6 1
. If 𝑠𝑖𝑛𝜃 + 𝑐𝑜𝑠𝜃 = 1, then the value of 𝑠𝑖𝑛2𝜃 is:
(A) 1 (B) √3 (C) 0 (D) – 1

7 If a complex number lies in second quadrant in the argand plane, then its conjugate lies in 1
quadrant.
(A) second (B) third (C) first (D) fourth

8 6 boys and 6 girls sit in a row at random. The probability that all the girls sit together 1
is______

( a ) 1/432 ( b ) 12/ 431 ( c ) 1/132 ( d ) none of these

9 The total number of terms in the expansion of (𝑥 + 𝑎)51 − (𝑥 − 𝑎)51 after simplification is 1

___
(A) 102 (B) 104 (C) 26 (D) none of these

10 Solution of −8 ≤ 5𝑥 − 3, for 𝑥𝜖𝑅 is ________ 1

(A) (– 1, 2) (B) [– 1, ∞) (C) [– 1, 2] (D) {– 1, 2}

18
11 The largest coefficient in the expansion of (𝑎 + 𝑏) is: 1
(A) 18𝐶18
(B) 18𝐶12
(C) 18𝐶9
(D) 18𝐶6

12 A box contains 10 good articles and 6 with defects. One item is drawn at random. The 1
probability that it is either good or has a defect is__________

( a) 64/64 ( b ) 49/64 (c ) 40/64 ( d ) 24/64


13 A line cutting off intercept – 3 from the y – axis and the tangent of angle to x – axis is 3/5, 1
then its equation is:
5

322
(A) 5𝑦 − 3𝑥 + 15 =0 (B) 3𝑦 − 5𝑥 + 15 = 0

(C) 5𝑦 − 3𝑥 − 15 = 0 (D) none of these

𝒙𝟐 𝒚𝟐
14 For hyperbola − 𝒔𝒊𝒏𝟐 = 𝟏 the absolute value of ae is: 1
𝒄𝒐𝒔𝟐 𝜶 𝜶

(A) √2 (B) √3 (C) 1 (D) 0

15 If L is the foot of perpendicular drawn from the point P (3, 4, 5) on the xy – plane, then the1111
coordinates of L are:

(A) (3, 0, 0) (B) (0, 4, 5) (C) (3, 0, 5) (D) none of


these

𝑒 𝑥 −𝑒 5
16 lim is equal to_____ 1
𝑥→5 𝑥−5

A) 𝑒−5 (B) 1 (C) 𝑒5 (D) 5𝑒

17 Two students Ajay and Vijay appeared in an examination. The probability that Ajay will qualify
the examination is 0.05 and that Vijay will qualify the examination is 0.10 . The probability that
both will qualify the examination is 0.02. The probability at least one of them not will not qualify
the examination is
A) 0.13 (B) 0.87 (C) 0.02 (D) 0.98
𝑑𝑦 𝜋
18 If 𝑦 = 𝑠𝑖𝑛𝑥 + cos 𝑥 then value of 𝑑𝑥 at 𝑥 = 6
𝑖𝑠 1
is:

√𝟑 √𝟑−𝟏
(A) 1 (B) 𝟐 (c) ½ (D ) 𝟐

Assertion and Reason questions

Directions: Each of these questions contains two statements : Assertion (A) and Reason
(R). Each of these questions has four alternative choices, any one of which is the correct answer. You have to select
one of the codes (A), (B), (C) and (D) given below.
(E) A is true, R is true; R is correct explanation for A.
(F) A is true, R is true; R is not correct explanation for A.
(G) A is true; R is false.
(H) A is false; R is true.

19 Assertion (A): If A = {8𝑛 − 7𝑛 − 1 ∶ 𝑛 ∈ ℕ} and B = {49𝑛 − 49 ∶ 𝑛 ∈ ℕ}


1

then A and B are equal sets.

323
Reason (R): Two sets A and B are equal sets if A and B have exactly the same elements.

20 Assertion (A) : 𝑡𝑎𝑛3𝑥𝑡𝑎𝑛2𝑥𝑡𝑎𝑛𝑥 = 𝑡𝑎𝑛3𝑥 − 𝑡𝑎𝑛2𝑥 −𝑡𝑎𝑛𝑥 1

𝑡𝑎𝑛𝐴− 𝑡𝑎𝑛𝐵
Reason (R) : tan(𝐴 − 𝐵) = 1−𝑡𝑎𝑛𝐴 𝑡𝑎𝑛𝐵

Section B (Very Short Answer Type Questions) 2 Marks Each

21 If A and B are two sets such that A ∩ B ≠ 𝜙, then draw the Venn diagram to 2
represent (A – B) ∪ (B – A).

22 Let 𝑓 = {(1, 1), (2, 3), (0, −1), (−1, −3)} be a function defined by 𝑓(𝑥) = 𝑎𝑥 + 2

𝑏 for some integers 𝑎 and 𝑏, then find 𝑓(𝑥).

23 If 5th , 8 th and 11th term of a G.P. are 𝑝, 𝑞 and 𝑟 respectively. Show that 𝑞2 = 𝑝𝑠 2

OR
If 4th, 10th and 16th term of a G.P. are 𝑥, 𝑦 and 𝑧 respectively. Prove that 𝑥, 𝑦, 𝑧 are in
G.P.

24 If a line joining two points A (2, 0) and B (3, 1) is rotated about A in anti-clock wise 2

direction through an angle of 15o. Find the equation of line in new position.

OR
Find the equation of the line which passes through the point (1, – 2) cuts off equal
intercepts from axes.

25 The number lock of suitcase has 4 wheels, each labelled with ten digits i.e. from 0 to 9. 2
The lock opens with a sequence of four digits with no repeat. What is the probability of
a person getting the right sequence to open the suitcase?

Section C (Short Answer Type Questions) 3 Marks Each

324
26 Let R be the relation from ℤ to ℤ defined by R = { (a, b) : a – b is divisible by 2, a, b ∈ 3
Z}. Show that:
(i) (a, a) ∈ R for all a ∈ Z
(ii) (a, b) ∈ R ⟹ (b, a) ∈ R
(iii) (a, b) ∈ R and (b, c) ∈ R ⟹ (a, c) ∈ R.

OR
Find domain of the function 𝑓(𝑥) = √𝑥 2 − 4

27 sin 4𝑥+sin 3𝑥+sin 2𝑥 3


Prove that : cos 4𝑥+cos 3𝑥+cos 2𝑥 = tan 3𝑥

28 If 𝛼 and 𝛽 are different complex numbers with |𝛽| = 1 then find value of | 𝛼−𝛽̅ | 3
1−𝛼𝛽

Or

𝑥 𝑦
If 𝑥 + 𝑖𝑦 = (𝑎 + 𝑖𝑏)3 , 𝑥, 𝑦, 𝑎, 𝑏𝜀𝑅 . Show that − = −2(𝑎2 + 𝑏 2 )
𝑎 𝑏

29 Find the equations of lines through (3, 2) which makes an angle of 45 o with the line 3
𝑥 − 2𝑦 − 3 = 0 .

√𝑥 2 +1−√5
30 Evaluate: lim 3
𝑥→2 𝑥−2

31 Find the derivative of xcosx by first principle. 3

Section D (Long Answer Type Questions) 5 Marks Each


32 𝜋 3𝜋 5𝜋 7𝜋 1 5
Prove that (1+cos 8 )( 1+cos 8 ) (1+cos 8 ) (1+cos 8 ) = 8

𝜋 𝜋 3
OR Prove that: 𝑠𝑖𝑛2𝑥 + 𝑠𝑖𝑛2 (𝑥 + ) + 𝑠𝑖𝑛2 (𝑥 − )=
3 3 2

33 1. A solution of 8% boric acid is to be diluted by adding 2% boric acid solution to it. The 5
resulting The resulting mixture to be more than 4% but less than 6% boric acid. If we
have 640 litres of the 8% solution, how many litres of 2% solution be added to the
8% solution.

34 If A and G be A.M. and G.M. respectively between two positive numbers, prove that 5

the numbers are 𝐴 ± √(𝐴 + 𝐺)(𝐴 − 𝐺)

35 Find the mean and variance for the following frequency distribution. 5
Class 00 – 30 00 – 30 00 – 30 00 – 30 00 – 30 00 – 30 00 – 30
Frequency 2 5 10 3 5 2
325
OR
The mean of the 5 observation is 4.4 and their variance is 8.24. If three of the
observations are1, 2, 6, find the other two observations.

Section E (Case Based Questions) 4 Marks Each

36 For a debate competition in a school the house master of Subhash House enrolled 4 girls
and 7 boys. From these 11 students the house master have to select 5 participants for the
competition. In how many ways can the house master select when
1.The team consists no girl (1)
2.The team consists at least three girls. (1)
3.The team consists at least one girl and one boy. (2)
37 In a class of 60 students, 30 opted for NCC, 32 opted for NSS and 24 opted for both NCC
and NSS. If one of the student is selected at random, then find the probability that
1.The student opted for NCC or NSS. (1)
2.The student opted neither NCC nor NSS. (1)
3.The student has opted NSS but not NCC.(2)
38
A suspension bridge is a type of bridge in which the deck is hung below suspension
cables on vertical suspenders. The basic structural component of a suspension bridge
system includes stiffening girders/trusses, the main suspension cables, main towers and
the anchorages for the cable at each end of the bridge. The main cables are suspended
between towers and are finally connected to the anchorage or the bridge itself, and
vertical suspenders carry the weight of the deck and the traffic load on it. Like other cable
supported bridge, the super structure of suspension bridges is constructed without false
work as the cable erection method is used. The main cables carrying members us the main
cables, which are tension members made of high – strength steel. The whole cross –
section of the main cable is highly efficient in carrying the loads and bucking is not

problem.

326
The main cables of a suspension bridge are hang in the form of a parabola
between towers of height 30 metre above the roadways and are 200 metre apart.
If the supporting cable is 5 metre above the roadway at the centre of the bridge.
C B

N R
O

L A Q M
30
m

Based on above information answer the following:

1.Find the equation of main cable. (2)


2.If the supporting cable PQ is 30 m from the mid – way find its length. (2)

327
MARKING SCHEME
SAMPLE PAPER 4
Q. No. Value Point/ Correct answer Marks
1. C
2. C
3. A
4. A
5. C
6. C
7. B
8. C
9. C
10. B
11. C
12. A
13. A
14. C
15. D
16. C
17. B
18. D
19. A
20. B
21. 1
A B U for showing sets A, B and U
For correct shading 1

22. 𝑓(1) = 1 ⟹ 𝑎 + 𝑏 = 1 ½
𝑓(0) = −1 ⟹ 𝑏 = −1 ½
½
𝑎=2
½
𝑓(𝑥) = 2𝑥 – 1
23. Let a be the first term and r be the common ratio of the G.P.
∴ 𝑎𝑟4 = 𝑝, 𝑎𝑟7 = 𝑞, 𝑎𝑟10 = 𝑠 ½
½
𝑝𝑠 = 𝑎𝑟4 × 𝑎𝑟10 = 𝑎2𝑟14
½
⟹ 𝑝𝑠 = (𝑎𝑟7)2 ½
⟹ 𝑝𝑠 = 𝑞2
OR
Let a be the first term and r be the common ratio of the G.P. ½
½
∴ 𝑎𝑟3 = 𝑥, 𝑎𝑟9 = 𝑦, 𝑎𝑟15 = 𝑧
𝑥𝑧 = 𝑎𝑟3 × 𝑎𝑟15 = 𝑎2𝑟18 ½
⟹ 𝑥𝑧 = (𝑎𝑟9)2 ½
⟹ 𝑥𝑧 = 𝑦2
⟹ 𝑥, 𝑦, 𝑧 are in GP

24 1−0 1
Slope of line AB=3−1 = 1, 𝜃 = 45°
Inclination of line in new position=45° + 15° = 60°

328
Slope of line in new position=𝑡𝑎𝑛45° = √3
1
Equation of line in new position :𝑦 − √3𝑥 + 2√3 = 0
OR
𝑥 𝑦
Let the line be 𝑎 + 𝑏=1 1

It passes through the point (1, −2)


1 −2
+ 𝑎 =1 1
𝑎
𝑎 = −1
Equation of the line is 𝑥 + 𝑦 + 1 = 0
25. Total number of outcomes = 10𝐶4 × 4! OR 10𝑃4 ½
= 5040 ½
No. of favorable outcomes = 1 ½
1
P(the lock will open)= ½
5040

26. (i) Since 𝑎 − 𝑎 = 0 which is divisible by 2 1


Hence (𝑎, 𝑎) ∈ 𝑅
(ii) (𝑎, 𝑏) ∈ 𝑅
⟹ 𝑎 − 𝑏 is divisible by 2
⟹ −(𝑏 − 𝑎) is divisible by 2
⟹ 𝑏 − 𝑎 is divisible by 2 1
⟹ (𝑏, 𝑎) ∈ 𝑅
(iii) (𝑎, 𝑏) ∈ 𝑅 ⟹ 𝑎 − 𝑏 is divisible by 2
(𝑏, 𝑐) ∈ 𝑅 ⟹ 𝑏 − 𝑐 is divisible by 2
On adding above two result we get
𝑎 − 𝑏 + 𝑏 − 𝑐 is divisible by 2 1
⟹ 𝑎 − 𝑐 is divisible by 2
⟹ (𝑎, 𝑐) ∈ 𝑅
OR 1
For real function f 𝑥2 − 4 ≥ 0
⟹ (𝑥 + 2)(𝑥 − 2) ≥ 0 1
⟹ either 𝑥 ≤ −2 or 𝑥 ≥ 2
⟹ 𝐷𝑓 = (−∞, −2] ∪ [2, ∞)
1
27. (sin 4𝑥+sin 2𝑥)+sin 3𝑥
LHS:(cos 4𝑥+cos 2𝑥)+cos 3𝑥 ½
2 cos 𝑥 sin 3𝑥 + sin 3𝑥 𝑠𝑖𝑛3𝑥(2 cos 𝑥 + 1) 1½
= =
2𝑜𝑠 3𝑥 cos 𝑥 + cos 3𝑥) cos 3𝑥(cos 𝑥 + 1)
sin 3𝑥
=cos 3𝑥 = tan 3𝑥 = 𝑅𝐻𝑆 1

28 𝛽−𝛼 ̅
(𝛽−𝛼)𝛽
|1−𝛼̅𝛽|=|(1−𝛼̅𝛽)𝛽̅| 1
̅
(𝛽−𝛼)𝛽
= | 𝛽̅−𝛼𝛽𝛽̅ | , we know that 𝑧𝑧̅ = |𝑧|2 = 1 and ̅̅̅̅̅̅̅̅̅=𝑧
𝑧1 − 𝑧2 ̅1 − 𝑧̅2
2
|𝛽̅ | = 1
OR
1
𝑥 + 𝑖𝑦 = (𝑎 + 𝑖𝑏)3

329
⟹ 𝑥 + 𝑖𝑦 = 𝑎3 + 3𝑎2𝑖𝑏 + 3𝑎𝑖2𝑏2 + 𝑖3𝑏3
= (𝑎3 − 3𝑎𝑏2) + 𝑖(3𝑎2 − 𝑏3) 1
⟹ 𝑥 + 𝑖𝑦 = 𝑎(𝑎2 − 3𝑏2) + 𝑏(3𝑎2 − 𝑏2)
Comparing real and imaginary parts
𝑥 𝑦
= 𝑎2 − 3𝑏2 𝑎𝑛𝑑 = 3𝑎2 − 𝑏2
𝑎 𝑏 1
𝑥 𝑦
− = 𝑎2 − 3𝑏2 − 3𝑎2 + 𝑏2
𝑎 𝑏
= −2𝑎2 − 2𝑏2
= −2(𝑎2 + 𝑏2)
29.
Let slope of required line be m
1
Slope of given line is 2 1
1
𝑚−
2
𝑡𝑎𝑛45° = 𝑚
1+ 2
2𝑚 − 1
= ±1
2+𝑚 1
For positive sign, 𝑚 = 3
1
For negative sign 𝑚 = − 3
Equations of lines are ½
3𝑥 − 𝑦 = 7 ½
𝑥 + 3𝑦 = 9

30. √𝑥 2 +1−√5
lim
𝑥→2 𝑥−2
√𝑥 2 +1−√5 √𝑥 2 +1+√5
= lim ×
𝑥→2 𝑥−2 √𝑥 2 +1+√5
𝑥 2 +1−5
1
= lim
𝑥→2 (𝑥−2)(√𝑥 2 +1+√5)
𝑥 2 −4
= lim 1
𝑥→2 𝑥−2)(√𝑥 2 +1+√5)
𝑥+2 4
= lim = 1
𝑥→2 √𝑥 2 +1+√5 2√5

31. Let 𝑓(𝑥) = 𝑥𝑐𝑜𝑠𝑥


𝑑𝑦 𝑓(𝑥 + ℎ) − 𝑓(𝑥) (𝑥 + ℎ) cos(𝑥 + ℎ) − 𝑥𝑐𝑜𝑠 𝑥 1
= lim = lim
𝑑𝑥 ℎ→0 ℎ ℎ→0 ℎ
𝑥 cos(𝑥+ℎ)−𝑥𝑐𝑜𝑠 𝑥+ℎ𝑐𝑜𝑠 (𝑥+ℎ) 𝑥 cos(𝑥+ℎ)−𝑥𝑐𝑜𝑠 𝑥 ℎ𝑐𝑜𝑠(𝑥+ℎ)
= lim =lim + lim 1
ℎ→0 ℎ ℎ→0 ℎ ℎ→0 ℎ
ℎ ℎ
cos(𝑥+ℎ)−𝑐𝑜𝑠 𝑥 −2 sin(𝑥+ ) sin( )
2 2
= 𝑥 lim + cos 𝑥 = 𝑥 lim + cos 𝑥
ℎ→0 ℎ ℎ→0 ℎ 1
= −𝑥𝑠𝑖𝑛 𝑥 + cos 𝑥

330
32. LHS:
𝜋 3𝜋 3𝜋 𝜋
(1 + cos 8 )(1 + cos 8 )(1 + cos(𝜋 − 8 ))(1+cos (𝜋 − 8 )) 1
𝜋 3𝜋 3𝜋 𝜋
=(1 + cos 8 )(1 + cos 8 )(1 − cos 8 ) (1 − cos 8 )
𝜋 3𝜋 1
=(1 − 𝑐𝑜𝑠 2 ) (1 − 𝑐𝑜𝑠 2 )
𝜋
8
3𝜋
8 1
=𝑠𝑖𝑛2 𝑠𝑖𝑛2
8 8
1 𝜋 3𝜋
= (1 − cos )(1 − cos ) 1
4 4 4
1 1
=4 (1 − 2)(1 + 2)
1 1
1 √ √
=8

OR
𝜋
1−cos 2(𝑥+ ) 1−cos 2(𝑥− )
𝜋 1
1−𝑐𝑜 2𝑥
LHS: + 3
+ 3
1
2 2 2
1 2𝜋 2𝜋
=2 (3 − (cos 2𝑥 + cos(2𝑥 + ) + cos(2𝑥 − )) 1
3 3
1
=2 (3 − (cos 2 + 2 cos 𝑥 cos
2𝜋
)) 1
3 1
1 1
=2 (3 − (cos 2𝑥 − 2 cos 2𝑥 × 2))

1 3
=2 (3 − 0) = 2

33. Let 𝑥 litres of 2% boric acid solution be added to the 8% solution


Therefore total quantity of solution = 𝑥 + 640 litres ½
∴ 2% 𝑜𝑓 𝑥 + 8% 𝑜𝑓 640 > 4% 𝑜𝑓 (640 + 𝑥)

𝑎𝑛𝑑 2% 𝑜𝑓 𝑥 + 8% 𝑜𝑓 640 < 6% 𝑜𝑓 (640 + 𝑥)
2𝑥 8 4
𝑜𝑟 + × 640 > × (640 + 𝑥)
100 100 100 1
⟹ 𝑥 < 1280
2𝑥 8 6
+ × 640 < × (640 + 𝑥) 1
100 100 100 ½
⟹ 𝑥 > 320
∴ 320 < 𝑥 < 1280 ½
We have to add more than 320 litres but less than 1280 litres of 2% solution
to fulfil the given condition.
34. Let two positive numbers be 𝑎 𝑎𝑛𝑑 𝑏
𝑎+𝑏
𝐴= , 𝐺 = √𝑎𝑏
2
𝐴 𝑎+𝑏
= 1
𝐺 2√𝑎𝑏
Applying Componendo and Dividendo
𝐴+𝐺 𝑎+𝑏+2√𝑎𝑏
= 𝑎+𝑏−2√𝑎𝑏
𝐴−𝐺

331
2
𝐴+𝐺 (√𝑎+√𝑏)
= 2
𝐴−𝐺 (√𝑎−√𝑏)
√𝐴+𝐺 √𝑎+√𝑏
=
√𝐴−𝐺 √𝑎−√𝑏
Applying componendo and dividend 1
√𝐴+𝐺+√𝐴−𝐺 √𝑎+√𝑏+√𝑎−√𝑏
=
√𝐴+𝐺−√𝐴−𝐺 √𝑎+√𝑏−√𝑎+√𝑏
√𝐴+𝐺+√𝐴−𝐺 2√ 𝑎
= 2√𝑏
√𝐴+𝐺−√𝐴−𝐺
Squaring both sides
2𝐴+2√𝐴+𝐺√𝐴−𝐺 𝑎 1
=𝑏
2𝐴−2√𝐴+𝐺√𝐴−𝐺
𝐴+√𝐴+𝐺√𝐴−𝐺 𝑎
=𝑏
𝐴−√𝐴+𝐺√𝐴−𝐺

Let 𝑎 = 𝑘(𝐴 + √𝐴 + 𝐺√𝐴 − 𝐺)


𝑏 = 𝑘(A−√𝐴 + 𝐺√𝐴 − 𝐺)
1

Put values of 𝑎 𝑎𝑛𝑑 𝑏 𝑖𝑛 𝐺 = √𝑎𝑏

𝐺 = √𝑘(𝐴 + √𝐴 + 𝐺√𝐴 − 𝐺)𝑘(A − √𝐴 + 𝐺√𝐴 − 𝐺)

𝐺 = 𝑘√𝐴2 − 𝐴2 + 𝐺 2
1
We get 𝑘 = 1
Therefore, numbers are 𝐴 ± √𝐴 + 𝐺√𝐴 − 𝐺)

35. Class Freq. Class 𝑥𝑖 − 105 𝑦𝑖2 𝑓𝑖𝑦𝑖 𝑓𝑖𝑦2


𝑖
Mark 𝑦𝑖 = 30
0 – 30 2 15 −3 9 −6 18
30 – 60 3 45 −2 4 −6 12
60 – 90 5 75 −1 1 −5 5
90 – 120 10 105 0 0 0 0
120 – 150 3 135 1 1 3 3
150 – 180 5 165 2 4 10 20
180 – 210 2 195 3 9 6 18 2
Total 30 2 76

∑𝑛
𝑖=1 𝑓𝑖 𝑥𝑖
𝑥̅ = 𝐴 + ∑𝑛
×ℎ 1
𝑖=1 𝑓𝑖
2
𝑥̅ = 105 + × 30 = 107
30
ℎ2 1
Var=𝜎 2 = {𝑁 ∑𝑛𝑖=1 𝑓𝑖 𝑦𝑖 2 − (∑𝑛𝑖=1 𝑓𝑖 𝑦𝑖 )2 }
𝑁2

302
𝜎2 = (30 × 76 − 22 )
302 1

𝜎2 = 2280 − 4 = 2276
OR

332
Let other two numbers be 𝑥 and 𝑦. ½
1+2+6+𝑥+𝑦
Mean (𝑥̅ ) = 5
𝑥 + 𝑦 = 13 − − − (1) ½
Variance =8.24

1
=𝑛2 (𝑛 ∑𝑛𝑖=1 𝑥 2 −
(∑𝑛𝑖=1 𝑥)2 )
1 1 ½
= (5(12 + 22 + 62 +
25
𝑥 2 + 𝑦 2 ) − 222 )
206 = 5(41 + 𝑥2 + 𝑦2) − 484
𝑥2 + 𝑦2 = 97 − − − (2) 11/2
From (1) and (2) and using(𝑥 + 𝑦)2 = 𝑥2 + 𝑦2 + 2𝑥𝑦
132 = 97 + 2𝑥𝑦
2𝑥𝑦 = 72 1
(𝑥 − 𝑦)2 = 𝑥2 + 𝑦2 − 2𝑥𝑦
(𝑥 − 𝑦)2 = 97 − 72
𝑥 − 𝑦 = ±5 − − − (3)
On solving (1) and (3)
Numbers are 4 and 9
36. 36.1 Since the team will not include no girl
∴ selection be made from boys only
1
∴ number of selections = 7𝐶5 = 21
36.2 To select at least 3 girls, 3 or 4 girls can be selected 1
∴ required number of selections 4𝐶3 × 7𝐶2 + 4𝐶4 × 7𝐶1 = 91
36.3 When at least 1 boy and 1 girl be selected
Number of selections = 4𝐶1 × 7𝐶4 + 4𝐶2 × 7𝐶3 + 4𝐶3 × 7𝐶2 + 4𝐶4 × 7𝐶1 1
= 140 + 210 + 84 + 7
1
= 441
37 A : Event that the student opted NCC.
B : Event that the student opted NSS.
30 32 24
∴ 𝑃(𝐴) = , 𝑃(𝐵) = , 𝑃(𝐴 ∩ 𝐵) = 1
60 60 60
37.1 𝑃(𝐴 ∪ 𝐵) = 𝑃(𝐴) + 𝑃(𝐵) − 𝑃(𝐴 ∩ 𝐵)
30 32 24 19 1
= + − =
60 60 60 30
37.2 P(student opted neither NCC nor NSS)
= 𝑃(𝐴′ ∩ 𝐵′) = 𝑃((𝐴 ∪ 𝐵)′)
= 1 − 𝑃(𝐴 ∪ 𝐵)
19 11
= 1 − 30 == 30 1

37.3 P(student opted NSS but not NCC)


= 𝑃(𝐵 − 𝐴)
32 24 2
= 𝑃(𝐵) − 𝑃(𝐴 ∩ 𝐵)=60 − 60 = 15
1

333
38
C Y B

X
X’ O N R

L A 30 m Q M

Y’

38.1 Let equation of main cable be 𝑥2 = 4𝑎𝑦 1


OA = NQ = BM = 5 m
BR = BM – RM = 30 – 5 = 25 m
Hence coordinates of B (100, 25)
B lies on the parabola 1
1002 = 4𝑎 × 25
𝑎 = 100
Therefore equation of main cable 𝑥2 = 400𝑦

38.2 Let PN = ℎ

Therefore coordinates of P (30, ℎ)


1
P(30, ℎ) lies on parabola

∴ 302 = 400ℎ
9
ℎ = 4=2.25

∴ PQ = 2.25 + 5 = 7.25 m 1
Ans: Length of supporting cable 30 m from the midway is 7.25 m

334
BLUE PRINT SAMPLE PAPER 5

SNO UNIT/TOPIC SECTION‘A’ SECTION SECTION‘C SECTION’ SECTION‘ TOTA UNI


(OBJECTIVETY ‘B’ ’ D’ E’ L TWI
PE) (VSA) (SA) (LA) (CBQ) SE
MA
RKI
NG
MCQ A&R (2M (3Ma (5M (4M
(1Mar (1Mar arks rks arks arks
kEach) kEach) Each Each) Each Eac
) ) h)
1 SETS 1(1) 1(5) 1(4) 3(10) 23 M
2 RELATIONSANDFUNCTIONS 2(2) 2(2) 4(6)
3 TRIGONOMETRICFUNCTIO 1(1) 1(1) 1(5) 3(7)
NS
4 COMPLEXNUMBERS 1(1) 1(2) 3(3) 25M
5 LINEAR INEQUALITIES 1(1) 1(3) 2(4)
6 PERMUTATIONS 1(3) 1(4) 2(7)
AND
COMBINATIONS
7 BINOMIALTHEOREM 2(2) 1(2) 3(4)
8 SEQUENCEAND SERIES 2(2) 1(5) 3(7)
9 STRAIGHTLINES 1(1) 1(3) 2(4) 12 M
10 CONIC SECTION 2(2) 1(3) 3(5)
11 3DIMENSIONALGEOMETRY 1(3) 1(3)
12 LIMITSANDDERIVATIVES 2(2) 1(1) 1(2) 1(3) 5(8) 08M
13 STATISTICS 1(1) 1(5) 2(6) 12M
14 PROBABILITY 2(2) 1(4) 3(6)
TOTAL 18(18) 2(2) 5(10) 6(18) 4(20) 3(12) 36(8 80
0)

335
SAMPLE PAPER - 5
Time: 3 hours Maximum marks 80
---------------------------------------------------------------------------------------------------------------------------------
General Instructions:
a) The question paper contains- five sections A, B, C, D and E. Each section is compulsory. However, there are
internal choices in some questions.
b) Section A has 18 MCQ’s and 02 Assertion- Reason based questions of 1 mark each.
c) Section B has 5 very short Answer (VSA)-type questions of 2 mark each.
d) Section C has 6 Short Answer (SA)-type questions of 3 marks each.
e) Section D has 4 Long Answer (LA)-type questions of 5 mark each.
f) Section E has 3 source based/ case based/ passage/ integrated units of assessment of 4 marks each with sub-
parts.
Section-A

336
1. If Let n(A)=m, n(B)=n, then the total number of non-empty relations that can be defined 1
from A to B is:
(A)𝑚𝑛 (B)𝑛𝑚 - 1 (C)2𝑚𝑛 -1 (D)None of these
2. A line passes through (2,2) and is perpendicular to the 3𝑥 + 𝑦 = 3.Find its y-intercept. 1
(A)4/3 (B) 1 (C) 1/3 (D) 2/3

𝒙
3. The domain of the function 𝒇(𝒙) = is ___________ 1
𝒙𝟐 +𝟑𝒙+𝟐
(a) [-2, -1] (b) R – {1, 2} (c) R – {-1, -2} (d) R – {2}

4. If |x+3|≥10 then 1
(a) x ∈ (-13,7] (b) x ∈(-10,7] (c) x∈ (-∞, −7) ∪ [13, ∞ ) (d) x ∈ (-∞, −13] ∪ [7, ∞ )
5. One card is drawn from a pack of 52 cards. The probability that it is the card of king or 1
spade is_____________
4 1 17 1
(a)13 (b13 (c)52 (d) 26
6. The standard deviation of the observation 5,5,5,5,5 is- 1

(a) 0 (b) 5 (c)25 (d) 20


7. If 8x+i(2x-y) = 3-8i and x , y ∈ R , then values of x and y are 1
(a) x=3/8,y=35/4 (b) x=-3/8,y=35/4 (c) x=3/8,y=-35/4 (d) x=-3/8,y=-35/4
8. 3 1 1
If f(x) = 𝑥 − 𝑥 3 then the value of f(𝑥)+ f(𝑥) is equal to
2
(a) 𝑥 3 (b) 2𝑥 3 (c) 1 (d)0

9. 1+7𝑖 1
If z=(2−𝑖)2 then these |𝑧| equals to __________
1 1
(a)√2 (b) (c) (d) 2
√2 2

10. The total number of terms in the expansion of (𝑥 + 𝑎)51 -(𝑥 − 𝑎)51 1
(a) 23 (b) 24 (c) 26 (d) 28
11 A pair of die is rolled , the probability of getting a total of 6: 1
(a) 4/36 (b) 5/36 (c) 5/18 (d) 1/18

12 The third term of the expansion (1 − 2𝑥 )5 is 1


a)-40𝑥 2 b) 40𝑥 2 c) 40x d) none of these

13 Which of the following is an example of a geometric sequence? 1

(a) 1,2,3,4 (b) 1,2,4,8 (c) 3,5,7,9 (d)9,20,21,28

14 𝑥 7 −𝑎 7 1
lim = 7 the the value of a is :
𝑥→𝑎 𝑥−𝑎
(a) 1 (b) -1 (c) 0 (d)±1

337
15 What is the angle through which a pendulum swings if its length is 75 cm and the tip 1
describe an arc of length 10cm
(a) 2/15 radian (b) 3/15 radian (c) 1/15 radian (d) 4/15radian
16 The length of transverse axis hyperbola is the distance between the ….. 1

(a) Two vertices (b) two foci (c) vertex and the origin (d) focus and the vertex
17 The derivative of 1/(1-x) w.r.t. x is : 1
(a) 1/(1-x)2 (b) -1/(1-x)2 (c) 2/(1-x)2 (d)none of these
18 Find the equations of the directrix & the axis of the parabola ⇒3𝑥 2 =8y 1

(a) 3y-4=0, x=0 (c) 3x-2=0, x=0


(b) 3y-4x=0 (d) none of these
Q. No 19 and 20 based on statement of Assertion (A) is followed by a statement of Reason
(R). choose the correct answer out of the following choices.
A) Both A and R are true and statement R is the correct explanation for A
B) Both A and R are true and but R is the not correct explanation for A.
C) A is true but R is false
D) A is false but R is true
19 Assertion(A) :If the third term of a G.P is 4,then the product of its first five term is 45 . 1
Reason (R) : The product of first five terms of a G.P is given as 𝑎(𝑎𝑟)(𝑎𝑟 2 )(𝑎𝑟 3 )(𝑎𝑟 4 )
20 Assertion (A) : The value of sin(-690°) cos(-300°) + cos (-750°)sin (-240°) = 1 1
Reason (R) : The value of sin and cos is negative in third and fourth quadrant
respectively.
SECTION:B

21 Let A={1,2} and B={3,4} Write A×B.How many subsets will A×B Have? 2
OR
Let 𝐴 = {𝑥: 𝑥 = 6𝑛, 𝑛 ∈ 𝑁} , 𝐵 = {𝑥: 𝑥 = 9𝑛, 𝑛 ∈ 𝑁}.find 𝐴 ∩ 𝐵

𝑛
22 Prove that ∑𝑛𝑟=0 3𝑟 𝐶𝑟 = 4n 2

23 Express (1-2i)-3 in the standard form. Also find its conjugate. 2


OR
𝑎−𝑖𝑏 1+𝑖
If 𝑎+𝑖𝑏 = 1−𝑖 , then find the value a+b.

24 sin 5𝑥 2
Write the value of lim
𝑥→0 sin 7𝑥

25 Draw the graph of f(x) = I x-2I. 2

SECTION:C
26 Find the centroid of a triangle, mid-points of whose sides are D(1,2,−3), E(3,0,1) 𝐹(−1,1,−4) 3

27 4𝑥+5𝑠𝑖𝑛𝑥 3
Find the derivative of 3𝑥+7𝑐𝑜𝑠𝑥

28 Solve the following system of inequalities: 3


2𝑥 − 3 4𝑥
2(2𝑥 + 3) − 10 < 6(𝑥 − 2), +6≥4+
4 3

338
29 Find the equations of the straight lines which pass through the origin and trisect the 3
portion of the straight line 2𝑥 + 3𝑦 = 6 which intercepted between the axes.
OR
Points A and B have coordinates (3,2), (7,6) respectively. Find
(i) The equation of right bisector of the segment AB.

(ii) The value of p if (−2, 𝑝) lies on it.


30 Find the point in XY-plane Which is equidistant from the points (2,0,3), (0,3,2) and (0,0,1). 3

31 A committee of 7 has to be formed from 9 boys and 4 girls. in how many ways this can be 3
done when the committee consists of at least 3 girls.
i. exactly 3 girls?
ii. at least 3 girls?
iii. almost 3 gilrs?

OR
If sin A + sin B=p , cos A + cos B =q , find cos(A-B)

SECTION:D

32 𝜋 𝜋 3 5
Prove that: 𝑐𝑜𝑠 2 𝑥 + 𝑐𝑜𝑠 2 (𝑥 + 3 ) + 𝑐𝑜𝑠 2 (𝑥 − 3 ) = 2

OR

4 5 𝜋
If cos(𝛼 + 𝛽) = and sin(𝛼 − 𝛽) = and 0≤ 𝛼, 𝛽 ≤ then find tan2 𝛼 .
5 13 4

33 For sets A, B and C using properties of sets , prove that: 5


(i) 𝐴 − (𝐵 ∪ 𝐶) = (𝐴 − 𝐵) ∩ (𝐴 − 𝐶)
(ii) 𝐴 − (𝐵 − 𝐶) = (𝐴 − 𝐵) ∪ (𝐴 ∩ 𝐶)
34 Find three numbers in G.P. whose sum is 13 and the sum of whose square is 91. 5
OR
If S be the sum ,P be the product and R be the sum of reciprocals of n terms of a G.P. then
𝑆
prove that (𝑅 )𝑛 =𝑃2

35 Find the variance and standard deviation for the following distribution: 5
Class 30-40 40-50 50-60 60-70 70-80 80-90 90-100
frequency 3 7 12 15 8 3 2

SECTION:E
[This section comprises of 3 case- study/passage based questions of 4 marks each

339
36
A Mathematics teacher of class XI write three sets A, B and C are such that
A= { x∈ N | x is an even number , x ≤10 }
B = { y∈ N | y is prime number , y≤ 10 }
C = {z ∈ N | z is divisible by 3 , z ≤ 10 }
Based on the above information answer the following

i) Among the set A, B and C which set contain maximum number of elements
ii) Write the number of element present in (A U C)
iii) Write the set B in tabular form.
iv) Write the set(A-B) in tabular form.
1
1
1
1
37 Each zone is allotted a specific non-zero digit which is to be used as first digit of
alltelephone numbers of that zone.Based on the above information, answer the
following questions.

Q . (i) How many different telephone numbers are there in each zone, if the digit on 1
first places not used again?
1
Q. (ii) How many different telephone numbers are there in the city, if there is no
restriction?
2
Q (iii) How many different telephone numbers are there in each zone with all digits
distinct?

340
38
Some friends went on a picnic and after moving round and having light snacks, they sat down
and started playing with a pair of dice each time they throw the dice they asked other friends
about the chance of getting the event which they have thought. You also try to help them
out.

(i) Find the probability of getting a number less than 3 on first dice. 1
(ii) Find the probability of getting an even total of numbers on two dice. 1
(iii) Find the probability of getting a number multiple of 3 on both dice. 2

OR
(III) Find the probability of getting a doublet.

341
MARKING SCHEME [SAMPLE PAPER 5]

Q.No. ANSWERS STEP


MARKS

1 (C) 1

2 (A) 1

3 ( C) 1

4 (D) 1

5 (A) 1

6 (A) 1

7 (B) 1

8 (D) 1

9 (D) 1

10 (C) 1

11 (B) 1

12 (B) 1

13 (B) 1

14 (D) 1

15 (A) 1

16 (A) 1

17 (A) 1

18 (D) 1

19 (A) 1

20 (C) 1

342
Section-B
21 A×B={(1,3), (1,4), (2,3), (2,4)} (1)
Since,
No. Of subsets of A×B is 24 =16 (1)
OR
𝐴= {6,12,18,24, }
… and 𝐵 = {9,18,27,36, …}
𝐴 ∩ 𝐵 = {18,36,54, … } (1)
(1)

22 30𝐶(𝑛, 0) + 31 𝐶(𝑛, 1) + 32 𝐶(𝑛, 2) + ⋯ + 3𝑛 𝐶(𝑛, 𝑛)=(1+3)n=4n (1+1)

23 =1/(1-2i)3=1/(-11+2i) (1)
−11 2 (1/2)
......= 125 – 125 i
−11 2
Conjugate is + i (1/2)
125 125
OR
𝑎 2 −𝑏2 2𝑎𝑏
By rationalising both sides 2 2 − 2 2 𝑖 = 0+i (1)
𝑎 +𝑏 𝑎 +𝑏
⇒ a²-b²=0→ 𝑎 = ±𝑏 (1/2)
2𝑎𝑏
But − 2 2=1 => 𝑎 = −𝑏 =>a+b=0
𝑎 +𝑏 (1/2)

24 sin 5𝑥 (1)
lim sin 7𝑥
𝑥→0 (1)
𝑠𝑖𝑛5𝑥 7𝑥 1
= lim × 5𝑥 × ×
𝑥→0 5𝑥 sin 7𝑥 7𝑥

sin 5𝑥 5 sin 𝑥
lim = 7 ( lim = 1)
𝑥→0 sin 7𝑥 𝑥→0 𝑥

25 f(x)= {𝑥 − 2, 𝑥 ≥ 1 2 − 𝑥, 𝑥 < 1 (1)


for Correct graph
(1)

343
26
𝐴(𝑥1 , 𝑦1 , 𝑧1 ), 𝐵(𝑥2 , 𝑦2 , 𝑧2 )𝑎𝑛𝑑 𝐶(𝑥3 , 𝑦3 , 𝑧3 )

D is the mid-point of BC
𝑥2 +𝑥3 𝑦2 +𝑦3 𝑧2 +𝑧3
= 1, = 2, = −3
2 2 2 1

𝑥2 + 𝑥3 = 2, 𝑦2 + 𝑦3 = 4, 𝑧2 + 𝑧3 = −6 --------(i)

E is the mid-point of CA

So, 𝑥1 + 𝑥3 = 6, 𝑦1 + 𝑦3 = 0, 𝑧1 + 𝑧3 = 2 -------(ii) 1

F is the mid-point of AB

So, 𝑥1 + 𝑥2 = −2, 𝑦1 + 𝑦2 = 2, 𝑧1 + 𝑧2 = −8 -------(iii)

By adding the three equations , we get


1

𝑥1 + 𝑥2 + 𝑥3 = 3, 𝑦1 + 𝑦2 + 𝑦2 = 3, 𝑧1 + 𝑧2 + 𝑧3 = −6

Coordinate of centroid(1,1,−2)

27 The given function is f(x) = (4x + 5 sin x) / (3x + 7 cos x)


Its derivative using the is,
1
𝑑 𝑑
𝑑 (3x + 7 cos x) (4x + 5 sin x)−(4x + 5 sin x) (3x + 7 cos x)
𝑑𝑥 𝑑𝑥
(f(x)) = (3x + 7 cos x)2
=
𝑑𝑥
35 + 15x cos x + 28x sin x – 15 sin x 2
(3x + 7 cos x)2
[As cos2x + sin2x = 1]

28 solving 1st inequality x>4 (1)


and 2nd equality x≤3/2 (1) (1)
Solution impossible (1) (1)
29

A(3,0) and B( 0,2) 1


The line 2𝑥 + 3𝑦 = 6 trisect by points P and Q
Therefore , AP=PQ=QB
P(2,2,3) and Q(1,4,3)
344
Clearly ,P and Q lie on line 𝑦 = 𝑚1 𝑥 and 𝑦 = 𝑚2 𝑥 respectively.
1 4 1
𝑦 = 𝑥 and 𝑦 = 𝑥
3 3
So, 𝑥 − 3𝑦 = 0 𝑎𝑛𝑑 4𝑥 − 3𝑦 = 0 1
OR
( I ) the mid point of AB=(5,4)
Line passes through mid-point of AB is perpendicular to AB 1
So,( slope of AB)( slope of line passes through mid point of AB)=−1
slope of AB=1, slope of line passes through mid point of AB=−1
equation of right bisector of AB is 𝑦 − 4 = −1(𝑥 − 5) 1
𝑥+𝑦 =9
( ii ) (−2, 𝑝) lies on 𝑥 + 𝑦 = 9
There fore 𝑝 = 11 1

30 Let the Point in XY Plane (x, y,o) is equidistant from A, B & C (1/2)
PB=PC ⇒ y= 2. (1)
PA=PC ⇒ x= 3. (1)
The point is A(3,2,0) (1/2)

31 No. of ways of selection=C(9,4)XC(4,3)=504 1


No. of ways of selection=C(4,3)XC(9,4)+C(4,4)XC(9,3)
=504+84=588 1
We have to select at most 3 girls. So, number of ways of selection
=C(4,0)XC(9,7)+C(4,1)XC(9,6)+C(4,2)XC(9,5)+C(4,3)XC(9,4)+C(4,4)XC(9,3)=
36+336+756+504=1632 1

345
32. = ½ (2cos2x+2cos2(x+π/3) +2cos²(x-π/3)) (1)
= (1/2)(1+cos2x + 1+ cos(2x+2π/3) + 1+ cos(2x-2π/3)) (1)
=(1/2)(3+ cos2x + cos(2x+2π/3) + cos(2x-2π/3)) (1)
=(1/2)(3+cos2x+2cos2x.cos2π/3) (1)
= 3/2 (1)
OR
3 5
𝑡𝑎𝑛 𝑡𝑎𝑛(𝐴 + 𝐵) = 4 , 𝑡𝑎𝑛 𝑡𝑎𝑛(𝐴 − 𝐵) = 12

𝑡𝑎𝑛 𝑡𝑎𝑛2𝐴 =𝑡𝑎𝑛 tan( (𝐴 + 𝐵) + (𝐴 − 𝐵)) (1)


1
𝑡𝑎𝑛 𝑡𝑎𝑛(𝐴 + 𝐵) +𝑡𝑎𝑛 tan(𝐴 − 𝐵) (12)
𝑡𝑎𝑛 𝑡𝑎𝑛2𝐴 =
1 −𝑡𝑎𝑛 𝑡𝑎𝑛(𝐴 + 𝐵) 𝑡𝑎𝑛 tan(𝐴 − 𝐵)
3 5
+ (1)
= 4 12
3 5 1
1− × (12)
4 12
56
=
33
33. ( 𝑖) 𝐴 − (𝐵 ∪ 𝐶) = 𝐴 ∩ (𝐵 ∪ 𝐶)′ (1)
= 𝐴 ∩ (𝐵′ ∩ 𝐶 ′ ) = (𝐴 ∩ 𝐵′ ) ∩ (𝐴 ∩ 𝐶 ′ ) (1)
= (𝐴 − 𝐵) ∩ (𝐴 − 𝐶) (1/2)
(II)
𝐴 − (𝐵 − 𝐶) = 𝐴 − (𝐵 ∩ 𝐶 ′ )
= 𝐴 ∩ (𝐵 ∩ 𝐶 ′ )′ = 𝐴 ∩ (𝐵′ ∪ 𝐶) = (𝐴 ∩ 𝐵′ ) ∪ (𝐴 ∩ 𝐶) (1)
= (𝐴 − 𝐵) ∪ (𝐴 ∩ 𝐶) (1)
(1/2)

346
34 Numbers in G.P are 𝑎, 𝑎𝑟 𝑎𝑛𝑑 𝑎𝑟 2
Given sum =13
𝑎 + 𝑎𝑟 + 𝑎𝑟 2 = 13 ---------(i) (1)
𝑎2 + 𝑎2 𝑟 2 + 𝑎2 𝑟 4 = 91
𝑎2 (1 + 𝑟 2 + 𝑟 4 ) = 91 (1)
𝑎2 (1 − 𝑟 + 𝑟 2 )𝑎(1 + 𝑟 + 𝑟 2 ) = 91

𝑎(1 − 𝑟 + 𝑟 2 )𝑎(1 + 𝑟 + 𝑟 2 ) = 91

91
𝑎(1 − 𝑟 + 𝑟 2 ) = 13 = 7 ------(ii)
By Dividing (i) by (ii), we get
𝑎+𝑎𝑟+𝑎𝑟 2 13 (1)
=
𝑎(1−𝑟+𝑟 2 ) 7
7(1 + 𝑟 + 𝑟 = 13(1 − 𝑟 + 𝑟 2 )
2)

6𝑟 2 − 20𝑟 + 6 = 0
(3𝑟 − 1)(𝑟 − 3) = 0
1
𝑟 = 3 𝑜𝑟 𝑟 = 3 (1)
When, 𝑟 = 3, 𝑎 = 1
1
When, 𝑟 = 3 , 𝑎 = 9
(1)
So, the numbers are 1,3 and 9 or 9,3 and 1

OR (1)
𝑎(𝑟 𝑛 −1)
S =Sum=𝑎 + 𝑎𝑟 + 𝑎𝑟 2 + ⋯ 𝑛 𝑡𝑒𝑟𝑚𝑠 = (𝑟−1)
(1)
𝑛(𝑛−1) (1)
P= Product =𝑎. 𝑎𝑟. 𝑎𝑟 2 … 𝑛 𝑡𝑒𝑟𝑚𝑠 = 𝑎𝑛 𝑟 2
1 1
1 1 1 (1−( )𝑛 )
R=Reciprocal=𝑎 + 𝑎𝑟 + 𝑎𝑟2 + ⋯ 𝑛 𝑡𝑒𝑟𝑚𝑠 = 𝑎 𝑟
1 =
(1− )
𝑟
1 𝑟 𝑛 −1 (2)
= 𝑎𝑟𝑛−1 { 𝑟−1 }
(S/R)n = (a2rn-1)n =.......=.P2

347
35 (3)
2 2
Class Frequency Mid fi xi (x i – 𝑥 ) fi (x i – 𝑥 )
(f i ) point
(x i)
30-40 3 35 105 729 2187
40-50 7 45 315 289 2023
50-60 12 55 660 49 588
60-70 15 65 975 9 135
70-80 8 75 600 169 1352
80-90 3 85 255 529 1587
90-100 2 95 190 1089 2178

50 3100 10050
Mean = 62 (½)

Standard Deviation = 𝜎 =√201 = 14.18 (1)

Variance = 𝜎 2 = 201 (½)

348
SECTION-E

36 i) A set contain maximum number of element


ii) n (A U C) =7 (1)
iii) B ={ 2,3,5,7 }. (1)
iv) ( A-B) = { 4,6,8,10 }
(1)
(1)
37 i) 10×96 different telephone numbers are their in each zone,if the digit
on the first place is not used again. If zero is not to be used on the first
(1)
place then number of ways are 97 .
different telephone numbers are there in the city, if there is no restriction. (1)
ii) Total number of ways are 604,800.
(2)

38 (1,1)(1,2)(1,3)(1,4)(1,5)(1,6)
(2,1)(2,2)(2,3)(2,4)(2,5)(2,6)
(3,1)(3,2)(3,3)(3,4)(3,5)(3,6)
(4,1)(4,2)(4,3)(4,4)(4,5)(4,6)
(5,1)(5,2)(5,3)(5,4)(5,5)(5,6)
(6,1)(6,2)(6,3)(6,4)(6,5)(6,6)
n(E) – 36
(I) Events for less than 3 on the first dice
(1,1)(1,2)(1,3)(1,4)(1,5)(1,6)(2,1)(2,2)(2,3)(2,4)(2,5)(2,6)
1
n (E)-12
Probability - 12/36. =1/3
(II)n(E)
(1,1)(1,3)(1,5)(2,2)(2,4)(2,6)(3,1)(3,3)(3,5)(4,2)(4,4)(4,6)(5,1)(5,3)(5,5)(6,2)(6,4)(6,6)
n(E)- 18
Probability - 18/36 = ½
(iii) 3,3)(3,6)(6,3)(6,6) 1
n(E)- 4
probability - 4/36 = 1/9
Or 2
n(E)- 6
probability =6/36=1/6

349
Class Room Art & Craft Room Dance Room

Indoor Sport Room Music Instrumental Room Computer Lab

Library Maths Lab Biology Lab

Chemistry Lab Physics Lab Joy Park

Prof. Ram Meghe Square, New Express Way, Amravati-Badnera Road.


Class XI Session 2024-25
Subject - Mathematics
Sample Question Paper - 6

Time Allowed: 3 hours Maximum Marks: 80

General Instructions:

1. This Question paper contains - five sections A, B, C, D and E. Each section is compulsory. However, there are

internal choices in some questions.

2. Section A has 18 MCQ’s and 02 Assertion-Reason based questions of 1 mark each.

3. Section B has 5 Very Short Answer (VSA)-type questions of 2 marks each.

4. Section C has 6 Short Answer (SA)-type questions of 3 marks each.

5. Section D has 4 Long Answer (LA)-type questions of 5 marks each.

6. Section E has 3 source based/case based/passage based/integrated units of assessment (4 marks each) with sub
parts.

Section A
1. If cot θ = 1

2
and sec ϕ = −5

3
, where θ lies in quadrant III and ϕ lies in quadrant II, then tan (θ + ϕ) = ? [1]

a) −6

11
b) 5

11

c) 2

11
d) 10

11

2. Let A and B be two sets containing four and two elements respectively.Then,the number of subsets of A×B,each [1]
having at least three elements is:

a) 256 b) 219

c) 510 d) 275
3. Two dice each numbered from 1 to 6 are thrown together. Let A and B be two events given by [1]
A: even number on the first die
B: number on the second die is greater than 4
What is the value of P(A ∪ B)?

a) b)
1 1

4 2

c) 1

6
d) 2

4. lim
sin x
is equal to [1]
x→0 √x+1 −√1−x

a) 1 b) 0

c) 2 d) -1
5. The reflection of the point (4, -13) about the line 5x + y + 6 = 0 is [1]

a) (1, 2) b) (0, 0)

Page 1 of 16
c) (3, 4) d) (-1, -14)
6. The set A = {x : x is a positive prime number less than 10} in the tabular form is [1]

a) {2, 3, 5, 7} b) {1, 2, 3, 5, 7}

c) {3, 5, 7} d) {1, 3, 5, 7, 9}
7. If z = 1
, then |z| = [1]
(1−i)(2+3i)

−−
a) 1 b) 1/√26

−− −−
c) 4/√26 d) 5/√26

8. The minimum value of sin x + cos x is [1]


– –
a) −2√2 b) √2


c) 0 d) − √2

[1]
3(x−2)
9. Solve the system of inequalities: −15 < 5
≤ 0

a) -13 < x < 13 b) -23 < x ≤ 2

c) -23 < x < 23 d) -13 < x < 2


10. cosec (-1110°) = ? [1]
−2
a) -2 b) √3

c) 2 d) √3
2

11. The number of subsets of a set containing n elements is [1]

a) 2n-1 b) 2n - 2

c) 2n d) n
1 1 1

12. If a, b, c are in GP and a x = b y


= c z then x, y, z are in [1]

a) GP b) AP

c) H.M. d) HP
13. {C0 + 3C1 + 5C2 + ...+ (2n + 1)Cn} = ? [1]

a) (n - 2) . 2n-2 b) (n - 1) (n + 2)

c) (n + 2) . 2n-1 d) (n + 1)2n

14. The solution set for (x + 3) + 4 > -2x + 5: [1]

a) (- ∞, 2) b) (
−2
, ∞)
3

c) (- ∞, - 2) d) (2, ∞)

15. If A and B are two given sets , then A ∩ (A ∩ B)c is equal to [1]

a) B b) A

c) A ∩ Bc d) ϕ

16. cos θ + sin( 270



+ θ) − sin( 270

− θ) + cos( 180

+ θ) is equal to [1]

a) 2 cos θ b) 0

Page 2 of 16
c) 2 sin θ d) 1
17. If z is any complex number, then z−z̄

2i
is [1]

a) either 0 or purely imaginary b) purely imaginary

c) purely real d) either 0 or purely real


18. In how many ways can the letters of the word 'APPLE' be arranged? [1]

a) 90 b) 6

c) 120 d) 60

19. Assertion (A): The expansion of (1 + x)n = n c0 + nc x + nc x


2
… + nc x
n
. [1]
1 2 n

Reason (R): If x = -1, then the above expansion is zero.

a) Both A and R are true and R is the correct b) Both A and R are true but R is not the
explanation of A. correct explanation of A.

c) A is true but R is false. d) A is false but R is true.


20. Assertion (A): The proper measure of dispersion about the mean of a set of observations i.e. standard deviation [1]
is expressed as positive square root of the variance.
Reason (R): The units of individual observations xi and the unit of their mean are different that of variance.

Since, variance involves sum of squares of (x - x̄).

a) Both A and R are true and R is the correct b) Both A and R are true but R is not the
explanation of A. correct explanation of A.

c) A is true but R is false. d) A is false but R is true.


Section B
21. Find the domain of the relation, R = {(x, y): x, y ∈ z, y = 4} [2]
OR
2
x −16
Find the domain and the range of the real function: f(x) = x−4

22. Differentiate sin3 x cos3 x w.r.t x [2]


23. Find the equation of the hyperbola, referred to its principal axes as axes of coordinates, in the following cases: [2]
Vertices at (± 5, 0), Foci at (± 7, 0).
OR
Find the vertex, focus, axis, directrix and latus-rectum of the following parabolas y2 - 4y + 4x = 0
24. Write E = {14, 21, 28, 35, 42, ..., 98} in set-builder form. [2]
25. The intercept cuts-off by a line from y-axis is twice than that from x-axis and the line passes through the point [2]
(1, 2). Find the equation of the line.
Section C
26. Let A = {1, 2, 3, 4}, B = {1, 4, 9, 16, 25} and R be a relation defined from A to B as, R = {(x, y) : x ∈ A, y ∈ B [3]

and y = x2}
i. Depict this relation using arrow diagram.
ii. Find domain of R.
iii. Find range of R.
iv. Write co-domain of R.

Page 3 of 16
27. Solve the following system of linear inequations: [3]
3x - 6 ≥ 0
4x - 10 ≤ 6
28. Show that the points A(4, 6, - 3), B(0, 2, 3) and C(-4, - 4, -1) form the vertices of an isosceles triangle. [3]
OR
Given that ,P(5, 4, - 2), Q(7, 6, -4) and R(11, 10, -8) are collinear points. Find the ratio in which Q divides PR.

29. Using binomial theorem, expand: (√x − √y) 3 3 6
[3]
OR

Find the coefficient of x5 in the product (1 + 2x)6 (1 -x)7 using binomial theorem.
30. Find the square roots: 7 - 24i. [3]
OR

If (x + iy)1/3 = a + ib, where x, y, a, b ∈ R, then show that = - 2 (a2 + b2).


x y

a b

31. Let A = {a, e, i, o, u}, B = {a, d, e, o, v) and C = {e, o, t, m] . Using Venn diagrams, verify that: A ∪ (B ∩ C) = [3]
(A ∪ B) ∩ (A ∪ C)
Section D
32. 20 cards are numbered from 1 to 20. One card is drawn at random. What is the probability that the number on [5]
the card drawn is,
i. A prime number
ii. An odd number
iii. A multiple of 5
iv. Not divisible by 3.
4 3

33. Solve: lim


x −3x +2

3 2
[5]
x→1 x −5x +3x+1

OR
Differentiate log sin x from first principles.
34. Find four numbers in GP, whose sum is 85 and product is 4096. [5]
35. Prove that cos 2π

15
⋅ cos

15
⋅ cos

15
⋅ cos
16π

15
=
1

16
[5]
OR
If α, β are two different values of x lying between 0 and 2π which satisfy the equation 6 cos x + 8 sin x = 9, find the
value of sin (α + β)
Section E
36. Read the following text carefully and answer the questions that follow: [4]
Arun is running in a racecourse note that the sum of the distances from the two flag posts from him is always 10
m and the distance between the flag posts is 8 m.

i. Path traced by Arun represents which type of curve. Find the length of major axis? (1)
ii. Find the equation of the curve traced by Arun? (1)

Page 4 of 16
iii. Find the eccentricity of path traced by Arun? (2)
OR
iv. Find the length of latus rectum for the path traced by Arun. (2)
37. Read the following text carefully and answer the questions that follow: [4]
A teacher conducted a surprise test of Mathematics, Physics and Chemistry for class XI on Monday.
The mean and standard deviation of marks obtained by 50 students of the class in three subjects are given below:

Subject Mathematics Physics Chemistry

Mean 42 32 40.9

Standard deviation 12 15 20

i. Which of the three subjects shows the highest variability? (1)


ii. What is the coefficient of variation of marks obtained by the students in Chemistry? (1)
iii. What is the coefficient of variation of marks obtained by the students in Physics? (2)
OR
What is the coefficient of variation of marks obtained by the students in Mathematics? (2)
38. A permutation is an act of arranging the objects or numbers in order. Combinations are the way of selecting [4]
the objects or numbers from a group of objects or collections, in such a way that the order of the objects does not
matter.

How many different words can be formed by using all the letters of the word ALLAHABAD?
i. In how many of them, vowels occupy the even position?
ii. In how many of them, both L do not come together?

Page 5 of 16
Solution

Section A
1.
(c) 11
2

Explanation: In quadrant III, sin θ < 0, cos θ < 0 and tan θ > 0
In quadrant II, sin ϕ > 0, cos ϕ < 0 and tan θ < 0
Now, cot θ = ⇒ tan θ = 2 1

2
−5 −3
sec ϕ = ⇒ cos ϕ =
3 5
−−
2 2 9 16 16 4
∴ sin θ = (1 − cos ϕ) = (1 − ) = ⇒ sin ϕ = √ =
25 25 25 5

4 5 −4
∴ tan ϕ = ( × ) =
5 −3 3

4 2
(2− ) ( )
tan θ+tan ϕ 3 3 2 3 2
∴ tan(θ + ϕ) = = = = ( × ) =
1−tan θ tan ϕ −4 8 3 11 11
{ 1−(2× )} (1+ )
3 3

2.
(b) 219
Explanation: n(A) = 4, n(B) = 2
n(A × B) = 8

∴ N umber of subsets having at least 3 elements

8 8 8
= 2 − (1 + C1 + C2 ) = 219

3.
(d) 2

Explanation: Let S be the sample space.


∴ n(S) = 36

A. even number on the first die


B. number on the second die is greater that 4
∴ n(A) = 18, n(B) = 12,
P(A) = = and P(B) =
18

36
1

2
12

36
=
1

Also, A ∩ B = {(2, 5), (2, 6), (4, 5), (4, 6), (6, 5), (6, 6)}
6 1
P(A ∩ B) = 36
=
6

P(A ∪ B) = P(A) + P(B) - P(A ∩ B)


1 1 1
= + −
2 3 6
3+2−1 4 2
= = =
6 6 3

4. (a) 1
Explanation: Given, lim sin x

x→0 √x+1− √1−x

sin x[ √x+1+ √1−x]


= lim
x→0 ( √x+1− √1−x)( √x+1+ √1−x)

sin x[ √x+1+ √1−x]


= lim
x+1−1+x
x→0

sin x[ √x+1+ √1−x] sin x −−−−− −−−−−


1
= lim = ⋅ lim [√x + 1 + √1 − x]
2x 2 x
x→0 x→0

Taking limits, we get


1 −−−− −−−− 1
= × 1 × [√0 + 1 + √1 − 0] = × 1 × 2 = 1
2 2

5.
(d) (-1, -14)
Explanation: Suppose (h, k) be the point of reflection of the given point (4, – 13) about the line 5x + y + 6 = 0.
h+4 k−13
The mid-point of the line segment joining points (h, k) and (4, – 13) is given by 2
,
2
h+4 k−13
This point lies on the given line, thus we have 5 2
+
2
+ 6 = 0 or 5h + k + 19 = 0 ... (1)

Page 6 of 16
k+13
Again the slope of the line joining points (h, k) and (4, –13) is given by h−4
.
k+3
This line is perpendicular to the given line and therefore, (-5)
h−4
= −1

This gives 5k + 65 = h – 4 or h – 5k – 69 = 0 ... (2)


On solving (1) and (2), we obtain h = –1 and k = –14.
Therefore thethe point (-1, -14) is the reflection of the given point.
6. (a) {2, 3, 5, 7}
Explanation: Prime no. less then 10 is 2, 3, 5, 7 so
Set A = {2, 3, 5, 7}
7.
−−
(b) 1/√26
−−
Explanation: 1/√26
Let z = (1−i)(2+3i)
1

1
⇒ z=
2
2+i−3i
1
⇒ z=
2+i+3

1 5−i
⇒ z= ×
5+i 5−i
5−i
⇒ z=
2
25−i

5−i
⇒ z=
25+1

5−i
⇒ z=
26
5 i
⇒ z= −
26 26
−−−−−−−−
25 1
⇒ |z| = √ +
676 676

1
⇒ z=
√26

8.

(d) −√2
Explanation: Let f (x) = sin x + cos x


∴ f (x) = cos x − sin x

′′
⇒ f (x) = − sin x − cos x

Now, f ′
(x) = 0

⇒ cos x − sin x = 0 ⇒ sin x = cos x ⇒ tan x = 1


π
⇒ x = nπ + , n ∈ z
4
π
At x = π +
4
,

π π
f'' (x) = − sin (π + ) − cos (π + )
4 4

π π 1 1 −

= sin( ) + cos( ) = + = √2 > 0
4 4 √2 √2

∴ x = π +
π

4
is point of minimum
Minimum value = sin(π +
π

4
) + cos(π +
π

4
)

π π
= − sin( ) − cos( )
4 4

1 1 2 –
= − − = − = − √2
√2 √2 √2

9.
(b) -23 < x ≤ 2
3(x−2)
Explanation: −15 < 5
≤ 0

3(x−2)
5 5 5
⇒ −15 ⋅ < ⋅ ≤ 0 ⋅
3 5 3 3

⇒ -25 < (x - 2) ≤ 0 + 2
⇒ -25 + 2 < x - 2 + 2 ≤ 2

⇒ -23 < x ≤ 2

10. (a) -2
c c

Explanation: 180 ∘
= π
c
⇒ 1110

= (
π

180
× 1110) = (
37π

6
)

Page 7 of 16
∴ cosec (-1110°) = -cosec 1110° = - cosec 37π

π π
= − cosec(6π + ) = − cosec = −2 [∵ cosec(2nπ + θ) = cosec θ]
6 6

11.
(c) 2n
Explanation: 2n
The total number of subsets of a finite set consisting of n elements is 2n.
12.
(b) AP
1

Then, a = kx, b = ky and c = kz.


1 1

Explanation: Let a = b x y
= c z = k

Since, a, b, c are in GP
2 y 2 x z
⇒ b = ac ⇒ (k ) = (k × k )

⇒ k2y = k(x+z) ⇒ 2y = x + z
⇒ x, y, z are in AP.

13.
(d) (n + 1)2n
Explanation: We have, C0 + 3C1 + 5C2 + ...+ (2n + 1)Cn
= (C0 + C1 + C2 + ... + Cn) + 2(C1 + 2C2 + ... + nCn)

= 2n + 2(n . 2n-1) = (n + 1) . 2n
14.
−2
(b) ( 3
, ∞)

Explanation: (x + 3) + 4 > -2x + 5


⇒ x + 7 > -2x + 5

⇒ x + 7 + 2x > -2x + 5 + 2x
⇒ 3x + 7 > 5
⇒ 3x + 7 - 7 > 5 - 7

⇒ 3x > -2
−2
⇒ x> 3

−2
⇒ x ∈ ( , ∞)
3

15.
(c) A ∩ Bc
Explanation: A ∩ Bc
A and B are two sets.
A ∩ B is the common region in both the sets.
(A ∩ Bc) is all the region in the universal set except A ∩ B
Now, A ∩ (A ∩ B)c = A ∩ Bc
16.
(b) 0
Explanation: sin(270 + θ) = − cos θ , sin(270
∘ ∘
− θ) = − cos θ , cos(180

+ θ) = − cos θ

∴ given exp = cos θ − cos θ + cos θ − cos θ = 0

17.
(c) purely real
Explanation: Let z = x + iy
Then z̄ = x - iy
∴ z - z̄ = (x + iy) - (x - iy) = 2iy

z− z̄
Now 2i
=y
z− z̄
Hence 2i
is purely real.

Page 8 of 16
18.
(d) 60
Explanation: There are in all 5 letters out of which there are 2P, 1A, 1L and IE
∴ required number of ways = 5!

(2!)(1!)(1!)(1!)
= 60.

19.
(b) Both A and R are true but R is not the correct explanation of A.
Explanation: Assertion:
(1 + x)n = n c0
+ nc x + nc x
1 2
2
… + nc x
n
n

Reason:
(1 + (-1))n = n c0 1
n
+ nc (1)
1
n−1
(−1)
1
+ nc (1)
2
n−2
(−1)
2
+... + n n−n
cn (1) (−1)
n

= n − n + n − n + ... (-1)nn
c8 c1 c2 c3 cn

Each term will cancel each other


∴ (1 + (-1))n = 0
Reason is also the but not the correct explanation of Assertion.
20. (a) Both A and R are true and R is the correct explanation of A.
Explanation: Assertion: In the calculation of variance, we find that the units of individual observations xi and the unit of their
mean x̄ are different from that of variance, since variance involves the sum of squares of (xi - x̄ ).
For this reason, the proper measure of dispersion about the mean of a set of observations is expressed as positive square-root of
the variance and is called standard deviation.
Section B
21. Given, R = {(x, y ) : x, y ∈ z, xy = 4}
= {(-4, -1), (-2, -2), (-1, -4), (1, 4), (2, 2), (4, 1)}
∴ Domain of R = {-4, -2, -1, 1, 2, 4}

OR
2
x −16
Here we are given that, f(x) = x−4

Need to find: where the function is defined.


2
x −16
Let, f(x) = x−4
= y ........(i)
To find the domain of the function f(x) we need to equate the denominator of the function to 0
Therefore,
x - 4 = 0 or x = 4
It means that the denominator is zero when x = 4
So, the domain of the function is the set of all the real numbers except 4
The domain of the function, D{f(x)} = (-∞ , 4) ∪ (4, ∞ )
Now if we put any value of x from the domain set the output value will be either (-ve) or (+ve), but the value will never be 8
So, the range of the function is the set of all the real numbers except 8
The range of the function, Rf(x) = (-∞ , 8) ∪ (8, ∞ )
22. We have, d

dx
(sin
3
x cos
3
x) = sin
3
x ⋅
dx
d
cos
3
x + cos
3
x ⋅
d

dx
(sin
3
x) [Using Product Rule of differentiation]
3 2 3 2
= sin x ⋅ 3 cos x(− sin x) + cos x ⋅ 3 sin x ⋅ cos x

= -3 sin4 x cos2 x + 3 cos4 x sin2 x


= 3 sin2 x cos2 x (-sin2 x + cos2 x)
2 2
= 3 sin x cos x ⋅ cos 2x
3 2 2 3 2
= ⋅ 4 sin x cos x ⋅ cos 2x = (2 sin x cos x) cos 2x
4 4
3 2
= sin 2x ⋅ cos 2x
4
2 2
x y
23. Since the vertices lie on the x-axis, so let the equation of the required hyperbola be 2

2
= 1 ... (i)
a b

The coordinates of its vertices and foci are (± a, 0) and (± ae, 0) respectively.
But, the coordinates of vertices and foci are given as (± 5, 0) and (± 7, 0) respectively.
∴ a = 5 and ae = 7 then 5e = 7 ⇒ e =
7

Now, b2 = a2 (e2 - 1) ⇒ b2 = 25 ( 49

25
− 1) = 24.

Page 9 of 16
2

Substituting the values of a2 and b2 in (i), we obtain


2 y
x
− = 1
25 24
2 2
y
Required equation of hyperbola is x

25

24
= 1

OR
We are given:
y2 - 4y + 4x = 0
⇒ (y - 2)2 - 4 + 4x = 0
⇒ (y - 2)2 = - 4 (x - 1)
Let Y = y - 2
X=x-1
Then, we have:
Y2 = - 4X
On comparing the given equation with Y2 = - 4aX
4a = 4 ⇒ a = 1
∴ Vertex = (X = 0, Y = 0) = (x = 1, y = 2)

Focus = (X = -a, Y = 0) = (x - 1 = - 1, y - 2 = 0) = (x = 0, y = 2)
Equation of the directrix:
x=a
i.e. x - 1 = 1 ⇒ x = 2
Axis = Y = 0
i.e. y - 2 = 0 ⇒ y = 2
Therefore, length of the latus rectum = 4a = 4 units
24. Now,
14 = 7 × 2
21 = 7 × 3
28 = 7 × 4
35 = 7 × 5
42 = 7 × 6
98 = 7 × 14
Therefore,the given set can be write as
E = {x: x = 7n, n ∈ N and 1 ≤ n ≤ 14}
25. The equation of a line intercept form is
y
x

a
+ =1
b

Given, b = 2a
x y
∴ (1) ⇒ a
+
2a
=1
⇒ 2x + y = 2a
since the line passes through the point (1, 2),
2 . 1 + 2 = 2a
⇒ a = 2

∴ Equation of the line is 2x + y - 4 = 0.

Section C
26. Given, A = {1,2,3,4 } and B = {1, 4, 9, 16, 25} and
R = {(x, y) : x ∈ A, y ∈ B and y = x2}
i. Relation R = {(1, 1), (2, 4), (3, 9), (4, 16)}

ii. Domian of R = {1, 2, 3, 4}

Page 10 of 16
iii. Range of R = {1, 4, 9, 16}
iv. Co-domain of R = {1, 4, 9, 16, 25}
27. The given system of inequations is 3x - 6 ≥ 0 ...(i)
4x - 10 ≤ 6 .....(ii)
Now 3x - 6 ≥ 0 ⇒ 3x ≥ 6 ⇒ ≥ ⇒ x ≥ 2
3x

3
6

Solution set of inequation (i) is [2, ∞ )


and, 4x -10 < 6 ⇒ 4x ≤ 16 ⇒ x < 4
∴ The solution set of inequation (ii) is (∞ , 4]

The solution sets of inequations (i) and (ii) are represented graphically on the real line in the above figure.
Clearly, the intersection of these solution sets is the set [2,4].
Hence, the solution set of the given system of inequations is the interval [2,4].
28. To prove: Points A, B, C form an isosceles triangle.
Formula: The distance between two points (x1, y1, z1) and (x2, y2, z2) is given by
−−−−−−−−−−−−−−−−−−−−−−−−−−−−
D = √(x 2 − x1 )
2
+ (y2 − y1 )
2
+ (z2 − z1 )
2

Here,
(x1, y1, z1) = (4, 6, -3)
(x2, y2, z2) = (0, 2, 3)
(x3, y3, z3) = (-4, -4, -1)
−−−−−−−−−−−−−−−−−−−−−−−−−−−−
2 2 2
Length AB = √(x 2 − x1 ) + (y2 − y1 ) + (z2 − z1 )
−−−−−−−−−−−−−−−−−−−−−−−−− −
2 2 2
= √(0 − 4) + (2 − 6) + (3 − (−3))
−−−−−−−−−−−−−−−− −
2 2 2
= √(−4) + (−4) + (6)
−−−−−−−−− −
= √16 + 16 + 36
−− −−
Length AB = √68 = 2√17
−−−−−−−−−−−−−−−−−−−−−−−−−−−−
Length BC = √(x 3 − x2 )
2
+ (y3 − y2 )
2
+ (z3 − z2 )
2

−−−−−−−−−−−−−−−−−−−−−−−−−− −
2 2 2
= √(−4 − 0) + (−4 − 2) + (−1 − 3)
−−−−−−−−−−−−−−−−− −
2 2 2
= √(−4) + (−6) + (−4)
−−−−−−−−− −
= √16 + 36 + 16
−− −−
Length BC = √68 = 2√17
−−−−−−−−−−−−−−−−−−−−−−−−−−−−
2 2 2
Length AC = √(x 3 − x1 ) + (y3 − y1 ) + (z3 − z1 )
−−−−−−−−−−−−−−−−−−−−−−−−−−−−− −
2 2 2
= √(−4 − 4) + (−4 − 6) + (−1 − (−5))
−−−−−−−−−−−−−−−−− −
2 2 2
= √(−8) + (−10) + (2)
−−−−−−−−− −
= √64 + 100 + 4
−−−
Length AC = √168
Here, AB = BC
∴ vertices A, B, C forms an isosceles triangle.

OR
Let Q divides PR in the ratio k : 1
11k+5 10k+4 −8k−2
Thus coordinates of Q are [ k+1
,
k+1
,
k+1
]

It is given that coordinates of Q are (7, 6, -4).


11k+5 10k+4 −8k−2
∴ = 7, = 6, = −4
k+1 k+1 k+1

Now solving these we get k = 1

2
1
Thus Q divides PR in the ratio 2
: 1 or 1 : 2

29. To find: Expension of 3 3
(√x − √ y)
6
by means of binomial theorem..
Formula used: n
Cr =
n!

(n−r)!(r)!

(a + b)n = n C
a + C 0
n n
1a
n−1
b +
n
C2 a
n−2 2
b +…… +
n
Cn−1 ab
n−1 n
+ nCn b

We have, (√x − √y)
3 3 6

Page 11 of 16
1 1

We can write √−
x , as x , and √y , as y
3
, 3 3 3

6
1 1

Now, we have to solve for (x 3


− y 3
)

6−0 6−1 1 6−2 2 6−3 3


1 1 1 1 1 1 1

⇒ [6C0 (x 3
) ] + [6C1 (x 3
) (− y 3
) ] + [6C2 (x 3
) (− y 3
) ] + [6C3 (x 3
) (− y 3
) ]

6−4 4 6−5 5 6
1 1 1 1 1

+ [6C4 (x 3 ) (− y 3 ) ] + [6C5 (x 3 ) (− y 3 ) ] + [6C6 (− y 3 ) ]

5 1 4 2 3 3
6
6 6
⇒ [ C0 ( )] − [ C1 (x 3 ) (y 3 )] + [6C2 (x 3 ) (y 3 )] − [6C3 (x 3 ) (y 3 )]
3
x

2 4 1 5

6 6
+ [ C4 (x 3
) (y 3
)] − [6C5 (x 3
) (y 3
)] + [6C6 ( )]
3
y

5 2 4 2
6! 2 6! 6!
⇒ [ (x )] − [ (x 3
) (y 3
)] + [ (x 2
) (x 3
)]
0!(6−0)! 1!(6−1)! 2!(6−2)!

2 4 1 5
6! 6! 6! 6! 2
−[ (x)(y)] + [ (x 3 ) (y 5 )]− [ (x 3 ) (y 3 )] + [ (y )]
3!(6−3)! 4!(6−4)! 5!(6−5)! 6!(6−6)!

5 1 4 2 2
2 4
⇒ [1 (x )] − [6 (x 3 ) (y 3 )] + [15 (x 3 ) (y 3 )] −[20(x)(y)] + [15 (x 3 )( )]
3
y

1 5
2
− [6 (x 3 ) (y 3 )] + [1 (y )]

5 1 4 2 2 4 1 5
2 2
⇒ x − 6x 2 y 3 + 15x 3 y 3 − 20xy +15x 3 y 3 − 6x 3 y 3 + y

Hence the result.


OR
Using binomial theorem
(1 + 2x)6 (1 - x)7 = [ 6 6 6
C0 + C1 (2x)+ C2 (2x) + C3 (2x) + C4 (2x) + C5 (2x) + C6 (2x) ]
2 6 3 6 4 6 5 6 6

7 7 7 2 7 3 7 4 7 5 7 6 7 7
[ C0 − C1 (x)+ C2 (x) − C3 (x) + C4 (x) − C5 (x) + C6 (x) − C7 (x) ]

= [1 + 12x + 60x2 + 160x3 + 240x4 + 192x5 + 64x6] [1 - 7x + 21x2 - 35x3 + 35x4 - 21x5+ 7x6 - x7]
∴ Coefficient of x5 in the product
= (1 × −21) + (12 × 35) + (60 × −35) +(160 × 21) + (240 × −7) + (192 × 1)

= -21 + 420 - 2100 + 3360 - 1680 + 192 = 171


−−−−−−
30. Let √7 − 24i = x + iy. Then
− −−−−−
√7 − 24i = x + iy

⇒ 7 - 24i = (x + iy)2
⇒ 7 - 24i = (x2 - y2) + 2i xy
⇒ x2 - y2 = 7 ... (i)
and 2xy = -24 ... (ii)
Now, (x2 + y2)2 = (x2 - y2)2 + 4 x2 y2
⇒ (x2 + y2)2 = 49 + 576 = 625 [∵ x2 + y2 > 0]
⇒ x2 + y2 = 25... (iii)
add (i) and (iii), we get
2x2 = 32
⇒ x2 = 16
⇒ x = ± 4

put value of x in (I), we get


y2 = 9 => y = ± 3
From (ii) we observe that 2xy is negative. So, x and y are of opposite signs.
−−−−−−
Hence, √7 − 24i = ± (4 - 3i)
OR
We have, (x + iy)1/3 = a + ib
⇒ x + iy = (a + ib)3 [cubing on both sides]
⇒ x + iy = a3 + i3 b3 + 3 iab (a + ib)

Page 12 of 16
⇒ x + iy = a3 - ib3 + i 3a2 b - 3ab2
⇒ x + iy = a3 - 3ab2 + i (3a2 b - b3)
On equating real and imaginary parts from both sides, we get
x = a3 - 3ab2 and y = 3a2b - b3
= a2 - 3b2 and = 3a2 - b2
x y

a b

a2 - 3b2 - 3a2 + b2
y
Now, x

a

b
=

= - 2a2 - 2b2 = - 2 (a2 + b2)


Hence proved.
31. Here,it is given: A = {a, e, i, o, u}, B = {a, d, e, o, v} and C = {e, o, t, m}.
B ∩ C = {e, o} and A ∪ (B ∩ C) = {a, e, i, o, u}
LHS

A ∪ (B ∩ C )

R.H.S: A ∪ B = {a, d, e, I, o, u, v} and A ∪ C = {a, e, I, o, u, t, m}

(A ∪ B) ∩ (A ∪ C) = {a, e, I, o, u}
L.H.S = R.H.S. [Verified]
Section D
32. Let S be the sample space
S = {1,2,3,4,5.....20}
Let E1, E2 and E3, E4 are the event of getting prime number, an odd number, multiple of 5 and not divisible by 3 respectively.
P (E1 ) =
8

20
=
2

5
, E1 = {2, 3, 5, 7, 11, 13, 17, 19 }
10 1
P (E2 ) =
20
=
2
, E2 = {1, 3, 5, 7, 9, 11, 13, 15, 17, 19 }
P (E3 ) =
4

20
=
1

5
, E3 = {5, 10, 15, 20 }
P (E4 ) =
14

20
=
7

10
, E4 = {1, 2, 4, 5, 7, 8, 10, 11, 13, 14, 16, 17, 19, 20 }

33. Dividing x4 − 3x3 + 2 by x3 − 5x2 + 3x + 1

4 3 2
x −3x +2 7x −7x
⇒ lim
3 2
= lim(x + 2) + lim 3 3
x→1 x −5x +3x+1 x→1 x→1 x −5x +3x+1

7x(x−1)
= lim x + 2 + lim 3 3
x→1 x→1 x −5x +3x+1

7x(x−1)
= lim x + 2 + lim 2
x→1 x→1 (x−1)(x −4x−1)

= lim x + 2 + lim 2
7x

x→1 x→1 (x −4x−1)

Page 13 of 16
=1+2+ 7

(1−4−1)

=3- 7

4
12−7
= 4

= 5

OR
Let f (x) = log sin x. Then, f (x + h) = log sin (x + h)
d f (x+h)−f (x)
∴ (f (x)) = lim
dx h
h→0

log sin(x+h)−log sin x



d

dx
(f (x)) = lim h
h→0

sin(x +h)
log{ }
sin x


d

dx
(f (x)) = lim h
h→0

sin(x +h)
log{1+ −1}
sin x


d

dx
(f (x)) = lim h
h→0

sin(x +h)−sin x
log{1+ }
sin x


d

dx
(f (x)) = lim h
h→0

sin(x +h)−sin x
log{1+ }
sin x sin(x+h)−sin x
d

dx
(f (x)) = lim sin(x +h)−sin x
× {
sin x
}
h→0
h{ }
sin x

sin(x +h)−sin x
log{1+ }
h sin(x+h)−sin x
d

dx
(f (x)) = lim sin(x +h)−sin x
×
h
×
1

sin x
h→0
{ }
h

sin(x +h)−sin x
h h
log{1+ }
2 sin cos(x+ )
h

(f (x)) = lim
d 2 2 1
⇒ × lim ×
dx sin(x +h)−sin x h sin x
h→0 h→0
{ }
h

sin(x +h)−sin x
h h
log{1+ }
h
sin( ) cos(x+ )

(f (x)) = lim
d 2 2 1
⇒ × lim ×
dx sin(x +h)−sin x h sin x
h→0 h→0
{ }
2
h

d

dx
(f (x)) = 1 × cos x × 1

sin x
= cot x.
34. Let the four numbers in GP be
a
,
a

r
, ar, ar3 ...(i)
r3

Product of four numbers = 4096 [given]


(ar) (ar3) = 4096
a a
⇒ ( )( )
3 r
r

⇒ a4 = 4096 ⇒ a4 = 84
On comparing the base of the power 4, we get

a
+
a

r
+ ar + ar3 = 85
r3

⇒ a[ 1

3
+
1

r
+ r + r ]
3
= 85
r

⇒ 8 [r3 + 1

3
] + 8 [r + 1

r
] = 85 [∵ a = 8]
r

= 85 [∵ a3 + b3 = (a + b)3 - 3 (a +b)]
3
⇒ 8 [ (r + 1

r
) − 3 (r +
1

r
)] + 8 (r +
1

r
)

3
⇒ 8 (r + 1

r
) - 16 (r + 1

r
) - 85 = 0 ...(ii)
On putiing (r + 1

r
) = x in Eq. (ii), we get
8x3 - 16x - 85 = 0
⇒ (2x - 5) (4x2 + 10x + 17) = 0
⇒ 2x - 5 = 0 [∵ 4x2 + 10x + 177 = 0 has imaginary roots]
⇒ x = ⇒ r + [put x = r + ]
5 1 5 1
=
2 r 2 r

⇒ 2r2 - 5r + 2 = 0
⇒ (r - 2) (2r - 1) = 0

⇒ r = 2 or r =
1

On putting a = 8 and r = 2 or r = 1

2
in Eq. (i), we obtain four numbers as
8 8 3
, , 8 × 2, 8 × 2
3
2 2

Page 14 of 16
3
or 8

3
,
8
,8 ×
1

2
,8 × (
1

2
)
(1/2) (1/2)

⇒ 1, 4, 16, 64 or 64, 16, 4, 1.


8π 16π
35. LHS = cos ⋅ cos

⋅ cos
15

15 15
⋅ cos
15

2π 2π 2π 2π
= cos cos2 ( )cos4 ( ) cos8 ( )
15 15 15 15


Put 15
= α

⇒ LHS = cos α ⋅ cos 2α ⋅ cos 4α ⋅ cos 8α


2 sin α[cos α⋅cos 2α⋅cos 4α⋅cos 8α]
= 2 sin α
[multiplying numerator and denominator by 2sin α]
(2 sin α⋅cos α)⋅cos 2α⋅cos 4α⋅cos 8α
= 2 sin α
2(sin 2α⋅cos 2α⋅cos 4α⋅cos 8α)
= [∵ 2sinα cosα = sin2 α and multiplying numerator and denominator by 2 ]
2(2 sin α)

(2 sin 2α⋅cos 2α)⋅cos 4α⋅cos 8α


= 4 sin α
2(sin 4α⋅cos 4α) cos 8α
= 2(4 sin α)
[∵2sinα cosα = sin2 α and multiplying numerator and denominator by 2 ]
2(sin 8α⋅cos 8α)
= 2(8 sin α)

sin 16α sin(15α+α)


= 16 sin α
=
16 sin α

Now, 15α = 2π ,
sin(2π+α) sin α 1
= 16 sin α
=
16 sin α
= 16
= RHS
∴ LHS = RHS
Hence proved.
OR
We have to find the value of sin (α + β)
It is given that
6 cos x + 8 sin x = 9
⇒ 6 cosx = 9 - 8 sinx

⇒ 36 cos2x = (9 - 8 sinx)2
⇒ 36(1 - sin2x) = 81 + 64 sin2x - 144 sinx
⇒ 100 sin2x - 144 sinx + 45 = 0
Now, α and β are the roots of the given equation;
therefore, cos α and cos β are the roots of the above equation.
45
⇒ sin α sin β =
100

(Product of roots of a quadratic equation ax2 + bx + c = 0 is c

a
)
Again, 6 cosx + 8 sinx = 9
⇒ 8 sinx = 9 - 6 cosx

⇒ 64 sin2x = (9 - 6 cos x)2


⇒ 64(1 - cos2x) = 81 + 36cos2x - 108 cosx
⇒ 100 cos2x - 108 cosx + 17 = 0
Now, α and β are the roots of the given equation;
therefore, sinα and sinβ are the roots of the above equation.
Therefore, cos α cos β = 17

100

Hence, cos(α + β) = cos α cos β − sin α sin β


17 45
= −
100 100
28
= −
100
7
= −
25
−−−−−−−−−−− −
2
sin(α + β) = √1 − cos (α + β)
−−−−−−−−−
2
−7
= √1 − ( )
25

−−

576
= √
625

24
=
25

Section E

Page 15 of 16
36. i. An ellipse is the set of all points in a plane, the sum of whose distances from two fixed points in the plane is a constant. Hence
path traced by Arun is ellipse.
Sum of the distances of the point moving point to the foci is equal to length of major axis =10m
ii. Given 2a = 10 & 2c = 8
⇒ a = 5 & c = 4

c2 = a2 + b2
⇒ 16 = 25 + b2
⇒ b2 = 25 - 16 = 9
2 2
x y
Equation of ellipse 2
+
2
= 1
a b
2 2
y
Required equation is x

25
+
9
= 1
2
2
y
iii. equation is of given curve is x

25
+
9
= 1

a = 5, b = 3 and given 2c = 8 hence c = 4


Eccentricity = = a
c 4

OR
2
2
x y
+ = 1
25 9

Hence a = 5 and b = 3
2
2b 2×9 18
Length of latus rectum of ellipse is given by a
=
5
=
5

37. i. The subject with greater C.V. is more variable than others.
Therefore, the highest variability in marks is in Chemistry.
ii. Standard deviation of Chemistry = 20
20
C.V. (in Chemistry) = 40.9
× 100 = 48.89
iii. Standard deviation of Physics = 15
The coefficient of variation, C.V. = Standard deviation

Mean
× 100

C.V. (in Physics) = 15

32
× 100 = 46.87
OR
Standard deviation of Mathematics = 12
Standard deviation
The coefficient of variation, C.V. = Mean
× 100

C.V. (in Mathematics) = 12

42
× 100 = 28.57
38. In a word ALLAHABAD, we have
Letters A L H B D Total

Number 4 2 1 1 1 9
9! 9×8×7×6×5
So, the total number of words = 4!2!
=
2×1
= 7560
i. There are 4 vowels and all are alike i.e., 4 A’s.
Also, there are 4 even places which are 2nd, 4th, 6th and 8th. So, these 4 even places can be occupied by 4 vowels in 4!

4!
=1
way. Now, we are left with 5 places in which 5 letters, of which two are alike (2 L’s) and other distinct, can be arranged in 5!

2!

ways.
Hence, the total number of words in which vowels occupy the even places = 5!

2!
×
4!

4!
=
5!

2!
= 60
ii. Considering both L together and treating them as one letter. We have,
Letters A LL H B D Total

Number 4 1 1 1 1 8
Then, 8 letters can be arranged in 8!

4!
ways.
So, the number of words in which both L come together = 8!
= 8 × 7 × 6 × 5 = 1680
4!

Hence, the number of words in which both L do not come together


= Total number of words - Number of words in which both L come together
= 7560 - 1680 = 5880
Hence, the total number of words in which both L do not come together is 5880

Page 16 of 16
Class XI Session 2024-25
Subject - Mathematics
Sample Question Paper - 7

Time Allowed: 3 hours Maximum Marks: 80

General Instructions:

1. This Question paper contains - five sections A, B, C, D and E. Each section is compulsory. However, there are

internal choices in some questions.

2. Section A has 18 MCQ’s and 02 Assertion-Reason based questions of 1 mark each.

3. Section B has 5 Very Short Answer (VSA)-type questions of 2 marks each.

4. Section C has 6 Short Answer (SA)-type questions of 3 marks each.

5. Section D has 4 Long Answer (LA)-type questions of 5 marks each.

6. Section E has 3 source based/case based/passage based/integrated units of assessment (4 marks each) with sub
parts.

Section A
1. sin 3x−sin x

cos x−cos 3x
is equal to [1]

a) cot 2x b) - cot 2x

c) - tan 2x d) tan 2x

2. Let f(x) = x3 . Then, dom (f) and range (f) are respectively [1]

a) R and R+ b) R+ and R

c) R+ and R+ d) R and R

3. A batsman scores runs in 10 innings as 38, 70, 48, 34, 42, 55, 63, 46, 54 and 44. The mean deviation about mean [1]
is

a) 7.6 b) 6.4

c) 8.6 d) 10.6
−−−−−−−−−−−−−−−−−−−−−−−
−−−−−−−−−−−−−−−−−−
4. If y =
−−−− −
√x + √x + √x + … + to∞ ∞ then
dy
= [1]
dx

a) 1

2y+1
b) 1

2y−1

−−−
c) d)
x x

y+1 y+1

5. The two lines ax + by = c and a′x + b′y = c′ are perpendicular if [1]

a) ab′ = ba′ b) aa′ + bb′ = 0

c) ab + a′b′ = 0 d) ab′ + ba′ = 0


6. A plane is parallel to yz-plane so it is perpendicular to: [1]

Page 1 of 17
a) y-axis b) none of these

c) z-axis d) x-axis
5 5

7. If z = (
√3
+
i
) + (
√3

i
) , then [1]
2 2 2 2

a) Re(z)=0 b) Re(z) > 0, Imz) >

c) Re(z) > 0, Irs(z) < 0 d) Im(z)=0

8. 0! is always taken as [1]

a) 1 b) 2

c) ∞ d) 0
9. lim
sin x

x
= [1]
x→∞

a) 2 b) 1

c) ∞ d) 0
10. tan 15° = ? [1]

a) (√2+1)
b) (√3+1)

(√2−1) (√3−1)

c) (√3−1)
d) (√2−1)

(√3+1) (√2+1)

11. The set of all prime numbers is [1]

a) an infinite set b) a singleton set

c) a multi set d) a finite set



12. The integral part of ( √2 + 1)
6
is [1]

a) 98 b) 96

c) 99 d) 100
13. ∑
n

r=0
r
4 .
n
Cr is equal to [1]

a) 6n b) 5-n

c) 4n d) 5n

14. Solve the system of inequalities: [1]


x+7 2x+1
> 2, > 5
x−8 7x−1

a) (4, 8) b) (3, 6)

c) no solution d) (2, 5)
15. If A = {1, 2, 3, 4, 5, 6} then the number of proper subsets is [1]

a) 63 b) 36

c) 64 d) 25
16. cos 15° = ? [1]

a) b)
(√3+1) (√3+1)

√2 2√2

c) (√3−1)
d) (√3−1)

2√2 √2

dy
17. If y = sin x+cos x

sin x−cos x
, then dx
at x = 0 is equal to [1]

Page 2 of 17
a) 0 b) -2

c) 1

2
d) Does not exist
18. The total number of 9 digit numbers which have all different digits is [1]

a) 9! b) 10 × 10!

c) 10! d) 9 × 9!
19. Assertion (A): Let A = {a, b} and B = {a, b, c}. Then, A ⊄ B. [1]
Reason (R): If A ⊂ B, then A ∪ B = B.

a) Both A and R are true and R is the correct b) Both A and R are true but R is not the
explanation of A. correct explanation of A.

c) A is true but R is false. d) A is false but R is true.


20. Assertion (A): A sequence is said to definite if it has finite no of terms. [1]
Reason (R): The sequence whose nth term if
n
2

n
if 2, 2, 8

3
, 4...

a) Both A and R are true and R is the correct b) Both A and R are true but R is not the
explanation of A. correct explanation of A.

c) A is true but R is false. d) A is false but R is true.


Section B
21. If A and B are any two non-empty sets, then prove that A × B = B× A ⇔ A = B [2]
OR
Find the domain of f(x) =
1

x+2
.
22. Evaluate: lim 1−cos 4θ

1−cos 6θ
. [2]
θ→0

23. Two dice are thrown simultaneously. Find the probability of getting a total of at least 10. [2]
OR
If A and B are two events associated with a random experiment such that P(A) = 0.25, P(B) = 0.4 and P(A or B) =
0.5, find the values of
i. P(A and B)
ii. P(A and B
¯
)
24. Is A = {x : x ∈ N, 1 < x ≤ 2} null set? [2]
25. Find the coordinates of the point which divides the join of A(-5, 11) and B(4, -7) in the ratio 2 : 7. [2]
Section C
n

[3]
2 [1⋅3⋅5.....(2n−1)]
26. Prove that : 2n
Cn =
n!
.
27. Find the point in yz-plane which is equidistant from the points A(3, 2, -1), B(1, -1, 0) and C(2, 1, 2). [3]

28. Using binomial theorem, expand: ( √x + √y)
8
[3]
OR

In the expansion of (x + a)n, sums of odd and even terms are P and Q respectively, prove that

i. 2(P2 +Q2) = (x + a)2n + (x - a)2n

ii. P2 - Q2 = (x2 - a2)n


29. If f(x) = mx + c and f(0) = f'(0) = 1. What is value of f(2)? [3]
OR

Page 3 of 17
Differentiate the function: 3x-5
30. The sum of three numbers in G.P. is 14. If the first two terms are each increased by 1 and the third term [3]
decreased by 1, the resulting numbers are in A.P. Find the numbers.
OR
The sum of first three terms of a G.P. is and their product is 1. Find the common ratio and the terms.
39

10

31. Out of 25 members in a family, 12 like to take tea, 15 like to take coffee and 7 like to take coffee and tea both. [3]
How many like
i. at least one of the two drinks
ii. only tea but not coffee
iii. only coffee but not tea
iv. neither tea nor coffee
Section D
32. Find the mean and standard deviation for the following data: [5]

Class interval 0-10 10-20 20-30 30-40 40-50 50-60 60-70 70-80 80-90 90-100

Frequency 3 2 4 6 5 5 5 2 8 5

33. Find the coordinates of the foci, the vertices, the length of major axis, the minor axis, the eccentricity and the [5]
length of the latus rectum of the ellipse.
2
2
x y
+ = 1
100 400

OR
Find the lengths major and minor axes, coordinates of the vertices, coordinates of the foci, eccentricity, and length of
the latus rectum of the ellipse 9x2 + y2 = 36.
34. Solve the following system of linear inequalities. [5]
2(2x + 3) - 10 < 6(x - 2)
and
2x−3 4x
+ 6 ≥ 2+
4 3

35. If sin x =
√5
and x lies in the 2nd quadrant, find the values of cos , sin
x x
and tan .x
[5]
3 2 2 2

OR

Prove that: cos 36o cos 42o cos 60o cos 78o = 1

16
.
Section E
36. Read the following text carefully and answer the questions that follow: [4]
Representation of a Relation
A relation can be represented algebraically by roster form or by set-builder form and visually it can be
represented by an arrow diagram which are given below
i. Roster form In this form, we represent the relation by the set of all ordered pairs belongs to R.
ii. Set-builder form In this form, we represent the relation R from set A to set B as R = {(a, b): a ∈ A, b ∈ B
and the rule which relate the elements of A and B}.
iii. Arrow diagram To represent a relation by an arrow diagram, we draw arrows from first element to second
element of all ordered pairs belonging to relation R.
Questions:
i. If n(A) = 3 and B = {2, 3, 4, 6, 7, 8} then find the number of relations from A to B. (1)
ii. If A = {a, b} and B = {2, 3}, then find the number of relations from A to B. (1)

Page 4 of 17
iii. If A = {a, b} and B = {2, 3}, write the relation in set-builder form. (2)
OR
Express of R = {(a, b): 2a + b = 5; a, b ∈ W} as the set of ordered pairs (in roster form). (2)
37. Read the following text carefully and answer the questions that follow: [4]
Two students Ankit and Vinod appeared in an examination. The probability that Ankit will qualify the
examination is 0.05 and that Vinod will qualify is 0.10. The probability that both will qualify is 0.02.
i. Find the probability that atleast one of them will qualify the exam. (1)
ii. Find the probability that atleast one of them will not qualify the exam. (1)
iii. Find the probability that both Ankit and Vinod will not qualify the exam. (2)
OR
Find the probability that only one of them will qualify the exam. (2)
38. Read the following text carefully and answer the questions that follow: [4]
The conjugate of a complex number z, is the complex number, obtained by changing the sign of imaginary part
of z. It is denoted by z̄ .
The modulus (or absolute value) of a complex number, z = a + ib is defined as the non-negative real number
−−−−−−
√a2 + b2 . It is denoted by |z|. i.e.
−−−−−−
2 2
|z| = √a + b

Multiplicative inverse of z is z̄

2
. It is also called reciprocal of z.
|z|

2
z z̄ = |z|

i. If f(z) = 7−z

2
, where z = 1 + 2i, then find |f(z)|. (1)
1−z

ii. Find the value of(z + 3)(z̄ + 3). (1)


iii. If (x - iy) (3 + 5i) is the conjugate of -6 - 24i, then find the value of x + y. (2)
OR
If z = 3 + 4i, then findz̄ . (2)

Page 5 of 17
Solution
Section A
1. (a) cot 2x
(C+D) (C−D)
Explanation: Using sin C - sin D = 2 cos 2
sin
2
(C+D) (C−D)
and cos C - cos D = −2 sin 2
sin
2
, we get
4x 2x
2 cos( ) sin( )
sin 3x−sin x
= cot 2x.
2 2 cos 2x sin x
= =
cos x−cos 3x 4x 2x sin 2x sin x
2 sin( ) sin( )
2 2

2.
(d) R and R
Explanation: f(x) = x3
f(x) can assume any value, so domain of f(x) is R
The Range of the function can be positive or negative Real numbers, as the cube of any number depends on the sign of the
number, So Range of f(x) is R
3.
(c) 8.6
Explanation: First we arrange score in the ascending order
Then scores are : 34, 38, 42, 44, 46, 48, 54, 55, 63, 70
As there are 10 items in this data,
So, median will be the mean of fifth and sixth term
46+48
∴ Median = 2
= 47
Now, deviation from median for each value
di = 13, 9, 5, 3, 1, 1, 7, 8, 16, 23
∑ di 86
∴ Required Mean deviation = 10
=
10
= 8.6

4.
(b) 2y−1
1

−−−−−−
Explanation: y = √(x + y)
y2 = x + y
2yy' = 1 + y'
5.
(b) aa′ + bb′ = 0

−a −a
Explanation: We know that Slope of the line ax + by = c is b
, and the slope of the line a′x + b′y = c′ is ′
The lines are
b

perpendicular if tan θ = 5−x


3
(1)

−a −a ′ ′
= −1 or aa + bb = 0
b ′
b

6.
(d) x-axis
Explanation: Any plane parallel to yz-plane is perpendicular to x-axis.
7.
(d) Im(z)=0
Explanation: From given ,
2 3
√3 √3 2 √3
5 5 i 5 i
z = 2 C0 + C2 + C4
2 2 2 2 2

Since t2 = -1 and t4 = 1, will not contain any i and hence I m


(z) = 0

8. (a) 1
Explanation: We have nPr = n!

(n−r)!
.....(i)

Page 6 of 17
Number of ways you can arrange n thing in n available spaces = n!
nP = n! ....(ii)
⇒ n

But from (i) we get nPn = n!


=
n!
.....(iii)
(n−n)! 0!

n!
Now from (ii) and (iii) we get 0!
= n! ⇒ 0! = 1

9.
(d) 0
Explanation: lim
sin x

x
x→∞

Let x = 1

x → ∞

∴ y → 0
1
sin
y
= lim
1
y→0
y

1
= lim y sin
y
y→0

1
= lim y × lim sin
y
y→0 y→0

1
= 0 × lim sin
y
y→0

=0
10.
( √3−1)
(c)
( √3+1)

1
(1− )
∘ ∘ √3 ( √3−1)
tan 45 −tan 30
Explanation: tan 15° = tan(45° - 30°) = 1+tan 45

tan 30
∘ = =
1 ( √3+1)
(1+ )
√3

11. (a) an infinite set


Explanation: Set A = {2, 3, 5, 7,...} so it is infinite.
12.
(c) 99
Explanation: We have (1 + x)n = 1 + C (x) + C (x) + ....+ (x)n n
1
n
2
2

– – – – – – 5 – 6
Hence (√2 + 1) = 1 + C (√2) + C (√2) + C (√2) + C (√2) + C
6 6
1
6
2
2 6
3
3 6
4
4 6
5 (√2) + (√2)
– – – –
⇒ (√2 + 1) = 1 + 6(√2) + 15 × 2 + 20 × 2(√2) + 15 × 4 + 6 × 4(√2) + 8
6


= 99 + 70√2

Hence integral part of (√2 + 1) = 99 6

13.
(d) 5n
Explanation: ∑ n

r=0
r
4 .
n
Cr=4 0

n
C0 + 4
1 n
⋅ C1 + 4
2 n
⋅ C2 + ... + 4 n n
⋅ Cn

= 1 + 4. C + 4
n
1
2
.
n
C2 + .... + 4 n n
⋅ Cn

= (1 + 4)n = 5n
14.
(c) no solution
x+7
Explanation: x−8
> 2

x+7
⇒ − 2 > 0
x−8

x+7−2(x−8)

x−8
>0
x+7−2x+16

x−8
>0
(23−x) a

x−8
> 0 [∵ b
> 0 ⇒ (a > 0 and b > 0) or (a < 0 and b < 0)]
⇒ (23 - x > 0 and x - 8 > 0) or (23 - x < 0 and x - 8 < 0)
⇒ (x < 23 and x > 8) or (x > 23 and x < 8)
⇒ 8 < x < 23 [Since x > 23 and x < 8 is not possible]
⇒ xϵ (8, 23)
2x+1
Now 7x−1
>5

Page 7 of 17
2x+1

7x−1
-5>0
2x+1−5(7x−1)

7x−1
>0
2x+1−35x+5

7x−1
>0
(6−33x) a

7x−1
> 0 [∵ b
> 0 ⇒ (a > 0 and b > 0) or (a < 0 and b < 0)]
⇒ (6 - 33x > 0 and 7x - 1 > 0) or (6 - 33x < 0 and 7x - 1 < 0)
⇒ (x < and x > ) or (x > and x < )
6 1 2 1

33 7 11 7
1 2
⇒ < x <
7 11

⇒ x ∈ (
1

7
,
11
2
) [Since x > 11
2
and x < 1

7
is not possible]
x+7 2x+1
Hence, the solution of the system x−8
> 2,
7x−1
> 5 will be (8, 23) ∩ ( 1

7
,
2

11
) = ϕ

15. (a) 63
Explanation: 63
The no. of proper subsets = 2n - 1
Here n(A) = 6
In case of the proper subset, the set itself is excluded that's why the no. of the subset is 63. But if it is asked no. of improper or
just no. of subset then you may write 64
So no. of proper subsets = 63
16.
( √3+1)
(b)
2√2

Explanation: cos 15° = cos (45° - 30°) = cos 45°cos 30° + sin 45° sin 30°
1 √3 1 1 ( √3+1)
= ( × )+ ( × ) =
√2 2 √2 2 2√2

17.
(b) -2
sin x+cos x
Explanation: Given, y = sin x−cos x

dy (sinx−cosx)(cosx−sinx)−(sin x+cos x)(cos x+sin x)


=
dx 2
(sin x−cos x)
2 2
−(sin x−cos x ) −(sin x+cos x )
=
2
(sin x−cos x)

2 2 2 2
−[sin x+ cos x−2 sin x cos x+ sin x+ cos x+2 sin x cos x]

=
2
(sin x−cos x)

−2
=
2
(sin x−cos x)

dy −2 −2
∴ ( ) = = = −2
dx 2 2
at x=0 (sin 0−cos 0) (−1)

18.
(d) 9 × 9!
Explanation: We have to form 9 digit numbers from 0, 1, 2, 3, 4, 5, 6, 7, 8, 9 and we know that 0 can not be put on extremely
left place. Therefore, first place from the left can be filled in 9 ways.
Now repetition is not allowed. Therefore, the remaining 8 places can be filled in 9!
∴ The required number of ways = 9 × 9!

19.
(d) A is false but R is true.
Explanation: Assertion A = {a, b}, B = {a, b, c}
Since, all the elements of A are in B. So,
A⊂B
Reason ∵ A ⊂ B
⇒ A ∪ B = B

Hence, Assertion is false and Reason is true.


20.
(b) Both A and R are true but R is not the correct explanation of A.
Explanation: Assertion is true.
Reason

Page 8 of 17
n

Let tn = 2

Putting n = 1, 2, 7, x
8
t1 = 2, t2 = 2, t3 = 3
, t4 = x
8
80 the sequence is 2, 2, 3
,4
Reason is also correct but not the correct explanation for Assertion.
Section B
21. First, let A = B. Then, we have to prove that A × B = B × A
Now, A = B
⇒ A × B = A × A and B × A = A × A

⇒ A× B= B× A

Conversely, let A × B = B × A . Then, we have to prove that A = B.


Let x be an arbitrary element of A. Then,
x ∈ A

⇒ (x, b) ∈ A × B for all b∈B.


⇒ (x, b) ∈ B × A

⇒ x ∈ B

∴ A⊆ B

Let y be an arbitrary element of A. Then,


y ∈ B

⇒ (a, y) ∈ A × B for all a ∈ A

⇒ (a, y) ∈ B × A

⇒ y ∈ A

∴ B⊆ A

Hence, A = B.
Hence, A × B = B × A ⇔ A = B
OR
Here f (x) = 1

x+2

f(x) assume real values for all real values of x except for x + 2 = 0 i.e. x = - 2.
Thus domain of f (x) =R - {-2}.
1−cos(4θ)
22. We have:lim [ 1−cos 6θ
]
θ→0

{∵ 1 - cos A = 2 sin2 (
2

lim [
2 sin

2

]
A

2
}
)
θ→0 2 sin 3θ

⎡ 2 ⎤
2 (2θ)
sin 2θ
lim ⎢ × ⎥
2 sin 2 3θ
θ→0 (2θ) 2
⎣ ×(3θ ) ⎦
2
(3θ)

2 2
sin 2θ 3θ 4
lim [( ) × ( ) × ]
2θ sin 3θ 9
θ→0

4 sin x
= [∵ lim = 1]
9 x
X→0

23. We know that in a single throw of two dice, the total number of possible outcomes is (6 × 6) = 36.
Let S be the sample space of the event and is given by
n(S) = 36.
Let E5 = event of getting a total of at least 10. Then,
E5 = event of getting a total of 10 or 11 or 12 = ((4, 6), (5, 5), (5, 6), (6, 4), (6, 5), (6, 6)}.
⇒ n(E5) = 6
n( E5 )
∴ P(E3) = =
36
6
=
1

6
n(S)

OR
i. It is given that
: P(A) = 0.25, P(A or B) = 0.5 and P(B) = 0.4
To find : P(A and B)
Formula used : P(A or B) = P(A) + P(B) - P(A and B)
Substituting the value in the above formula we get,

Page 9 of 17
0.5 = 0.25 + 0.4 - P(A and B)
0.5 = 0.65 - P(A and B)
P(A and B) = 0.65 - 0.5
P(A and B) = 0.15
ii. Given : P(A) = 0.25, P(A and B) = 0.15 ( from part (i))
To find : P(A and B¯
)
Formula used : P(A and B ¯
) = P(A) - P(A and B)
Substituting the value in the above formula we get,
P(A and B
¯
) = 0.25 - 0.15
P(A and B
¯
) = 0.10
P(A and B) = 0.10
¯

24. We know that,Natural numbers = 1, 2, 3, 4, 5, 6, 7,…


Natural number greater than 1 (1 < x) = 2, 3, 4, 5, ...
Natural number less than or equal to 2 (x ≤ 2) = 2
⇒ one element in this set

∴ It is not a null set.

25. Let P(x, y) be the point that divides the join of A(-5, 11) and B(4, -7) in the ratio 2 : 7
We know that: If m1 : m2 is the ratio in which the join of two points is divided by another point (x, y), then
m1 x2 + m2 x1
x=
m1 + m2
m1 y + m2 y
2 1
y=
m1 + m2

Here, x1 = -5, x2 = 4, y1 = 11 and y2 = -7


Substituting,we get
2×4+7×−5
x =
2+7
8−35
x =
9
−27
x =
9

⇒ x = -3
2×−7+7×11
y =
2+7
−14+77
y =
9
63
y =
9

⇒ y=8
Thus, the coordinates of the point which divided the join of A(-5, 11) and B(4, -7) in the ratio 2 : 7 is (-3, 8).
Section C
n
2 [1⋅35.....(2n−1)]
26. 2n
Cn =
n!
(2n)!
=
n!n!

(2n)(2n−1)(2n−2)(2n−3).....

4⋅3⋅2⋅1
=
n!n!
n
[2 ....4⋅2][(2n−1)....3⋅1]
=
n!n!
n
2 [1⋅2⋅....n][1⋅3⋅5.....⋅(2n−1)]
=
n!n!
n
2 ×n![1⋅3⋅5....⋅(2n−1)]
=
n!n!
n
2 [1⋅3⋅5....⋅(2n−1)]
=
n!

27. The general point on yz plane is D(0, y, z).


Consider this point is equidistant to the points A(3, 2, -1), B(1, -1, 0) and C(2, 1, 2).
∴ AD = BD
−−−−−−−−−−−−−−−−−−−−−− − −−−−−−−−−−−−−−−−−−−−−− −
√(0 − 3)2 + (y − 2)2 + (z + 1)2 = √(0 − 1)2 + (y + 1)2 + (z − 0)2

Squaring both sides,


(0 - 3)2 + (y - 2)2 + (z + 1)2 = (0 - 1)2 + (y + 1)2 + (z - 0)2
9 + y2 - 4y + 4 + z2 + 2z + 1 = 1 + y2 + 2y + 1+ z2
-6y + 2z + 12 = 0 ….(1)
Also, AD = CD
−−−−−−−−−−−−−−−−−−−−−− − −−−−−−−−−−−−−−−−−−−−−− −
√(0 − 3)2 + (y − 2)2 + (z + 1)2 = √(0 − 2)2 + (y − 1)2 + (z − 2)2

Page 10 of 17
Squaring both sides,
(0 - 3)2 + (y - 2)2 + (z + 1)2 = (0 - 2)2 + (y - 1)2 + (z - 2)2
9 + y2 - 4y + 4 + z2 + 2z + 1 = 4 + y2 - 2y + 1+ z2 - 4z + 4
-2y + 6z + 5 = 0 ….(2)
By solving equation (1) and (2) we get
−3
y= 31

16
z= 16
31 −3
The point which is equidistant to the points A(3, 2, -1), B(1, -1, 0) and C(2, 1, 2) is ( 16
, 16
).

28. We hand to find value of (√x + √y)
8

Formula used: n
Cr =
n!

(n−r)!(r)!

(a + b)n = C a + C n
0
n n
1a
n−1
b +
n
C2 a
n−2 2
b +…… +
n
Cn−1 ab
n−1
+
n
Cn b
n

We have, (√−
x + √y)
8

1 1

We can write √−
x as x 2 and √y as y 2

8
1 1

Now, we have to solve for (x 2 + y 2 )

8−0 8−1 1 8−2 2 8−3 3


2 1 2 1 1 1 1
8
= [8C0 (x 2
) ] + [8C1 (x 2
) (y 2
) ] + [8C2 (x 2
) (y 2
) ] + [ C3 (x 2
) (y 2
) ]

8−4 8−5 5 8−6 6


1 4 1 2 1 1
8 1 8 8
+ [ C4 (x 2 ) ( ) ] + [ C5 (x 2 ) (y 2 ) ]+ [ C6 (x 2 ) (y 2 ) ]
y2

8−7 7 8
1 1 1

+ [8C7 (x 2 ) (y 2 ) ] + [8C8 (y 2 ) ]

5 2 1 6 2
8! 8! 8!
= [ (x 2 )] + [ (x 2 ) (y 2 )] + [ (x 2 ) (y 2 )]
0!(8−0)! 1!(8−1)! 2!(8−2)!

5 3 4 4
8! 8!
+[ (x 2 ) (y 2 )] + [ (x 2 ) (y 2 )]
3!(8−3)! 4!(8−4)!

2 5 2 1 5
8! 8! 6 8! 2 8!
+[ (x 2 ) (y 2 )] + [ (x 2 )( )] + [ (x 2 )( )] + [ (y 2 )]
2 2
5!(8−5)! 6!(8−6)! y 7!(8−7)! y 8!(8−8)!

7 1 5
4 3 2
= [1 (x )] + [8 (x 2 ) (y 2 )] + [28 (x ) (y)] + [56 (x 2 )( )]
2
y

3 5 1 2
2 2 2 3 4
+ [70 (x ) (y )] + [56 (x 2
) (y 2
)] + [28 (x ) (y )] + [8 (x 2
) (y 2
)] + [1 (y )]

OR
Here (x + a)n =n C 0x
n n
+ C1 x
n−1
a+ C2 x
n n−2 2
a +. . . +
n
Cn a
n

= P + Q . . . (i)
where P = C x n
0
n n
+ C3 x
n−3 3
a +. . . .

n n−1 n n−3 3
Q= C1 x a+ C3 x a +. . . .

nn
Also (x − a) = C x − C x a+ C x
n n
0
n n n−1 n
2
n−2 2
a +. . . + (−1) Cn a
n
. . . (ii)
=P-Q
(i) Squaring and adding (i) and (ii) we have
(x + a)2n + (x - a)2n = (P + Q)2 + (P - Q)2
= P2 + Q2 + 2PQ + P2 + Q2 - 2PQ
= 2P2 + 2Q2 = 2(P2 +Q2)
(ii) Multiplying(i) and (ii) we have
(x + a)n (x - a)n = (P + Q)(P - Q)
(x2 - a2)n = P2 - Q2
29. We have,
f(x) = mx + c ....(i)
Differentiating with respect to x, we get
f'(x) = m.1 + 0
⇒ f'(x) = m ....(ii)

Put, x = 0 in (i) and (ii), we get


f(0) = c and f'(0) = m

Page 11 of 17
⇒ 1 = c and 1 = m [ ∴ f(0) = f'(0) = 1]
Put the values of m and c in f(x) = mx + c, we get f(x) = x + 1.
∴ f(2) = 2 + 1 = 3. [Put x = 2 in f(x) = x +1]

OR
We have,
d n n−1
x = nx
dx

Therefore,
d

dx
3x
−5
= 3(-5)x-5-1
= -15x-6
a
30. Let three number in G.P. are r
, a, ar
Here,
a
× a × ar = 729
r

3
⇒ a = 729

⇒ a=9
From the given conditions we can write ,
a a
( × a) + (a × ar) + ( × ar) = 819
r r
81
⇒ + 81r + 81 = 819
r
9
⇒ + 9r + 9 = 91
r

⇒ 9 + 9r2 + 9r = 91r
⇒ 9r2 - 82r + 9 = 0
⇒ 9r2 - 81r - r + 9 = 0
⇒ 9r(r - 9) -1(r - 9) = 0

r = 9, 1

So, required G.P. are


81, 9, 1, ....
or, 1, 9, 81, ....
OR
Let a

r
, a, ar be first three terms of the given G.P.
a

r
+ a + ar =
39

10
...(i)
(
a

r
)(a)(ar) = 1 ...(ii)
From (ii) we obtain a3 = 1 ⇒ a = 1 (considering real roots only)
Substituting a = 1 in equation (i), we obtain
1 39
+ 1 + r=
r 10

2 39
⇒ 1 + r+ r = r
10

⇒ 10 + 10r + 10r2 - 39r = 0


⇒ 10r2 - 29r + 10 = 0
⇒ 10r2 - 25r - 4r + 10 = 0
⇒ 5r(2r - 5)-2(2r - 5) = 0
⇒ (5r - 2)(2r - 5) = 0

⇒ r= or 2

5
5

corresponding terms of the G.P


i. when r = 2

5
5 2

2
, 1, 5
5
ii. when r = 2


2

5
, 1, 5

31. Given that, n(T) = 12


n(C) = 15
n(T ∩ C) = 7
i. n(T ∪ C) = n(T) + n(C) - n(T ∩ C)
= 12 + 15 - 7

Page 12 of 17
n(T ∪ C) = 20
20 members like at least one of the two drinks.
ii. Only tea but not coffee
= n(T) - n(T ∩ C)
= 12 - 7
=5
iii. Only coffee but not tea
= n(C) - n(T ∩ C)
= 15 - 7
=8
iv. Neither tea nor coffee
= n(U) - n(T ∪ C)
= 25 - 20
=5
Section D
32. We make the table from the given data:

Class marks Mid value (xi) di = xi -a = xi - 45 fi fidi d


i
2
fid 2
i

0-10 5 -40 3 -120 1600 4800

10-20 15 -30 2 -60 900 1800

20-30 25 -20 4 -80 400 1600

30-40 35 -10 6 -60 100 600

40-50 45 0 5 0 0 0

50-60 55 10 5 50 100 500

60-70 65 20 5 100 400 2000

70-80 75 30 2 60 900 1800

80-90 85 40 8 320 1600 12800

90-100 95 50 5 250 2500 12500

∑ fi = 45 ∑ fidi = 460 ∑ fid = 38400


2
i

Let a = 45.
∑ fi di
∴ Mean = a + ∑f
i
460
= 45 + 45

= 45 + 10.22 = 55.22
−−−−−−−−−−−−−−−
2 2
∑ fi d ∑ fi di
∴ Standard deviation = √ i
− ( )
∑ fi ∑ fi

−−−−−−−−−−−−−
=√ 38400

45
− (10.22)
2

−−−−−−−−−−−−−
= √853.33 − 104.45
−−−−−
= √748.88
= 27.36
2 2
y
33. The equation of given ellipse is x

100
+
400
= 1

Now 400 > 100 ⇒ a2 = 400 and b2 = 100


2
y 2

So the equation of ellipse in standard form is 2


+
x

2
= 1
a b

∴ a2 = 400 ⇒ a = 20 and b2 = 100 ⇒ b = 10


−−−−−−
We know that c = √a 2
− b
2

−−−−−− −− −−− –
∴ c = √400 − 100 = √300 = 10√3

∴ Coordinates of foci are (0, ±c) i.e. (0, ±10√3)
Coordinates of vertices are (0, ±a) i.e. (0, ±20)

Page 13 of 17
Length of major axis = 2 a = 2 × 20 = 40
Length of minor axis = 2 b = 2 × 10 = 20
10√3 √3
Eccentricity (e) = c

a
=
20
=
2
2
2b 2×100
Length of latus rectum = a
=
20
= 10

OR
Given that:
9x2 + y2 = 36.
After divide by 36 to both the sides, we get
2 2
y
9

36
2
x +
1

36
y
2
= 1 ⇒
x

4
+
36
= 1 ... (i)
Now, the above equation is of the form,
2 2
y
x

2
+
2
= 1 ... (ii)
b a

Comparing Eq. (i) and (ii), we get


a2 = 36 and b2 = 4 ⇒ a = √36 and b = √4 ⇒ and a = 6 and b = 2
−− –

i. Length of major axes


∴ Length of major axes = 2a = 2 × 6 = 12 units

ii. Length of major axes


∴ Length of major axes = 2b = 2 × 2 =4 units

iii. Coordinates of the Vertices


∴ Coordinates of the Vertices = (0, a) and (0, -a) = (0, 6) and (0, -6)

iv. Coordinates of the foci


As we know that, Coordinates of foci = (0, ±c) where c2 = a2 - b2
Now
c2 = 36 - 4 ⇒ c2 = 32 ⇒ c = √32 ... (iii)
−−

−−
Coordinates of foci = (0, ±√32)

v. Eccentricity
√32
As we know that, Eccentricity = c

a
⇒ e=
6
[from (iii)]
vi. Length of the Latus Rectum
2
2 2×(2)
As we know that, Length of Latus Rectum = 2b

a
=
6
=
8

6
4
=
3

34. The given system of linear inequalities is


2(2x + 3) -10 < 6(x - 2) ....(i)
2x−3 4x
and 4
+ 6 ≥ 2 +
3
...(ii)
From inequality (i), we get
2(2x + 3) - 10 < 6(x - 2)
⇒ 4x + 6 - 10 < 6x - 12

⇒ 4x - 4 < 6x - 12

⇒ 4x - 4 + 4 < 6x - 12 + 4 [adding 4 on both sides]

⇒ 4x < 6x - 8

⇒ 4x - 6x < 6x - 8 - 6x [subtracting 6x from both sides]


⇒ -2x < -8
⇒ 2x > 8 [dividing both sides by - 1 and then inequality sign will change]

⇒ > [dividing both sides by 2]


2x

2
8

∴ ...(iii)
x > 4

Thus, any value of x greater than 4 satisfies the inequality.


∴ Solution set is x ∈ (4, ∞)

The representation of solution of inequality (i) is

From inequality (ii), we get

Page 14 of 17
2x−3 4x 2x−3+24 6+4x
+ 6 ≥ 2 + ⇒ ≥
4 3 4 3
2x+21 6+4x
⇒ ≥ ⇒ 3(2x + 21) ≥ 4(6 + 4x)
4 3

⇒ 6x + 63 ≥ 24 + 16x

⇒ −10x ≥ −39 ⇒ 10x ≤ 39


10x 39
⇒ ≤
10 10

⇒ ...(iv)
x ≤ 3.9

Thus, any value of x less than or equal to 3.9 satisfies the inequality.
∴ Solution set is x ∈ (−∞, 3.9] .
Its representation on number line is

From Eqs. (iii) and (iv), it is clear, that there is no common value of x, which satisfies both inequalities (iii) and (iv).
Hence, the given system of inequalities has no solution.
35. We know,
sin2 x + cos2 x = 1
cos2 x = 1 - sin2 x
−−−−−
cos x = ±√1 − 5

9
=± 2

Since, x ∈ ( π

2
, π)

cos x will be negative in second quadrant


therefore. cos x = − 2

We know,
cos 2x = 2 cos2 x - 1
−−−−−
x 1−cos x
cos 2
=±√ 2
−−−−−−
−2
1+( ) −


Now, cos = ±√
x 3 1
= ±
2 2 6

Since, x ∈ ( π

2
, π) ⇒
x

2
∈ (
π

4
,
π

2
)

cos x

2
will be positive in 1st quadrant.
x 1
So, cos 2
=
√6

We know,
cos 2x = 2 cos2 x - 1
cos x = 1 - 2 sin2 x

2
... [∵ cos x = − ] 2


2

3
= 1 - 2 sin2 x

2 sin2 x 2
= +12

sin2
2+3
2 x

2
= 3

sin2 = x 5

2 6


sin2
x 5

2
= ±√ 6

π x π π
Since, x ∈ ( 2
, π) ⇒
2
∈ (
4
,
2
)

x
sin 2
will be positive in 1st quadrant
So,


sin x

2
=√ 5

We know,
x
sin

tan x

2
= 2

x
cos
2

5

6

tan x

2
= 1

√6

Page 15 of 17

tan =√5 x

2


x x x 5 –
Hence, values of cos , sin , tan 2 2 2
are 1
,√ 6
and √5
√6

OR
We have to prove that cos 36o cos 42o cos 60o cos 78o = 1

16
.
LHS = cos 36o cos 42o cos 60o cos 78o
By regrouping the LHS and multiplying and dividing by 2 we get,
= cos 36o cos 60o (2 cos 78o cos 42o)
1

But 2 cos A cos B = cos (A + B) + cos (A - B)


Then the above equation becomes,
= cos 36o cos 60o (cos(78o + 42o) + cos(48o - 42o))
1

= cos 36o cos 60o (cos(120o) + cos(36o))


1

= cos 36o cos 60o (cos(180o - 60o) + cos(36o))


1

But cos(90o - θ) = sinθ and cos(180o - θ)= -cos(θ).


Then the above equation becomes,
= cos 36o cos 60o (-cos(60o) + cos(36o))
1

Now, cos(36o) =
√5+1

cos(60o)
1
= 2

Substituting the corresponding values, we get


√5+1 √5+1
= 1

2
(
4
)(
1

2
) (−
1

2
+
4
)

√5+1 √5+1−2
=( 16
)(
4
)

2 2
( √5) − 1
=( 16×4
)

5−1
=( 64
)

16

LHS = RHS
Hence proved.
Section E

36. i. Number of relations = 2mn


= 23× 6 = 218
ii. Number of relations = 2mn
= 22× 2 = 24 = 16
iii. R = {(x, y): x ∈ P, y ∈ Q and x is the square of y}
OR
Here, W denotes the set of whole numbers.
We have 2a + b = 5 where a, b ∈ W
∴ a = 0 ⇒ b = 5

⇒ a = 1 ⇒ b = 5 − 2 = 3

and a = 2 ⇒ b = 1
For a > 3, the values of b given by the above relation are not whole numbers.
∴ A = {(0, 5), (1, 3), (2, 1)}

37. i. Let E1 and E2 denotes the events that Ankit and Vinod will respectively qualify the exam.
P (E1 ∪ E2 ) = P(E1) + P(E2) − P(E1 ∩ E2)
= 0.05 + 0.10 - 0.02 = 0.13
ii. Let E1 and E2 denotes the events that Ankit and Vinod will respectively qualify the exam.
Probability of atleast one of them does not qualify
′ ′ ′
= P (E ∪ E ) = P ((E1 ∩ E2 ) )
1 2

= 1 - p(E1 ∩ E2) = 1 - 0.02 = 0.98

Page 16 of 17
iii. Let E1 and E2 denotes the events that Ankit and Vinod will respectively qualify the exam.
′ ′ ′
= P (E ∩ E ) = P ((E1 ∪ E2 ) )
1 2

= 1 - P(E1 ∪ E2) = 1 - 0.13 = 0.87


OR
Let E1 and E2 denotes the events that Ankit and Vinod will respectively qualify the exam.
The probability that Vinod will not qualify the exam.
Probability that only one of them will qualify the exam = P((E1 - E2) ∪ (E2 - E1))
= P(E1 - E2) + P(E2 - E1)
= P(E1 ∪ E2) - P(E1 ∩ E2)
= 0.13 - 0.02 = 0.11
38. i. Let z = 1 + 2i
−−−− –
⇒ |z| = √1 + 4 = √5
7−z 7−1−2i
Now, f(z) = =
1−z 2 1−(1+2i)
2

6−2i 6−2i
= 1−1−4i2−4i
=
4−4i

(3−i)(2+2i)
=
(2−2i)(2+2i)
2
6−2i+6i−2i 6+4i+2
= 2
=
4+4
4−4i
8+4i
= 8
= 1 +
1

2
i

f(z) = 1 + 1

2
−−−−− −−−
4+1 √5 |z|
∴ |f(z)| = √1 + 1

4
= √
4
=
2
=
2

ii. Given that: (z + 3)(z̄ + 3)


Let z = x + yi
So (z + 3)(z̄ + 3) = (x + yi + 3)(x - yi + 3)
= [(x + 3) + yi][(x + 3) - yi]
= (x + 3)2 - y2i2
= (x + 3)2 + y2
= |x + 3 + iy|2
= |z + 3|2
iii. The conjugate of -6 - 24i is -6 + 24i.
It is given that -6 + 24i = (x – iy) (3 + 5i)
-6 + 24i = 3x + 5xi - 3iy -5yi2
-6 + 24i = (3x + 5y) + i(5x - 3y)
Comparing the real and imaginary parts,
3x + 5y = -6
5x - 3y = 24
Solving these two equations we get x = 3 and y = -3.
Therefore, x = 3 and y = -3
Then x + y = 3 - 3 = 0
OR
z = 3 + 4i
⇒ z̄ = 3 - 4i

Page 17 of 17
Class XI Session 2024-25
Subject - Mathematics
Sample Question Paper - 8

Time Allowed: 3 hours Maximum Marks: 80

General Instructions:

1. This Question paper contains - five sections A, B, C, D and E. Each section is compulsory. However, there are

internal choices in some questions.

2. Section A has 18 MCQ’s and 02 Assertion-Reason based questions of 1 mark each.

3. Section B has 5 Very Short Answer (VSA)-type questions of 2 marks each.

4. Section C has 6 Short Answer (SA)-type questions of 3 marks each.

5. Section D has 4 Long Answer (LA)-type questions of 5 marks each.

6. Section E has 3 source based/case based/passage based/integrated units of assessment (4 marks each) with sub
parts.

Section A
1. sin π

12
=? [1]

a) (√3+1)
b) (√3−1)

2√2 2√2

c) (2√3+1)
d) −(√3−1)

3√2 2√2

2. If n(A) = 10, n(B) = 6 and n (C) = 5 for three disjoint sets A, B and C, then n(A ∪ B ∪ C ) = [1]

a) 11 b) 21

c) 1 d) 9
3. The mean of the series x1, x2, ..., xn is X . If x2 is replaced by λ , then what is the new mean?
¯¯¯
¯
[1]
¯ ¯
a) X −x2 −λ
b) nX −x2 −λ

n n

¯
c) X −x2 +λ
d) ¯¯¯
¯
X − x2 + λ
n

4. If f(x) = x sinx, then f' ( is equal to [1]


π
)
2

a) 1 b) 1

c) -1 d) 0
5. The coordinates of the foot of perpendicular from (0, 0) upon the line x + y = 2 are [1]

a) (1, 1) b) (1, -2)

c) (-1, 2) d) (1, 2)
6. The length of the foot of perpendicular drawn from the point P (3, 4, 5) on y-axis is [1]
−−
a) √34 b) 10

Page 1 of 19
−−− –
c) √113 d) 5√2

7. Mark the correct answer for 3i34 + 5i27 - 2i38 + 5i41 = ? [1]

a) 1 b) -1

c) -4i d) 10i
8. A fair dice is rolled n times. The number of all the possible outcomes is [1]

a) 6n b) n6

c) 6n d) 6+n
2 3 dy
9. If y = 1 +
x
+
x
+
x
+ ..., then = [1]
1! 2! 3! dx

a) y2 b) y + 1

c) y d) y - 1
−−−−−−−−−−−
10. If

< α < π ,then √2 cot α +
1

2
is equal to [1]
4 sin α

a) - 1 + cot α b) - 1 - cot α

c) 1 - cot α d) 1 + cot α

11. Each set Xr contains 5 elements and each set Yr contains 2 elements and
20 n

⋃ xr = S = ⋃ Yr . If each element [1]


r=1 r=1

of S belong to exactly 10 of the Xr’s and to exactly 4 of the Yr’s, then n is

a) 10 b) 20

c) 50 d) 100

12. In the expansion of (x + a)n, if the sum of odd terms be P and the sum of even terms be Q, then 4PQ = ? [1]

a) (x + a)n - ( x - a )n b) (x + a)2n - (x - a)2n

c) (x + a)n + ( x - a )n d) (x + a)2n + (x - a)2n


n
13. If (1 − x + x 2
) = a0 + a1 x + a2 x
2
+ … + a2n x
2n
, then a0 + a2 + a4 + … +a2n equals. [1]
n

a) 3 n
+
1

2
b) 3

2
+1

n n

c) 3

2
−1
d) 1−3

14. If x is a real number and | x | < 3 , then [1]

a) - 3 < x < 3 b) x ≥ − 3

c) x ≥ 3 d) − 3 ≤ x ≤ 3

15. Which of the following is a set? [1]


A. A collection of vowels in English alphabets is a set.
B. The collection of most talented writers of India is a set.
C. The collection of most difficult topics in Mathematics is a set.
D. The collection of good cricket players of India is a set.

a) B b) D

c) A d) C
16. If 3 sin x + 4 cos x = 5, then 4 sin x - 3 cos x = [1]

Page 2 of 19
a) 1 b) 5

c) 3 d) 0
17. lim
sin x

x−π
is equal to [1]
x→π

a) 1 b) -1

c) 2 d) -2
18. The value of ( 7
C0 +
7
C1 ) +( 7 7
C1 + C2 ) + .... + ( 7
C6 +
7
C7 ) is [1]

a) 28 - 2 b) 28 - 1

c) 27 - 1 d) 28

19. Assertion (A): The collection of all natural numbers less than 100' is a set. [1]
Reason (R): A set is a well-defined collection of the distinct objects.

a) Both A and R are true and R is the correct b) Both A and R are true but R is not the
explanation of A. correct explanation of A.

c) A is true but R is false. d) A is false but R is true.


20. Assertion (A): If the sum of first two terms of an infinite GP is 5 and each term is three times the sum of the [1]
succeeding terms, then the common ratio is 1

4
.
Reason (R): In an AP 3, 6, 9, 12 ........ the 10th term is equal to 33.

a) Both A and R are true and R is the correct b) Both A and R are true but R is not the
explanation of A. correct explanation of A.

c) A is true but R is false. d) A is false but R is true.


Section B
−−−−− −−−−−
21. Let f, g be two real functions defined by f(x) = √x + 1 and g(x) = √9 − x
2
Then describe each of the [2]
following functions: f + g.
OR
|x−4|
Find the domain and the range of the real function: f(x) =
x−4
3

22. Evaluate lim


x −1
. [2]
x−1
x→1

23. If A and B are two events associated with a random experiment such that P(A) = 0.25, P(B) = 0.4 and P(A or B) [2]
= 0.5, find the values of
i. P(A and B)
ii. P(A and B
¯
)
OR
Check whether the probabilities P(A) and P(B) are consistently defined P(A) = 0.5, P(B) = 0.4, P (A ∪ B) = 0.8

24. In a group of students, 100 students know Hindi, 50 know English and 25 know both. Each of the students [2]
knows either Hindi or English. How many students are there in the group?
25. Find the length of perpendicular from the origin to the lines 7x + 24y = 50. [2]
Section C
26. In how many ways can six persons be seated in a row? [3]
27. Find the point in yz-plane which is equidistant from the points A(3, 2, -1), B(1, -1, 0) and C(2, 1, 2). [3]
28. Find (x + 1)6 + (x - 1)6. Hence or otherwise evaluate
– 6

( √2 + 1) + ( √2 − 1)
6 [3]

Page 3 of 19
OR
Using g binomial theorem, expand {(x + y) and hence find the value of
5 5
+ (x − y) }

– 5
– 5
{( √2 + 1) + ( √2 − 1) }

29. Evaluate the following limits: lim . [3]


2x

a+x−√a−x
x→0 √

OR

Differentiate eax+b from first principle.


30. The sum of three numbers a, b, c in A.P. is 18. If a and b are each increased by 4 and c is increased by 36, the [3]
new numbers form a G.P. Find a, b, c.
OR
If A.M. and G.M. of roots of a quadratic equation are 8 and 5 respectively then obtain the quadratic equation.

31. Are the E = {x : x ∈ Z, x2 ≤ 4} and F = {x : x ∈ Z, x2 = 4} pairs of equal set? [3]


Section D
32. The mean and standard deviation of 100 observations were calculated as 40 and 5.1, respectively by a student [5]
who took by mistake 50 instead of 40 for one observation. What are the correct mean and standard deviation?
33. Fine the lengths major and minor axes, coordinates of the vertices, coordinates of the foci, eccentricity, and [5]

length of the latus rectum of the ellipse 25x2 + 4y2 = 100.


OR
A visitor with sign board 'DO NOT LITTER' is moving on a circular path in an exhibition. During the movement he
stops at points represented by (3, - 2) and (-2, 0). Also, centre of the circular path is on the line 2x - y = 3. What is the
equation of the path? What message he wants to give to the public?
34. Solve the following system of linear inequalities [5]
4x

3

9

4
<x+ 3

4
and 7x−1

3
- 7x+2

6
> x.
35. Prove that: cot x + cot(
π
+ x) + cot(

+ x) = 3 cot 3x. [5]
3 3

OR
sin 2A+sin 2B+sin 2C
If A + B + C= π, prove that = 8 sin
A B C
sin sin
sin A+sin B+sin C 2 2 2

Section E
36. Read the following text carefully and answer the questions that follow: [4]
Method to Find the Sets When Cartesian Product is Given
For finding these two sets, we write first element of each ordered pair in first set say A and corresponding
second element in second set B (say).
Number of Elements in Cartesian Product of Two Sets
If there are p elements in set A and q elements in set B, then there will be pq elements in A × B i.e. if n(A) = p
and n(B) = q, then n(A × B) = pq.
i. The Cartesian product A × A has 9 elements among which are found (-1, 0) and (0, 1). Find the set A and
the remaining elements of A × A. (1)
ii. A and B are two sets given in such a way that A × B contains 6 elements. If three elements of A × B are (1,
3), (2, 5) and (3, 3), then find the remaining elements of A × B. (1)
iii. If the set A has 3 elements and set B has 4 elements, then find the number of elements in A × B. (2)
OR
If A × B = {(a, 1), (b, 3), (a, 3), (b, 1), (a, 2), (b, 2)}. Find A and B. (2)
37. Read the following text carefully and answer the questions that follow: [4]

Page 4 of 19
On her vacation, Priyanka visits four cities. Delhi, Lucknow, Agra, Meerut in a random order.

i. What is the probability that she visits Delhi before Lucknow? (1)
ii. What is the probability she visit Delhi before Lucknow and Lucknow before Agra? (1)
iii. What is the probability she visits Delhi first and Lucknow last? (2)
OR
What is the probability she visits Delhi either first or second? (2)
38. Two complex numbers Z1 = a + ib and Z2 = c + id are said to be equal, if a = c and b = d. [4]

i. If (x + iy)(2 - 3i) = 4 + i then find the value of (x, y). (1)


2
(1+i)
ii. If 2−i
= x + iy, then find the value of x + y. (1)
100
1−i
iii. If ( 1+i
) = a + ib, then find the values of a and b. (2)
OR
If (a - 2, 2b + 1) = (b - 1, a + 2), then find the real values of a and b. (2)

Page 5 of 19
Solution
Section A
1.
( √3−1)
(b)
2√2

π π π π π π π
Explanation: sin 12
= sin(
4

6
) = sin
4
cos
6
− cos
4
sin
6

√3 ( √3−1)
1 1 1
= ( × ) − ( × )=
√2 2 √2 2 2√2

2.
(b) 21
Explanation: Since A, B, C are disjoint
∴ n(A ∪ B ∪ C ) = n(a) + n(B) + n(C)

= 10 + 6 + 5 = 21
3.
¯
nX− x2 −λ
(b) n
x1 + x2 + … + xn ¯¯¯
¯
Explanation: We know, X
¯
=
n
⇒ x1 + x2 + ... + xn = nX
¯¯¯
¯
⇒ x1 + x2 + ... + xn = nX - x2
¯¯¯
¯
⇒ x1 + x3 + ... + xn + λ = nX - x2 + λ
x1 + x3 + … + xn + λ
⇒ Mean = Sum of all values

Total number of values


=
n
¯
nX− x2 −λ
= n

4. (a) 1
Explanation: f′ (x) = x cosx + sinx
So, f ( ) = cos + sin = 1
′ π

2
π

2
π

2
π

5. (a) (1, 1)
Explanation: The equation of the line perpendicular to the given line is x - y + k = 0
Since it passes through the origin,
0-0+k=0
Therefore, k = 0
Hence the equation of the line is x - y = 0
On solving these two equations we get x = 1 and y = 1
The point of intersection of these two lines is (1, 1)
Hence the coordinates of the foot of the perpendicular is (1, 1)
−−
6. (a) √34
Explanation: Let l be the foot of the perpendicular from point P on the y-axis. Therefore, its x and z-coordinates are zero, i.e.,
−−−−− −−
(0, 4, 0). Therefore, the distance between the points (0, 4, 0) and (3, 4, 5) is √9 + 25 = √34 .
7.
(b) -1
Explanation: 3i34 + 5i27 - 2i38 + 5i41 = 3× (i4)8× i2 + 5× (i4)6× i3 -2× (i4)9× i2 + 5× (i4)10× i
= 3× 1× (-1) + 5× 1× (-i)-2× 1× (-1) + 5× 1× i
= -3 - 5i + 2 + 5i = -1
8.
(c) 6n
Explanation: Each time there are 6 possibilities, therefore for n times there are 6n possibilities.
9.
(c) y

Page 6 of 19
2 3

Explanation: y = 1 + x

1!
+
x

2!
+
x

3!
+ ...
dy 2 2

Differentiating both sides with respect to x, we get dx


=
d

dx
(1 +
x
+
x
+
x
+ …)
1! 2! 3!

2 3 4

= d

dx
(1) +
dw
d
(
11
x
)+
dw
d
(
x

2!
) +
dw
d
(
x

3!
) +
d

dx
(
x

4!
) + ...
= d

dx
(1) +
1

1! dx
d
(x) +
1

2! dw
d
(x ) +
2 1

3! dw
d 3
(x ) +
1

4! dw
d
(x )
4
+ ...
dy
=0+ 1
× 1+ 1
× 2α +
1
× 3α
2
+
1
× 4α
3
+ ... (y = α 2
⇒ = nα n−1
)
1! 2! 3! 4! ∂α
2 3

=1+ x

1!
+
x

2!
+
x

3!
+ ... [ x

n!
=
1

(n−1)!
]

=y
dy

dx
=y

10.
(b) - 1 - cot α
Explanation: We have:
−−−−−−−−−−−−
1
√2 cot α + 2
sin α
−−−−−−−−−−−
2 cos α 1
= √ +
sin α 2
sin α
−−−−−−−−−
2 sin α cos α+1
= √
2
sin α
−−−−−−−−−−−−−−−−
2 2
2 sin α cos α+ sin α+ cos α
= √
2
sin α

−−−−−−−−−
2
(sin α+cos α)
= √
2
sin α

−−−−−−−− −
2
= √(1 + cot α)

= |1 + cot α|

= −(1 + cot α) [When 4
< α < π, cot α < −1 ⇒ cot α + 1 < 0]
= - 1 - cot α
11.
(b) 20
Explanation: The correct answer is (B)
20

Since, n(Xr) = 5, ⋃ Xr = S, we obtain n(S) = 100


r=1

But each element of S belong to exactly 10 of the X ’s


Thus, = 10 are the number of distinct elements in S.
100

10

Also each element of S belong to exactly 4 of the Yr’s and each Yr'scontain 2 elements. If S has n number of Yr in it.
Then 2n

4
= 10
which gives n = 20
12.
(b) (x + a)2n - (x - a)2n
Explanation: P + Q = (x + a) and P - Q = (x - a) n n

⇒ 4PQ = (P + Q)2 - (P - Q)2 = (x + a)2n - (x - a)2n


13.
n
3 +1
(b) 2
n
Explanation: (1 − x + x 2
) = a0 + a1 x + a2 x
2
+ … + a2n x
2n
..(1)
Put x=1 in (1),we get
1 = a0 + a1 + a2 + a3 + … + a2n ..(2)
Put x=-1 in(1),we get
3
n
= a0 − a1 + a2 − a3 + … + a2n ..(3)
Adding(1) and(2),we get
n
3 + 1 = 2 (a0 + a2 + a4 + … + a2n )
n
3 +1
Thus, a 0 + a2 + a4 + … + a2n =
2

14. (a) - 3 < x < 3


Explanation: We have |x| < a ⇔ −a < x < a

Page 7 of 19
15.
(c) A
Explanation: The set is {a, e, i, o, u}
16.
(d) 0
Explanation: 3 sinx + 4 cosx = 5
3 4
sin x + cos x = 1
5 5

Let cos α = 3

5
and sin α = 4

∴ cos α sin x + sin α cos x = 1


π
⇒ sin(α + x) = sin
2

π
⇒ α + x =
2

⇒ x =
π

2
− α .... (i)
We have to find the value of 4 sin x - 3 cos x
− α) .... From eq. (i)
π π
4sin( − α) − 3cos(
2 2

= 4 cos α − 3 sin α

3 4 3 4
= 4 × − 3 × (∵ cos α = and sin α = )
5 5 5 5

0
17.
(b) -1
sin(π−x)
Explanation: Given, lim
sin x

x−π
= lim
x→π x→π −(π−x)

sin x
= −1 [∵ lim = 1 and π − x → 0 ⇒ x → π]
x
x→0

18. (a) 28 - 2
Explanation: ( 7
C0 +
7
C1 ) + ( C1 +
7 7 7
C2 ) + ( C2 +
7
C3 ) +(7 7
C4 +
7 7
C5 ) + ( C5 +
7 7
C6 ) + ( C6 + C7 )

=1+2× 7
C1 + 2 ×
7
C2 + 2 ×
7
C3 +2× 7
C5 + 2 ×
7
C6 + 1

=1+2× 7
C1 + 2 ×
7
C2 + 2 ×
7
C3 +2× 7
C2 + 2 ×
7
C6 + 1

=2+2 ( 2 7
C1 +
7
C2 +
7
C3 )

=2+2 2
(7 +
7

2
× 6 +
7

3
×
6

2
× 5)

= 2 + 252
= 254
= 28 - 2
19. (a) Both A and R are true and R is the correct explanation of A.
Explanation: Assertion The collection of all natural numbers less than 100', is a well-defined collection. So, it is a set.
20.
(c) A is true but R is false.
Explanation: Assertion Let a be the first term and r(|r| < 1) be the common ratio of the GP.
∴ The GP is a, ar, ar2,...
According to the question,
T1 + T2 = 5 ⇒ a + ar = 5 ⇒ a(1 + r) = 5
and Tn = 3(Tn+1 + Tn+2 + Tn+3 +...)

⇒ arn-1 = 3(arn + arn+1 + arn+2 + ...)


⇒ arn-1 = 3arn(1 + r + r2 + ...)
1
⇒ 1 = 3r( 1−r
)

⇒ 1 - r = 3r
⇒ r =
1

Reason: Given, 3, 6, 9, 12 ...


Here, a = 3, d = 6 - 3 = 3
∴ T10 = a + (10 - 1)d

Page 8 of 19
=3+9×3
= 3 + 27 = 30
Section B
21. According to the question , we can state,
We know the square of a real number is never negative.
Clearly, f(x) takes real values only when x + 1 ≥ 0
= x > -1
∴ x ∈ [−1, ∞)

Thus, domain of f = (−1, ∞)


Similarly, g(x) takes real values only when 9 - x2 ≥ 0
= 9 > x2
= x2 < 9
= x2 - 9 < 0
= x2 - 32 < 0
= (x + 3) (x - 3) < 0
= x ≥ -3 and x < 3
x ∈ [−3, 3]

Thus, domain of g = = [−3, 3]


i.f + g
We know (f + g)(x) = f(x) + g(x)
−−−−− −−−−−
2
∴ (f + g)(x) = √x + 1 + √9 − x

Domain of f + g = Domain of f ∩ Domain of g


= Domain of f + g = [−1, ∞) ∩ [−3, 3]
Domain of f + g = [-1, 3]
−−−−− −−−−−
Thus, f + g : [-1, 3] R is given by (f + g)(x) = √x + 1 + √9 − x 2

OR
|x−4|
Here we have, f(x) = x−4

We need to find where the function is defined.


To find the domain of the function f(x) we need to equate the denominator of the function to 0
Therefore,
x - 4 = 0 or x = 4
It means that the denominator is zero when x = 4
So, the domain of the function is the set of all the real numbers except 4
The domain of the function, D{f(x)} = (-∞ , 4) ∪ (4, ∞ )
The numerator is an absolute function of the denominator.
So, for any value of x from the domain set, we always get either +1 or -1 as the output.
So, the range of the function is a set containing -1 and +1
Therefore, the range of the function, Rf(x) = {-1, 1}
3
x −1
22. Given, lim x−1
x→1

Therefore, (x - 1) is a common factor of (x3 - 1) and (x


3
x −1
If we put x = 1, then expression x−1
becomes the indeterminate form 0

0
.

- 1).
Factorising the numerator and denominator, we have
3
x −1 0
lim
x−1
[ 0
form]
x→1
2
(x−1)(x +x+1)

= lim
x→1 (x−1)

= 2
lim (x + x + 1) = 12 + 1 + 1 = 3
x→1

23. i. It is given that


: P(A) = 0.25, P(A or B) = 0.5 and P(B) = 0.4
To find : P(A and B)
Formula used : P(A or B) = P(A) + P(B) - P(A and B)

Page 9 of 19
Substituting the value in the above formula we get,
0.5 = 0.25 + 0.4 - P(A and B)
0.5 = 0.65 - P(A and B)
P(A and B) = 0.65 - 0.5
P(A and B) = 0.15
ii. Given : P(A) = 0.25, P(A and B) = 0.15 ( from part (i))
To find : P(A and B
¯
)
Formula used : P(A and B¯
) = P(A) - P(A and B)
Substituting the value in the above formula we get,
P(A and B ¯
) = 0.25 - 0.15
P(A and B) = 0.10
¯

P(A and B ¯
) = 0.10
OR
Given that P(A) = 0.5, P(B) = 0.4 and P (A ∪ B) = 0.8
Applying the general addition rule,
P (A ∪ B) = P (A) + P (B) − P (A ∩ B)

∴ 0.8 = 0.5 + 0.4 − P (A ∩ B)

⇒ P (A ∩ B) = 0.9 − 0.8 = 0.1

∴ P (A ∩ B) < P (A) and P (A ∩ B) < P (B)

Thus the given probabilities are consistently defined.


24. Let H be the set of students who know Hindi and E be the set of students who know English.
Here n(H) = 100, n(E) = 50 and n(H ∩ E) = 25
We know that n(H ∪ E) = n(H) + n(E) − n(H ∩ E)
= 100 + 50 - 25 = 125.
25. Here, it is given: Point (0,0) and line 7x + 24y = 50
We have to find: The length of the perpendicular from the origin to the line 7x + 24y = 50
We know that the length of the perpendicular from P (m,n) to the line ax + by + c = 0 is given by,
|am+bn+c|
D=
√a2 + b2

The given equation of the line is 7x + 24y - 50=0


Here m= 0 and n = 0, a = 7, b = 24, c = -50
|7(0)+24(0)−50|
D=
√72 + 242

|0+0−50| |−50| |−50| 50


D= = = = = 2
√49+576 √625 25 25

D=2
Therefore, the length of perpendicular from the origin to the line 7x + 24y = 50 is 2 units.
Section C
26. Given: Six persons are to be arranged in a row.
Assume six seats, now in the first seat, any one of six members can be seated, so the total number of possibilities is 6C1

Similarly, in the second seat, any one of five members can be seated, so the total number of possibilities is 5C1

In the third seat, any one of four members can be seated, so the total number of possibilities is 4C1

In the fourth seat, any one of three members can be seated, so the total number of possibilities is 3C1

In the fifth seat, any one of two members can be seated, so the total number of possibilities is 2C1

Page 10 of 19
In the sixth seat, only one remaining person can be seated, so the total number of possibilities is 1C1

Hence the total number of possible outcomes = 6C1 × 5C1 × 4C1 × 3C1 × 2C1 × 1C1 = 6 × 5 × 4 × 3 × 2 × 1 = 720
27. The general point on yz plane is D(0, y, z).
Consider this point is equidistant to the points A(3, 2, -1), B(1, -1, 0) and C(2, 1, 2).
∴ AD = BD
−−−−−−−−−−−−−−−−−−−−−− − −−−−−−−−−−−−−−−−−−−−−− −
√(0 − 3)2 + (y − 2)2 + (z + 1)2 = √(0 − 1)2 + (y + 1)2 + (z − 0)2

Squaring both sides,


(0 - 3)2 + (y - 2)2 + (z + 1)2 = (0 - 1)2 + (y + 1)2 + (z - 0)2
9 + y2 - 4y + 4 + z2 + 2z + 1 = 1 + y2 + 2y + 1+ z2
-6y + 2z + 12 = 0 ….(1)
Also, AD = CD
−−−−−−−−−−−−−−−−−−−−−− − −−−−−−−−−−−−−−−−−−−−−− −
√(0 − 3)2 + (y − 2)2 + (z + 1)2 = √(0 − 2)2 + (y − 1)2 + (z − 2)2

Squaring both sides,


(0 - 3)2 + (y - 2)2 + (z + 1)2 = (0 - 2)2 + (y - 1)2 + (z - 2)2
9 + y2 - 4y + 4 + z2 + 2z + 1 = 4 + y2 - 2y + 1+ z2 - 4z + 4
-2y + 6z + 5 = 0 ….(2)
By solving equation (1) and (2) we get
−3
y= 31

16
z= 16
31 −3
The point which is equidistant to the points A(3, 2, -1), B(1, -1, 0) and C(2, 1, 2) is ( 16
, 16
).
28. (x + 1)6 + (x - 1)6 == [ 6 6 6 5 6 4 6
C0 x + C1 x + C2 x + C3 x + C4 x + C5 x+ C6 ]
3 6 2 6 6

6 6 6 5 6 4 2 6 3 3 6 2 4 6 5 6 6
+ [ C0 x + C1 x (−1)+ C2 x (−1) + C3 x (−1) + C4 x (−1) + C5 x(−1) + C6 (−1) ]

= [x6 + 6x5 + 15x4 + 20x3 + 15x + 6x + 1] + [x6 - 6x5 + 15x4 - 20x3 + 15x2 - 6x + 1]
2

= 2x6 + 30x4 + 30x2 + 2


= 2(x6 + 15x4 + 15x2 + 1)

Putting x = √2
– 6
– 6
– 6 – 4 – 2
(√2 + 1) + (√2 − 1) = 2[(√2) + 15(√2) + 15(√2) + 1]

= 2 [8 + 15 × 4 + 15 × 2 + 1]

= 2 [8 + 60 + 30 + 1]
= 2 × 99 = 198

OR
W e have

5 5 5 5 5 3 2 5 1 4
(x + y) + (x − y) = 2 [ C0 x + C2 x y + C4 x y ]

5 3 2 4
= 2 (x + 10x y + 5xy

P utting x = √2 and y = 1, we get

– 5
– 5
– 5 – 3 –
(√2 + 1) + (√2 − 1) = 2 [ (√2) + 10(√2) + 5√2]

– – –
= 2 [4 √2 + 20 √2 + 5√2]


= 58√2

29. Given: lim a+x− √a−x


2x

x→0 √

Rationalizing the given equation,


( √a+x+ √a−x)
= lim 2x

x→0 ( √a+x− √a−x) ( √a+x+ √a−x)

Formula: (a + b)(a - b) = a2 - b2
2x( √a+x+ √a−x)
= lim a+x−a+x
x→0

2x( √a+x+ √a−x)


= lim 2x
x→0

( √a+x+ √a−x)
= lim 1
x→0

Page 11 of 19
Now we can see that the indeterminant form is removed, so substituting x as 0
Therefore, lim a+x− √a−x
= √−

2x −

a + √a = 2√a

x→0 √

OR
We need to find derivative of f(x) = eax + b
Derivative of a function f(x) is given by
f (x+h)−f (x)
f’(x) = lim h
{where h is a very small positive number}
h→0

∴ derivative of f(x) = eax + b is given as


f (x+h)−f (x)
f’(x) = lim h
h→0
a(x +h)+b ax +b
′ e −e
⇒ f (x) = lim
h
h→0
ax +b ah ax +b
′ e e −e
⇒ f (x) = lim
h
h→0

ax +b ah
e (e −1)

⇒ f (x) = lim
h
h→0

ah

eax + b× lim
′ e −1
⇒ f (x) = lim
h
h→0 h→0
ah
e −1
As one of the limits × lim h
can’t be evaluated by directly putting the value of h as it will take 0

0
form.
h→0

So we need to take steps to find its value.


eax + b× lim
ah
′ e −1
⇒ f (x) = lim ×a
ah
h→0 h→0
x
e −1
Use the formula: lim x
=logee = 1
x→0

⇒ f’(x) = eax + b × (a)


⇒ f’(x) = aeax + b
Hence,
Derivative of f(x) = eax + b = aeax + b
30. Let the first term of the A.P. be a and the common difference be d.
∴ a = a, b = a + d and c = a + 2d

a + b + c = 18
⇒ 3a + 3d = 18

⇒ a + d = 6 ...(i)
Now, according to the question, a + 4, a + d + 4 and a + 2d + 36 are in G.P.
∴ (a + d + 4)2 = (a + 4)(a + 2d + 36)
⇒ (6 - d + d + 4)2 = (6 - d + 4) (6 - d + 2d + 36) [using(i)]
⇒ (10)2 = (10 - d)(42 + d)
⇒ 100 = 420 + 10d - 42d - d2
⇒ d2 + 32d - 320 = 0
⇒ (d + 40)(d - 8) = 0

⇒ d = 8, -40

Now, substituting d = 8, -40 in equation (i), we obtain, a = -2, 46, respectively.


For a = -2 and d = 8, we obtain
a = -2, b = 6, c = 14
And for a = 46 and d = -40, we obtain
a = 46, b = 6, c = -34
OR
Let a and b be the roots of required quadratic equation.
a+b
Then A.M. = 2
=8

a + b = 16
−−
And G.M. = √ab = 5
⇒ ab = 25
Now, Quadratic equation x2 - (Sum of roots) x + (Product of roots) = 0

Page 12 of 19
⇒ x2 - (a + b)x + ab = 0
⇒ x2 - 16x + 25 = 0
Therefore, required equation is x2 - 16x + 25 = 0
31. We know two sets A and B are said to be equal if they have exactly the same elements & we write A = B
We have, E = {x : x ∈ Z, x2 ≤ 4}
Here, x ∈ Z and x2 ≤ 4
If x = -2, then x2 = (-2)2 = 4 = 4
If x = -1, then x2 = (-1)2 = 1 < 4
If x = 0, then x2 = (0)2 = 0 < 4
If x = 1, then x2 = (1)2 = 1 < 4
If x = 2, then x2 = (2)2 = 4 = 4
Therefore, E = {-2, -1, 0, 1, 2}
and F = {x : x ∈ Z, x2 = 4}
Here, x ∈ Z and x2 = 4
If x = -2, then x2 = (-2)2 = 4 = 4
If x = 2, then x2 = (2)2 = 4 = 4
Therefore, F = {-2, 2}
∴ E ≠ F because the elements in the both the sets are not equal.

Section D
32. We have, n = 100, x = 40 and σ = 5.1
¯¯
¯

¯¯
¯ 1
∴ x = Σxi
n

⇒ Σxi = nx = 100 × 40 = 4000


¯¯
¯

∴ Incorrect Σ xi = 4000
and,
σ = 5.1

⇒ σ
2= 26.01
xi2 - (mean)2 = 26.01
1
⇒ Σ
n


100
1
Σ xi2 - 1600 = 26.01

⇒ Σ xi2 = 1626.01 × 100

∴ Incorrect Σ xi2 = 162601


To correct the ∑ x , we need to subtract the incorrect observation 50 and add correct observation is 40.
i

We have, incorrect Σ xi = 4000


∴ Correct Σ xi = 4000 - 50 + 40 = 3990
and,
Similarly, to obtain correct ∑ x we need to subtract 502 and add 402 to incorrect one.
2
i

Incorrect Σ xi2 = 162601

∴ Correct Σ xi2 = 162601 - 502 + 402 = 161701


3990
Now, Correct mean = 100
= 39.90
(Correct Σ xi2) - (Correct mean)2
1
Correct variance = 100

⇒ Correct variance = 161701

100
− (
3990

100
)

2
161701×100−(3990)
⇒ Correct variance = 2
(100)

16170100−15920100
⇒ Correct variance = 10000
= 25
−−
∴ Correct standard deviation = √25 =5
33. Given that:
25x2 + 4y2 = 100

Page 13 of 19
after divide by 100 to both the sides, we get
2 2
y
25

100
x
2
+
100
4
y
2
= 1 ⇒
x

4
+
25
= 1 ... (i)
Now, above equation is of the form,
2
2 y
x

2
+
2
= 1 ... (ii)
b a

Comparing eq. (i) and (ii), we get


a2 = 25 and b2 = 4 ⇒ a = √25 and b = √4 ⇒ a = 5 and b = 2
−− –

i. Length of major axes


∴ Length of major axes = 2a = 2 × 5 = 10 units

ii. Length of minor axes


Length of minor axes = 2b = 2 × 2 = 4 units
iii. Coordinates of the vertices
Coordinates of the vertices = (0, a) and (0, -a) = (0, 5) and (0, -5)

iv. Coordinates of the foci


As we know that,
Coordinates of foci = (0, ±c) where c2 = a2 - b2
Now
c2 = 25 - 4 ⇒ c2 = 21 ⇒ c = √21 ... (iii)
−−


∴ Coordinates of foci = (0, ± √21)

v. Eccentricity
√21
As we know that, Eccentricity = c

a
⇒ e=
5

vi. Length of the Latus Rectum


2 2
2×(2)
As we know that Length of Latus Rectum = 2b

a
=
5
=
8

OR
Let the equation of circle whose centre (- g, - f) be
x2 + y2 + 2 gx + 2 fy + c = 0 ...(i)
Since, is passes through points (3, - 2) and (- 2, 0)
∴ (3)2 + (- 2)2 + 2g (3) + 2f (- 2) + c = 0
and (- 2)2 + (0)2 + 2g (- 2) + 2f (0) + c = 0
⇒ 9 + 4 + 6g - 4f + c = 0

and 4 + 0 - 4g + 0 + c = 0
⇒ 6g - 4f + c = - 13

and c = 4g - 4 ...(ii)
∴ 6g - 4f + (4g - 4 ) = - 13

⇒ 10g - 4f = - 9 ...(iii)

Also, centre (- g, - f) lies on the line 2x - y = 3


∴ - 2g + f = 3 ...(iv)

On solving Eqs. (iii) and (iv), we get


3
g = and f = 6
2

On putting the values of g and f in Eq. (ii), we get


c = 4( 3

2
) -4=6-4=2
On putting the values of g, f and c in Eq. (i), we get
x2 + y2 + 2 (
3

2
) x + 2(6) x + 2 = 0

⇒ x2 + y2 + 3x + 12x + 2 = 0
which is the required equation of the path
The message which he wants to give to the public is 'Keep your place clean'.
34. We have, − < x + ... (i)
4x

3
9

4
3

4
7x−1 7x+2
and 3

6
> x ... (ii)
From inequality (i), we get
16x−27 4x+3
4x

3
− <x+ ⇒ 9

4
< 3

4 12 4

Page 14 of 19
⇒ 16x - 27 < 12x + 9 [multiplying both sides by 12]
⇒ 16x - 27 + 27 < 12x + 9 + 27 [adding 27 on both sides]
⇒ 16x < 12x +36

⇒ 16x - 12x < 12x + 36 - 12x [ subtracting 12x from bot sides]
⇒ 4x < 36 ⇒ x < 9 [dividing both sides by 4]
Thus, any value of x less than 9 satisfies the inequality. So, the solution of inequality (i) is given by x ∈ (−∞, 9)

From inequality (ii) we get,


7x−1 7x+2 14x−2−7x−2

3
- 6
>x⇒ 6
>x
⇒ 7x - 4 > 6x [multiplying by 6 on both sides]
⇒ 7x - 4 + 4 > 6x + 4 [adding 4 on both sides]
⇒ 7x > 6x + 4

⇒ 7x - 6x > 6x + 4 - 6x [subtracting 6x from both sides]

∴ x>4
Thus, any value of x greater than 4 satisfies the inequality.
So, the solution set is x ∈ (4, ∞)

The solution set of inequalities (i) and (ii) are represented graphically on number line as given below:

Clearly, the common value of x lie between 4 and 9.


Hence, the solution of the given system is, 4 < x < 9 i.e., x ∈ (4, 9)
π 2π
35. We have to prove cot x + cot( 3
+ x) + cot( 3
+ x) = 3 cot 3x.

LHS = cot x + cot( π

3
+ x) + cot( 2π

3
+ x)

We know,
cot( 2π

3
+ x) = cot(π − ( π

3
− x)) = -cot( π

3
− x) ... (as - cotθ = cot (180o - θ)
Hence the above LHS becomes
= cot x + cot( π

3
+ x) - cot( π

3
− x)

= 1

tan x
+ 1

π
- 1

π
tan( +x) tan( −x)
3 3
π π
1−tan x tan 1+tan x tan
tan A+tan B tan A−tan B
= +( -( ... [∵ tan(A + B) = ( and tan(A - B) = ( ]
1 3 3

π
) π
) ) )
tan x 1−tan A tan B 1+tan A tan B
tan +tan x tan −tan x
3 3

1− √3 tan x 1+ √3 tan x
= 1

tan x
+( ) -( )
√3+tan x √3−tan x

(1− √3 tan x)( √3−tan x)−(1+ √3 tan x)( √3+tan x)


= 1

tan x
+( )
( √3+tan x)( √3−tan x)

2 2
(√3−tan x−3 tan x+ √3 tan x)−(√3+3 tan x+tan x+ √3 tan x)
= 1

tan x
+( 2
)
(3− tan x)

(0−4 tan x−4 tan x+0)


= 1

tan x
+( 2
)
(3− tan x)

= 1

tan x
-( 8 tan x

2
)
((3− tan x))

2 2 2 2
(3− tan x)−8 tan x (3− tan x)−8 tan x
=( 2
) =( 2
)
tan x(3− tan x) tan x(3− tan x)

2
1−3 tan x
= 3( 3
)
(3 tan x− tan x)

3
3 tan x− tan x
=3× 1

tan 3x
... (as tan 3x = 2
)
1−3 tan x

= cot 3x
LHS = RHS
Hence proved.
OR

Page 15 of 19
Here it is given that, A + B + C = π
sin 2A+sin 2B+sin 2C
We need to prove that, sin A+sin B+sin C
= 8 sin A

2
sin
B

2
sin
C

Proof: Taking LHS, we have,


sin 2A+sin 2B+sin 2C
L. H. S =
sin A+sin B+sin C

Where, sin 2A + sin 2B + sin 2C = 2sin A cos A + 2sin(B + C)cos(B - C)


A+B A−B
[ By using, sin A + sin B = 2 sin( 2
) cos(
2
)

and sin 2A = 2sin A cos A]


Since A + B + C = π
⇒ B + C = 180 - A
∴ sin 2A + sin 2B + sin 2C = 2sin A cos A + 2sin(π - A)cos(B - C )

= 2sin Acos A + 2sin A cos(B - C)


= 2sin A{cos A + cos(B - C)}
( but cos A = cos { 180 - ( B + C ) } = - cos ( B + C )
And now using
A+B −A+B
cos A - cos B = 2 sin( 2
) sin(
2
)

So, sin 2A + sin 2B + sin 2C = 2sin A{2sin B sin C}


= 4sin A sin B sin C
= 32 sin cos sin cos sin cos
A

2
A

2
B

2
B

2
c

2
c

Now, take denominator we have


B+C B−C
sin A + sin B + sin C = sin A + {2 sin( 2
) cos(
2
)}

π−A B−C
= sin A + {2 sin( 2
) cos(
2
)}

B−C
= sin A + {2 cos( A

2
) cos(
2
)}

B−C
= 2 sin A

2
cos
A

2
+ {2 cos( A

2
) cos(
2
)}

B−C
= 2 cos A

2
{sin
A

2
+ cos(
2
)}

B+C B−C
= 2 cos A

2
{cos(
2
) + cos(
2
)}

= 2 cos A

2
{2 cos(
B

2
) cos(
c

2
)}

= 4 cos A

2
cos(
B

2
)cos(
c

2
)

Therefore,
A A B B C C
32 sin cos sin cos sin cos
sin 2A+sin 2B+sin 2C
L. H. S =
sin A+sin B+sin C
= 2 2

A
2

B
2

C
2 2

4 cos cos cos


2 2 2

= 8 sin A

2
sin
B

2
sin
c

= R.H.S
Section E
36. i. n(A × A) = 9
⇒ n(A) ⊂ n(A) = 9 ⇒ n(A) = 3

(-1,0) ∈ A × A ⇒ -1 ∈ A, 0 ∈ A
(0,1) ∈ A × A ⇒ 0 ∈ A, 1 ∈ A
⇒ -1, 0, 1 ∈ A

Also, n(A) = 3 ⇒ A = (-1, 0, 1)


Hence, A = {-1, 0, 1}
Also, A × A = {-1, 0, 1} × {-1, 0, 1}
= {(-1, -1), (-1, 0), (-1, 1), (0, -1), (0, 0), (0, 1), (1, -1), (1, 0), (1, 1)}
Hence, the remaining elements of A × A are
(-1, -1), (-1, 1), (0, -1), (0, 0), (1, -1), (1, 0) and (1, 1).
ii. Given, (A × B) = 6 and (A × B) = {(1, 3), (2, 5), (3, 3)}
We know that Cartesian product of set A = {a, b} & B = {c, d} is A × B = {(a, c), (a, d), (b, c), (b, d)}
Therefore, A = {1, 2, 3} & B = {3, 5}
⇒ A × B = {(1, 3), (1, 5), (2, 3), (2, 5), (3, 3), (3, 5)}

Thus, remaining elements are A × B = {(1, 5), (2, 3), (3, 5)}

Page 16 of 19
iii. If the set A has 3 elements and set B has 4 elements, then the number of elements in A × B = 12
OR
Clearly, A is the set of all first entries in ordered pairs in A × B and B is the set of all second entries in ordered pairs in A × B
∴ A = {a, b} and B = {1, 2, 3}
37. i. Let the Priyanka visits four cities Delhi, Lucknow, Agra, Meerut are respectively A, B, C and D. Number of way's in which
Priyanka can visit four cities A, B, C and D is 4! i.e. 24
∴ n(S) = 24

Clearly, sample space for this experiment is


⎧ ABC D, ABDC , AC BD, AC DB, ADBC , ADC B
⎪ ⎫

⎪ ⎪
⎪ ⎪
BAC D, BADC , BC AD, BC DA, BDAC , BDC A
S= ⎨ ⎬
⎪ C ABD, C ADB, C BAD, C BDA, C DAB, C DBA, ⎪

⎩ ⎪

⎪ ⎪
DABC , DAC B, DC AB, DC BA, DBAC , DBC A

Let E1 be the event that Priyanka visits A before B.


Then,
E1 = {ABCD, ABDC, ACBD, ACDB, ADBC, ADCB, CABD, CADB, CDAB, DABC, DACB, DCAB}
⇒ n(E1) = 12
n( E1 )
∴ P(she visits A before B) = P(E1) = =
12

24
=
1

2
n(S)

ii. Let the Priyanka visits four cities Delhi, Lucknow, Agra, Meerut are respectively A, B, C and D. Number of way's in which
Priyanka can visit four cities A, B, C and D is 4! i.e. 24
∴ n(S) = 24

Clearly, sample space for this experiment is


⎧ ABC D, ABDC , AC BD, AC DB, ADBC , ADC B
⎪ ⎫

⎪ ⎪
⎪ ⎪
BAC D, BADC , BC AD, BC DA, BDAC , BDC A
S= ⎨ ⎬
⎪ C ABD, C ADB, C BAD, C BDA, C DAB, C DBA, ⎪

⎩ ⎪

⎪ ⎪
DABC , DAC B, DC AB, DC BA, DBAC , DBC A

E1 = {ABCD, ABDC, ACBD, ACDB, ADBC, ADCB, CABD, CADB, CDAB, DABC, DACB, DCAB}
⇒ n(E1) = 12
n( E1 )
∴ P(she visits A before B) = P (E 1) = =
12

24
=
1

2
n(S)

iii. Let the Priyanka visits four cities Delhi, Lucknow, Agra, Meerut are respectively A, B, C and D. Number of way's in which
Priyanka can visit four cities A, B, C and D is 4! i.e. 24
∴ n(S) = 24

Clearly, sample space for this experiment is


⎧ ABC D, ABDC , AC BD, AC DB, ADBC , ADC B ⎫
⎪ ⎪
⎪ ⎪
⎪ ⎪
BAC D, BADC , BC AD, BC DA, BDAC , BDC A
S= ⎨ ⎬
⎪ C ABD, C ADB, C BAD, C BDA, C DAB, C DBA, ⎪


⎪ ⎪


DABC , DAC B, DC AB, DC BA, DBAC , DBC A

Let E3 be the event that she visits A first and B last.


Then,
E3 = {ACDB, ADCB}
n(E3) = 2
∵ P(she visits A first and B last) = P(E3)
n( E3 ) 2 1
= = =
n(S) 24 12

OR
Let the Priyanka visits four cities Delhi, Lucknow, Agra, Meerut are respectively A, B, C and D. Number of way's in which
Priyanka can visit four cities A, B, C and D is 4! i.e. 24
∴ n(S) = 24

Clearly, sample space for this experiment is


⎧ ABC D,

ABDC , AC BD, AC DB, ADBC , ADC B ⎫

⎪ ⎪
⎪ ⎪
BAC D, BADC , BC AD, BC DA, BDAC , BDC A
S= ⎨ ⎬
⎪ C ABD, C ADB, C BAD, C BDA, C DAB, C DBA, ⎪
⎪ ⎪

⎪ ⎭

DABC , DAC B, DC AB, DC BA, DBAC , DBC A

Page 17 of 19
Let E4 be the event that she visits A either first or second. Then,
E4 = {ABCD, ABDC, ACBD, ACDB, ADBC, ADCB, BACD, BADC, CABD, CADB, DABC, DACB}
⇒ n(E4) = 12
Hence, P(she visits A either first or second)
n( E4 )
12 1
= P (E4 ) = = =
n(S) 24 2

38. i. (x + iy)(2 - 3i) = 4 + i


2x − (3x)i + (2y)i − 3yi2 = 4 + i
2x + 3y + (2y − 3x)i = 4 + i
Comparing the real & imaginary parts,
2x + 3y = 4 ...(i)
2y − 3x = 1 ...(ii)
Solving eq (i) & eq (ii), 4x + 6y = 8
−9x + 6y = 3
5
13x = 5 ⇒ x = 13

y= 14

13
5 14
∴ (x, y) = ( 13
, 13
)
2
(1+i)
ii. x + iy = 2−i
2 2 2
(1+i) 1+2i+i 2i(2+i) 4i+2i
x + iy = 2−i
=
2−i
=
2−i
2i
= =
2
(2−i)(2+i) 4−i

4i−2 −2
= 4+1
=
5
+
4i

5
−2 −2
⇒ x= 5
,y= 4

5
⇒ x+y= 5
+
4

5
=
2

5
100
1−i
iii. We have ( 1+i
) = a + bi
100
1−i 1−i
⇒ (
1+i
×
1−i
) = a + bi
2 100
1+ i −2i
⇒ (
2
) = a + bi
1−i
100
1−1−2i
⇒ (
1+1
) = a + bi
100
−2i
⇒ (
2
) = a + bi

⇒ (-i)100 = a + bi
⇒ i100 = a + bi
⇒ (i4)25 = a + bi
⇒(1)25 = a + bi
⇒ 1 = a + bi

⇒ 1 + 0i = a + bi

Comparing the real and imaginary parts,


We have a = 1, b = 0
Hence (a, b) = (1, 0)
OR
Given, (a - 2, 2b + 1) = (b - 1, a + 2)
Comparing x coordinates of both the sides, we get,
a-2=b-1
∴ a - b = 1 ...(1)

Comparing y coordinates of both the sides, we get,


2b + 1 = a + 2
∴ a - 2b = -1 ...(2)
Subtracting equation (2) from (1), we get,
(a - a) + (-b - (-2b)) = 1 - (-1)
∴ (-b + 2b) = 1 + 1

∴ b = 2

Put this value in equation (1), we get,

Page 18 of 19
a-2=1
∴ a = 3

Page 19 of 19

You might also like